nclex rn qbank princeton review

अब Quizwiz के साथ अपने होमवर्क और परीक्षाओं को एस करें!

The nurse in the primary care office is speaking with a client who has contact dermatitis on both hands. The client wants to know how to manage the condition. Which interventions should the nurse recommend to the client? Select all that apply. Applying a cold pack to the area can help Corticosteroid cream is acceptable to use Correct Answer (Blank) Using soap without fragrance is recommended Correct Answer (Blank) Avoid heat that can exacerbate symptoms Correct Answer (Blank) Rubbing the area can alleviate symptoms Frequent handwashing is important

Rationale: Exposure to heat or cold may cause or exacerbate contact dermatitis. Rubbing the area may also exacerbate or spread symptoms. While washing hands after exposure to possible irritants is recommended, frequent handwashing is not. Soap with fragrance is an external irritant and may exacerbate symptoms, so fragrance-free soap is recommended. A barrier cream containing a corticosteroid is the most frequently prescribed topical ointment.

An off-duty nurse arrives at a park and is told by a bystander that a child is choking and needs assistance. The bystander has already called 911. The nurse observes an approximately 8-year-old child with cyanosis and an inability to breathe who remains conscious and standing. What should the nurse do next? A Deliver two rescue breaths. B Stand behind the child and administer abdominal thrusts. Correct Answer (Blank) C Instruct the child to lay down and begin CPR. D Check the child's carotid pulse.

Rationale: For a conscious choking victim, according to basic life support (BLS) guidelines by the American Heart Association (AHA), the next action by the nurse should be to perform abdominal thrusts (i.e., the Heimlich Maneuver) to attempt to clear the airway obstruction. Attempting to deliver rescue breaths or checking the carotid pulse of a conscious choking victim would not be indicated. If the child were to become unconscious, then chest compressions should be initiated.

A client with a known large abdominal aortic aneurysm develops a sudden change in level of consciousness and tachycardia. The client's blood pressure is 72/48 mm Hg. What should the nurse do first? A Activate the hospital's emergency response team. Correct Answer (Blank) B Obtain a 12-lead electrocardiogram. C Conduct a complete head-to-toe physical assessment. D Page the client's health care provider.

Rationale: The client is exhibiting signs and symptoms of an abdominal aortic aneurysm (AAA) rupture. The nurse's first action should be to activate the hospital's emergency response team, as this client needs immediate advanced care. The nurse is anticipating the need for rapid action and surgical intervention to avoid the death of the client. While notifying the client's health care provider and obtaining a 12-lead electrocardiogram (ECG) may be needed, activating the emergency response team should be done first. Similarly, the nurse does not have time complete a head-to-toe physical assessment before activating the emergency response team.

An adult client arrives at the clinic after being stung by a bee. The nurse notes that the client is having difficulty breathing, is audibly wheezing and has swollen lips. What is the nurse's highest priority? A Administer epinephrine. Correct Answer (Blank) B Administer a bronchodilator. C Obtain an arterial blood gas. D Obtain a home medication list.

Rationale: The client's condition indicates the high likelihood of a life-threatening anaphylactic reaction to a bee sting, with an obstructed airway due to bronchoconstriction and a high potential for hypoxemia. While obtaining a home medication list and obtaining arterial blood gases may be part of the care provided to the client, the highest priority is to administer epinephrine. Epinephrine is a critical drug in the treatment of anaphylaxis. Relieving the vasoconstriction effects on bronchial muscles with epinephrine could be life-saving in this situation. A bronchodilator may also be prescribed, but not before epinephrine has been administered.

The nurse in the primary health care provider's office is speaking with a 40-year-old male client whose most recent hemoglobin A1C level was 9%. The client states that he is motivated to make lifestyle changes to better manage his disease. What interventions should the nurse recommend for this client? Select all that apply. Minimize intake of caffeinated beverages. Start a weight loss program until BMI is below 25. Correct Answer (Blank) Schedule an appointment with a registered dietitian. Correct Answer (Blank) Engage in regular physical activity, such as walking. Correct Answer (Blank) Eliminate all consumption of alcohol. Check the blood sugar several times a day, ideally before eating. Correct Answer (Blank)

Rationale: The client's hemoglobin A1C level indicates the client is not managing their diabetes well. An A1C level of 7% or less is the goal for clients with diabetes. Effective diabetes management should include daily or more frequent blood sugar monitoring, learning how to count carbohydrates and eat appropriately-sized meals, maintaining a healthy weight (i.e., a BMI between 18.5 and 24.9, per CDC guidelines) and engaging in regular physical activity. Avoiding caffeine or eliminating all alcoholic beverages is not required.

A 3-year-old child is being treated in the emergency department after ingestion of one ounce of a liquid narcotic. Which action should the nurse perform first? A Assess airway, breathing, circulation and level of consciousness Correct Answer (Blank) B Start the ordered intravenous fluids C Obtain blood and urine samples D Prepare for gastric lavage

Rationale: The first step in treatment of a toxic exposure or ingestion is to assess the airway, breathing and circulation (ABCs), particularly when the substance is known to cause CNS depression, such as a narcotic. The level of consciousness will also be an important indicator of the CNS-depressant effects of the medication. The other actions are correct and also important, but they are not the first priority.

The nurse is caring for a client diagnosed with chronic obstructive pulmonary disease. The client reports persistent dyspnea. Which action should the nurse take? A Assist the client with pursed-lip breathing. Correct Answer (Blank) B Place the client in low-Fowler's position. C Lower the rate of oxygen flow. D Instruct the client to breathe into a paper bag.

Rationale: The nurse should assist the client with pursed-lip breathing. Pursed-lip breathing during periods of dyspnea in clients with chronic obstructive pulmonary disease (COPD) helps to control the rate and depth of respirations. This will also help to prevent alveolar collapse and improve oxygenation. The other actions are not appropriate for this client.

The nurse is caring for a client who has a wound with a closed drainage system in place. Which of the following assessments findings should be documented each time the collection chamber is emptied? A Color of drainage Correct Answer (Blank) B Urine output amount C Auscultation of bowel sounds D Size of wound measurement

Rationale: When a drain is emptied, the nurse should assess and document the drainage color and amount as well as if there are clots or sediment present in the drainage. Urine output, bowel sounds, and wound measurements are assessed frequently but are not necessary to be assessed each time the collection chamber is emptied.

The nurse is caring for a client who experienced an atonic seizure while in bed. Which action should the nurse take? Reorient the client Administer prescribed phenytoin Request a CT of the head Turn the client on their side

Reorient the client Rationale: An atonic seizure is characterized by loss of muscle tone and a period of confusion after the seizure occurs. The nurse should reorient the client after the seizure. Administering phenytoin is indicated for status epilepticus or for the prevention of seizures. This does not address the effects experienced after the seizure. The client's seizure occurred while in bed. Requesting a CT scan of the head to assess for injuries is not indicated. Turning the client on their side will prevent aspiration. However, an atonic seizure is not characterized by vomiting or salivation.

The nurse is preparing to perform a prescribed gastric lavage on a conscious client. How will the nurse position the client to facilitate the procedure? Trendelenburg Supine High-Fowler's Prone

High-Fowler's Rationale: A gastric lavage includes introducing fluid into the stomach via a nasogastric tube. Clients should be positioned sitting up (high-Fowler's) to avoid aspiration of fluid. Placing the client in a Trendelenburg position, with the head of the bed lower than the foot of the bed, increases the risk of aspiration. A supine (flat back) and a prone (face down) position increase the risk of aspiration and do not promote gastric emptying.

The nurse is caring for a client with increased intracranial pressure who is mechanically ventilated. Which of the following actions by the nurse is appropriate? Hyperventilating the client with 100% oxygen prior to suctioning Educating the client to cough to clear respiratory secretions Assessing oxygen saturation every 6 hours Increasing the positive end-expiratory pressure

Hyperventilating the client with 100% oxygen prior to suctioning Rationale: Suctioning can increase intracranial pressure (ICP) but hyperoxygenation prior to suctioning reduces the increase in pressure. Coughing should be avoided, and positive end-expiratory pressure (PEEP) should be as low as possible to avoid increasing the ICP further. Oxygen saturation should be monitored continuously rather than every 6 hours.

A nurse is providing care to a client who has completed hemodialysis treatment. Which assessment finding indicates a complication that should be reported to the healthcare provider? A Weight loss of 1 kilogram B Absence of a bruit Correct Answer (Blank) C Blood pressure of 99/50 mmHg D Bleeding at the access site

Rationale: A bruit to the AV fistula is a normal finding. The absence of a bruit may indicate decreased blood flow to the area. Weight loss is common due to the removal of fluid during hemodialysis. A blood pressure of 100/50 mmHg is on the lower side of normal. A decrease in blood pressure is not uncommon after hemodialysis. The nurse should monitor the client's vital signs closely. Bleeding at the access site is not uncommon, particularly if anticoagulants were given during therapy. The nurse should apply pressure to the site and monitor for uncontrolled bleeding.

A nurse is developing a teaching plan for a 60-year-old client with chronic obstructive pulmonary disease (COPD). The teaching will include ways to reduce the frequency of exacerbations. Which of the following will the nurse include in the instructions? Obtain the pneumococcal polysaccharide 23 vaccine Use the prescribed long-acting beta-agonist whenever there is shortness of breath Decrease the amount of fluids consumed Engage in outdoor activities in the spring and fall when it is not as hot outside

Obtain the pneumococcal polysaccharide 23 vaccine Rationale: An exacerbation of COPD is defined as an event in the course of the disease characterized by worsening in the patient's respiratory symptoms beyond the normal day-to-day variations. An exacerbation also leads to a change in medication. Primary causes of an acute exacerbation include tracheobronchial infection and air pollution, including pollen. Clients with COPD experience thickened mucus and therefore should maintain fluid intake. Studies have demonstrated that vaccination against influenza and pneumonia decreases the risk for exacerbation in patients with COPD. The Centers for Disease Control (CDC) recommends pneumococcal polysaccharide 23 (PPSV23) for all adults 65 years or older, people 2 through 64 years old with certain medical conditions, and adults 19 through 64 years old who smoke cigarettes. Long-acting bronchodilator treatment is the cornerstone for the treatment of stable COPD, and it significantly decreases the risk for exacerbations but is not used for rescue therapy.

A nurse is assessing a 48-hour post-surgical client with an abdominal wound. Which clinical manifestation indicates that the client may be developing a wound infection? A The wound edges are separated. B The client's oral temperature is 99.4°F (37.4°C). C The wound drainage is brown and thick. Correct Answer (Blank) D The client's heart rate is 105 beats/min.

Rationale: Thick, brown drainage is considered purulent and is indicative of infection. Normal wound drainage should be thin and clear or serosanguinous. Separation of wound edges is indicative of dehiscence, a serious wound complication. An oral temperature of 99.4°F (37.4°C) is within normal range. An elevated heart rate is not a specific sign of wound infection. Tachycardia can be due to pain or other physiological responses.

The nurse is developing as plan of care for a postoperative client following a radical left mastectomy. Which nursing problem should be the priority for this client? A Acute pain related to the surgery Correct Answer (Blank) B Risk of infection of the surgical site C Anxiety related to the cancer diagnosis D Impaired left arm circulation (lymphedema)

Rationale: A radical mastectomy is performed to treat invasive breast cancer and involves the removal of the breast, the nipple and areola, as well as a portion of the axillary lymph nodes. Using Maslow's hierarchy of needs to prioritize nursing care and interventions, the acute post-surgical pain (a basic, physiological need) is the priority problem. Next, the nurse should focus on prevention of lymphedema, alleviating the client's anxiety and monitoring for signs of infection at the surgical site.

The nurse is assessing a client with diverticulitis. Which finding would the nurse report immediately? A Abdomen appears distended upon inspection. B Bowel sounds are hypoactive upon auscultation. C Abdomen is rigid upon light palpation. Correct Answer (Blank) D Pain is felt in the left lower quadrant upon deep palpation.

Rationale: A rigid abdomen upon light palpation is a hallmark indication of peritonitis. The nurse should report this finding immediately to prevent sepsis. Diverticulitis is an infection and inflammation of the bowel mucosa. A distended abdomen is an expected finding in clients with diverticulitis. Constipation and hypoactive bowel sounds are expected findings with diverticulitis due to the inflammatory response. Diverticulitis occurs in the colon. Pain in the left lower quadrant of the abdomen is an expected response due to the anatomical location of the colon.

A client is transferred from the post-anesthesia care unit to the medical-surgical unit after an appendectomy. Which action should the nurse on the medical-surgical unit perform first? A Orient the client to the unit B Take the client's vital signs Correct Answer (Blank) C Review the postoperative orders D Ask the client about pain

Rationale: Although all these actions are appropriate, the first assessment or data collected should be the client's vital signs. After surgery, a client may still experience side effects from the surgery and the anesthetic agents used. Therefore, vital signs provide important information about the client's hemodynamic and respiratory status. Then, the nurse should evaluate the client's level of pain and implement interventions to alleviate the client's pain.

The nurse is caring for a client in the late stages of amyotrophic lateral sclerosis. Which finding is consistent with this diagnosis? A Confusion B Shallow respirations Correct Answer (Blank) C Tonic-clonic seizures D Loss of half of visual field

Rationale: Amyotrophic lateral sclerosis (ALS) is a progressive neurodegenerative disease that affects nerve cells in the brain and spinal cord. In ALS, upper and lower motor neurons degenerate and stop sending messages to muscles. All muscles eventually weaken and atrophy, including the muscles needed to maintain effective respirations. People eventually lose their ability to speak, eat, move and breathe. The other findings are not typically seen with ALS.

Which assessment findings should make the nurse suspect that the client might have amyotrophic lateral sclerosis (ALS)? A Complaints of double vision and light sensitivity B Loss of sensation in the extremities C Progressive hearing loss in the last month D Fatigue, progressive muscle weakness and twitching Correct Answer (Blank)

Rationale: Amyotrophic lateral sclerosis (ALS) typically has a gradual onset, which is generally painless. Progressive muscle weakness is the most common initial symptom in ALS. Other early symptoms vary but can include tripping, dropping things, abnormal fatigue of the arms and/or legs, slurred speech, muscle cramps and twitches and/or uncontrollable periods of laughing or crying.

A child is injured on the school playground and appears to have a fractured leg. What is the first action that the school nurse should take? A Immobilize the limb and joints above and below the injury B Call for emergency transport to the hospital C Assess the child and the extent of the injury Correct Answer (Blank) D Apply cold compresses to the injured area

Rationale: Application of the nursing process dictates that assessment is the first step in the provision of care. The 6 Ps of vascular impairment (pain, pulse, pallor, paresthesia, paralysis and poikilothermia (coolness) can be used as a guide for assessment of the injured leg. The other options would be done in this sequence —immobilize, call 911 and then apply ice as indicated.

The nurse is educating a client with chronic obstructive pulmonary disorder (COPD) on diaphragmatic breathing. What will the nurse include in the teaching? A "Take a deep breath and hold it for a few seconds." B "Put both hands on your chest and breathe in deeply." C "Purse your lips and take in a deep breath." D "Breathe in through your nose and let your abdomen rise." Correct Answer (Blank)

Rationale: Diaphragmatic breathing focuses on using the abdominal muscles to guide the breath. The abdomen should rise upon inhalation through the nose and contract upon exhalation through the mouth. Holding the breath for a few seconds upon inhalation is a technique used with incentive spirometry. One hand should be placed on the middle of the chest and the other on the abdomen. The client should be instructed to purse their lips upon exhalation.

The nurse is caring for a client who is receiving continuous tube feedings and medication administration through a nasogastric tube. The nurse will include which of the following in the plan of care? A Verify tube placement every shift. B Flush the tube with sterile saline after administering medications. C Assess the skin around the patient's nares. Correct Answer (Blank) D Discard the gastric aspirate.

Rationale: Discarding gastric contents may lead to an electrolyte imbalance. Gastric aspirate should be emptied into a clean container and re-instilled once the volume is measured. Tube placement should be verified every 4 hours or per hospital policy. Gastric tubes are flushed with water, sterile or tap, based on hospital policy. The skin around the nares should be assessed for breakdown.

The nurse is providing the client who takes digoxin and furosemide with dietary instructions. The nurse should reinforce that the combination of these medications can result in which outcome? A Edema B Irritability C Arrhythmias Correct Answer (Blank) D Oliguria

Rationale: Furosemide is an effective diuretic but electrolyte depletion may occur. Concurrently taking furosemide and digoxin exaggerates the metabolic effects of hypokalemia, especially alterations in cardiac rate and rhythm, and contributes to digitalis toxicity. Digitalis toxicity may stimulate almost every known type of dysrhythmia. The effects of hypokalemia include fatigue (not excitability) and polyuria (not oliguria); digitalis toxicity can cause nausea, vomiting, anorexia and weight loss (not weight gain). Foods rich in potassium include avocados, bananas, peas and beans, spinach and tomatoes.

The nurse is caring for a client who received intravenous furosemide for a serum potassium level of 6.0 mEq/L. Which of the following findings on the client's electrocardiogram indicates that the treatment has been effective? A Tall, peaked T waves B PR interval of 0.18 seconds Correct Answer (Blank) C Absent P waves D QRS duration of 0.20 seconds

Rationale: Hyperkalemia can cause several electrocardiogram abnormalities, such as tall, peaked T waves, prolonged PR intervals, flat or absent P waves, and wide QRS complexes. A normal PR interval is 0.12 - 0.20 seconds; therefore, a PR interval of 0.18 seconds on the client's electrocardiogram indicates the effective resolution of the client's hyperkalemia. Tall, peaked T waves, absent P waves, and a prolonged QRS of 0.20 seconds indicate treatment for hyperkalemia has not been effective.

The nurse is providing care for a 40-year-old client suspected of having Guillain-Barré syndrome. Which intervention should the nurse plan for? A Implementation of airborne precautions B Administration of immunoglobulins Correct Answer (Blank) C A bone marrow biopsy D Genetic testing of the client's children

Rationale: Guillain-Barré Is a syndrome with an unknown etiology occurring after a bacterial or viral infection. It is characterized by muscle weakness and paralysis that occurs in an ascending manner. It may result in paralysis of the respiratory muscles requiring mechanical ventilation for the client. Intravenous immunoglobulins (IV Ig) are used to treat Guillain-Barré in the early phase. They are believed to interfere with antigen presentation and help to modulate the body's immune response. The other interventions are not appropriate or necessary for this condition.

The nurse is planning care for a client diagnosed with Guillain-Barré syndrome. Which problem should the nurse identify as a priority? A Altered bowel elimination B Partial or total immobility C Nutritional deficits D Difficulty breathing Correct Answer (Blank)

Rationale: Guillain-Barré syndrome (GBS) is an acute autoimmune disorder characterized by varying degrees of motor weakness and paralysis. Difficulty breathing is the priority problem because acute respiratory failure due to muscle weakness and respiratory paralysis can occur, requiring ventilatory support. The other problems are also potential problems but are not life-threatening. The nurse should prioritize using the ABC decision-making approach.

A nurse is assessing a client with chronic pulmonary disease. The chest x-ray reveals hyperinflation of the alveoli. Which clinical manifestation does the nurse expect to assess? A Anterior to posterior ratio of 1:2 B Dullness upon percussion of the chest C Clubbing of the fingernails Correct Answer (Blank) D PaO2 level of 85 mmHg

Rationale: Hyperinflation of the alveoli prevents adequate gas exchange in the lungs. Impaired oxygenation leads to chronic hypoxia and clubbing; an increased angle of the nail of more than 160 degrees. Clients with chronic pulmonary disease will develop a barrel chest with an AP:lat ratio of 1:1. Hyperinflation of the alveoli results in air trapping. Hyperresonance is expected with percussion. Decreased gas exchange in the alveoli results in hypoxemia and partial pressure of oxygen (PaO2) levels below 80 mmHg.

The nurse is educating an older client with osteoporosis on dietary management. What should the nurse include in the teaching? A "Drink no more than 1 alcohol drink per day." B "Regulate the amount of green leafy vegetables in your diet." C "Consume at least 700 mg of calcium a day." D "Limit the amount of carbonated beverages you drink." Correct Answer (Blank)

Rationale: Osteoporosis is characterized by low bone density and decreased calcium levels. Carbonated beverages contain high amounts of phosphorus. Phosphorus increases the rate of calcium loss in the body. High alcohol intake decreases bone formation and increases bone absorption. Clients should drink no more than 2 to 3 drinks per day. Green leafy vegetables contain vitamin K, a vitamin essential for bone metabolism. Green leafy vegetables should be encouraged. Older adults should consume at least 1,000 mg of calcium daily.

The nurse is assessing a client following the placement of a right subclavian central line. The nurse notes the client has an oxygen saturation of 88%, respiratory rate of 32, and a client report of sudden onset of shortness of breath. Which finding observed by the nurse would indicate the client is experiencing a pneumothorax? A Absent breath sounds over the right upper lobe Correct Answer (Blank) B Paradoxical chest wall movement on the right C Sudden loss of consciousness D Muffled heart sounds

Rationale: Pneumothorax is a potential iatrogenic complication from subclavian central line insertion. The apex of the lung is directly beneath the insertion site and can be punctured by the needle. Paradoxical chest wall movement is associated with flail chest. Sudden loss of consciousness is not associated with central line insertion. Muffled heart sounds are often associated with cardiac tamponade.

The nurse is planning care for a 3-month-old infant in the immediate postoperative period after placement of a ventriculoperitoneal shunt for hydrocephalus. In anticipation of complications of the procedure, the nurse should take which action? A Pump the shunt at intervals to assess for proper function B Maintain the infant in supine position C Assess for abdominal distention or taut abdominal wall Correct Answer (Blank) D Begin formula feedings when infant is alert

Rationale: The nurse should observe for abdominal distention or a taunt abdominal wall because cerebrospinal fluid could cause peritonitis or a postoperative ileus as a complication of distal catheter placement. The child does not need to remain supine and can be placed in an upright position. The infant would be started on clear liquids initially, not formula. The shunt will not be pumped.

The nurse is planning care for a client with tuberculosis. Which intervention will the nurse implement to prevent transmission of the disease? A Infuse prescribed antibiotics to the client B Ensure the client wears an N95 mask when transported C Administer humidified oxygen to the client D Place the client in a negative-airflow room Correct Answer (Blank)

Rationale: Tuberculosis is an infectious disease transmitted via the airborne route. Airborne particles must be contained in the client's room using negative airflow. Infusing antibiotics will treat the disease but will not prevent transmission. The N95 mask should be worn by healthcare providers. The client should wear a surgical mask if transportation to another unit is required. Administering oxygen to the client will assist with oxygenation but will not prevent disease transmission.

The nurse is taking care of a client experiencing dyspnea. The client becomes visibly anxious and begins to hyperventilate. Which intervention will best promote airway clearance? A Voluntary coughing B Deep breathing exercises C Incentive spirometry D Pursed-lip breathing Correct Answer (Blank)

Rationale: Pursed-lip breathing helps to prolong expiration and decrease airway collapse. Clients should be instructed to purse their lips to create a narrow opening for air to flow through. This will improve air exchange and decrease the shortness of breath. Voluntary coughing helps to clear secretions but will not correct the existing dyspnea. Deep breathing exercises are beneficial for clients experiencing hypoventilation. Incentive spirometry helps clients practice deep breathing and expand their lungs. This intervention is not indicated for clients who are presently experiencing shortness of breath.

To evaluate the effectiveness of antiretroviral therapy for a client infected with human immunodeficiency virus (HIV), which laboratory test result will the nurse plan to review? A Nucleic acid amplification test B Rapid HIV antibody test C Viral load test Correct Answer (Blank) D Western blot test

Rationale: Viral load refers to the amount of HIV circulating in the blood. The effectiveness of antiretroviral therapy (ART) is measured by the decrease in the amount of HIV virus, i.e., viral load, detectable in the blood. The goal is for the viral load to be so low that it is deemed undetectable. An undetectable viral load does not mean that the client is cured or can no longer transmit the disease. The other tests are used to detect HIV antibodies, which remain positive even with effective ART. A nucleic acid amplification test (NAAT) is commonly used to diagnose a gonorrhea infection.

The nurse is assessing a client's healing wound to the coccyx area. The nurse notes moist, light red tissue surrounding the wound. The nurse knows the wound is in which phase of healing? A Inflammatory B Maturation C Proliferation Correct Answer (Blank) D Hemostasis

Rationale: Wound healing occurs in four phases. The proliferation phase is characterized by new tissue formation that appears red in color. Granulation tissue should be protected to promote healing. The inflammatory phase is the second phase and lasts 2 to 3 days. It is characterized by swelling and erythema around the wound. The maturation phase is the last phase of wound healing and is characterized by collagen deposits that result in wound closure and scar formation. Hemostasis is the initial phase of wound healing and is characterized by liquid exudate containing blood and plasma.

A nurse enters a client's room to discover that the client has no pulse or respirations. After calling for help, what is the first action the nurse should take? A Start a peripheral IV B Establish an airway C Obtain the crash cart D Initiate chest compressions Correct Answer (Blank)

A nurse enters a client's room to discover that the client has no pulse or respirations. After calling for help, what is the first action the nurse should take? A Start a peripheral IV B Establish an airway C Obtain the crash cart D Initiate chest compressions Correct Answer (Blank)

The nurse is caring for a client with a serum calcium level of 15 mg/dL. Which of the following actions should the nurse take? A Reduce stimulation by keeping the room quiet. B Request a prescription for an antidiarrheal medication. C Provide foods that are rich in calcium to the client. D Request a prescription for an intravenous normal saline infusion. Correct Answer (Blank)

Rationale: A serum calcium level of 15 mg/dL indicates severe hypercalcemia. Hypercalcemia is treated by fluid volume replacement with normal saline to correct hypercalcemia-induced urinary salt wasting. Additionally, correcting the hypovolemia and the sodium in the IV fluid enhances kidney excretion of calcium. An antidiarrheal medication may be needed for clients who are experiencing hypocalcemia, not hypercalcemia, due to the increased peristalsis that the electrolyte abnormality causes within the gastrointestinal tract. Additionally, clients with hypocalcemia may have increased excitability and would benefit from reduced stimuli. Clients with hypercalcemia should have high-calcium and vitamin D foods removed from their diet until the electrolyte abnormality resolves.

The nurse is auscultating the heart of a client who has dilated cardiomyopathy. Which finding should the nurse expect to hear? A Diastolic murmur B Ventricular gallop of S3 Correct Answer (Blank) C Apical click D Split S2

Rationale: A ventricular gallop, S3 is caused by blood flowing rapidly into a distended noncompliant ventricle. This is the most common sound with left-sided heart failure. Increased left heart pressures may cause dilation of the mitral valve in the client with heart failure resulting in a systolic murmur.

The emergency room nurse is assessing a client admitted with unstable angina. Which lab test is the priority for this client? A Serum creatinine B Serum potassium C Troponin Correct Answer (Blank) D Hemoglobin

Rationale: Cardiac-specific troponin is a heart muscle protein released into circulation after injury or infarction. Normally, the level in the blood is very low, so a rise in level is diagnostic of myocardial infarction (MI) or injury. Troponin is the priority biomarker of choice in the diagnosis of acute coronary syndromes.

A nurse working in a nursing home is caring for an older adult client who has been diagnosed with a urinary tract infection. Which finding should be of greatest concern to the nurse? A Low blood pressure Correct Answer (Blank) B Cloudy urine C Suprapubic pain D Confusion

Rationale: Having a low blood pressure should be the greatest concern to the nurse. Clients with a urinary tract infection (UTI) are at risk of developing urosepsis, an infection of the blood, which can quickly lead to septic shock. Low blood pressure can be a sign of urosepsis and the beginning stage of shock. Confusion, suprapubic pain and cloudy urine are expected signs and symptoms of a UTI.

The nurse is caring for a client with a history of advanced peripheral vascular disease. Which of the following findings should be reported to the healthcare provider immediately? A Dull pain in both calves B Pedal edema 3+ by the end of the day C Redness of a unilateral extremity Correct Answer (Blank) D Diminished sensation in the lower extremities

Rationale: Increased redness in one extremity is a sign of impaired venous return and possibly venous thromboembolism. This is a reportable assessment finding. All other findings are expected in the client with PVD.

The nurse is caring for a client with altered mental status, hypotension, and decreasing urine output. Urosepsis is suspected. Which of the following urinalysis findings would support this diagnosis? A Specific gravity of 1.021 B Positive for nitrites Correct Answer (Blank) C WBC <5 D Negative for leukocyte esterase

Rationale: Normal urine contains nitrates, which are converted to nitrites by bacteria. Therefore, when nitrites are present, it is indicative of a urinary tract infection. Normal specific gravity values range from 1.010 to 1.030, therefore, this is a normal finding. WBCs less than 5 is normal as well as a negative leukocyte esterase.

The nurse in a pediatrician's office is completing a health history on an infant. The parent reports that the infant vomits forcefully after almost every feeding, has hard infrequent stools and seems very sleepy. The nurse suspects that the infant might be experiencing which condition? A Sickle cell disease B Pyloric stenosis Correct Answer (Blank) C Rotavirus D Diphtheria

Rationale: Pyloric stenosis is a condition where the pylorus increases in size causing a partial to complete obstruction between the stomach and small intestine. Manifestations include projectile vomiting, dehydration, weight loss, constipation and lethargy. The other disorders do not cause the reported symptoms.

The nurse is caring for a client who had bariatric surgery. When removing the staples from the abdominal wound, which of the following techniques should the nurse use? A Remove every other staple first and monitor wound edges. Correct Answer (Blank) B Start from the top and move downward to remove all staples. C Begin at the bottom of the incision and move upward to remove all staples. D Remove several staples from the top and bottom of the incision and monitor wound edges.

Rationale: Removing alternative staples provides strength to the incision line during removal and prevents accidental separation of the incision. If the wound edges remain approximated, remove remaining staples. This may occur on a different day based on the prescription.

A nurse is providing early postoperative care to a client. Which intervention will the nurse perform to prevent cardiovascular complications? A Encourage the client to ambulate B Teach cough and deep breathing exercises C Apply prescribed sequential compression devices Correct Answer (Blank) D Explain the use of an incentive spirometer

Rationale: Sequential compression devices are applied to the lower extremities to promote adequate blood flow and prevent cardiovascular complications, such as a blood clot. Ambulation should occur once the client has fully recovered from anesthesia. During the early postoperative period, the client requires continuous monitoring. Cough and deep breathing exercises and the use of an incentive spirometer primarily prevent respiratory complications.

The nurse is providing discharge information to a client with glaucoma. Which instruction should the nurse include? A "Avoid overusing your eyes." B "Decrease your fluid intake to control the intraocular pressure." C "Eye medications will need to be administered lifelong." Correct Answer (Blank) D "Decrease the amount of salt in your diet."

Rationale: The administration of eye drops is a critical component of the treatment plan for the client with glaucoma. The client needs to be instructed that medications will need to be taken for the rest of their life. Limiting fluid and salt intake will decrease systemic fluid volume but is usually not indicated for glaucoma. Overuse of eyes can be misinterpreted and difficult to adhere to.

The nurse is caring for a client who had abdominal surgery 4 days ago. Which of the following client findings is a priority to follow up? A Client reports a popping sensation in the abdomen. Correct Answer (Blank) B Small amounts of purulent drainage at the incision site. C Client reports increased sharp, continuous abdominal pain. D Increased swelling and redness at the incision site.

Rationale: The client's report of a popping sensation is a symptom of possible dehiscence (partial or complete separation of the tissue layers during the healing process) or evisceration (total separation of the tissue layers, allowing the protrusion of visceral organs through the incision). These surgical complications are an emergency and require immediate follow-up from the nurse. The increased swelling and drainage may indicate infection and the pain also requires assessment; however, the key is the priority finding.

The nurse is caring for a client with elevated intracranial pressure. Which of the following actions is appropriate by the nurse? A Administering osmotic diuretics Correct Answer (Blank) B Increasing intravenous fluids C Placing the client in the Trendelenburg position D Promoting the Valsalva maneuver

Rationale: The nurse should complete interventions that reduce intracranial pressure (ICP), such as administration of diuretics and implementing fluid restriction. The client's head should be placed at a 30-45 degree elevation, and extreme rotation of the neck should be avoided. The Valsalva maneuver increases ICP and should be avoided.

A nurse is caring for a client with a ventricular pacemaker who is on electrocardiogram (ECG) monitoring. When interpreting the ECG strip, proper pacemaker functioning is based on which of the following findings? A Pacemaker spikes after each QRS complex B Pacemaker spikes before each P wave C Pacemaker spikes before each QRS complex Correct Answer (Blank) D Pacemaker spikes after each P wave

Rationale: The pacemaker fires, which stimulates the ventricle to depolarize, and the QRS complex appears. Pacemaker firing is indicated by the spike on the ECG. If the spike occurs before each P wave, it indicates atrial pacing. Undersensing occurs when the pacemaker fails to detect spontaneous myocardial depolarization, which results in asynchronous pacing. Atrial or ventricular pacing spikes arise regardless of P waves or QRS complex.

The parents of a 5-month-old infant report that the infant has "vomited 9 times in the past six hours." Which complication should the nurse monitor the infant for? A Hemodilution B Metabolic alkalosis Correct Answer (Blank) C Respiratory acidosis D Hypervolemia

Rationale: Vomiting results in a loss of acid from the stomach. Prolonged vomiting results in excess loss of acid and leads to metabolic alkalosis. Manifestations of metabolic alkalosis include irritability, increased activity, hyperactive reflexes, muscle twitching and elevated pulse.

The nurse is caring for a client who has a Jackson-Pratt drain. Which of the following actions should the nurse take when emptying the drain reservoir? A Measure the amount of drainage in a graduated cylinder Correct Answer (Blank) B Remove the drain before assessing output C Flush the drain with normal saline after emptying D Hang the drain reservoir above the wound site

Rationale: When emptying the client's Jackson-Pratt (JP) drain, it is important to assess drainage (amount, color, etc.). The JP drain should not be removed by the nurse. Flushing the drain may contaminate the wound site, and the drain should be secured to the client's gown below the level of the wound site.

The nurse is examining a 2-year-old child with a tentative diagnosis of Wilm's tumor. Which statement by the child's parent should the nurse follow-up on? A "My child seems to be urinating less over the past 2 days." Correct Answer (Blank) B "All of my child's pants have become tight around the waist." C "My child has lost 3 pounds in the last month." D "My child prefers some salty foods more than others."

Rationale: Wilm's tumor is a malignant tumor of the kidney that can lead to kidney dysfunction. A recent decrease in urinary output should be investigated further as it may be a sign of renal dysfunction. Increasing abdominal girth is a common finding with a Wilm's tumor, but does not require immediate intervention by the nurse.

The nurse is preparing to perform a prescribed closed system gastric lavage. Which action should the nurse take before starting the procedure? A Obtain a 50-ml catheter tip syringe B Apply sterile gloves to both hands C Attach a Y-connecter to the nasogastric tube Correct Answer (Blank) D Set the regulator to 20 mmHg intermittent suction

Rationale: A Y-connector is necessary for a closed system gastric lavage. One port is used to infuse the irrigating solution while the other port is used to suction out stomach contents. A catheter tip syringe is necessary for an intermittent open system. Sterile gloves are not necessary for the procedure. Clean technique may be used. The suction canister should be set to continuous suction and regulated by the nurse by turning it off or clamping when appropriate.

A client is evaluated in the emergency department for an ankle sprain. Which discharge instructions should the nurse provide? Select all that apply. "It is important to avoid standing or walking without crutches until after your follow-up visit." Correct Answer (Blank) "You should apply an ice pack to your ankle for 20 to 30 minutes at a time, 3 to 4 times a day." Correct Answer (Blank) "Rest your foot and ankle tonight, but start strengthening exercises tomorrow morning." "To help decrease swelling, it is important to keep your ankle elevated when you are resting." Correct Answer (Blank) "Wrap your foot and ankle tightly with the elastic wrap and leave on overnight."

Rationale: A sprain is a common musculoskeletal injury resulting from the excessive stretching of a ligament. Initial treatment of a sprain would include rest, ice, compression, and elevation. Strengthening exercises may induce a re-injury and should not be performed. Education should include rest, including using the proper method of walking with crutches and resting the extremity. Education should also include ice. Cold therapy should be applied for 20 to 30 minutes at a time, 3 to 4 times a day. Teach the client about appropriate compression. The elastic wrap should be tight enough to provide support but not so tight that it impedes circulation. Elastic wraps should be removed when sleeping. Elevating the extremity above the level of the heart will help reduce swelling.

The nurse in an outpatient surgery center is caring for a client after cataract surgery. Which statement by the client indicates an understanding of the discharge instructions? A "I will not take tub baths until my incisions are fully healed." B "I will not drive until I have permission from my doctor." Correct Answer (Blank) C "I will follow a soft diet at home for the next five days." D "I will refrain from wearing my sunglasses for one week."

Rationale: After cataract surgery to the eyes, the client will experience changes in their vision. The teaching priority after cataract surgery pertains to safety due to impaired vision. The client should be advised to refrain from driving, operating machinery and participating in certain sports until given specific permission from the ophthalmologist. Clients should wear sunglasses to protect their eyes when outdoors and in brightly-lit rooms. Eating a modified diet or refraining from tub baths isn't necessary following this type of surgery.

The nurse is talking with the parents of a toddler who is newly diagnosed with retinoblastoma. Which point is a priority when discussing this diagnosis with the parents? A Suggest that total blindness may follow surgery B Prepare them for their child's permanent disfigurement C Inform them that even aggressive treatment is usually ineffective D There is a need for genetic counseling Correct Answer (Blank)

Rationale: Aggressive treatment of retinoblastoma can be effective. If the tumor does not respond to chemotherapy and/or radiation therapy, the eye may need to be removed; however, that does not necessarily mean the child will be permanently disfigured. Regardless, the oncologist is the person who will discuss treatment options and anticipated outcomes with the parents. The parents should be prepared for the effects of cancer on their child, but they should also understand that retinoblastoma is a rare cancer that runs in families, and there is a high risk for future offspring to be affected.

A child with Tetralogy of fallot visits the clinic several weeks before a scheduled surgery. The nurse should give priority attention to which focus? A Observation for developmental delays B Assessment of oxygenation Correct Answer (Blank) C Maintenance of adequate nutrition D Prevention of infection

Rationale: All of the responses would be important for a child diagnosed with tetralogy of Fallot. However, persistent hypoxemia causes acidosis, which further decreases pulmonary blood flow. Additionally, low oxygenation leads to development of polycythemia and may result in neurological complications.

The nurse in an emergency department is assessing a client who fell at home 24 hours ago. Which finding requires the nurse's immediate attention? A Heart rate of 98 bpm B Baseline blood pressure of 150/90 mmHg C Large bruise behind one ear Correct Answer (Blank) D Atrial fibrillation on the ECG monitor

Rationale: Although all of the findings are important, the bruising behind one ear (over the mastoid process) requires immediate attention. Known as "Battle's sign," this sign appears 1 to 2 days post skull fracture. Other signs of a skull fracture can include bruising around the eyes, blood leaking from the ear, headache, changes in orientation and level of consciousness, and nausea and vomiting.

During an initial home visit, a nurse is discussing with family members the care of their parent who is newly diagnosed with Alzheimer's disease. Which of these interventions would be most helpful at this time? A List actions to improve the client's socialization with friends B Leave a pamphlet about relaxation techniques C Discuss communication strategies to use with the client Correct Answer (Blank) D Write out an exercise routine for them to assist the client with

Rationale: Alzheimer's disease is a progressive chronic illness that affects memory and cognition. Communication is often the most challenging issue between caregivers and clients. At this initial visit, the nurse should discuss effective communication strategies that support the client and family. The other choices are important but do not address the most significant and current need.

A nurse is assessing a client who is receiving one unit of packed RBCs to treat blood loss from a duodenal ulcer. The client reports chest tightness, dyspnea, and low back pain. Which action should the nurse take? A Slow the rate of infusion, and medicate the client for pain as prescribed B Stop the blood transfusion, and begin an infusion of normal saline Correct Answer (Blank) C Stop the blood transfusion, and obtain a prescription for an antihistamine D Slow the rate of the transfusion, and obtain a prescription for a diuretic

Rationale: An acute hemolytic transfusion reaction is an immune-mediated reaction that is the result of recipient antibodies present to blood donor antigens. Antibodies already present in the recipient's plasma rapidly combine with antigens on donor erythrocytes, and hemolysis occurs. Symptoms include fever, chills, low back pain, nausea, chest tightness, dyspnea, and anxiety. Hypotension, bronchospasm, and vascular collapse may result. When a transfusion reaction is suspected, the transfusion should be immediately stopped, and the intravenous line should be kept open using appropriate fluids (usually 0.9% saline). In this type of reaction, slowing the infusion is of no benefit.

The nurse is caring for a client who has decreased adrenal function. Which intervention should the nurse include in the client's plan of care? A Prevent constipation B Encourage physical activity C Limit the number of visitors Correct Answer (Blank) D Place the client in reverse isolation

Rationale: Any stress, either physical or emotional, places additional stress on the adrenal glands, which could precipitate an Addisonian crisis in this client. The plan of care should protect the client from stress by avoiding the emotional stress of (too many) visitors and by reducing physical activity until the client's condition stabilizes.

A nurse is caring for a client in the coronary care unit. The display on the cardiac monitor indicates ventricular fibrillation. What should the nurse do first? A Perform defibrillation B Assess the level of consciousness Correct Answer (Blank) C Assess the pulse D Initiate CPR

Rationale: Artifact (interference) can mimic ventricular fibrillation on a cardiac monitor. Always treat the patient, not the monitor. If the client is truly in ventricular fibrillation, the client will be unresponsive, and no pulse will be present. The standard of care is to verify the monitor display with an assessment of the client's level of consciousness, shaking, and shouting to arouse followed by a carotid pulse check. If the client is unresponsive without a pulse in ventricular fibrillation, the most effective treatment will be electricity or defibrillation. This should be the priority: supplementing circulation using chest compressions until the defibrillator is set up and ready to deliver the shock.

Which finding should alert the nurse to the possible presence of a cataract in a client? A Farsightedness and loss of central vision B Blurred vision and reduced color perception Correct Answer (Blank) C Nearsightedness and loss of peripheral vision D Dull aching in the eye and eyelids

Rationale: As the lens becomes opaque and less able to refract light appropriately, the client will experience blurred vision and a reduced ability to distinguish among different colors. The development of a cataract does not typically cause loss of peripheral or central vision and visual acuity, nor does it result in aching of the eye or eyelids.

The nurse is caring for a client who has a wound on the leg from a motorcycle accident. During a home visit, the nurse should use which assessment parameter as an indication that this client is experiencing normal wound healing? A Green drainage from the center of the wound B Pebbled red tissue in the wound base Correct Answer (Blank) C White patches on the outside edges of the wound D Eschar over the surface of the wound

Rationale: As the wound granulates, pebbled red tissue in the wound base indicates healing. The other findings indicate the wound is not healing properly or could be indicative of infection.

The nurse in the emergency department is assessing a client diagnosed with an acute asthma attack. Which assessment finding would support this diagnosis? A Fever and chills B Sharp pain during inspiration C Loose, productive cough D Diffuse expiratory wheezes Correct Answer (Blank)

Rationale: Asthma is characterized as a hyper-responsive inflammatory disorder of the terminal bronchioles. The inflammation causes constriction of the smooth muscle around the bronchioles (bronchoconstriction). These changes make it difficult for air to enter the lungs, resulting in wheezes. The other findings are not typically seen with an acute asthma attack.

The nurse is caring for a client who is experiencing increased intracranial pressure. Which of the following should be included in the client's plan of care? A Increase intracranial blood volume. B Monitor for temperature elevation. Correct Answer (Blank) C Hold sedation medications. D Perform neurological assessments every 30 minutes.

Rationale: Avoiding a spike in body temperature is important to avoid increasing the client's intracranial pressure (ICP) therefore frequent monitoring should be performed. For the client who has an increased ICP, the nurse should strive to reduce intracranial blood volume. Sedation is a common method of minimizing cerebral oxygen demand and is often used for clients with elevated ICP. Lastly, while neurological assessments are often performed on these clients, every 30 minutes is excessive and would mean that the nurse would be waking the client frequently.

A 14-month infant is brought to the emergency department with irritability, lethargy for two days, dry skin and increased pulse rate. What additional questions should the nurse ask to assist the health care provider with determining the proper diagnosis? A Use of daycare B Change in eating habits C Reverse of sleep-wake cycles D The number of wet diapers in the past two days Correct Answer (Blank)

Rationale: Based on these clinical findings, the nurse might suspect that the infant is dehydrated. Asking about the number of wet diapers would assess for decreased urine output, a key finding in dehydration. Asking about increased concentration of the urine would also be appropriate. The other questions, while appropriate, would not provide the most helpful diagnostic information.

The nurse is caring for a client who was admitted for hyperglycemic hyperosmolar state (HHS). Which clinical finding would support this diagnosis? A Blood sugar > 600 mg/dL Correct Answer (Blank) B Positive urine ketones C Serum pH level < 7.35 D Deep, rapid breathing pattern

Rationale: Clients who suffer from type 2 diabetes mellitus are at risk for having hyperglycemic hyperosmolar state (HHS). HHS has a gradual onset with precipitating factors including poor fluid intake, infections or stress. While these are similar etiologies to diabetic ketoacidosis (DKA), there are some differences. Blood sugar levels with HHS are generally much higher (> 600 mg/dL) when compared to DKA (> 300 mg/dL). A deep, rapid breathing pattern (i.e., Kussmaul's) is associated with DKA. The serum pH for HHS is usually normal, since there is an absence of ketones and acidosis.

The nurse in a primary care office is examining a 15-month-old child with suspected otitis media. Which group of findings should the nurse anticipate? A Periorbital edema, absent light reflex and translucent tympanic membrane B Irritability, rhinorrhea, and bulging tympanic membrane Correct Answer (Blank) C Vomiting, pulling at ears and pearly white tympanic membrane D Diarrhea, retracted tympanic membrane and enlarged parotid gland

Rationale: Clinical manifestations of otitis media include irritability, rhinorrhea, bulging tympanic membrane, and pulling at the ears.

The nurse is caring for a group of assigned clients with closed-chest drainage systems in place. Which of the following clients should the nurse assess first? A The client who has continuous bubbling in the water seal chamber and respirations of 36 Correct Answer (Blank) B The client who had a chest tube placed 1 hour ago and is reporting moderate pain at the insertion site C The client who has tidaling in the water seal chamber and small amounts of crepitus around the insertion site D The client who had a thoracotomy 1 day ago and has a chest tube output of 100 mL in the past 2 hours

Rationale: Continuous bubbling in the water seal chamber indicates an air leak which places the client at risk for developing a tension pneumothorax. In addition, the client in the key is tachypneic and should be assessed immediately for airway compromise. It is normal to experience pain or crepitus at the insertion site and have an output of 100 mL in the past 2 hours.

A nurse is caring for a client diagnosed with Cushing's syndrome. While reviewing the client's medical record, which risk factor most likely caused the client to have this syndrome? A Long-term use of steriods Correct Answer (Blank) B Chronic kidney disease C Tumor in the hypothalamus D Decreased levels of cortisol

Rationale: Cushing's syndrome is different from Cushing's disease and most commonly develops as a side effect of long-term use of corticosteroids. This syndrome refers to the clinical manifestations caused by excessive levels of cortisol, including hyperglycemia and fluid retention, which can subsequently cause edema, hypertension, weight gain, glucose intolerance and protein wasting. Chronic kidney disease is not a risk factor for Cushing's syndrome.

A client is admitted for hypovolemia associated with multiple draining wounds. Which is the best method for the nurse to use to evaluate the client's fluid balance? A Hourly urine output B Presence of edema C Skin turgor D Daily weight Correct Answer (Blank)

Rationale: Daily weight is the most easily obtained and accurate means of assessing a client's fluid volume status. Skin turgor varies considerably with age. Marked excess fluid volume may already be present before fluid moves into the interstitial space and causes edema. Although very important, hourly urine outputs do not take into account fluid intake or fluid loss through insensible loss, sweating, or loss from the gastrointestinal (GI) tract or wounds.

The triage nurse in a pediatrician's office is speaking with the parent of a young child that has been having frequent episodes of diarrhea. Which instruction is most important to provide to the parent? A "Call the office immediately if your child becomes difficult to arouse." Correct Answer (Blank) B "Use a barrier ointment to prevent a diaper rash." C "Make sure to have the entire family wash their hands frequently." D "Give your child extra bottles of water throughout the day."

Rationale: Dehydration occurs when the loss of fluids is greater than the intake. Diarrhea is a common cause of dehydration in children. Rehydration and prevention of dehydration are the primary goals of care. A decrease in level of consciousness such as lethargy and difficulties arousing the child indicate severe cellular dehydration and should be reported immediately; therefore, this instruction is most important to provide to the parent. The other instructions are also appropriate to be given to the parent, but they are not as important.

The nurse is caring for a client who is undergoing radiation therapy. Which of the following assessment findings would indicate that the client is experiencing an adverse effect of this treatment? A Mild fatigue after treatment. B Desquamation of the treatment area. Correct Answer (Blank) C Reduction of urine output. D Increase in white blood cell count.

Rationale: Desquamation is an alteration of the skin that indicates radiation dermatitis. This should be monitored closely, and if severe, may necessitate interruption or cessation of radiation therapy. Fatigue is an expected finding and is not considered an adverse effect. White blood cell count may remain the same or decrease but does not increase with radiation therapy. Urine output is not affected by radiation therapy.

The nurse is evaluating the growth and development of a toddler who has acquired immunodeficiency syndrome (AIDS). The nurse should anticipate which finding in this child? A Accelerated development of fine motor skills B Delay in achievement of most developmental milestones Correct Answer (Blank) C Achieve developmental milestones at an erratic rate D Difficulty with color recognition

Rationale: Developmental delays are common in children with AIDS, and after achievement of normal development, there may be loss of milestones. The majority of children with AIDS have neurological involvement. There is decreased brain growth as evidenced by microcephaly and abnormal neurologic findings.

A client is receiving external radiation therapy for a neck tumor. The nurse assesses the client after the therapy is completed. Which finding requires prompt intervention from the nurse? A Dysgeusia B White blood cell (WBC) count of 9,000/mm³ C Platelet count of 350,000/mm³ D Dysphagia Correct Answer (Blank)

Rationale: Dysphagia, or trouble swallowing, is an injury that can occur after radiation. The nurse should assess the airway to prevent aspiration. Dysgeusia (altered taste) is a common finding after external radiation. The nurse should encourage a diet without red meat as it can be unpalatable. Decreased white blood cell (WBC) and platelet counts are common effects after radiation. The normal range of WBCs is 5,000 to 10,000/mm³. The normal platelet count is 150,000 to 400,000/mm³.

The nurse is caring for a client in the postanesthesia care unit (PACU) following corrective surgery for scoliosis. Which action should receive priority in the plan of care? A Assess the sensation and movement of the lower extremities Correct Answer (Blank) B Assist to stand at the bedside within the first few hours post-surgery C Teach client isometric exercises for the legs D Initiate the prescribed antibiotic therapy

Rationale: Following corrective surgery for scoliosis, the neurological status of the extremities requires priority attention in the PACU as well as the postoperative surgical units. Initiation of antibiotic therapy should begin as soon as possible after neurological status is obtained. Getting the patient out of bed and teaching isometric exercises will be done after the client is neurologically cleared.

The nurse is developing a plan of care for a postoperative client following the surgical creation of an ileostomy. Which intervention should the nurse implement first? A Providing emotional support B Assessing the appearance of the stoma Correct Answer (Blank) C Teaching the management of the pouch D Addressing concerns with body image

Rationale: Following the nursing process, assessing the appearance of the stoma is the intervention that should be implemented first. The nurse needs to monitor for stoma health and possible complications. Examples of complications include bleeding, infection, or lack of blood flow to the stoma. As the stoma starts to function, the nurse would explain the care of the stoma to the client. Providing emotional support, teaching how to care for the ostomy, and addressing body image concerns are important but should be implemented after assessing the ostomy.

The nurse is providing individualized nutrition counseling with a client who has chronic obstructive pulmonary disease (COPD). The client is experiencing unintended weight loss. What instructions should the nurse include in the teaching? A "Buy products that contain whole grains." B "Increase your intake of fruits and fruit juices." C "Eat high- protein snacks between meals and at bedtime." Correct Answer (Blank) D "Choose a variety of vegetables throughout the day."

Rationale: For a client with COPD, nutrition is part of the therapeutic regimen. Cachexia and muscle-wasting is a frequent, but partly reversible, complication in patients with COPD and affects the disease progression and prognosis. Weight loss in COPD is a consequence of increased energy requirements unbalanced by dietary intake. Metabolism of carbohydrates produces the most carbon dioxide for the amount of oxygen used; metabolism of fat produces the least. For some people with COPD, eating a diet with fewer carbohydrates and more protein helps them breathe easier and maintain muscle mass.

The nurse is caring for a 14-year old child who has been tentatively diagnosed with hyperthyroidism. Which of these findings noted on the initial nursing assessment require intervention by the nurse? A A 10% weight loss in the last month despite an excellent appetite B An apical heart rate of 190 bpm Correct Answer (Blank) C A comment by the client, "I just can't sit still." D A report of irritability worsening in the past two weeks

Rationale: Hyperthyroidism is the result of an overactive thyroid gland and is characterized by increased metabolism. The clinical manifestations of hyperthyroidism may include goiter (enlarged thyroid), hyperactivity, heat intolerance, tachycardia, warm skin, exophthalmos, and weight loss. The parents may notice the child has difficulty sleeping and poor school performance related to distractibility. A sudden increase in the client's heart rate, blood pressure, or level of irritability may be associated with thyroid storm. Thyroid storm is caused by a sudden release of thyroid hormones and is a medical emergency that could progress to heart failure and shock. The other options are expected related to the tentative diagnosis of hyperthyroidism.

The nurse is caring for a client who received intravenous potassium chloride for a serum potassium level of 2.2 mEq/L. Which of the following client findings indicates that the treatment has been effective? A Presence of U waves on the client's electrocardiogram. B Hyporeflexive deep tendon reflexes. C Increased rate and depth of respirations. Correct Answer (Blank) D Hypoactive bowel sounds.

Rationale: Hypokalemia can cause severe disturbances in the cardiovascular, respiratory, neurological, and musculoskeletal systems. A major cause of death from hypokalemia is respiratory insufficiency due to respiratory muscle weakness. An increase in rate and depth of respirations indicates that treatment with intravenous potassium has been effective. Hypoactive bowel sounds, hyporeflexia, and U waves are all signs of severe hypokalemia, which indicate the treatment has not been effective in reversing the client's hypokalemia.

The nurse in the postanesthesia unit is assessing a postoperative infant following a hypospadias repair. The infant's legs are drawn up to his chest, he is crying, and urine is leaking around his urinary catheter. What intervention is the priority? A Administer the scheduled antibiotic B Apply a cold compress C Feed the infant D Administer the prescribed oxybutynin Correct Answer (Blank)

Rationale: Hypospadias is a congenital condition where the opening of the urethra is on the underside of the penis. This condition is most often surgically corrected. The procedure involves placement of a urinary stent to the bladder for drainage while the newly formed urethra heals. One common side effect of this procedure is bladder spasms. Pediatric clients often present during a bladder spasm with knees pulled up to the chest, back arched and urine leakage around the catheter. Using Maslow's hierarchy of needs to prioritize, the priority is to manage the infant's pain caused by the bladder spasm with an anticholinergic medication such as oxybutynin.

The nurse is caring for a client who has a new colostomy. Which question by the nurse is appropriate to assess the client's adaptation to this new change? A "Can I listen to your bowel sounds?" B "When was the last time that your colostomy bag was emptied?" C "Are you experiencing pain at the ostomy site?" D "Do you feel comfortable completing your own ostomy care?" Correct Answer (Blank)

Rationale: In order to assess the client's adaptation, the nurse should ask questions about feelings, behavioral changes, or the effect that the illness will have on the client's life. In this case, asking if the client feels comfortable with the ostomy care allows the client to discuss their feelings and express concerns. Asking about pain, the time when the colostomy was emptied, and listening to bowel sounds assesses the function of the colostomy but not if the client is adapting to the change.

The nurse is planning discharge teaching for an adult client with a newly inserted permanent pacemaker. The client plans to return to work in an automobile plant after the recovery period. What teaching point should be included? A "Reaching for objects overhead will help maintain range of motion." B "Avoid lifting anything heavier than 5 pounds for 4 weeks." C "Strong magnets or motors may affect the pacemaker." Correct Answer (Blank) D "Pacemaker insertion will have no impact on the plan to return to work."

Rationale: In workplaces that contain welding equipment or strong motor-generator systems, it is recommended that a person with an implanted cardiac device remain at least two feet from external electrical equipment and leave the immediate locale if lightheadedness or other symptoms develop. The client needs to avoid lifting anything heavier than 10 pounds or reaching for objects overhead for 4 weeks after insertion.

The nurse is teaching a client who is in the third trimester of pregnancy. Which factor should the nurse include that can increase the neonate's risk for development of sepsis? A Maternal gestational diabetes B Precipitous vaginal birth C Cesarean delivery D Premature rupture of membranes Correct Answer (Blank)

Rationale: Premature rupture of the membranes (PROM) means the spontaneous rupture of the amniotic sac and leakage of amniotic fluid before the onset of labor. PROM can lead to intrauterine infection and neonatal sepsis, especially if the PROM occurred at an early gestational age. Typically, the mother is prescribed a 7-day course of a broad-spectrum antibiotic to reduce the risk for infection. The other factors are not typically associated with an increased risk for neonatal sepsis.

The nurse is placing the ventilated client with acute respiratory distress syndrome (ARDS) in prone position. Which finding indicates that the intervention is having the intended effect? A Peak airway pressure has increased B Partial pressure of oxygen has increased Correct Answer (Blank) C V/Q ratio declines D Oxygen saturation remains stable

Rationale: Proning has become a standard treatment in the management of patients with ARDS who have difficulty achieving adequate oxygen saturation. Prone positioning is used to improve V/Q mismatch (increasing the ratio) and oxygenation. Increasing PaO2 is an indication of improved oxygenation. Proning is usually not implemented in clients with O2 saturations above 92%. Therefore, a goal of prone positioning would be improved oxygen saturation. Increasing peak airway pressures indicate a worsening of ARDS.

The home health nurse is visiting a 3-year-old client when the child begins to have a generalized seizure. Which intervention should the nurse implement first? A Place the child on their side Correct Answer (Blank) B Restrain the child C Place an oral airway in the mouth D Give the prescribed anticonvulsant

Rationale: Protecting the airway is the priority during a seizure, and the first action should be to roll the client on their side to open the airway and prevent aspiration in case of vomiting. Nothing should be placed in the mouth when the client is having a seizure and the client should not be restrained. Administration of the prescribed anticonvulsant would be appropriate after the seizure has ended.

The nurse is monitoring a client who received external radiation to their chest wall for early signs of skin breakdown. The nurse should observe the client for which finding? A Abscess B Scarring C Dryness Correct Answer (Blank) D Fibrosis

Rationale: Radiation damage to normal tissues can cause an inflammatory response. This can lead to dryness, redness, pain, blisters, swelling, fibrosis, and scarring.

The nurse is caring for a client who is receiving a 3% hypertonic saline infusion for a serum sodium level of 120 mEq/L. Which of the following client findings indicates the treatment has been effective? A A change in deep tendon reflexes from hyperactive to diminished B A change in Glasgow Coma Score from 8 to 11 Correct Answer (Blank) C A change in respirations from 22 to 10 D A change in bowel sounds from hypoactive to hyperactive

Rationale: Severe hyponatremia can cause changes in the level of consciousness. An increase in the Glasgow Coma Score from 8 to 11 indicates improvement in the level of consciousness, and therefore, the effectiveness of treatment of hyponatremia with hypertonic saline. Diminished deep tendon reflexes, hyperactive bowel sounds, and bradypnea are all symptoms of severe hyponatremia and do not indicate treatment has been effective.

A client has herpes simplex I with visible cold sores on the lips. Which intervention is most important for the client to implement to prevent spreading the infection? A Do not scratch the affected area B Avoid sharing towels Correct Answer (Blank) C Wash hands frequently D Take antiviral medication as prescribed

Rationale: Sharing any items (towels, lipstick, toothbrush, utensils, cups, etc.) that may touch the mouth has the highest risk of spreading infection from one individual to another. Washing hands and not touching/scratching the affected area are proactive measures to prevent spreading the infection but are not the priority. Taking antiviral medication as prescribed will promote healing.

The nurse is teaching the parents of a client diagnosed with sickle cell anemia. Which statement made by the nurse accurately explains their child's diagnosis? A "There is a reduced number of red blood cells due to inadequate iron in the diet." B "Red blood cells are abnormally shaped, preventing adequate oxygen delivery to the tissue." Correct Answer (Blank) C "Sickle-shaped red blood cells carry carbon dioxide to the tissues instead of oxygen." D "There is a depression of the platelets and also the red and white blood cells."

Rationale: Sickle cell anemia is caused by an abnormal type of hemoglobin, which changes the shape of red blood cells from a round to a sickle shape. These fragile abnormal blood cells carry less hemoglobin and can get stuck in the smaller blood vessels, depriving the tissue of oxygen and causing severe pain and tissue damage. Inadequate dietary iron causes iron-deficiency anemia. Platelets, red blood cells and white blood cells are all depressed in pancytopenia.

The nurse is reviewing lab results for a client that has been taking anti-tuberculosis medication. Which diagnostic result would indicate that the treatment is having the desired effect? A Sputum culture Correct Answer (Blank) B Chest X-ray C Tuberculin skin test D Gold test

Rationale: Sputum culture results are monitored for acid-fast bacilli to evaluate the effectiveness of the treatment regimen and adherence to therapy. The other tests will not indicate the effectiveness of treatment.

The nurse is evaluating a client with status asthmaticus. Which finding best indicates that interventions were effective? A The client's wheezes have decreased in intensity. B The client's respiratory rate is 20. C The client's pulse oximeter reads 94%. Correct Answer (Blank) D The client denies shortness of breath.

Rationale: Status asthmaticus means a severe, acute asthma exacerbation. Management of an acute asthma exacerbation focuses on correcting hypoxemia and improving ventilation. Supplemental oxygen is usually administered to achieve an O2 saturation of greater than 90%. The client's reading of 94% is the best indicator that interventions were effective. Wheezing, the respiratory rate and the client's subjective report are unreliable signs to gauge if the client's oxygenation status is adequate.

The nurse auscultates bibasilar inspiratory crackles in a 68-year-old client with systolic heart failure and an ejection fraction of 30%. Which other finding should the nurse expect based on this diagnosis? A Peripheral edema B Nail clubbing C Chest pain D Fatigue Correct Answer (Blank)

Rationale: Systolic heart failure is the result of a pumping problem, which is why the ejection fraction is reduced (normal is 60%). Heart failure can be caused by a heart attack, but chest pain is not normally a finding in heart failure. Nail clubbing is usually associated with disorders of the lungs. Exertional dyspnea and fatigue are common in clients with left-sided (systolic) heart failure due to fluid backing up into the lungs and pulmonary congestion. Peripheral edema is more commonly seen with right-sided (diastolic) heart failure.

The nurse is caring for a client diagnosed with testicular cancer. Which risk factor supports this diagnosis? A Undescended testis Correct Answer (Blank) B Genital herpes C Older than 60 years of age D Benign prostatic hyperplasia

Rationale: Testicular cancer is a rare cancer of the male testes that is common in males between the ages of 15 to 45. Factors that increase a male's risk of testicular cancer include an undescended testicle or testi (cryptorchidism), abnormal testicle development, family history, age (typically occurs in younger males, median age 33), and race (occurs more often in whites). The other conditions are not generally associated with an increased risk for testicular cancer.

The nurse in a primary care office is performing an assessment on an older adult client. The client appears tired, pale, and malnourished. The client reports drinking several alcoholic beverages a day and worsening dyspnea on exertion. Based on the assessment findings, which lab test is the priority? A Complete blood count Correct Answer (Blank) B Ammonia level C Chest x-ray D Liver enzymes

Rationale: The assessment findings and client reports point to anemia as the cause for the client's symptoms. Anemia is a reduction in the number of red blood cells (RBCs) and the amount of hemoglobin and hematocrit. It is most commonly caused by nutritional deficits as seen with malnutrition, chronic alcoholism and an iron- and folic acid-deficient diet. Therefore, a complete blood count or CBC lab test which includes an RBC, hemoglobin and hematocrit level, is the priority.

The nurse is caring for a client who is scheduled for a jejunostomy tube placement. The client states "Why is the tube not being placed in my stomach?" Which statement by the nurse would be appropriate to make? A "The jejunum can tolerate more feedings than the stomach." B "It is safer to place the tube in the jejunum as opposed to the stomach." C "The tube is more secure in the jejunum than in the stomach." D "The jejunum absorbs nutrients better than the stomach." Correct Answer (Blank)

Rationale: The jejunum is part of the small intestine and is the site of maximum nutrient absorption. The stomach prepares food for absorption. The jejunum is the largest part of the small intestine. Jejunostomies are placed to enhance absorption, not because of size. Insertion of both feeding tubes carries the same risks. Jejunostomies and gastrostomies can be done via percutaneous endoscopy or a surgical procedure. Enteral tubes are not easily dislodged. They are secured with internal and external crossbars.

The nurse is caring for a client who is 16 weeks pregnant. During the assessment, the client reports blurred vision and epigastric pain. The nurse should identify these as clinical manifestations of which gestational complication? Placenta previa Hypoglycemia Hypertension Placental abruption

Hypertension Rationale: Blurred vision and epigastric pain are indicative of gestational hypertension. The nurse should further assess the client's vital signs and accompanying symptoms. Placenta previa is a condition in which the placenta blocks the cervix during labor. Placental abruption is a serious complication that occurs due to premature separation of the placenta. Placenta previa and placental abruption are characterized by vaginal bleeding. Hypoglycemia is not characterized by the given symptoms. Hypoglycemia results in weakness, lightheadedness, and irritability.

The nurse is caring for a client who suddenly develops a wide QRS complex. The client's blood pressure is 82/40 mm Hg and respiratory rate is 22. The client is unarousable, and the nurse cannot palpate a pulse. Which action would be most appropriate for the nurse to take? Administer a prescribed intravenous IV fluid bolus Prepare the client for synchronized cardioversion Prepare to defibrillate the client Administer prescribed metoprolol

Prepare to defibrillate the client Rationale: The client is experiencing ventricular tachycardia, which is a lethal rhythm. The nurse should prepare to defibrillate the client. Synchronized cardioversion is indicated for tachycardic dysrhythmias but will not convert vTach. Metoprolol is a beta-blocker that has anti-dysrhythmic effects, but for vTach, defibrillation is the priority action.

The emergency room nurse is caring for a client admitted with a cervical spinal cord injury. Which assessment is the priority? A Blood pressure B Muscle weakness C Ability to urinate D Respiratory function Correct Answer (Blank)

Rationale: A spinal injury at the cervical level can result in quadriplegia with impairment of the phrenic nerve. As a result, the client is at high risk for respiratory insufficiency and failure; therefore, assessing and close monitoring of respiratory function is the priority.

A client is diagnosed with mitral regurgitation. The nurse would expect to encounter which finding? A Crushing chest pain B Platelet count C Low red blood cell count D Exertional dyspnea Correct Answer (Blank)

Rationale: Mitral regurgitation is the backflow of blood from the left ventricle into the left atrium. The clinical manifestations include exertional dyspnea, fatigue, orthopnea and pulmonary edema. The client with mitral regurgitation may experience palpitations but not crushing chest pain. The red blood cells and the platelet count are not impacted by mitral regurgitation

The nurse is caring for a client receiving continuous renal replacement therapy (CRRT) for acute kidney injury of prerenal origin. Which of the following findings indicates that the CRRT is having the intended effect? Serum concentrations of urea are reduced Urine output is increased Neurological status is improved Blood pressure is increased

serum concentrations of urea are reduced Rationale: Prerenal acute kidney injury results from poor perfusion to the kidney. The aim of CRRT is to gently remove waste and fluid while reducing the burden on the kidney to aid in recovery. Therefore, BUN/Cr and potassium levels should all decrease after the initiation of CRRT. While urine output may increase due to a recovery in kidney function, it is not the result of CRRT. Blood pressure stability is typically the result of vasoactive medication or treatment of sepsis in clients with prerenal acute kidney injury.

The nurse is caring for a client with diabetes who was admitted for intractable vomiting. The nurse notes that the client's skin is cool to the touch, and the fingerstick blood sugar result is 55 mg/dL. What intervention should the nurse implement first? A Administer glucagon. Correct Answer (Blank) B Recheck the blood sugar in 15 minutes. C Administer an antiemetic. D Offer the client a warm blanket.

Rationale: A client with diabetes receiving anti-hyperglycemic agents or insulin who is unable to eat or digest food is at risk for hypoglycemia (blood sugar level less than 70 mg/dL). Cool, clammy skin and a decreased level of consciousness are additional signs of hypoglycemia. To treat the hypoglycemia, the client requires glucose in the form of glucagon or another carbohydrate to increase the blood glucose to an acceptable level, typically greater than 70 mg/dL. After addressing the hypoglycemia first, the nurse should implement the other interventions.

A 50-year-old male client with a family history of prostatic hyperplasia asks the nurse how the health care provider will screen him for the disease. Which is the best response by the nurse? A A prostate-specific antigen test B A digital rectal exam Correct Answer (Blank) C A prostate biopsy D A history of symptoms

Rationale: A digital rectal exam is the most effective way to determine if the prostate gland is enlarged. The prostate-specific antigen (PSA) test is a blood test used primarily to screen for prostate cancer, not benign prostatic hyperplasia (BPH). A history of symptoms will also be completed, however many symptoms of BPH are similar to other conditions. A biopsy is usually done to determine the presence of cancer.

A nurse is developing the plan of care for a neonate diagnosed with hyperbilirubinemia with a new prescription for phototherapy. What intervention should the nurse include in the plan of care? A Keep in the fetal position B Ensure the neonate is covered C Apply opaque eye covers Correct Answer (Blank) D Transition the neonate to formula feedings

Rationale: Absorption of light through the skin converts unconjugated bilirubin into bilirubin photoproducts that are excreted in the stool and urine. During phototherapy, the newborn's eyes are covered to protect from the lights. The newborn is turned every 2 hours to expose all areas of the body to the lights and is kept undressed, except for the diaper area, to provide maximum body exposure to the lights. Fluid intake is increased to allow for added fluid, protein, and calories but transitioning to formula is not required.

The nurse is reviewing the arterial blood gas (ABG) for a client. The nurse notes the results are pH 7.49, CO2 32 mm/Hg, and HCO3 24 mEq/L. Which of the following interventions should the nurse implement? A Administer prescribed bronchodilator B Teach the client how to use the incentive spirometer C Apply prescribed oxygen via nasal cannula D Instruct the client to breathe into a paper bag Correct Answer (Blank)

Rationale: An ABG result of pH 7.49 indicates the client has alkalosis, and a CO2 of 32 mm/Hg indicates he is losing carbon dioxide, which is documented as respiratory alkalosis. The nurse should implement interventions to promote the client to retain CO2, such as breathing in a paper bag. After breathing in a paper bag, the client will re-breathe in a higher concentration of CO2. For a client who is experiencing respiratory acidosis, the nurse will teach the client to use the incentive spirometer, administer a prescribed bronchodilator, or apply oxygen.

A nurse is caring for a client with a traumatic brain injury who has a tympanic temperature of 104°F. Which action would be most appropriate for the nurse to take? A Administer prescribed acetaminophen Correct Answer (Blank) B Apply a cooling blanket C Place ice packs in the axilla area D Adjust the environmental temperature

Rationale: Antipyretics (acetaminophen, aspirin, non-steroidal anti-inflammatory agent) are given to reset the hypothalamus thermoregulatory mechanism or thermostat for temperatures greater than 38.5°C. The use of cooling blankets and ice packs are used when antipyretics are not effective. Caution is used with cooling devices, which can cause shivering and increase intercranial pressure. Adjusting the environmental temperature will not impact the client's core temperature.

The nurse is caring for a client admitted to the telemetry unit. The client has been in sinus rhythm, but the monitor is showing what appears to be new onset atrial fibrillation. What action should the nurse take first? A Obtain a 12-lead electrocardiogram. B Attempt to reposition the leads. C Measure the client's blood pressure. Correct Answer (Blank) D Print a strip for interpretation.

Rationale: Atrial fibrillation can lead to hypotension and poor client outcomes, so measuring the blood pressure is the nurse's priority intervention. All other interventions are important and part of the nurse's responsibilities, but the blood pressure is first.

A nurse is caring for a client who experienced head trauma from a motor vehicle collision. While assessing the client, the nurse notes a serosanguinous fluid leaking from the client's ear. Which is the priority action for the nurse to take? A Obtain a culture of the drainage B Observe the drainage for a halo sign Correct Answer (Blank) C Document the drainage D Cleanse the area to remove the drainage

Rationale: Basal skull fractures are suspected when CSF escapes from the ears (CSF otorrhea) and the nose (CSF rhinorrhea). Drainage of CSF is a serious problem because meningeal infection can occur if organisms gain access to the cranial contents via the nose, ear, or sinus through a tear in the dura. The "halo" or "double-ring" sign is a method for determining whether bloody discharge from the ears or nose contains cerebrospinal fluid (CSF). Culture, documentation, and hygiene are all important components of care, but identifying a CSF leak is the priority intervention.

A client presents at an urgent care center after burning their hand while cooking. The client's burn wound has an intact skin surface with redness and blistering that covers their posterior hand. How should the nurse describe this wound when documenting it in the client's medical record? A A partial-thickness wound Correct Answer (Blank) B A superficial-thickness wound C A full-thickness-wound D A deep full-thickness wound

Rationale: Burn wounds are classified as superficial-thickness, partial-thickness, full-thickness, and deep full-thickness. The wound described here is a partial-thickness wound. It involves the entire epidermis and varying depths of the dermis. These wounds are red, moist, and blanch when pressure is applied. When small vessels are damaged, they may leak plasma causing blister formation. The correct answer is a partial-thickness wound.

The nurse is taking care of a client with a spinal cord injury at the level of L3. Which intervention will the nurse perform to promote muscle tone? A Administer prescribed anti-spasticity medications. B Transfer the client to a chair. C Apply heel protectors to bilateral feet. D Assist the client with passive range of motion. Correct Answer (Blank)

Rationale: Clients with spinal cord injuries will experience loss of sensation below the level of injury. A lumber injury at the level of L3 will result in flaccid paralysis. Assisting the client with passive range of motion will help muscles maintain their tone. Anti-spasticity medications, or muscle relaxants, relieve spasms associated with injuries above the level of L1 and L2 but do not promote muscle tone. Transferring the client to a chair is beneficial for mobility but does not increase muscle tone. Applying heel protectors to bilateral feet prevents pressure ulcers and maintains skin integrity. However, it does not promote muscle tone.

The nurse is removing staples from a wound. During the process, the nurse notes that the edges of the wound begin to separate, and the client reports increased pain. What is the appropriate action? A Allow the client to have time to deep breath and relax and then continue to remove the staples. B Stop removing the staples and apply steri-strips to the open area. Correct Answer (Blank) C Pause to medicate the patient for pain before continuing to remove the remainder of the staples. D Decrease the rate at which each staple is removed.

Rationale: Dehiscence is a complication of suture/staple removal, especially with abdominal wounds. If the nurse notices that the wound edges are separating, it is essential to stop removing the staples, apply steri-strips to the open areas, and notify the healthcare provider. Pain is a symptom of dehiscence and therefore it is more important to focus on the cause at this time. Decreasing the rate of staple removal will not reduce the risk of dehiscence.

The nurse is providing care to a client with liver cancer. The client has been receiving external beam radiation therapy (ERBT). Which symptom assessed by the nurse indicates an adverse effect to ERBT? A Ascites Correct Answer (Blank) B Diarrhea C Anorexia D Nausea

Rationale: External beam radiation therapy (ERBT) can cause radiation-induced liver disease (RILD). RILD can be fatal and is characterized by jaundice, fluid accumulation in the abdomen (ascites), and abnormal blood liver tests. Diarrhea is a common side effect of radiation therapy. The nurse should ensure proper hydration. Nausea and anorexia or loss of appetite, are also common side effects of radiation. These side effects should subside within a few weeks or after treatment is completed.

A client diagnosed with gouty arthritis is admitted with severe pain and cellulitis of the right foot. Which intervention would be essential for the nurse to include in the client's plan of care? A High protein diet of beef B Hot compresses to affected joints C Active range of motion exercises D Fluid intake of at least 3000 mL/day Correct Answer (Blank)

Rationale: Fluid intake should be increased to prevent precipitation of urate in the kidneys; a lack of sufficient fluids enhances the formation of urate renal calculi or kidney stones. Treatment for acute attacks include supportive measures, such as applying ice and resting the affected joint. The client should avoid eating foods high in purines, such as organ meats (liver), and limit eating beef, pork and lamb.

The nurse is assessing a client with heart failure. Which finding observed by the nurse would indicate a decrease in cardiac output? A Crackles in bilateral lung bases B Hypoactive bowel sounds Correct Answer (Blank) C Jugular vein distension D Lower extremity edema

Rationale: Heart failure, which is the impairment of the ventricles to adequately fill and contract, can lead to a decrease in cardiac output. A decrease in cardiac output will lead to a decrease in perfusion to organs. Decreased perfusion will impair the function of the organs, such as decreased peristalsis in the intestines, cool skin, confusion, and pain. When the heart is unable to adequately pump the blood, vascular pressure will back up into the lungs and the systemic circulation, leading to crackles in the lungs, edema, and vein distention.

A nurse is caring for a client who has developed cardiac tamponade. Which finding would the nurse anticipate? A Bradycardia B Pleural friction rub C Widening pulse pressure D Distended neck veins Correct Answer (Blank)

Rationale: In cardiac tamponade, intrapericardial pressures prevent adequate filling of the heart from the vena cava, and reduce cardiac output. As a result, venous pressures rise and the neck veins become distended.

The nurse is caring for a client who is three hours postoperative after abdominal surgery. Which of the following actions should the nurse take to prevent respiratory complications? A Teaching the client to avoid coughing B Encouraging incentive spirometer use every two hours Correct Answer (Blank) C Assessing the client's incision site every eight hours D Holding the administration of pain medications

Rationale: In order to prevent postoperative respiratory complications, the nurse should encourage the client to turn, cough, and deep breathe as well as use an incentive spirometer every two hours. Pain medications should be administered as needed. If the client is experiencing acute pain, they are less likely to ambulate, turn, cough, or deep breathe. While the nurse should assess the incision site, this does not prevent respiratory complications.

The community health nurse is developing a plan of care for afemale adolescent with a body mass index greater than 40. Thenurse should recognize that the client has the highest risk for whichproblem? A Learning difficulties B Social isolation Correct Answer (Blank) C Sexually transmitted infection D Developmental delays

Rationale: A body mass index (BMI) greater than 40 indicates morbid obesity. During adolescence, individuals go through rapid physical and psychological desired physical appearance. Most often this desired appearance is being thin and physically fit. A morbidly obese adolescent is at risk for bullying, peer pressure, a poor body image and low self-esteem which can put them at a high risk to distance themselves from others, i.e., socially isolate.

The client was admitted two days ago with a diagnosis of myocardial infarction (MI). When assessing this client, the nurse notes the client's temperature is now 101.1°F (38.5°C). What is the most appropriate nursing intervention? A Increase the client's fluid intake B Send blood, urine and sputum for cultures C Call the health care provider (HCP) immediately D Administer PRN acetaminophen as ordered Correct Answer (Blank)

Rationale: After a myocardial infarction, leukocytosis can occur on day 2, because the body is undergoing a systemic inflammatory response. Nursing interventions should be to reduce the temperature, which would be the administration of the acetaminophen. It is not necessary to do further cultures or to notify the HCP at this time.

The nurse admits a 3-week-old infant to the special care nursery with a diagnosis of bronchopulmonary dysplasia. As the nurse reviews the birth history, which data would be most consistent with this diagnosis? A Meconium was cleared from the airway at delivery B Phototherapy was used to treat Rh incompatibility C The infant received mechanical ventilation for two weeks Correct Answer (Blank) D Gestational age assessment suggested growth retardation

Rationale: Bronchopulmonary dysplasia (BPD) is an iatrogenic disease caused by mechanical ventilation. When the prematurely born infant is treated with mechanical ventilation, over time the pressure from the ventilation and excess oxygen can injure the infant's lungs, causing BPD.

The home health nurse is developing a plan of care for a 3-year-old client diagnosed with cerebral palsy (CP). Which goals are the priority for this client? Select all that apply. Prevent seizures Correct Answer (Blank) Treat muscle spasms Correct Answer (Blank) Arrange for genetic counseling Select appropriate school environment

Rationale: Cerebral palsy (CP) is defined as a disorder of posture and movement from static brain injury perinatally or postnatally, which limits activity. In addition to motor disorders, the condition often involves disturbances of sensation, perception, communication, cognition, and behavior. Some of the disabilities associated with CP are visual impairment, hearing impairment, behavioral problems, communication and speech difficulties, seizures, and intellectual impairment. The priority goals at this age should include the prevention of seizure activity and correction of any associated physical defects and physical/occupational therapy to promote mobility and movement or the ability to move from one place to another (locomotion). Children with CP often suffer from muscle spasms and seizures and typically require pharmacotherapy for both. The other interventions are not appropriate at this age. CP is not a genetic disease.

The nurse is caring for a client who is experiencing an exacerbation of ulcerative colitis. Which manifestations would the nurse expect to see with this client? Select all that apply. Abdominal pain relieved by defecation Correct Answer (Blank) Mucous noted in the stool Correct Answer (Blank) Crackles in the lower lung fields Frequent bloody stools Correct Answer (Blank) Fever of 104°F (40°C)

Rationale: Due to the inflammatory nature of ulcerative colitis (UC), clients suffering from this illness will experience frequent, bloody stools that often contains mucous. Clients will often report lower abdominal pain that is relieved by defecation. Anemia can be associated with prolonged intestinal bleeding and dehydration may occur related to decreased absorption. Clients with UC may have a low-grade fever, but a high fever such as 104°F (40°C), would be more likely to be associated with an infection or peritonitis.

A nurse is caring for a client who has end-stage kidney disease (ESKD) and is receiving prescribed epoetin alfa. Which of the following findings would indicate that the treatment is having the intended effect? A An increase in leukocytes B An increase in platelets C An increase in hematocrit Correct Answer (Blank) D An increase in erythrocyte sedimentation rate (ESR)

Rationale: In end-stage renal disease, erythropoietin production decreases and profound anemia results, producing fatigue, angina, and shortness of breath. Anemia associated with ESKD is treated with erythrocyte-stimulating agents (recombinant human erythropoietin). Erythrocyte stimulation therapy is initiated to achieve a hematocrit of 33% to 38% and target hemoglobin of 12 g/dL, which generally reduces the symptoms of anemia. Epoetin alfa is administered IV or subcutaneously three times a week in ESKD. It may take 2 to 6 weeks for the hematocrit to increase; therefore, the medication is not indicated for immediate treatment of severe anemia. Leukocytes and platelets are unaffected by the administration of epoetin. Increases in the erythrocyte sedimentation rate (ESR) indicate inflammation in the body.

The nurse is planning care for a client with pulmonary edema due to severe fluid overload. Which independent nursing action will assist in managing this condition? A Applying a low-flow nasal cannula at 2 L/min B Administering a loop diuretic C Positioning the head of the bed at 90 degrees Correct Answer (Blank) D Obtaining the client's oxygen saturation

Rationale: Independent nursing actions can be initiated without a prescription from the healthcare provider. Pulmonary edema is fluid accumulation in the alveoli that prevents adequate gas exchange in the lungs. Positioning the head of the bed at 90 degrees will help to maximize ventilation. Applying a low-flow nasal cannula at 2 L/min will not help manage this condition. High-flow oxygen is recommended and is a dependent nursing action if the oxygen is administered continuously. Administration of a loop diuretic is a dependent nursing action. Obtaining the client's oxygen saturation level is important. However, this assessment does not directly manage the condition.

The nurse receives change-of-shift report on an 80-year-old client diagnosed with middle-stage Alzheimer's disease. Which information should be of highest concern? A A change in the color and temperature of the client's fingers and toes. B An increase in the client's basal heart rate by 10 bpm. C A 10 mm Hg drop in the client's diastolic blood pressure. D Reports of increased confusion, agitation and withdrawal. Correct Answer (Blank)

Rationale: Infections and pain can quickly exacerbate common symptoms of Alzheimer's disease, including confusion, agitation or withdrawal. A urinary tract infection (UTI) is one of the most common causes of sudden behavior changes in older clients. Because a UTI can quickly progress to urosepsis, the neurologic changes are of highest concern.

The nurse is caring for a child diagnosed with Kawasaki disease. The nurse should monitor the child for which potential complication? A Pulmonary embolism B Occlusions at the vessel bifurcations C Coronary artery aneurysm Correct Answer (Blank) D Chronic vessel plaque formation

Rationale: Kawasaki disease (mucocutaneous lymph node syndrome or infantile polyarteritis), affects the mucous membranes, lymph nodes, walls of the blood vessels and the heart. It can cause inflammation of the arteries, especially the coronary arteries of the heart, which can lead to aneurysms and possible myocardial infarction in the child. The other complications are not typically seen with Kawasaki disease.

The nurse in a pediatrician's office is performing a physical assessment on a 3-year-old with suspected Kawasaki disease in the acute stage. Which findings should the nurse anticipate? Select all that apply. Persistent diarrhea Peeling skin on hands and feet Correct Answer (Blank) Enlarged cervical lymph nodes Correct Answer (Blank) Rash triggered by sunlight Correct Answer (Blank) Strawberry tongue Correct Answer (Blank) Bilateral conjunctivitis Correct Answer (Blank) Intermittent low-grade fever

Rationale: Kawasaki disease is an acute illness that causes systemic inflammation in the blood vessels. The illness typically occurs in children under the age of five. Diagnosis is based on clinical presentation. There is no specific test to diagnose Kawasaki disease. Clinical manifestations in the acute phase include a high grade, persistent fever that is unresponsive to antipyretics or antibiotics. Other clinical manifestations include erythema of the hands and feet, peeling of the hands and feet, bilateral conjunctivitis, erythema in the oral cavity including lips and tongue (strawberry tongue), rash triggered by sun exposure, and enlarged lymph node(s). Diarrhea is not typically seen with Kawasaki disease.

The nurse is providing education on medication resistance to a client with tuberculosis. Which statement made by the client indicates further teaching is needed? A "I will need to take a combination of medications to eliminate the bacteria." B "I will need to take these medications until I feel better." Correct Answer (Blank) C "Some of these medications may change the color of my urine." D "I will need to have liver function tests before and after treatment."

Rationale: Medications prescribed for tuberculosis should be taken for the entire course of treatment (between 6-12 months) to decrease the risk of resistant strains. Two or more medications are recommended to decrease the risk of resistance. Rifampin, a bacteriostatic and bactericidal antibiotic used to treat tuberculosis changes the color of urine and other bodily secretions to orange. Isoniazid, rifampin, and pyrazinamide are three primary medications used to treat tuberculosis. All medications can cause hepatotoxicity, so liver function studies should be performed before and after treatment.

The nurse is caring for a group of assigned clients. The nurse should understand that which of the following clients has the highest risk for developing metabolic acidosis? A The client with diabetes who has had nausea for a week Correct Answer (Blank) B The client with pneumonia who reports shortness of breath C The client with an ileus who has a prescription for nasogastric suctioning D The client with an infection who has a documented temperature of 102°F

Rationale: Metabolic acidosis has a pH greater than 7.45 and HCO3 less than 22 and occurs when the client loses bicarbonate through excessive diarrhea or with renal failure and diabetes. A client with pneumonia is at risk for developing respiratory acidosis. A client with an infection who has a fever is at risk of developing respiratory alkalosis. The client with an ileus who has a prescription for nasogastric suctioning is at risk for developing metabolic alkalosis.

As a client is being discharged following resolution of a spontaneous pneumothorax, the client tells the nurse, "I'm going on a beach vacation next week." The nurse should instruct the client to avoid which activity? A Surfing B Scuba diving Correct Answer (Blank) C Swimming D Sun bathing

Rationale: The nurse would strongly emphasize the need for the client with a history of spontaneous pneumothorax problems to avoid high altitudes, flying in an unpressurized (open) aircraft and scuba diving. The negative pressure associated with diving could cause the lung to collapse again.

The nurse is evaluating a client after initiation of a warm bath to the hands due to frostbite. Which client response indicates the treatment is effective? Erythema is noted to bilateral hands Client reports increasing pain to bilateral hands. Client's oral temperature is 37°C (98.6°F) Bilateral radial pulses are +1

Client reports increasing pain to bilateral hands. Rationale: Frostbite of the extremities leads to numbness and paresthesia. When the tissues rewarm and thaw, the client will experience burning pain to the extremities, signaling improved circulation. Erythema is due to vasodilation and is an expected finding for superficial frostbite. Rewarming results in white or yellow skin tone. The client's oral temperature is a normal finding; however, this does not directly evaluate the effectiveness of therapy to the extremities. Bilateral radial pulses of +1 do not indicate effective treatment. The normal pulse strength is +2.

The nurse is assessing a client who has recently been diagnosed with diabetes type II. Which of the following questions by the nurse is appropriate to determine how the client is adapting to their new diagnosis? "Do you feel like your blood glucose is high today?" "Do carbohydrate restrictions change how you feel about food?" "Did someone come in to test your blood glucose earlier?" "When do you plan to make your follow-up appointment?"

"Do carbohydrate restrictions change how you feel about food?" Rationale: In order to assess the client's adaptation, the nurse should ask questions about feelings, behavioral changes, or the effect that the illness will have on the client's life. In this case, asking if the client feels differently about food due to the dietary restrictions allows the client to discuss their feelings related to this lifestyle change. All other responses do not assess adaptation.

The nurse is caring for a client experiencing diarrhea after radiation therapy. Which of the following would the nurse include in a teaching plan to help the client manage this side effect? "Try to avoid food and drinks that are high in sodium and potassium." "Eat three large meals each day." "Increase the amount of clear liquids consumed each day." "Look for foods that are high in fiber."

"Increase the amount of clear liquids consumed each day." Rationale: Severe diarrhea can cause fluid and electrolyte imbalances, so clients should increase fluid intake and look for sources of sodium and potassium as these electrolytes are lost from the gastrointestinal tract. Often clients will have nausea or decreased appetite when experiencing diarrhea; therefore, it is best to encourage frequent small meals. Foods that are high in fiber can make diarrhea worse. The nurse should recommend low-fiber foods, such as bananas, white rice, or white toast when the client is experiencing diarrhea.

The charge nurse is observing a newly hired nurse instruct a client who requires endotracheal intubation for status asthmaticus. Which of the following statements by the newly hired nurse requires intervention? "You will not be able to communicate once the tube is in place." "The mechanical ventilator may alarm, which does not always indicate a problem is occurring." "The tube will assist with your work of breathing and improve gas exchange." "You will be administered medications to maintain comfort and reduce anxiety as needed."

"You will not be able to communicate once the tube is in place." Rationale: Clients who are undergoing endotracheal intubation should be instructed, if possible, about what to expect during and after the procedure. Clients should be taught that their ability to talk will be eliminated, however, alternative forms of communication, such as a whiteboard or pen and paper will be provided to ensure client needs are met. Clients should be educated that ventilator sounds may occur and reassured that each sound does not always indicate a problem, which can cause anxiety in clients. Clients should be educated on the purpose of the tube insertion, which is to reduce the work of breathing and improve gas exchange. Clients should be assured that their comfort will be managed with medications, such as anxiolytics, as needed.

The nurse is caring for a client who reports the sudden onset of palpitations. The nurse reviews the client's electrocardiogram strip. Which prescribed medication should the nurse anticipate administering to the client? A Metoprolol B Digoxin C Lisinopril D Adenosine

Adenosine Rationale: Supraventricular tachycardia occurs when there is a reentry pathway in the cardiac conduction resulting in a rhythm that is regular, without clear p-wave, and heart rate greater than 150. For SVT, the nurse should prepare to administer adenosine. Beta-blockers, calcium channel blockers, and digoxin are preferred medications for atrial fibrillation.

The nurse is reviewing prescriptions for a client who has a stage 4 pressure injury and is receiving negative pressure wound therapy (NPWT). Which prescription from the healthcare provider should the nurse question? Perform foam dressing changes every 72 hours Administer cefazolin, 1 gram every 6 hours, for a positive wound culture and sensitivity (C&S) Administer enoxaparin, 40mg subcutaneously daily, for venous thromboembolism (VTE) prophylaxis Perform dressing and tubing changes every 24 hours

Administer enoxaparin, 40mg subcutaneously daily, for venous thromboembolism (VTE) prophylaxis Rationale: Anticoagulation therapy is used cautiously in clients with negative pressure wound therapy as it places the clients at a higher risk for complications of bleeding. The nurse should follow up with the primary healthcare provider regarding this prescription. The other prescriptions are standard care and correct for the client.

The charge nurse is observing a newly hired nurse who is caring for a client with hypomagnesemia and is receiving an intravenous magnesium sulfate infusion. Which of the following actions by the newly hired nurse requires intervention? Requesting to discontinue the client's prescribed furosemide Assessing the client's deep tendon reflexes every 4 hours Requesting a prescription for a stool softener Initiating continuous cardiac monitoring

Assessing the client's deep tendon reflexes every 4 hours Rationale: Clients with hypomagnesemia who are receiving intravenous magnesium sulfate should have deep tendon reflexes monitored hourly to monitor effectiveness and prevent hypermagnesemia. Clients with hypomagnesemia should be started on continuous cardiac monitoring to assess for dysrhythmias, have stool softeners ordered due to decreased gastrointestinal peristalsis, and have high-ceiling diuretics discontinued to prevent further magnesium losses. These actions all indicate correct management of a client with hypomagnesemia.

A nurse is caring for an older adult client following knee surgery who received a spinal anesthetic. What assessment finding requires immediate intervention? Pulse oximetry fluctuates between 98% and 96% Client reports knee pain increasing from 3/10 to 5/10 Current blood pressure 100/52 compared to 130/65 in recovery Temperature changed from 99.0°F (37.2°C) to 99.4°F (37.5°C)

Current blood pressure 100/52 compared to 130/65 in recovery Rationale: A drop in systolic blood pressure after surgery and recovery from anesthesia is a sign of possible hemorrhage. Low-grade fever and moderate pain are expected findings in the inflammatory stage of wound healing. Pulse oximetry fluctuations that remain in the expected range are unremarkable.

The nurse is caring for a client with an arterial line for blood pressure monitoring and frequent blood gases. To obtain accurate readings, which of the following actions should be taken? Level the transducer to the phlebostatic axis Raise the limb with the site of arterial catheter access to the level of the heart Turn the stopcock off to the transducer and "zero" it to calibrate the equipment Turn the stopcock off to the transducer and perform the square wave test

Level the transducer to the phlebostatic axis Rationale: Arterial line transducers are leveled and calibrated to the phlebostatic axis, which locates the right atrium. Moving the limb will result in unreliable readings. The stopcock should be positioned off towards the client for calibrating the equipment to atmospheric pressure. A square wave test of the transducer system is performed by fast flushing the tubing for 1-2 seconds. A normal (optimally damped) waveform will be a perfect square with 1-2 oscillations. The stopcock will be closed to the atmosphere for this test.

The charge nurse is observing a newly hired nurse perform a wet-to-dry dressing change for a client's wound. Which of the following actions by the newly hired nurse requires intervention? Squeezing excess moisture out of the gauze prior to packing the wound Applying a secondary absorbent dressing over the primary dressing Packing the wound tightly to apply pressure against the wound-bed Avoiding covering the wound edges with moist gauze

Packing the wound tightly to apply pressure against the wound-bed Rationale: The charge nurse should intervene if the newly hired nurse is observed packing the wound tightly with gauze. Packing the wound tightly increases pressure on the wound leading to wound damage. Wounds should be packed gently and loosely to fill the wound. It is the correct technique to squeeze out excess moisture from the gauze prior to packing the wound. The gauze should be moist, not dripping wet, to allow the gauze to absorb drainage. A secondary absorbent dressing should be applied over the primary dressing to absorb excess drainage, promote a more secure dressing, and better protect the wound. It is the correct technique to avoid covering the wound edges with moist gauze as this prevents further skin breakdown on otherwise healthy tissue.

The nurse is assigned to care for a client diagnosed with HIV/AIDS. A first-semester nursing student asks the nurse how a diagnosis of AIDS is determined, other than a positive HIV test. What response by the nurse is the best explanation for how AIDS is diagnosed? A "Having a CD4+ lymphocyte count less than 400 and a positive Western Blot test." B "The presence of any number of opportunistic infections and testing positive on the viral load test." C "Having symptoms of anxiety, dementia, depression and insomnia, along with a low viral load." D "Having a CD4+ lymphocyte count less than 200 and one or more AIDS defining conditions." Correct Answer (Blank)

Rationale: A CD4+ lymphocyte count is normally 600 to 1000 cells per cubic millimeter of blood. The Centers for Disease Control defines AIDS as someone who has a positive HIV blood test, one or more opportunistic infections (such as candidiasis and Kaposi's sarcoma) and a CD4+ lymphocyte count of less than 200. The ELISA Test is used to detect HIV infection; the Western Blot test is used to confirm a positive ELISA test. A viral load test measures the amount of virus in the blood; individuals with higher viral loads are at greatest risk for progressing from HIV infection to AIDS.

Several hours after a total gastrectomy, the client's nasogastric tube (NGT) stops draining. After referring to the postoperative orders, which order will the nurse implement first? A Notify the surgeon B Irrigate the nasogastric tube Correct Answer (Blank) C Reposition the tube until it begins to drain D Increase the amount of suction by 5 mmHg

Rationale: After surgery, the nurse should closely monitor the nasogastric tube (NGT) to ensure it is suctioning appropriately. Irrigating the NGT is appropriate because these tubes may become clogged. A clogged NGT could lead to acute gastric dilation after surgery. This intervention should be performed first.

The nurse is assisting with the placement of a central venous catheter into the subclavian vein. Which action should the nurse take? A Place the client in Trendelenburg position and turn head away from insertion site B Obtain full sterile drape and personal protection equipment (PPE) Correct Answer (Blank) C Soak sterile cotton in povidone-iodine and place in the sterile field D Obtain a prescription for an ultrasound to confirm line placement

Rationale: A central venous catheter/line (CVC or CVL) is a large-bore central venous catheter that is placed using a sterile technique (unless an urgent clinical scenario prevents sterile technique placement) in certain clinical scenarios. There are three possible sites for CVL placement in the adult patient, including the internal jugular (IJ) vein, femoral vein, and subclavian (SC) vein. The client is placed in reverse Trendelenburg for SC and IJ insertion. The client is supine for femoral insertion. CVC insertion bundles require that the provider wear head-to-toe sterile PPE and that the client be maximally draped with a sterile barrier. Chlorhexidine is the preferred antiseptic for CVC insertion due to its antibacterial efficacy. CVC insertion may be performed with the aid of ultrasound; however, placement is confirmed with a chest radiograph.

A nurse is caring for a client who has a central venous catheter and suddenly develops chest pain, dyspnea, dizziness, and tachycardia. The nurse suspects air embolism and clamps the catheter immediately. What other action should the nurse do next? A Administer high flow oxygen. B Place client on left side in Trendelenburg position. Correct Answer (Blank) C Remove the catheter. D Replace the infusion system.

Rationale: A central venous line is placed with the catheter tip in the subclavian or right atrium. An open infusion line could lead to an air embolism entering the central vein, which can flow into the right atrium. If an air embolism enters the right atrium, it can push up against the pulmonary artery blocking the flow of blood. The nurse should close the system, then place the client in the left side Trendelenburg position. This position will force the air embolism to the top of the right atrium, eventually dissipating due to blood flow. Once the client is in position, then the nurse will either replace a leaking infusion system, remove the catheter if indicated, and apply high flow oxygen.

A nurse is assessing a child at a clinic visit for a mild respiratory infection. Koplik spots are noted on the oral mucous membranes. What should the nurse assess next? A Urine B Lungs C Skin Correct Answer (Blank) D Sputum

Rationale: A characteristic sign of rubeola is Koplik spots (tiny white spots). These are found on the buccal mucosa in the mouth about a few days before the onset of the measles rash (which appears as small red, irregularly shaped spots with a bluish-white center). Although the nurse should assess the child's lungs with any reports of a respiratory infection, these spots would indicate that the skin should be checked for the presence of a rash. Sometimes a complication of measles is pneumonia, but it may be a bit premature to do a sputum culture.

The nurse is caring for a client with a dry chest tube drainage system due to a left tension pneumothorax. Two hours ago, the health care provider (HCP) changed the chest tube prescription to water seal only. When entering the client's room, the nurse finds the client to be short of breath, tachypneic, and with an oxygen saturation of 84%. On auscultation, the nurse notes absent breath sounds to the left upper lobe. What action should the nurse take first? A Apply oxygen via nasal cannula Correct Answer (Blank) B Request a chest x-ray C Document all interventions in the client's medical record D Notify the appropriate HCP

Rationale: A chest tube system is usually changed from wall suction to water seal only when the pleural space has re-expanded after a pneumothorax and in preparation for removing the chest tube. Shortness of breath, tachypnea, absent breath sounds, and hypoxemia indicate a recurrence of the pneumothorax. This is a life-threatening medical emergency that requires the nurse to prioritize. Using the Airway-Breathing-Circulation (ABC) decision-making strategy, the nurse should first provide the client with supplemental oxygen and then immediately notify the HCP. A chest x-ray will be required to determine the cause of the client's change in condition. After the client's urgent needs have been addressed, the nurse should then document all interventions implemented in the client's medical record.

The nurse in a long-term care facility is reviewing the plan of care for a client with quadriplegia. Which risk assessment scale should be included for this particular client? A The Hendrich Scale B The Wong-Baker Scale C The Braden Scale Correct Answer (Blank) D The Hamilton Scale

Rationale: A client who has paralysis of all four limbs (quadriplegia) is at risk of developing a pressure ulcer. A pressure ulcer is tissue damage caused when the skin and underlying soft tissue are compressed between a bony prominence and an external surface for an extended period of time. The Braden Scale is used for predicting pressure ulcer risk and should be included in this client's plan of care. The Hendrich Scale is used for fall risk. The Wong-Baker Scale uses visual faces to assess pain. The Hamilton Scale is used to rate anxiety.

The nurse is assigned to a client who is newly diagnosed with active tuberculosis. Which intervention is the priority? A Place the client in a private, negative pressure room. Correct Answer (Blank) B Reinforce hand washing before and after entering the room. C Have the client dispose of soiled tissues in a separate bag. D Collect several sputum samples for testing.

Rationale: A client with active tuberculosis should be hospitalized in a negative pressure room, i.e., airborne precautions, to prevent spread of the disease. Placing the client on on airborne precautions is the priority because this bacteria can be suspended in the air for long periods of time and may be carried for long distances on air currents, infecting others.

A client diagnosed with hypoparathyroidism would be most likely to display which of the following symptoms? A Pruritus Correct Answer (Blank) B Decreased reflexes C Polydipsia D Flank pain

Rationale: A client with hypoparathyroidism can present with the following laboratory anomalies: hypocalcemia, hyperphosphatemia and hypomagnesemia. Therefore, the nurse should identify signs and symptoms associated with these laboratory changes. Pruritus is often associated with increased levels of serum phosphorous and is common in patients with hypoparathyroidism. Kidney stones and associated flank pain are sometimes seen in clients with hypercalcemia, which is not traditionally seen in clients with hypoparathyroidism. In fact, clients with hypoparathyroidism often have low levels of serum calcium. Clients with hypermagnesemia can present with decreased reflexes. However, serum magnesium is often low in clients with hypoparathyroidism. Polydipsia is often seen in clients with diabetes mellitus secondary to hyperglycemia, and glucose levels are not typically impacted by hypoparathyroidism.

The nurse is caring for a client with hypotension who has a prescription to administer an infusion of isotonic IV fluids. Which of the following fluids should the nurse anticipate administering to the client? A 0.33% normal saline B 0.9% normal saline Correct Answer (Blank) C 5% dextrose in normal saline D 0.45% normal saline

Rationale: A client with hypotension requires fluid volume replacement in the vascular space. Isotonic solution, such as 0.9 normal saline and lactated Ringer's, is used to increase vascular volume. Isotonic solutions do not create a fluid shift, which will keep the volume in the vascular space increasing the blood pressure. Hypertonic solutions, such as 5% dextrose in 0.9% normal saline, will shift fluid from the intracellular space into intravascular space. Hypotonic solutions, such as 0.33% normal saline and 0.45% normal saline, shift fluid from the vascular space to the intracellular space.

The nurse at the outpatient surgery center is speaking with a client who is scheduled for a colonoscopy the next morning. Which information about the procedure should the nurse make sure to include? Select all that apply. "Make sure to drink the entire bowel preparation liquid." Correct Answer (Blank) "You should only consume clear liquids for the next 12 to 24 hours." Correct Answer (Blank) "You will have an intravenous catheter inserted prior to the procedure." Correct Answer (Blank) "Remember to stop eating any food six hours before you come to the center." Correct Answer (Blank) "You will be required to lay still for 6 to 8 hours after the procedure."

Rationale: A colonoscopy is used to visualize the large bowel. It is used to detect sources of bleeding, colon cancer, polyps or other abnormalities. To help cleanse the bowel, the client should be on a clear liquid diet for 12 to 24 hours prior to the procedure. In addition, the client will be required to drink an oral liquid preparation (e.g., sodium phosphate, Phospho-Soda or GoLYTELY). Watery diarrhea usually begins in about an hour after starting the bowel preparation process. Intravenous access is necessary for the administration of moderate sedation during the procedure. The client should be NPO 4 to 6 hours before the procedure. The client will not be required to lay still for 6 to 8 hours after the colonoscopy.

The nurse is caring for a client who has a history of peptic ulcer disease. The nurse notes the abdomen is rigid and the client complains of severe pain with palpation. What is the priority action by the nurse? A Review the client's record for NSAID use. B Notify the health care provider of the findings. Correct Answer (Blank) C Record the findings in the client's record. D Ask the client about dietary habits.

Rationale: A complication of peptic ulcer disease (PUD) is perforation. When perforation occurs, gastrointestinal contents will leak into the peritoneal cavity, causing peritonitis. This is a surgical emergency. While the client with PUD may complain of epigastric pain or tenderness, severe pain and a rigid abdomen are not expected finding. Past medication use and dietary habits are important to note, but alerting the HCP is the priority.

The nurse is assessing a client who is 29 weeks pregnant. During the assessment, the client states, "My baby finally stopped kicking so much throughout the day." What is an appropriate nursing response to this statement? A "How strong are the baby's kicks?" B "It must be a relief to not feel pain from so much kicking." C "Tell me what else has improved besides the kicking." D "When did you notice a decrease in the kicking?" Correct Answer (Blank)

Rationale: A decrease in fetal movement may indicate fetal distress. The nurse should ask follow-up questions to determine the timeline of the possible complication. Asking how strong the fetal movements are does not address the decrease in fetal movement. Reassuring the client that the decrease in fetal movement is a relief is not an appropriate statement. Decreased fetal movement is not a sign of improvement during the third trimester of pregnancy.

The nurse is caring for a client with a deep vein thrombosis. Which finding would require the nurse's immediate attention? A Pulse rate of 98 beats per minute B Respiratory rate of 32 Correct Answer (Blank) C Temperature of 102 F (38.8 C) D Blood pressure of 94/50

Rationale: A deep vein thrombosis (DVT) is a blood clot formed in a vein deep in the body. Typical location of a DVT is in the lower leg or thigh and causes increased swelling, redness, and pain. If left untreated, the clot can travel to the lungs and cause respiratory distress. A clot that forms in the lung is called a pulmonary embolism (PE). The clot lodges in one of the pulmonary arteries and can cause lung damage and hypoxia. The most common symptoms of a PE are sudden shortness of breath, a rapid respiratory rate, and chest pain.

The nurse is preparing to perform prescribed gastric lavage for a client who ingested a toxic chemical. Which action should the nurse take first? A Take the client's blood pressure B Connect the client to a cardiac monitor C Start a secondary intravenous line (IV) D Set up suction equipment Correct Answer (Blank)

Rationale: A gastric lavage irrigates the stomach and removes its contents. The solution is administered via an oral or nasogastric tube and removed via suction. The nurse should ensure suction equipment is set up properly before the procedure. Monitoring the client's blood pressure is important for assessing circulatory status but is not specific to gastric lavage. Connecting the client to a cardiac monitor will ensure the client is reacting well to the procedure. However, a gastric lavage requires suction equipment to function. Starting a secondary intravenous line is not related to the procedure of gastric lavage.

The nurse is performing prescribed gastric lavage for a client with gastrointestinal bleeding. Which action should the nurse take? A "Administer oral activated charcoal." B "Irrigate the NG tube with normal saline." Correct Answer (Blank) C "Infuse water through the NJ tube." D "Provide syrup of ipecac for ingestion."

Rationale: A gastric lavage requires the solution to be infused via a nasogastric (NG) tube. Irrigation with normal saline will stop an active gastric bleed. Activated charcoal is given to clients who ingest toxic agents. A nasojejunal (NJ) tube is used primarily for feeding. Gastric lavages are administered into the stomach. Syrup of ipecac is an expectorant that induces vomiting. This drug was used in clients who ingested toxic agents but is no longer administered in clinical practice.

The nurse witnesses a client having a seizure. Which observation is important to note to determine the type of seizure? A The exact time from beginning to end. B Loss of bowel or bladder control. C Identifying the pattern of breathing. D The sequence and types of muscle movement. Correct Answer (Blank)

Rationale: All behaviors observed during and after the seizure need to be reported and recorded. However, accurate descriptions of seizure activity and a system for recording and reporting activity is essential to seizure management. For example, during the seizure event, the nurse needs to observe the client's facial expression, muscle tone, movements (e.g. jerking or twitching) the parts of the body involved, and any automatic or repeated movement (e.g. lipsmacking, chewing, swallowing).

The nurse admits a 7-year-old to the emergency department after a leg injury. The X-ray reveals a fracture to the growth plate. While speaking with the child's parents, what response by the nurse is most appropriate when discussing the outcome of this injury? A "In some instances this type of injury can cause stunted bone growth." Correct Answer (Blank) B "Bone growth is stimulated in the affected leg as therapy is initiated." C "This type of injury shows more rapid union than that of younger children." D "The injury is expected to heal quickly because of the bone's strong outer surface."

Rationale: A growth plate fracture affects the layer of growing tissue near the end of the bone. This area is the softest and weakest section of the skeletal system and any injury may affect how the bone will grow. If left untreated or treated improperly, the injury could result in a crooked or shorter than its opposite limb. Serious injuries usually require a cast or a splint. If the injury is not aligning into the joint, surgery may be necessary. Out of the other options, this is the most appropriate response.

The nurse is responding to a low-pressure alarm for a client with acute respiratory distress syndrome (ARDS) and is being mechanically ventilated. The nurse should understand that which finding would indicate the reason for the alarm? A Inadequate inflation of the tube cuff Correct Answer (Blank) B Kink in the tube C Pulmonary congestion and secretions D Bronchospasm

Rationale: A low-pressure alarm indicates that pressure within the ventilator circuit has decreased. Low-pressure alarms are usually caused by a leak or disconnect in the circuit. Secretions, water in the tubing, bronchospasm, or kinks in the tubing can cause a high-pressure alarm.

The nurse in a neonatal unit is caring for a newborn with a myelomeningocele with an intact sac. Which type of dressing should the nurse use to cover the sac? A Hydrocolloid dressing B Kurlix gauze, wrapped around the spine and abdomen C Sterile pressure dressing D Sterile, moist nonadherent dressing Correct Answer (Blank)

Rationale: A meningocele is a neural tube defect where a sac is protruding from the spinal column. If not covered, there is a high risk of infection and it is essential to protect the exposed area of the spine. The nurse should apply a moist, sterile nonadherent dressing to prevent drying of the area. Dressings are to be changed frequently to keep the area moist. Treatment includes surgical closure and closing the overlying meninges and the skin. The other dressing choices are not appropriate for this condition.

The nurse is caring for a client who had a small bowel resection two days ago. The client reports that the pain has significantly increased over the last two hours and does not get better after receiving an analgesic. Which additional findings are indicative of a postoperative complication the client might be experiencing? Select all that apply. Serosanguineous fluid in the surgical drain Nausea and vomiting Correct Answer (Blank) Taut, distended abdomen Correct Answer (Blank) Hyperactive bowel sounds Correct Answer (Blank) Tenderness at the incision site

Rationale: A non-mechanical obstruction, or paralytic ileus, is a complication of bowel surgery due to the manipulation of the intestines during surgery. The pain associated with a non-mechanical obstruction is constant and diffuse. The client may also have distention in the upper abdominal or epigastric region, decreased or absent bowel sounds, and nausea and vomiting. Increased bowel sounds, especially loud, gurgling sounds, result from increased motility of the bowel (borborygmus) and are sometimes heard above a complete intestinal obstruction. Tenderness at the incision site and serosanguineous fluid in the surgical drain are expected two days after a bowel resection.

The nurse is reviewing the medical record of a client with recurring, nonhealing venous stasis ulcers to the lower extremities. Which findings are most likely contributing to the nonhealing of the client's wounds? Select all that apply. The client is 74-years-old. Correct Answer (Blank) The client's body mass index (BMI) is 16.5. Correct Answer (Blank) The client has a history of seasonal allergies. The client's ethnicity is Asian American. The client has a history of benign prostatic hyperplasia. The client smokes one pack of cigarettes per day. Correct Answer (Blank)

Rationale: A number of factors can affect the skin's ability to heal once injured. Factors include chronic disease, age, presence of an infection (systemic or local), nutritional status, substance abuse, and smoking. The client's medical record shows a number of factors that are affecting the healing of the wounds for this client. The client is undernourished or malnourished (BMI less than 18), a smoker (nicotine causes vasoconstriction, increased coagulability, and decreased oxygen delivery to tissues), and older (the aging process causes a decrease in collagen synthesis and epithelialization). The client's ethnicity and history of benign prostatic hyperplasia and allergies do not affect wound healing.

A nurse is reviewing the telemetry strip for a client with a newly placed pacemaker set for ventricular pacing. Which finding would indicate to the nurse the pacemaker is functioning properly? A Pacemaker spikes after each ST segment. B Pacemaker spikes before each P wave. C Pacemaker spike before each QRS complex. Correct Answer (Blank) D Pacemaker spike after the R wave.

Rationale: A pacemaker is inserted when a client has dysrhythmias that cause slow or fast heart rates. The pacemaker provides electrical stimulation that will control the client's heart rate. Pacemakers are designed to provide electrical stimulation in the atria, ventricles, or both. For a ventricular pacemaker, the pacemaker spike will appear before the QRS, which represents ventricular contraction. A pacemaker spike will appear before the P wave for atria pacing. A pacemaker spike after the R wave or ST-segment indicates a malfunction of the pacemaker.

The post-anesthesia care nurse is monitoring a client who had a laparoscopic cholecystectomy. Which findings would indicate to the nurse the client is safe to discharge? A SaO2 of 95% on room air, vital signs stable for last 30 minutes, baseline neurological status Correct Answer (Blank) B Pain level reported as 3 out of 10, ability to move extremities, dressing dry and intact C Urinary output of 30 mL/hr awake and alert, turning from side to side D Temperature 99.2⁰F, occasional rhonchi on auscultation, strong cough

Rationale: A patient remains in the PACU until fully recovered from the anesthetic agent. Priority indicators of recovery include stable blood pressure, adequate respiratory function, and adequate oxygen saturation level compared with baseline. Additional indications are adequate pain control, baseline ambulatory status, minimal nausea/vomiting, and minimal surgical site drainage/bleeding.

A nurse is caring for a client recently diagnosed with Addison's disease. While the nurse is reinforcing education, which statement by the client indicates the need for additional teaching? A "I will take my methylprednisolone when I start to feel sick." Correct Answer (Blank) B "I will wash my hands often to prevent infections." C "I will eat six small meals a day instead of three." D "I may experience changes in my mood."

Rationale: A person with Addison's disease suffers from low levels of circulating cortisol. The client will be required to take exogenous corticosteroids. A commonly prescribed medication is methylprednisolone, which the client should take every day, not only when they feel sick. This medication suppresses the immune system, so the client should wash their hands often to prevent infection. As their condition progresses, clients may experience anorexia, nausea, vomiting and diarrhea. To ease gastrointenstinal discomfort, the client should eat small meals throughout the day and drink sufficient amounts of fluids. Low-dose, long-term therapy with steroids can cause depression or other psychologic disturbances. The client should be educated about potential psychologic reactions and when to notify their health care provider.

The nurse is caring for a client with hypervolemia who has a serum potassium level of 2.5 mEq/L. Which of the following actions should the nurse take? A Administer prescribed furosemide, 40 mg IV. B Initiate continuous pulse oximetry monitoring. Correct Answer (Blank) C Request a prescription for 40mEq potassium chloride, IV push. D Request a prescription to infuse 1 L normal saline over 4 hours.

Rationale: A potassium level of 2.5 mEq/L indicates hypokalemia. The nurse should initiate continuous pulse oximetry monitoring so that the client's fluid and electrolyte status can be monitored closely for the development of pulmonary edema and hypoxia due to respiratory muscle weakness from hypokalemia. The nurse should hold the prescribed furosemide and notify the primary healthcare provider, as furosemide can cause worsening of the client's hypokalemia. Intravenous potassium should never be given IV push; this could cause cardiac arrest. There is no indication for a fluid bolus of 1 L NS at this time as the client is experiencing volume overload.

The emergency department nurse is caring for a client with a laceration to the right leg that is bleeding profusely. The client has a heart rate of 115 beats/min and a capillary refill of 4 seconds to the right toes. Which action does the nurse perform to promote circulation in the client? A Place the client in a prone position B Infuse prescribed isotonic intravenous fluids Correct Answer (Blank) C Apply a gauze dressing to the right leg D Administer prescribed opioid analgesics

Rationale: A primary survey for trauma clients consists of the ABCDE criteria. "C" stands for circulation, and interventions are aimed at restoring effective blood flow. Isotonic intravenous fluids maintain fluid volume and prevent shock. Placing the client in a prone position will not promote circulation. In clients with impending shock, the lower extremities should be elevated to shunt blood towards vital organs. Applying a wound dressing to the right leg will not promote circulation. Hemorrhage control includes applying direct pressure to the wound. Administering prescribed analgesics will help the client with pain management but will not promote circulation.

The nurse is reviewing the plan of care for a client with an internal radiation implant placed. Which action on the plan should be revised? A Applying gloves when emptying the client's bedpan B Keeping all the linens in the room until the implant is removed. C Wearing a lead apron when providing direct care D Placing the client in a semiprivate room at the end of the hallway Correct Answer (Blank)

Rationale: A private room is necessary for a client with a radiation implant. The client emits radioactivity and needs to limit exposing other people. The nurse should wear gloves when emptying a bedpan, wear a lead apron, and keep all linens in the room to prevent contamination.

The nurse in a labor and delivery unit is assessing a client who is in the first stage of labor. The client reports that they felt their "water break" a few moments ago. On visual inspection, the nurse notes a short loop of the umbilical cord protruding from the vagina. Which action should the nurse take first? A Administer hi-flow oxygen to the mother. B Place the client in a knee-chest position. C Insert a gloved hand into the vagina and hold the presenting part off the cord. Correct Answer (Blank) D Notify the health care provider immediately.

Rationale: A prolapsed cord is a medical emergency. The prolapsed part of the cord is being compressed by the presenting part of the fetus, causing occlusion of blood flow and fetal hypoxia. If not relieved within a few minutes, it will result in central nervous damage or death of the fetus. Pressure on the cord must be relieved first and can be accomplished by the nurse placing a gloved hand or fingers into the vagina and holding the presenting part off the umbilical cord.

The nurse is assessing a client who is in active labor. The nurse notes the protrusion of the umbilical cord from the introitus. Which action should the nurse take first? A Initiate intravenous (IV) access and administer a bolus of IV fluids. B Apply an oxygen face mask on the client and set the rate at 8 L/min. C Insert a gloved hand into the vagina and elevate the fetal presenting part. Correct Answer (Blank) D Leave the room and immediately notify the healthcare provider.

Rationale: A prolapsed umbilical cord can result in compression and decreased circulation to the fetus. The nurse should relieve cord compression by elevating the fetal presenting part. Administering IV fluids is an important intervention. However, the priority is to maintain fetal circulation. Applying an oxygen mask to the client will improve oxygenation to the fetus. However, relieving cord compression is the priority intervention. The nurse should not leave the client unattended. The nurse should call for immediate assistance while in the room.

The labor and delivery nurse is assessing a client in labor and notes a loop of the umbilical cord protruding from the vagina. Which action should the nurse take first? A Place the client in a knee-chest position Correct Answer (Blank) B Apply oxygen by mask C Notify the health care provider D Check the fetal heart rate

Rationale: A prolapsed umbilical cord is a medical emergency, which can result in brain damage or death to the fetus if not treated promptly and properly. Immediate action is needed to relieve pressure on the cord to prevent the risk of fetal hypoxia. A Trendelenburg or knee-chest position accomplishes this and should be done first. Then the nurse should implement the other actions.

The nurse is caring for a client who is in a second-degree heart block and reports dizziness and shortness of breath. The nurse notes the client's blood pressure is 90/40 mmHg. Which action should the nurse take? A Prepare for transcutaneous pacing Correct Answer (Blank) B Perform synchronized cardioversion C Administer prescribed atropine D Administer prescribed vasopressin

Rationale: A second-degree block occurs when the electrical conduction is interrupted, usually at the AV node, and does conduct through the Purkinje fibers. This results in the failure of the ventricles to contract, which decreases cardiac output. To treat a second-degree block, the nurse should prepare the client for transcutaneous pacing, which will increase cardiac output. Synchronized cardioversion is used to treat tachycardia dysrhythmias, such as SVT or atrial fibrillation. Atropine contraindicating in treating a second-degree block. Vasopressin causes vasoconstriction, which increases blood pressure but will not treat second-degree block.

The nurse in a pediatric clinic is caring for a 10-year-old child with suspected COVID-19 respiratory infection. Which assessment finding requires immediate intervention by the nurse? A Rapid, bounding pulse B Profuse diaphoresis C Temperature of 101.3°F (38.5°C) D Slow, irregular respirations Correct Answer (Blank)

Rationale: A slow and irregular respiratory rate is a sign of respiratory fatigue and impending acute respiratory failure in the child. Respiratory failure can rapidly lead to respiratory and cardiac arrest. Immediate interventions are required, such as supplemental oxygen, intubation and mechanical ventilation to support the child's respiratory status.

The nurse in a pediatric clinic is caring for a 10-year-old child with a suspected COVID-19 respiratory infection. Which assessment finding requires immediate intervention by the nurse? A Slow, irregular respirations Correct Answer (Blank) B Temperature of 101.3°F (38.5°C) C Profuse diaphoresis D Rapid, bounding pulse

Rationale: A slow and irregular respiratory rate is a sign of respiratory fatigue and impending acute respiratory failure in the child. Respiratory failure can rapidly lead to respiratory and cardiac arrest. Immediate interventions are required, such as supplemental oxygen, intubation, and mechanical ventilation to support the child's respiratory status.

The nurse is caring for a 22- year-old female client who experienced a spontaneous abortion at 17 weeks gestation. The client appears to be crying and avoids eye contact with the nurse. Which statement by the nurse would be appropriate? A "It's a tragic event, and you need to move on." B "This happens to a lot of women, so you are not alone." C "You are young and will be able to get pregnant again." D "Talk to me about what this loss means to you." Correct Answer (Blank)

Rationale: A spontaneous abortion can cause intense feelings of grief and loss. The nurse should acknowledge the client's condition and encourage communicating her feelings. Telling the client that they need to move on does not allow the client to experience the stages of grief and loss. Telling the client that a spontaneous abortion is common does not promote patient-centered care. The nurse should acknowledge the client's own feelings. Telling the client that they are young and will get pregnant again does not promote empathy and does not allow the client to experience grief.

The nurse in an urgent care clinic is evaluating a client's understanding of discharge instructions for a second-degree ankle sprain. Which statement by the client requires follow-up by the nurse? A "I will apply ice intermittently for the first 24 to 48 hours." B "I will do gentle stretching and range of motion exercises daily." Correct Answer (Blank) C "I will elevate my ankle to decrease pain and swelling." D "I will apply a compression bandage and wear an ankle brace."

Rationale: A sprain is excessive stretching of the ligament with tearing of the ligament fibers. Twisting motions from a fall or sports activity typically precipitate the injury. A second-degree sprain is classified as moderate. Second-degree sprains require immobilization with an elastic bandage and ankle brace, splint or cast. Recommendations for caring for a client with a sprain include rest, use of ice for the first 24 to 48 hours, application of a compression bandage for a few days to reduce swelling and provide joint support and elevation of the affected extremity (RICE). It is recommended not to stretch or use the sprained joint for approximately a week, sometimes longer, to allow it to heal properly. The nurse should follow up and advise the client not to perform stretching and range of motion exercises.

The nurse in the urgent care center is caring for a 20-year-old client who sustained a sprained ankle while playing sports. Which instructions should the nurse give the client to prevent a future sprain injury? Select all that apply. Take ibuprofen 30 minutes before starting any sports activity. Wear snug, well-fitting shoes that go up to the ankle. Correct Answer (Blank) Warm up for several minutes before starting the activity. Correct Answer (Blank) Use appropriate protective equipment with the activity. Correct Answer (Blank) Encourage stretching before and after any sports activity. Correct Answer (Blank)

Rationale: A sprain occurs when there is a sudden, abnormal movement around the joint that can lead to stretching and/or tearing of the ligaments attached to the joint. Stretching before and after exercising increases the ligaments pliability and decreases the risk for injury. Gradually warming up prior to engaging in physical activity provides the muscles with increased circulation and loosens up joints; both will decrease the risk for strains or sprains. Wearing proper fitting shoes enhances stability and wearing appropriate protective gear provides protection and decreases the likelihood of sprains. Taking ibuprofen, a nonsteroidal anti-inflammatory drug (NSAID), to decrease inflammation would be appropriate after a musculoskeletal injury, but will not help to prevent one.

The nurse is assessing a postoperative client with a new colostomy. The client states that the stoma protruded into the collection bag 30 minutes ago. What action does the nurse perform next? A Notify the healthcare provider. Correct Answer (Blank) B Tell the client that this is expected after surgery. C Document the findings. D Reassess the client in an hour.

Rationale: A stoma protruding into the collection bag is known as a prolapse. A prolapsed stoma can lead to twisting of the bowel and impaired circulation. Given the timeframe of when the prolapse occurred, the nurse should notify the healthcare provider immediately. This is not an expected finding after surgery. Documenting the findings is a required intervention; however, the nurse should notify the healthcare provider before documenting interventions. Reassessing the client in an hour can lead to complications. The nurse should intervene immediately.

The nurse is performing a neurological assessment on a client who suffered a right hemispheric stroke. Which finding requires immediate attention? A Emotional lability B Loss of bladder control C Decrease in level of consciousness Correct Answer (Blank) D Visual spatial deficits

Rationale: A stroke is caused by an interruption of tissue perfusion or blood flow to any part of the brain. Any interruption that lasts for more than a few minutes will cause cerebral tissue to die (infarction). Brain metabolism and blood flow after a stroke can be affected around the infarction and in the contralateral (opposite side) hemisphere. Regardless of the location of the stroke, the nurse should follow the airway-breathing-circulation (ABC) approach to prioritize. A client with a decrease in level of consciousness (LOC) may be unable to maintain an open airway; therefore, this assessment finding could be life-threatening and requires immediate attention.

The nurse assesses several postpartum women. Which of these women is at the highest risk for a puerperal infection? A Five days postpartum, temperature is 99.6°F (37.6°C) since undergoing cesarean section B Twelve hours postpartum, temperature is 100°F (37.7°C) following vaginal delivery C Three days postpartum, temperature is 100.8°F (38.2°C) for two days after undergoing cesarean section Correct Answer (Blank) D Seven days postpartum, temperature is 99°F (37.2°C) since vaginal delivery

Rationale: A temperature of 100.4°F (38°C) or higher on two successive days (not counting the first 24 hours after birth) indicates a postpartum infection. Puerperal infections can be due to endometritis, wound, and other infections; the risk of endometritis increases after cesarean delivery. The other women are not at risk for infection because their temperatures are within the expected normal findings for the time period.

The nurse in a neonatal intensive care unit is performing a physical assessment on a newborn with a large ventricular septal defect (VSD). Which findings are consistent with this diagnosis? Select all that apply. Wheezes Correct Answer (Blank) Retractions Correct Answer (Blank) Hypertension Tachycardia Correct Answer (Blank) Heart murmur Correct Answer (Blank) Cool extremities Correct Answer (Blank) Increased urinary output

Rationale: A ventricular septal defect (VSD) is a congenial heart disorder that results in a hole in the tissues that separate the left and right ventricles. This hole allows blood to abnormally shunt between the right and left ventricle decreasing the hearts ability to pump effectively. Heart murmurs are often the first clinical finding. Babies with large VSDs commonly experience excess fluid in their lungs leading to congestive heart failure (CHF). Clinical findings of congestive heart failure include impaired myocardial function, pulmonary congestion, and systemic venous congestion. A heart murmur, tachycardia, retractions, cool extremities, and wheezing are all assessment findings with VSD. Increased urinary output and hypertension are not typical findings with VSD.

The recovery room nurse is caring for an infant following the surgical correction of a ventricular septal defect. Which nursing assessment is the priority? A Blanch nail beds for color and refill B Auscultate for pulmonary congestion C Monitor for the equality of peripheral pulses D Observe for postoperative dysrhythmias Correct Answer (Blank)

Rationale: A ventricular septal defect (VSD) is an abnormal opening between the right and left ventricles of the heart. Surgical repair of this defect focuses on closing the abnormal opening between the ventricles. Because this area is shared by the atrioventricular bundle (bundle of His), which is part of the cardiac electrical conduction system, the priority is to monitor for postoperative dysrhythmias. The other assessments are also important but do not take priority over monitoring for dysrhythmias.

The nurse is evaluating a stage III pressure ulcer. Which assessment finding would indicate that the prescribed treatment is working? A Soft yellow tissue seen in wound bed B The periwound texture is moist and soft C The edge of the wound appears rolled or curled under D The size of the wound is decreasing Correct Answer (Blank)

Rationale: A wound that is decreasing in size is healing. "Slough" is yellow, tan or green tissue that is not healing. Soft and denuded tissue in the periwound area indicate tissue breakdown due to excessive moisture from wound drainage. Curled or rolled wound edges (epibole) prevents epithelial cells from migrating to close the wound, preventing the wound from healing.

A client is diagnosed with amyotrophic lateral sclerosis (ALS). Which nursing action will help prevent the complications of atelectasis and pneumonia in this client? A Active and passive range of motion exercises twice a day B Repositioning every two hours around the clock C Use of the incentive spirometer every two hours while awake D Chest physiotherapy twice a day Correct Answer (Blank)

Rationale: ALS is a progressive neurodegenerative disease. Early symptoms include increasing muscle weakness, especially involving the arms and legs, speech, swallowing or breathing; eventually there is total paralysis. As the chest muscles and diaphragm become weaker, it will be more difficult to cough and clear secretions. This will predispose clients to develop recurrent pulmonary infections. Chest physiotherapy and airway clearance devices will help prevent and/or treat these secondary infections. Deep breathing and coughing exercises may not be realistic for this client. Repositioning is targeted to circulation issues and is not specific for prevention of complications associated with the lung.

The nurse is caring for a client who is 9 weeks pregnant. Which statement made by the client indicates a need for further assessment by the nurse? A "I wake up every morning with so much nausea." B "I get bad stomach cramps throughout the day." Correct Answer (Blank) C My breasts feel tender at the slightest touch." D "I feel the need to urinate very often."

Rationale: Abdominal cramping during the first trimester is a concerning symptom and may be indicative of an ectopic pregnancy. The nurse should further assess the client's statement. Nausea and breast tenderness are common discomforts during the first trimester of pregnancy. Urinary frequency may occur during the first and third trimesters of pregnancy. The nurse should encourage frequent bladder emptying and Kegel exercise to reduce stress incontinence.

The nurse is caring for a client who has gastroesophageal reflux disease (GERD). The primary health care provider's orders include omeprazole twice a day, Maalox prior to meals, elevation of the head of the bed, an acid-reflux diet, and no alcohol. Which order should the nurse question? A Bed position B Schedule for antacid Correct Answer (Blank) C Prescribed diet D Schedule for the proton-pump inhibitor

Rationale: All of the options listed are potential recommendations but the schedule for antacids should be one to three hours after eating and at bedtime as needed.

The nurse in the emergency department is caring for a pregnant client at 32-weeks gestation with suspected placental abruption. Which order from the health care provider should the nurse implement first? A Type and crossmatch for 2 units of packed RBCs B Administer 100% oxygen via nonrebreather mask Correct Answer (Blank) C Initiate continuous fetal heart rate monitoring D Start a normal saline IV infusion at 150 mL per hour

Rationale: Abruptio placentae, or placental abruption, means the premature detachment of the placenta, wholly or partially, from the uterine wall. Immediate birth, i.e., delivery, is the treatment of choice if the fetus is near term or the bleeding is moderate to severe or the life of the mother or fetus is in jeopardy. In this scenario, the nurse should use the Airway-Breathing-Circulation (A-B-C) approach to prioritize the ordered interventions. The nurse should first administer the oxygen to prevent hypoxia in the fetus. Then the nurse should implement the other interventions.

An adult client in the waiting room of an outpatient clinic is found to have become unresponsive. The nurse is unable to palpate the client's carotid pulse. Emergency medical services have been requested by calling 911. What should the nurse do next? A Wait for the emergency medical services technicians to arrive. B Use a jaw-thrust maneuver to open the client's airway. C Begin chest compressions. Correct Answer (Blank) D Deliver two rescue breaths.

Rationale: According to basic life support (BLS) guidelines by the American Heart Association (AHA), chest compressions are the next step in initiating cardiopulmonary resuscitation (CPR) for an unresponsive client in whom a carotid pulse cannot be palpated. After the initial round of 30 chest compressions, the nurse should open the client's airway with a head tilt-chin lift maneuver (or a jaw-thrust maneuver if spinal cord injury is suspected) and deliver two breaths. It would not be appropriate to wait to start CPR until emergency medical services technicians arrive because immediate action is needed.

Upon entering a client's room, the client is found to be unresponsive and is not breathing. After immediately calling for help, what is the next action the nurse should take? A Maintain an open airway B Check for a carotid pulse Correct Answer (Blank) C Deliver 30 chest compressions D Give two rescue breaths

Rationale: According to the American Heart Association's basic life support, the first step after determining a victim is unresponsive is to call for help. The next step is to check for a pulse (for no more than 10 seconds). If there is no pulse, the rescuer should begin CPR (30 chest compressions followed by 2 ventilations).

A nurse witnesses a child lose consciousness from choking on a hotdog in a public park. What should the nurse do first? A Attempt a single finger sweep to remove the food B Start 100 to 120 chest compressions per minute C Open the airway and give two rescue breaths D Activate the emergency response system Correct Answer (Blank)

Rationale: Activating the emergency response system should be done first so that emergency medical services (EMS) personnel can arrive quickly and support the nurse's resuscitation efforts.

The nurse is caring for a 6-year-old child with edema and hypertension associated with acute glomerulonephritis (AGN). Which of the following interventions should be the highest priority for the nurse? A Establish seizure precautions Correct Answer (Blank) B Administer prescribed antibiotics C Relieve boredom through physical activity D Encourage protein-rich foods

Rationale: Acute glomerulonephritis (AGN) is the inflammation of the nephrons and glomeruli caused by a previous streptococcal infection. In AGN, there is a leakage of red blood cells and protein from the inflamed glomeruli. Dietary restrictions should include fluids, sodium, protein, and potassium due to the edema and low urine output. A child with edema and severe hypertension may be at risk for complications such as hypertensive encephalopathy. This complication occurs due to decreased kidney function and low urine output. Findings with this complication include headache, confusion, and vomiting. Seizure precautions should be instituted in this client. Although antibiotics may be indicated if a bacterial infection is still present, this is not the priority action. The child should be on bed rest during the acute phase until they start to recover.

The nurse is reviewing the medical record of a client admitted with acute kidney injury. Which findings would support this diagnosis? Select all that apply. Hematuria Correct Answer (Blank) Decreased glomerular filtration rate Correct Answer (Blank) Hypokalemia Elevated creatinine level Correct Answer (Blank) Proteinuria Correct Answer (Blank) Decreased blood area nitrogen

Rationale: Acute kidney injury (AKI) is the rapid loss of kidney function due to some form of damage. A clinical manifestation of AKI includes an elevated blood urea nitrogen level due to the breakdown of protein. Protein is then released into the bloodstream and is filtered through the kidneys. Through a urine analysis protein can be found, which is not typically present. Increased levels of protein can damage the kidney, causing an elevated creatinine level, a decreased glomerular filtration rate and hematuria and can cause the release of cellular potassium into body fluids. This can cause hyperkalemia, not hypokalemia.

A client is admitted to the emergency department with acute onset of left hemiplegia. The nurse should prepare the client for which diagnostic procedure? A Computerized tomography scan Correct Answer (Blank) B Cerebral arteriogram C Lumbar puncture D Positron emission tomography scan

Rationale: Acute onset hemiplegia is indicative of a stroke and a computerized tomography (CT) scan without contrast is the most commonly used diagnostic test to quickly determine if a client suffered a stroke. The other diagnostic tests are not indicated at this time.

The nurse is caring for a client with adrenal insufficiency. The nurse understands that the hormone cortisol is controlled by a cascade of events in specific organs. Beginning with the organ that responds first to low cortisol levels, place the following events in the correct order. 1. Adrenal cortex is stimulated. 2. Hypothalamus is stimulated 3 Cortisol level returns to adequate range 4 Anterior pituitary is stimulated 5 Adrenocorticotropic hormone is released 6 Cortisol is released 7 Cortisol releasing hormone is released A 2, 7, 4, 5, 1, 6, 3 Correct Answer (Blank) B 1, 2, 4, 6, 5, 7, 3 C 2, 1, 6, 4, 3, 5, 7 D 7, 1, 2, 4, 5, 6, 3

Rationale: Adrenal insufficiency (Addison's disease) is caused by a decrease in adrenocortical hormones, such as cortisol and aldosterone. Adrenal insufficiency is characterized by muscle weakness, fatigue, hypotension and electrolyte imbalances. This hormone cascade is controlled by the hypothalamic-pituitary axis. In this negative feedback loop, the hypothalamus is stimulated by low cortisol levels, which causes a release of cortisol releasing hormone (CRH). CRH stimulates the anterior pituitary to release adrenocorticotropic hormone (ACTH). ACTH stimulates the adrenal cortex to release cortisol. Finally, adequate levels of cortisol cause the hypothalamus to stop releasing CRH.

The nurse is reviewing the plan of care for a client with acute adrenocortical insufficiency. Which intervention should be a priority for this client? A Administration of insulin B Administration of potassium supplements C Implementation of a low-sodium diet D Electrocardiogram monitoring Correct Answer (Blank)

Rationale: Adrenocortical insufficiency is caused by an insufficiency of both cortisol and aldosterone. Reduced aldosterone secretion causes a reduction in potassium excretion and an increase in sodium and fluid excretion, ultimately resulting in hyperkalemia and subsequent hyponatremia. Hyperkalemia can result in cardiac arrhythmias which can be fatal. Therefore, the priority intervention is to complete electrocardiogram monitoring to assess for the presence of dangerous arrhythmias. Additionally, the other interventions are clearly inappropriate. Implementation of sodium restriction is inappropriate, as patients may be experiencing hyponatremia. Additionally, administration of potassium supplements is also inappropriate, as patients with adrenocortical insufficiency are likely to experience hyperkalemia. Some patients with adrenocortical insufficiency present with hypoglycemia, so the administration of insulin may worsen their condition.

The nurse on a postpartum nursing unit is receiving report about a client who had a normal spontaneous vaginal delivery the night before. The client has been passing golf ball-sized clots on her peri-pad for the last few hours. The client's most recent blood pressure is 88/56 mm Hg, and her heart rate is 118 bpm. The nurse enters the client's room and notices blood oozing from her intravenous insertion site. Which action should the nurse take first? A Perform peri-care and change the client's peri-pad. B Notify the client's health care provider. Correct Answer (Blank) C Palpate and massage the client's uterus. D Encourage breastfeeding to promote uterine contractions.

Rationale: After a normal spontaneous vaginal delivery (NSVD), it is normal for a client to have vaginal bleeding on their peri-pad. Postpartum hemorrhage (PPH) is defined as blood loss greater than or equal to 500 mL after birth. If a patient is saturating more than one peri-pad in an hour or passing several large clots, the patient could be experiencing PPH. This is an obstetric emergency. Signs and symptoms of PPH include dizziness, hypotension, tachycardia, large clots passed vaginally and heavy bleeding on the peri-pad. PPH can progress to a life-threatening condition called disseminated intravascular coagulation (DIC). This can occur after an injury or childbirth. Proteins in the blood that form blood clots travel to the injury site to help stop bleeding. If these proteins become abnormally overactive throughout the body, DIC can ensue. Small blood clots form in blood vessels throughout the body, and can clog the vessels and cut off the normal blood supply to the organs. Signs and symptoms of DIC include severe bleeding, oozing from puncture sites, hypotension, tachycardia, dizziness and hypoxia. The nurse should suspect DIC and should notify the primary health care provider (HCP) immediately. Nursing measures to monitor and control normal postpartum uterine bleeding can include uterine massage, breastfeeding and peri-care. The client in this scenario may be experiencing a medical emergency (e.g., DIC), therefore the nurse should first notify the HCP

A nurse is caring for a client after a tonsillectomy. The nurse observes the client swallowing frequently between sips of water. The nurse understands that this could be a sign of which complication? A Bleeding Correct Answer (Blank) B Postnasal drip C Aspiration D Anxiety

Rationale: After a tonsillectomy, bleeding from the surgical site is a potential complication. Clients who are bleeding from that area tend to "swallow" the blood. Postnasal drip may also cause frequent swallowing, but unless the client also had an adenoidectomy performed, a postnasal drip should not be present. Anxiety symptoms typically don't include frequent swallowing. Frequent swallowing is not a sign of aspiration.

The nurse performs a heel stick for a blood glucose check on a 1-hour-old, full-term newborn who weighed 9 pounds (4.1 kg) at birth. The serum glucose reading is 45 mg/dL. Which action should the nurse take? A Check the pulse oximetry reading B Start an IV infusion of 5% dextrose C Ask another nurse to perform a second blood glucose check D Repeat the test in two hours Correct Answer (Blank)

Rationale: After birth, the infant's glucose supply from the placenta is no longer available, and the newborn's glucose levels will decline. Hypoglycemia is a frequent problem encountered in newborns. However, serum glucose of 45 mg/dL is within the expected newborn range of 40 to 90 mg/dL. Newborns who weigh more than 8.8 pounds (4 kg), are large for their gestational age, or have a gestational age of fewer than 37 weeks are considered at risk for hypoglycemia. Due to the weight of this newborn, a repeat blood glucose test is indicated. The other actions are not appropriate for this infant.

The nurse is interviewing a client who was recently exposed to pulmonary tuberculosis (TB). Which finding(s) would the nurse most likely see with this client? A Hemoptysis and respiratory failure B A rash to the trunk and back C A positive Mantoux skin test Correct Answer (Blank) D A fever requiring hospitalization

Rationale: After exposure to TB, the most likely finding would be a positive skin (Mantoux) test. The other findings would more likely be seen with an acute respiratory infection or advanced TB. A rash is not a typical finding with TB.

A client is recovering from an acute myocardial infarction. Which action by the nurse would best prevent complications associated with the Valsalva maneuver in this client? A Maintain the client on strict bed rest B Administer antiarrhythmic medications PRN as ordered C Administer stool softeners every day as ordered Correct Answer (Blank) D Assist the client with use of the bedside commode

Rationale: After myocardial infarction, the Valsalva maneuver can cause cardiac arrhythmias. Administering stool softeners every day will prevent the client from straining or bearing down on defecation (the Valsalva maneuver). If constipation occurs, laxatives would be necessary to prevent Valsalva. If the client experiences cardiac arrhythmias associated with straining on defecation, then administering antiarrhythmics would be appropriate. Maintaining bed rest with use of a bedpan can increase the likelihood of straining and difficulty with defecation as well as increased myocardial oxygen consumption, so use of the bedside commode is also appropriate to achieve this goal in this client.

The nurse is assisting with endotracheal intubation for a client with respiratory failure. Which of the following findings provides the best confirmation of tube placement? A Presence of symmetrical lung sounds B Absence of breath sounds over the stomach C Color change from purple to yellow on the end-tidal carbon dioxide monitor D Chest X-ray indicating distal tip is proximal to the carina Correct Answer (Blank)

Rationale: After placing the endotracheal tube, it is essential to confirm its placement in the trachea and position proximal to the carina. End-tidal carbon dioxide monitoring provides gold-standard bedside confirmation of tube placement. However, post-intubation chest X-ray confirms that the endotracheal tube's tip is 2 to 4 cm proximal to the carina and rules out mainstem bronchus intubation. The nurse should also auscultate for symmetric bilateral breath sounds and the absence of breath sounds over the stomach.

The nurse is caring for a 68-year-old male client who had a transurethral resection of the prostate (TURP) 12 hours ago. The client has an indwelling 3-way catheter with continuous bladder irrigation. Which finding requires the nurse's immediate intervention? A Minimal drainage into the urinary collection bag Correct Answer (Blank) B Light-pink urine with a continuous stream into the collection bag C Occasional suprapubic cramping about every hour D Reports of a feeling of discomfort from the urinary catheter

Rationale: All of the options, except the lack of drainage into the collection bag, are expected findings after this procedure. Urine will be bright red from bleeding immediately after the procedure, lightening over time as bleeding decreases. A lack of drainage needs to be reported immediately because minimal urinary drainage puts the client at risk for bladder rupture. The cause of this is likely to be a blood clot in the catheter or obstructing the catheter tip, which requires sterile irrigation of the catheter to restore its patency. The flow rate of the continuous irrigation would need to be slowed until urine flow has been restored. In some facilities, an order for syringe bladder irrigation as needed is a standing order accompanying the orders for continuous bladder irrigation

The nurse and a student nurse are discussing the health issues related to a laboring HBsAg-positive client. Which of these comments by the student is incorrect and indicates a need for further instruction? A "The infant will receive the hepatitis B immune globulin within 12 hours after birth." B "The HBsAg-positive mother should not breastfeed her baby." Correct Answer (Blank) C "The HBsAg-positive mother should be reported to the state or local health department." D "The infant will receive the hepatitis B vaccine within 12 hours after birth."

Rationale: All persons with HBsAg-positive laboratory results should be reported to the state or local health department. The newborn should receive the hepatitis B immune globulin and hepatitis B vaccine within 12 hours after birth, using different sites (the second vaccine is given between 1 and 2 months; the last vaccine is given between 6 and 18 months). HBV is not spread by breastfeeding, kissing, hugging, coughing, or casual contact.

A 12-year-old child with cancer is distraught about the alopecia that occurred after the last chemotherapy treatment. Which nursing interventions are appropriate to address this side effect of chemotherapy? Select all that apply. Practice and teach thorough hand washing Allow the child to choose a cap, scarf, wig or other head cover to use Correct Answer (Blank) Encourage visits from friends before discharge from the hospital Correct Answer (Blank) Administer prescribed antiemetic medication before nausea is too severe

Rationale: Alopecia is the loss of hair, which is a frequent side effect of certain types of chemotherapy. Although it is not life-threatening, the body image change is difficult for many individuals, particularly children and adolescents. Encouraging visits from friends before discharge helps the young client and friends adjust. Wearing preferred forms of head cover-ups increases comfort and decreases embarrassment. The other options are proper interventions for chemotherapy, but do not help the client with hair loss.

The nurse is reviewing the medical record of a client with acute pancreatitis. The nurse should recognize which information as the most likely risk factor for the client's illness? A Diabetes B Intravenous heroin use C Obesity D Gallstones Correct Answer (Blank)

Rationale: Although a number of factors can cause acute pancreatitis, the most common cause in the U.S. is gallbladder disease and gallstones. The second most common cause is chronic alcohol intake. Obesity is a risk factor for developing gallbladder disease. Elevated blood glucose levels can occur with pancreatitis due to impaired insulin metabolism. Intravenous IV drug use is not typically associated with acute pancreatitis.

The nurse is performing an assessment on a college student who comes to the health clinic with symptoms of meningitis. The student lives in the school dormitory on campus with hundreds of other students. What is the priority action the nurse should take? A Alert the college's administration and dormitory staff. Correct Answer (Blank) B Administer acetaminophen for the headache. C Perform a focused neurological assessment. D Obtain the client's immunization history.

Rationale: Although all of the actions are appropriate for this client, the priority is to notify school officials and the dormitory staff. If the client turns out to have bacterial meningitis, it is important to identify other students who might have been exposed.

The nurse is providing care to an 80-year-old client with the diagnosis of advanced Parkinson's disease. The nurse should know that the greatest risk to the client is related to which finding? A Extreme weakness in the lower extremities B Dizziness and syncopal episodes C Difficulties with reading and seeing at night D Drooling and coughing when eating Correct Answer (Blank)

Rationale: Although all of the findings pose a safety risk to the client, drooling and coughing while eating are indicative of dysphagia. Dysphagia, a common finding with advanced Parkinson's disease, puts the client at an increased risk for aspiration of oral secretions or choking on food, which can cause airway and/or breathing problems. Using the ABC prioritization strategy, the nurse should recognize this finding as the greatest risk.

The nurse is assessing a client with left-sided heart failure who exhibits fatigue, hypotension, crackles to auscultation and dyspnea on exertion. Which nursing problem is the priority for this client? A Decreased cardiac output B Fluid volume excess C Pain D Impaired gas exchange Correct Answer (Blank)

Rationale: Although all of the problems are important for the nurse to address, the nurse should use the airway-breathing-circulation (ABC) approach to prioritize. Due to the heart or "pump" failure, the client will experience excess fluid volume, a decrease in cardiac output which causes a decrease in blood pressure and pulmonary congestion due to the left ventricle's inability to pump and empty effectively. This leads to fluid backing up into the lungs (pulmonary edema). The pulmonary congestion or edema will interfere with the much needed exchange of oxygen, causing hypoxemia, dyspnea and general activity intolerance. Therefore, the priority will be to provide the client with supplemental oxygen to support the heart and lungs in meeting the body's tissue perfusion needs.

A nurse is suctioning a tracheostomy tube of a client. In order to prevent unnecessary hypoxia during the procedure, what action should the nurse take? A Apply suction for no more than 10 seconds Correct Answer (Blank) B Lubricate three to four inches of the catheter tip C Withdraw catheter in a circular motion with intermittent suction D Maintain sterile technique throughout the procedure

Rationale: Although all of the responses are correct actions during the suctioning process, hypoxia can result from applying suction for more than 10 seconds. The nurse should apply oxygen immediately before and after suctioning to allow the client to rest if more suctioning is indicated.

The nurse is caring for a client who was diagnosed with a deep vein thrombosis (DVT). The client reports sudden shortness of breath and the oxygen saturation decreases to 87% on room air. Which intervention is a priority action by the nurse? A Administer oxygen to maintain a saturation of 92% Correct Answer (Blank) B Call the health care provider (HCP) C Administer the PRN albuterol nebulizer D Begin continuous cardiac monitoring

Rationale: An acute onset of dyspnea and hypoxia is a classic finding of pulmonary embolism (PE). A client with a DVT has a risk for part of the clot breaking off and traveling to the lungs. The administration of oxygen to correct hypoxia is the highest priority. After administering oxygen, the HCP would need to be notified and the nurse should anticipate orders for diagnostic tests (Pulmonary Angiogram, d-dimer, CT scan). Albuterol nebulization is a standard treatment for respiratory distress related to asthma, COPD and anaphylaxis. However, it is not used for dyspnea due to a PE.

The nurse is teaching a client with diabetes mellitus about diabetes management during an acute illness. Which statement by the client demonstrates an understanding of the nurse's teaching? A "I will double my insulin dose while I'm sick." B "I should not be alarmed if I need less insulin because that is expected when I am sick." C "It is important I check my blood glucose more frequently when I'm sick." Correct Answer (Blank) D "It is normal for my blood sugar to be lower than normal while I'm sick."

Rationale: An acute, short-term illness tends to raise blood sugar levels. This means that clients usually need more, not less, insulin. The client should not "double" their insulin dose without first checking their blood sugar level and discussion with their health care provider (HCP). During an acute illness, clients should check their blood sugar more frequently to monitor for greater fluctuations in normal levels and adjust medication doses as instructed by their HCP.

The nurse is caring for a client with sepsis who has a femoral arterial line placed. Which would be the priority action for the nurse to take when monitoring the line? A Palpate pedal pulses. Correct Answer (Blank) B Inspect insertion site. C Flush line tubing with normal saline. D Clean insertion site with chlorhexidine.

Rationale: An arterial line placed in the femoral artery requires the nurse to monitor the client's neurovascular and perfusion function. The nurse should palpate pedal pulses to evaluate that the arterial line is not impeding blood flow to the lower extremity which can lead to impaired perfusion. The nurse will flush the line with normal saline, clean the insertion site with chlorhexidine, and inspect the insertion site, but the priority is to evaluate perfusion.

A nurse is assessing a client after an arterial puncture to obtain a blood gas sample. Which observation prompts the nurse to immediately contact the healthcare provider? A Absent radial pulse Correct Answer (Blank) B Pain to the wrist C Bleeding to the site D Bruising to the antecubital area

Rationale: An arterial puncture for a blood gas sample is typically performed on the radial artery. An absent radial pulse indicates injury to the blood vessel and must be reported immediately. Pain to the wrist is common after an arterial puncture. The nurse should provide the client with comfort measures to relieve pain. Bleeding to the site is common due to arterial flow. The nurse should ensure that pressure is applied to the site for at least 5 minutes. An arterial puncture is performed on either the radial or ulnar arteries. Bruising to the antecubital area is not expected.

The nurse is teaching a client with atelectasis about how to use an incentive spirometer. Which statement should the nurse include in the teaching? A "Hold your breath for 3 seconds after inhaling through the mouthpiece." Correct Answer (Blank) B "You should perform this exercise every 2 hours." C "This device measures the amount of fluid in your lungs." D "Place your lips tightly over the mouthpiece before exhaling."

Rationale: An incentive spirometer helps clients improve their deep breathing and prevent lung collapse after medical procedures or long episodes of immobility. Holding the breath for 3 seconds maximizes lung expansion and prevents further alveolar collapse. Incentive spirometry should be performed 10 times every hour while the client is awake. The incentive spirometer does not have the capability to measure the amount of fluid in the lungs. The client should be told to inhale slowly after placing their lips around the mouthpiece.

The nurse is caring for a client with a pneumothorax. Which intervention will the nurse perform to improve gas exchange? A Request a prescription for analgesics. B Educate the client on incentive spirometry. Correct Answer (Blank) C Place the client in a protective lateral position. D Monitor arterial blood gases.

Rationale: An incentive spirometer improves airflow and encourages deep breathing and lung expansion. The nurse should educate the client on performing incentive spirometry 10 times an hour while awake to improve gas exchange. Analgesics are helpful in controlling pain. However, these medications do not improve gas exchange. Clients should be placed in high Fowler's to maximize lung expansion and improve gas exchange in the lungs. Monitoring arterial blood gases is an important assessment for clients with a pneumothorax. However, it does not help with improving gas exchange.

A client is admitted to an ambulatory surgery center and underwent a right inguinal orchiectomy. Which goal is the priority before the client should be discharged home? A The client's psychological counseling is scheduled. B The client is able to tolerate a general diet. C The client is able to ambulate in the hallway with assistance. D The client's postoperative pain is well-managed. Correct Answer (Blank)

Rationale: An orchiectomy is the surgical removal of one or both testicles. It is usually performed to treat cancer (testicular, prostate, or cancer of the male breast). Due to the location of the incision, pain management is the priority. Most men will be able to eat regularly when they get home. They should at least tolerate liquids before discharge. The client should be able to walk without assistance prior to discharge. Psychological counseling may be needed as part of long-term aftercare; however, this is not the priority prior to discharge.

The nurse is caring for a client with aortic valve stenosis who reports dizziness and shortness of breath with ambulation. Which would be the priority for the nurse to assess? A Heart rate B Pulse oximetry C Blood pressure Correct Answer (Blank) D Respiratory rate

Rationale: Aortic stenosis is a hardening of the aortic valve leaflets resulting in an impairment of the aortic valve to open and close. The left ventricle has to work harder to push blood through the stenotic valve, which can result in a decreased cardiac output. A client who is engaging in activity and reports dizziness and shortness of breath could be experiencing a decrease in cardiac output. The nurse should assess the client's blood pressure to evaluate decreased cardiac output. Heart rate, pulse oximetry, and respiratory rate can assess an alteration in cardiac and respiratory function but does not directly assess cardiac output.

The nurse is providing education to a patent diagnosed with chronic kidney disease, stage 5. Which statement made by the client indicates that teaching has been effective? A "I know I have a high risk of clot formation since my blood is thick from too many red cells." B "I can expect to have periods of little urine and then sometimes a lot of urine." C "I have to go for epoetin (Procrit) injections at the health department." Correct Answer (Blank) D "My bones will be stronger with this disease since I will have higher calcium than normal."

Rationale: Anemia in end-stage renal failure is caused by reduced endogenous erythropoietin production in the kidney. Anemia in primary end-stage renal disease is treated with subcutaneous injections of Procrit or Epogen to stimulate the bone marrow to produce red blood cells. With kidney failure, too much phosphorus can build up in the blood and calcium is pulled from the bones, resulting in weakened bones. The statement about producing variable amounts of urine is incorrect, as the client will produce little to no urine at this stage of the disease.

The nurse is caring for a client with a history of unstable angina who reports chest pain. What accompanying assessment finding would be the priority to include when reported to the healthcare provider? A Elevated blood pressure B ST-segment elevation Correct Answer (Blank) C Jaw pain D Dizziness

Rationale: Angina pectoris refers to chest pain that is brought about by myocardial ischemia. Unstable angina is characterized by attacks that increase in frequency and severity and are not relieved by rest and administering nitroglycerin. Patients with myocardial ischemia may present with a variety of symptoms other than chest pain. Pain can cause a transient elevation in blood pressure. Some complain of epigastric distress and pain that radiates to the jaw or left arm. Patients who are older or have a history of diabetes or heart failure may report shortness of breath. Many women have been found to have atypical symptoms, including indigestion, nausea, palpitations, and numbness. Patients presenting with acute coronary syndrome without ST-segment elevation on the electrocardiogram are classified as having unstable angina. Once ST-segment elevation occurs, this indicates a potential progression to myocardial infarction and should be reported immediately.

The nurse is caring for a female, long-distance runner who is diagnosed with anorexia nervosa. Which of the following concerns should the nurse determine to be the priority when planning the client's care? A Electrolyte imbalance B Blood disorders C Digestive problems D Amenorrhea Correct Answer (Blank)

Rationale: Anorexia nervosa is considered an eating disorder that is characterized by low body weight, a fear of gaining weight, and distorted reality of weight. Clients with anorexia nervosa control their weight through caloric restriction and starvation. Anorexia nervosa affects the whole body. However, female athletes with this condition can experience a decrease in hormones, which causes irregular periods or even amenorrhea. Low estrogen levels and poor nutrition, especially low calcium intake, can lead to premenopausal osteoporosis. Young women athletes are at high risk of stress fractures and other bone pathology. The three conditions (eating disorder, amenorrhea, and osteoporosis) are sometimes referred to as the female athlete triad. Clients who suffer from anorexia are at risk for malnutrition, digestive problems, and blood disorders. However, the question pertains to a female, long-distance runner. The nurse's priority would be to focus on the client's periods.

The nurse is caring for a client newly diagnosed with atrial fibrillation. The atrial heart rate is 250 and the ventricular rate is controlled at 75. Which finding is a cause for the most concern? A Tachypnea with movement B Difficulty speaking Correct Answer (Blank) C Loss of appetite D Diminished bowel sounds

Rationale: Anticoagulant therapy is usually given to patients with atrial fibrillation to prevent blood clots and stroke. A new finding of difficulty speaking may indicate that the client has suffered a stroke. The nurse should assess for any other cognitive changes, assess lung function, and immediately contact the provider and possibly the stroke team. The atrial rate of 250 is normal for atrial fibrillation and is of no concern; the ventricular rate of 75 indicates that the cardiac rate is well-controlled (a ventricular rate above 100 would not be adequately controlled, necessitating additional rate control medications such as a beta blocker, calcium channel blocker or digoxin).

The nurse is planning care for a client admitted to the hospital with influenza. Which interventions should the nurse include in the client's plan of care? Select all that apply. Administer the prescribed oseltamivir. Correct Answer (Blank) Maintain droplet precautions. Correct Answer (Blank) Limit visitors who show signs of a respiratory infection. Correct Answer (Blank) Administer the influenza vaccine. Instruct the client on proper cough etiquette.

Rationale: Antiviral agents, such as oseltamivir, are used to shorten the course and reduce symptoms of the flu. Droplet transmission-based precautions are indicated to prevent the spread of the flu. To avoid further transmission of the illness, visitors with signs/symptoms of a respiratory illness should not be permitted on the unit. It is important to ensure that clients understand how to prevent transmission of infections such as the flu through proper hand hygiene and cough etiquette. The flu vaccine should not be given while the client is acutely ill.

A 14-year-old with a history of sickle cell disease is admitted to the hospital with a diagnosis of vaso-occlusive crisis. Which statement by the client would most likely indicate the cause of this crisis? A "I really enjoyed my fishing trip yesterday. I caught two fish." B "I used cold medicine last week and I have gotten worse." Correct Answer (Blank) C "I knew this would happen. I've been eating too much red meat lately." D "I have really been working hard practicing with the debate team at school."

Rationale: Any condition that increases the body's need for oxygen or alters the transport of oxygen, including infection, dehydration or even cold weather (due to vasospasm) may result in a sickle cell crisis. Sickle cell crisis is also called vaso-occlusive crisis and pain is the most distinguishing clinical feature. Clients should recognize the earliest signs of a vaso-occlusive crisis and seek help, treat all febrile illness promptly and identify environmental hazards that may precipitate a crisis.

The nurse is caring for a client on the antepartum unit who is experiencing late decelerations. Which action by the nurse is appropriate at this time? A Increasing the oxytocin infusion B Applying supplemental oxygen Correct Answer (Blank) C Administering intravenous pain medication D Preparing the room for a vaginal delivery

Rationale: Application of supplemental oxygen, increasing intravenous fluids, and position changes are all appropriate interventions in the event of late decelerations. If oxytocin is being administered, it should be discontinued rather than increased and pain medications should not be given until the fetus recovers. Preparing the room for a vaginal delivery is only warranted if the client has progressed into the second stage of labor.

The nurse in a long-term care facility is reviewing the plan of care for a female client diagnosed with a urinary tract infection. To reduce the risk of recurrence, which interventions should the nurse include in the plan of care? Select all that apply. Have the client void every 2 to 3 hours. Correct Answer (Blank) Bathe the client rather than have her shower. Provide the client with at least 1 liter of water a day. Discourage the client from drinking coffee or black tea. Correct Answer (Blank) Assist the client with wiping the perineum front to back. Correct Answer (Blank)

Rationale: Appropriate interventions include having the client void every 2 to 3 hours during the day to prevent retention and ensure frequent emptying of the bladder. Drinking caffeinate beverages can cause irritation to the bladder, increasing the risk for an infection. To help reduce pathogens from entering the urethral opening, the nurse should assist the client in wiping the perineum from front to back. The client should take a shower, rather than bathe, as the bacteria in the tub water may enter the urethra. The client should drink at least 2 to 3 liters of water a day, if not contraindicated, to help flush out bacteria in the urinary tract.

The nurse is caring for an adult client with septic shock who has a right radial arterial line. The nurse recognizes an overdamped waveform on the monitor. Which of the following is the likely cause? A The pressure on the flush bag is at 300 mmHg. B The pressure tubing is kinked under the client. Correct Answer (Blank) C The client has a 10-point drop in systolic blood pressure. D The transducer is improperly leveled

Rationale: Arterial pressure monitors provide continuous information on a patient's hemodynamics. This information is invaluable to assist in timely clinical decision-making and intervention. If the system is overdamped, there will be a falsely low systolic pressure, but the diastolic pressure is usually accurate. A system that is not optimally damped will be apparent in waveform analysis. An overdamped trace will show less than 1 1/2 oscillations below the baseline with an unclear dicrotic notch. Overdamping can be due to a clot or buildup of fibrin in the catheter tip, loose connections, air in the system, or kinks in the pressure tubing. The pressure on the flush bag should be set to 300 mmHg. If there is fluid loss or air entry, a damped waveform could occur. Transducer leveling is essential for accurate readings but does not affect damping.

The nurse is caring for a client with acute coronary syndrome when the client's cardiac monitor indicates artifact. What action should the nurse take? A Provide skin care to electrode sites Correct Answer (Blank) B Request that the family members encourage the client to rest C Remove the client's blankets from over the leads D Assist the client to take deep breaths and cough

Rationale: Artifact is a common finding in patients requiring an electrocardiogram (ECG). Muscle twitching is a prominent cause (i.e., shivering, tremors); however, improper lead placement and lead displacement from sweating can also be causes. Artifact can also be caused by hiccups and internal medical devices. Removing the blankets will cause the client to lose heat and possibly lead to shivering. Blankets rarely cause artifact. Cough can lead to temporary artifact but is not a common cause.

The nurse is caring for a client who is diagnosed with Hodgkin's disease and is scheduled for radiation therapy to the whole body. The nurse would expect the client to experience which side effect? A High fever B Nausea Correct Answer (Blank) C Neutropenia D Night sweats

Rationale: As a result of radiation therapy, which is at the lymph nodes throughout the body, nausea often results (radiation sickness). Night sweats are an expected finding with Hodgkin's disease. These clients are not likely to have a high fever because the lymphatic or immune system is not fully functional. Neutropenia is a side effect of chemotherapy.

The home health nurse is reviewing the medical record of a client with closed-angle glaucoma in both eyes. Which statement by the client would support this diagnosis? A "I can't see out of my left eye." B "I have constant blurred vision." C "I have specks floating in my eyes." D "I have to turn my head to see around the room." Correct Answer (Blank)

Rationale: As intraocular pressure rises in glaucoma, there is a slow, progressive loss of the peripheral visual field in the affected eye(s). If untreated or uncontrolled, it eventually can lead to blindness. The client's statement that they have to turn their head to be able to see indicates a loss of peripheral vision. Tiny, painless particles floating inside the eye are called floaters that can be harmless or signal retinal detachment. Blurred vision can have many causes, including refractive errors, chronic dry eyes, cataracts and macular degeneration. Complete, unilateral loss of vision is typically seen with a stroke or other intracerebral process.

The home health nurse is reviewing the medical record of a client with closed-angle glaucoma in both eyes. Which statement by the client would support this diagnosis? A "I have constant blurred vision." B "I have specks floating in my eyes." C "I have to turn my head to see around the room." Correct Answer (Blank) D "I can't see out of my left eye."

Rationale: As intraocular pressure rises in glaucoma, there is a slow, progressive loss of the peripheral visual field in the affected eye(s). If untreated or uncontrolled, it eventually can lead to blindness. The client's statement that they have to turn their head to be able to see indicates a loss of peripheral vision. Tiny, painless particles floating inside the eye are called floaters that can be harmless or signal retinal detachment. Blurred vision can have many causes, including refractive errors, chronic dry eyes, cataracts and macular degeneration. Complete, unilateral loss of vision is typically seen with a stroke or other intracerebral process.

The nurse is performing wound care for a client with an abdominal wound. Which action would prevent infection? A Obtaining a new sterile gauze for each stroke of the wound during cleansing Correct Answer (Blank) B Cleansing the wound from the outer edges to the inner bed with each stroke C Using wet-dry technique with sterile water for dressing application D Reapplying sterile gloves before applying the new sterile dressing

Rationale: Aseptic technique decreases the chances of transmitting pathogens and interrupts the chain of infection. In order to minimize cross-contamination of the wound, a new sterile gauze is utilized to not reintroduce contaminants into other areas of the wound bed. Wounds should be cleansed from the inner bed to the outer to avoid contamination. Normal saline is used for the wet-dry dressing technique. Sterile gloves minimize the introduction of pathogens but not the cross-contamination of pathogens pre-existing.

An adolescent client is hospitalized with hemarthrosis from a hemophilia A bleeding episode. Which order from the provider should the nurse question? A Desmopressin acetate 0.3 mcg/kg IV infused over 30 minutes. B Ibuprofen 400 mg as needed for pain. Correct Answer (Blank) C Immobilization of the joint in a splint. D Passive range of motion every shift.

Rationale: Aspirin and NSAIDs such as ibuprofen could increase the risk of additional bleeding in this client due to the potential effects on platelet aggregation; therefore, the nurse should contact the provider to clarify the order for pain relief. Hemophilia A results from a deficiency of clotting factor VIII. Desmopressin acetate promotes the release of von Willebrand's antigen from the platelets, which carries factor VIII, helping to control bleeding associated with factor VIII deficiency. Muscle strengthening and range of motion exercises are always indicated. Splinting of joints may provide pain relief.

The nurse is caring for a client with a central venous line who has a prescription for intravenous potassium chloride to treat severe hypokalemia. When attempting to administer the medication, the nurse encounters significant resistance to flushing. What is the appropriate procedure in this situation? A Forcefully flush the line and begin the infusion B Aspirate and reattempt flushing Correct Answer (Blank) C Clamp and remove the line D Insert a peripheral access, and administer the infusion

Rationale: Assessing for blood return is important in determining the patency of a central line. If blood return is present, flush the line, and begin the infusion. Forceful flushing of a line may dislodge a thrombus attached to the end of the line leading to embolization. If a blockage is identified, the line may be cleared with tPA; therefore, it is not appropriate to immediately remove the line. Administration of IV potassium via peripheral IV is discouraged due to phlebitis.

The nurse is caring for a client who has a tracheostomy to a bedside ventilator. The nurse notes the client attempting to speak through the tracheostomy. Which action should the nurse take? A Assess cuff pressure Correct Answer (Blank) B Provide tracheostomy care C Suction the client D Assess lung compliance

Rationale: Assessment of the tracheostomy cuff should be completed to assure the ventilatory processes are being achieved to the maximum potential. Tracheostomy tube cuff pressures should be routinely monitored. Cuff pressure should be maintained between 15-30 cm H2O (up to 22 mm Hg). When cuff pressures are low, it can be due to inadequate cuff inflation, positioning, damaged cuff, or tracheomalacia.

The nurse is caring for a client who is frequently admitted for acute exacerbations of asthma. The client admits that she does not use her medications as prescribed because she often does not feel short of breath. Which explanation by the nurse best describes the long-term consequences of uncontrolled airway inflammation? A Lung remodeling and permanent changes in lung function will result Correct Answer (Blank) B The alveoli will degenerate and balloon out C Chronic bronchoconstriction of the large airways will occur D The client will experience frequent bouts of pneumonia

Rationale: Asthma is categorized as a chronic, hyper-responsive disorder affecting the terminal bronchioles. Exacerbation of asthma or an "asthma attack" is an acute event. However, the effects of increased number of exacerbations and not using the medication is lung remodeling. This lung remodeling results in more narrow airways and increased mucous. By explaining the consequences of not using the medication, the nurse is reinforcing the need for daily management. Degeneration of alveoli causing increased expansion is a result of emphysema. Asthma does increase the risk of pneumonia, but this option does not address the permanent long-term issues associated with not taking the medication as prescribed. Chronic bronchoconstriction of the large airways is not associated with asthma.

A nurse is caring for a client who has been on bed rest after a partial gastrectomy. On assessment, the nurse notes diminished breath sounds in the bases of the lungs and oxygen saturation of 92% on 2 L. The client reports pain 2 out of 10 and that "it doesn't hurt if I don't move". Which action would be a priority for the nurse to take? A Administer the prescribed pain medication B Assist the client to use the incentive spirometer Correct Answer (Blank) C Reposition the client for comfort D Obtain a prescription for an arterial blood gas (ABG)

Rationale: Atelectasis and pneumonia are prevalent postoperative pulmonary complications. When a mucus plug obstructs one of the bronchi entirely, the pulmonary tissue beyond the plug collapses, resulting in atelectasis. To clear secretions, the nurse encourages the patient to turn frequently, take deep breaths, cough, and use the incentive spirometer at least every 2 hours. If the oxygenation does not improve after ICS, it is appropriate to request a prescription for an ABG or CXR. Pain medication may worsen respiratory status at this point and should be delayed until pulmonary hygiene is performed.

The nurse is reviewing the chart of a client who was recently diagnosed with coronary artery disease due to atherosclerosis. Which factors most likely contributed to the development of this disease? Select all that apply. Follows the D.A.S.H. diet Low-density lipoprotein (LDL) level of 149 mg/dL Correct Answer (Blank) Mother died of a myocardial infarction Correct Answer (Blank) History of diabetes mellitus Correct Answer (Blank) Used to smoke 40 packs per year until one year ago Correct Answer (Blank) Drinks one glass of red wine 3 to 4 days per week

Rationale: Atherosclerosis arises when plaque (fat, cholesterol and other substances) accumulates inside arterial walls. The plaque limits blood flow through arteries and can eventually lead to tissue and organ ischemia. Atherosclerosis can develop in any artery in the body. Smoking contributes to the development of atherosclerosis by damaging artery walls and triggering vasoconstriction. A family history of heart disease (mother) is a risk factor for the development of atherosclerosis. Diabetes and elevated low-density lipoprotein (LDL) cholesterol levels are closely tied to the development of atherosclerosis. The target LDL level for a client is less than 100 mg/dL. The client's current alcohol consumption is within current daily recommendations of no more than 1 to 2 drinks per day. The D.A.S.H. diet is rich in fruits, vegetables, whole grains and legumes, which can reduce the development and progression of atherosclerosis.

The nurse is reviewing the cardiac rhythm strip of a client who has a history of heart failure. The nurse notes an irregular rhythm with no identifiable P waves and a rate of 130. The nurse should understand the client is experiencing which abnormal cardiac rhythm?

Rationale: Atrial fibrillation is an abnormal cardiac rhythm that occurs when multiple areas of cells in the atria initiate electrical conductions. Atrial fibrillation will have a fast, irregular rate with no identifiable P waves. Supraventricular tachycardia will have a regular rhythm with a rate over 150 and no identifiable P waves on the rhythm strip. Sinus tachycardia will have a fast regular rate, but identifiable P waves. Ventricular fibrillation will have no rate, no P waves, and no QRS.

The nurse is caring for a client who experienced a tonic-clonic seizure. The client had autonomic manifestations during the seizure. Which intervention should the nurse implement to ensure the safety of the client post-seizure activity? A Restrain the client's lower extremities. B Raise the siderails of the client's bed. C Check the client's vital signs. D Place the client in a lateral recumbent position. Correct Answer (Blank)

Rationale: Autonomic symptoms include vomiting, salivation, and incontinence. The nurse should place the client in a side-lying position to avoid aspiration of oral secretions. Clients should not be restrained during or after a seizure. Restraining the limbs during a seizure can cause injury. Raising the side rails of the bed is considered a restraint if all of them are raised. The side rails should be padded to ensure safety and prevent injury. Checking the vital signs is an appropriate intervention after a seizure. However, this assessment does not promote safety in this situation.

The nurse is assessing a child with bacterial epiglottitis. The nurse notes inspiratory stridor and substernal retractions. After placing the child on oxygen, which action should the nurse perform next? A Place the client on airborne precautions B Obtain a throat culture C Prepare an intubation tray Correct Answer (Blank) D Administer prescribed corticosteroids

Rationale: Bacterial epiglottitis is an inflammation of the epiglottis and is considered a medical emergency. Swelling of the epiglottis blocks airflow as indicated by stridor and substernal retractions. The nurse should prepare an intubation tray and be prepared to assist in the procedure. Clients should be placed on droplet precautions after initiation of antibiotic therapy. Throat cultures should be avoided with epiglottitis to prevent further obstruction of the airway. Administration of corticosteroids is an expected treatment. However, the airway must first be secured.

The nurse is admitting a 10-month-old infant with suspected bacterial meningitis. Which intervention should the nurse implement first? A Initiate droplet precautions Correct Answer (Blank) B Establish a peripheral venous access device C Administer analgesics and antipyretics as needed D Measure head circumference

Rationale: Bacterial meningitis is an acute inflammation of the meninges and cerebrospinal fluid (CSF). Meningitis is contagious and can be transmitted by droplets from nasopharyngeal secretions of infected individuals. The first intervention should be to place the child on droplet precautions to prevent the transmission to others and spread the infection.

A client is recently diagnosed with Barrett's esophagus. Which of the following statements made by the client demonstrates that further teaching is needed about this illness? A "I will buy a wedge pillow to raise the head of my bed." B "I will cut back on my smoking to 1 pack a day." Correct Answer (Blank) C "I should avoid eating anything for two hours before I go to sleep." D "I will need regular endoscopies to monitor this illness."

Rationale: Barrett's esophagus is a complication of gastroesophageal reflux disease (GERD) and is associated with an increased risk for esophageal cancer. Endoscopies are used to monitor the progression of the disease and catch any cancer in its earliest stages. Treatment for Barrett's esophagus is the same as for GERD. Lifestyle changes include weight loss, avoiding acidic foods and fluids, not eating 90-120 minutes before bedtime, and sleeping with the head of the bed elevated or in a left side-lying position. Cutting back on smoking is too ambiguous. Since smoking aggravates GERD and is linked to the development of cancer, this client should be advised about smoking cessation programs.

The nurse is caring for a client who just delivered a term neonate vaginally. Upon assessment, the nurse notes a saturated perineal pad, hypotension, and tachycardia. Which action by the nurse is appropriate? A Massaging the uterine fundus Correct Answer (Blank) B Removing the infant from the room C Assessing deep tendon reflexes D Assisting the client with ambulation

Rationale: Based on the assessment findings of hypotension and excessive vaginal bleeding, this client is likely experiencing a hemorrhage. Interventions that would be appropriate for the hemorrhaging client include massaging the uterine fundus, emptying the client's bladder, and administration of uterotonic drugs. The infant should be kept with the mother unless the mother is unable to care for the baby, deep tendon reflexes are not directly related to hemorrhage, and ambulation would be unsafe until the client's bleeding is controlled.

A 76-year-old client who smokes one pack of cigarettes per day is diagnosed with chronic obstructive pulmonary disease (COPD). The nurse is teaching the client and family members about the use of oxygen by nasal cannula in the home. Which information is most important for the nurse to include in the discharge instructions? A The client should not smoke while wearing oxygen Correct Answer (Blank) B Adjust the liter flow to 5 L as needed for shortness of breath C Turn the oxygen off during every meal D The client will need to make arrangements for portable oxygen when traveling

Rationale: Because oxygen supports combustion, there is a risk of fire if anyone smokes near the oxygen equipment. The client should take off the oxygen, turn off the flow meter and go to another part of the home or outside to smoke. Smoking cessation should be encouraged and supported in ways that are appropriate for the client's readiness to quit. The most important teaching point at discharge is to stress not smoking while wearing oxygen for the client's safety.

The nurse is reinforcing teaching for a client with Bell's palsy. Which statement by the client indicates that additional teaching is needed? A "I will wear an eye patch at night while I sleep." B "I will eat and drink using the strong side of my mouth." C "I will rest my facial muscles because they are recovering." Correct Answer (Blank) D "I will apply warm, moist heat to my face."

Rationale: Bell's palsy is a form of acute facial paralysis. The disorder is characterized by a drawing sensation and the paralysis of all facial muscles on the affected side. The client cannot close the eye, wrinkle the forehead, smile, whistle or grimace. Nursing care is directed toward managing the major neurologic deficits and providing psychosocial support. Because the eye does not close, the client should wear an eyepatch or tape their eye closed at bedtime to protect their cornea from drying and developing subsequent ulceration or abrasion. The client should eat and drink using the unaffected side of their mouth. Simple massage techniques, the application of warm, moist heat and facial exercises should be explained to the client. The client's statement about resting their facial muscles is incorrect because they should be taught to exercise their facial muscles to aid in the recovery.

The nurse is developing a plan of care for a client with benign prostatic hyperplasia. Which nursing interventions should the nurse include for this client? Select all that apply. Monitor for bladder distention. Correct Answer (Blank) Calculate accurate intake and output. Catheterize as needed for post-void residual urine. Correct Answer (Blank) Limit caffeinated and alcoholic beverages. Correct Answer (Blank) Void every 1 to 2 hours to empty the bladder.

Rationale: Benign prostatic hyperplasia (BPH) is a benign enlargement of the prostate gland. This is common among aging men and can cause urinary difficulties including obstruction, retention, infection and incontinence. Nursing management includes assessing/monitoring for bladder distention. With urinary retention, this can cause pain and increased risk of an acute kidney injury. If the client is retaining urine, the nurse might need to perform straight catheterization to empty the bladder. The client should attempt to void every 4 to 6 hours to prevent retention. The client should avoid or limit the use of caffeinated and alcoholic beverages as these can cause irritation to the bladder and worsen symptoms. Monitoring intake and output would be appropriate to evaluate a client's fluid balance and kidney function, not BPH.

The nurse is caring for a 4-year-old child with a greenstick fracture. The nurse is teaching the parents about the child's fracture. How should the nurse describe this type of fracture? A "A child's bone is more flexible and can be bent 45 degrees before breaking." B "Your child's bones are weak and will break more easily." C "Bones of children are more porous than adults', leading to incomplete breaks." Correct Answer (Blank) D "Fractures in children are harmless and tend to heal quickly."

Rationale: Bones in children are generally more porous than adult bones. This allows the pliable bones of growing children to bend, buckle, and break in a "greenstick" manner. A greenstick fracture occurs when a bone is angulated beyond the limits of bending. The compressed side bends and the tension side develops an incomplete fracture. The other statements are not correct.

The nurse notices bone growths on the distal interphalangeal joints of a client with osteoarthritis. How should the nurse document these findings? A Bouchard's nodes B Heberden's nodes Correct Answer (Blank) C Dermatofibromas D Neurofibromatosis

Rationale: Bony outgrowths found on the distal interphalangeal joint (closest to the fingernail and furthest away from the body) are called Heberden's nodes. If the bony outgrowth was found on the proximal interphalangeal joint (the middle joint of the finger, closest to the body), they would be Bouchard's nodes.

The nurse is caring for a client with bronchiolitis. What is the priority nursing intervention? A Administering 100% oxygen B Maintaining a soft diet C Maintaining a patent airway Correct Answer (Blank) D Administering antiviral medications

Rationale: Bronchiolitis (tracheobronchitis) is the swelling and mucus buildup in the bronchioles, usually due to a viral infection. Using the Airway-Breathing-Circulation decision-making approach, the priority is to ensure a patent airway for this client. The scenario does not provide data that administering 100% oxygen is required. Antiviral medications are indicated but would be used as supportive care. A soft diet might be helpful, but that is not the priority.

An ICU nurse is caring for a client with a suspected foreign body in the airway. The client is scheduled for a bedside bronchoscopy. How should the nurse prepare this client? A Place the client in a supine position B Apply local anesthetic to the back of the client's throat as prescribed Correct Answer (Blank) C Administer the prescribed oral anxiolytic D Obtain consent from the client

Rationale: Bronchoscopy is the direct inspection and examination of the larynx, trachea, and bronchi through either a flexible fiberoptic bronchoscope or a rigid bronchoscope. Before the procedure, the nurse should verify that informed consent has been obtained, but it is not the role of the nurse to obtain consent. The patient must remove dentures and other oral prostheses. The client is placed in a semi-recumbent or lateral position to prevent aspiration. The examination is usually performed under local anesthesia or moderate sedation, which is administered intravenously not orally. General anesthesia may be used for rigid bronchoscopy. A topical anesthetic such as lidocaine is normally sprayed on the pharynx or dropped on the epiglottis and vocal cords and into the trachea to suppress the cough reflex and minimize discomfort.

The home health nurse is reviewing the medical record of a client with recurring oral candidiasis. Which prescribed medication is most likely causing the client's condition? A Fluticasone (nasal spray) B Detemir insulin (injection) C Budesonide (inhaler) Correct Answer (Blank) D Metformin (tab)

Rationale: Budesonide is an inhaled glucocorticoid used in the treatment of asthma and chronic obstructive pulmonary disease (COPD). The most common adverse effects are oropharyngeal (oral) candidiasis and dysphonia (hoarseness and difficulty speaking). Both effects result from the local deposition of inhaled glucocorticoids. To minimize these effects, clients should rinse the mouth with water and gargle after each administration. Using a spacer device can help too. If candidiasis develops, it can be treated with an antifungal drug. The other medications are not likely to cause oral candidiasis.

The client is a 16-year-old with full-thickness burns involving 20% total body surface area. After the initial 24 hours of treatment to replace fluids, which factor is used to determine if the client's fluid needs are being met? A Hourly urine output Correct Answer (Blank) B Daily hematocrit results C Daily weight measurements D Parkland formula for fluids

Rationale: Burn victims are at risk for deficient fluid volume. The Parkland formula for fluid replacement is used during the initial fluid resuscitation for burn victims. Thereafter, hourly urine output is used to guide fluid management. The desired urine output is 30-50 mL/hour for most adults and older children. Fluid replacement formulas (like Parkland) depend on the client's weight on admission, and daily weights are more commonly used to determine if caloric intake is enough to meet increased metabolic needs. Hematocrit (and hemoglobin) can be used to help identify blood loss and RBC destruction, but they are is not used to determine fluid replacement needs.

The nurse is caring for a client who had a closed reduction of a fractured wrist followed by the application of a fiberglass cast 12 hours ago. Which finding requires the nurse's immediate attention? A Client reports burning and tingling in the affected hand and arm Correct Answer (Blank) B Capillary refill of fingers on affected hand is about three seconds C Skin warm to touch and normally colored D Slight swelling of fingers of affected hand

Rationale: Burning and tingling, as well as intense pain out of proportion to the injury, may be an indication of compartment syndrome requiring immediate action by the nurse to prevent permanent muscle damage. The other findings are normal for a client in this situation.

The nurse is caring for a client who has suffered third-degree burns in a motor vehicle accident. The spouse of the client asks the nurse to clarify what is meant by third-degree burn. Which is the best response by the nurse? A "The top layer of the skin is destroyed, exposing the dermis." B "All layers of the skin were destroyed in the burn." Correct Answer (Blank) C "Muscle, tissue, and bone have been injured." D "The skin layers are inflamed. Blisters will appear and may weep."

Rationale: Burns are categorized based on the level of tissue damage. A first-degree burn is a superficial burn that may be pink or red, warm to the touch and painful. An example of a first-degree burn is a sunburn. A second-degree or partial thickness burn is characterized by a blistered appearance, red or pink and painful. An example of a second-degree burn could be a severe sunburn that has blisters. A third-degree burn or full thickness burn includes damage to all layers of the skin and underlying tissues. The area will appear leathery and the color could range from red to black. The area may lack sensation. A fourth-degree burn is also termed a full-thickness burn, but involves muscle, tissue and bone.

A client has been diagnosed with emphysema. Which intervention should the nurse implement when caring for this client? A Reassure the client that the lung damage is usually reversible. B Schedule a lung cancer screening for the client. C Assist the client with enrolling in a smoking cessation program. Correct Answer (Blank) D Inquire if the client has a power of attorney for health care.

Rationale: By the time the client is diagnosed with emphysema, lung damage is usually permanent and a common cause of disability. Smoking is the most common risk factor for developing emphysema and the client should stop smoking. Participating in a structured program increases the client's chance for successful smoking cessation. Scheduling a lung cancer screening and asking about a power of attorney are not appropriate interventions for the client at this time.

The nurse is caring for a client newly diagnosed with chronic obstructive pulmonary disease (COPD). The nurse reviews the client's medical record and notes which risk factors? Select all that apply. History of smoking tobacco products Correct Answer (Blank) Seasonal allergic rhinitis Hyperlipidemia History of childhood asthma Correct Answer (Blank) History of pulmonary embolus A family history of COPD Correct Answer (Blank)

Rationale: COPD is a pulmonary disease that is characterized by chronic airway inflammation, mucus hypersecretion, bronchospasms, destruction of alveoli and airflow limitations, leading to chronic carbon dioxide retention. It is primarily caused by cigarette smoking. Other risk factors include genetics, asthma and exposure to occupational chemicals and air pollution. Hyperlipidemia is a risk factor for cardiovascular diseases. Pulmonary embolism and allergic rhinitis are not risk factors for COPD.

The nurse is caring for a client who is postoperative two hours coronary artery bypass graft. The nurse notes that the client's central venous pressure (CVP) monitoring is 14 mm Hg. Which of the following actions should the nurse take? A Auscultate lung sounds Correct Answer (Blank) B Check the chest tubes for blockage C Increase prescribed IV fluids D Administer prescribed dopamine

Rationale: CVP, which has a normal reading of 4-12 mmHg, measures preload in the right side of the heart. An elevated CVP can indicate fluid volume overload. A client with a CVP of 14 mmHg should be assessed for signs of increased volume and heart failure, including auscultating lung sounds. A decrease in CVP could indicate bleeding, which would require the nurse to administer IV fluids, check tubes, and administer vasopressors.

The nurse is initiating transcutaneous pacing. After turning on the pacemaker and setting the rate to 60, the nurse notes pacer spikes with failure to capture on the electrocardiogram. Which intervention is appropriate? A Increase the current output by 10% Correct Answer (Blank) B Change the rate from 60 to 65 C Change the pacemaker batteries D Increase the sensing threshold

Rationale: Capture failure occurs when the generated pacing stimulus does not initiate myocardial depolarization. Pacing spikes are present, but they are not followed by a QRS complex in the event of ventricular non-capture or by the lack of P waves in the event of atrial non-capture. The most common reason for not obtaining capture is not adequately increasing the current. Increasing the rate will not affect capture, and because the nurse is able to see pacemaker spikes, the pacemaker is functioning properly. Increasing the sensing threshold would be done to manage undersensing, which is not occurring here.

Paramedics are transporting a client suspected of carbon monoxide poisoning to the hospital. Which treatment should the emergency room nurse plan for? A Hyperbaric oxygen therapy Correct Answer (Blank) B Therapeutic hypothermia C Chelation therapy D Naloxone administration

Rationale: Carbon monoxide poisoning leads to displaced oxygen from hemoglobin as carbon monoxide has a stronger affinity for the hemoglobin molecule. The binding of carbon monoxide to hemoglobin leads to reduced oxygen delivery to tissues and organs. Clients with carbon monoxide poisoning require 100% supplemental oxygen and, in some cases, hyperbaric oxygen therapy may be prescribed. Hyperbaric oxygen therapy increases the dissociation of carbon monoxide from the hemoglobin molecule. Chelation therapy is used for poisoning with mercury or lead. Naloxone is an opioid antagonist. Therapeutic hypothermia is typically used after a cardiopulmonary arrest.

The nurse suspects that the client is in cardiogenic shock, following a massive myocardial infarction. Which finding would support the nurse's suspicion? A Bradycardia B Decreased or muffled heart sounds Correct Answer (Blank) C Bounding pulses D Increased cardiac output

Rationale: Cardiogenic shock involves decreased cardiac output and evidence of tissue hypoxia in the presence of adequate intravascular volume; it is the leading cause of death in acute myocardial infarction. Findings of cardiogenic shock include hypotension, rapid and faint peripheral pulses, distant-sounding, decreased heart sounds, cool and mottled skin, oliguria and altered mental status.

The nurse is caring for a client who is receiving external beam radiation to the mediastinum for treatment of bronchial cancer. Which problem should be addressed as a priority in planning care? A Fatigue B Leukopenia Correct Answer (Blank) C Skin irritation D Esophagitis

Rationale: Clients being treated by radiation over the sternum, which is a bone marrow producing area, develop leukopenia due to the depressant effect of radiation therapy on the bone marrow function. With the resultant low white counts, infection is a potential outcome. The other options are possible complication outcomes of radiation therapy on this part of the body. However, they are not the priority because leukopenia is a threat to the entire body and the other options are more of a local problem.

The‌ ‌home‌ ‌health‌ ‌nurse‌ ‌is‌ ‌visiting‌ ‌a‌ ‌client‌ ‌with‌ ‌sickle‌ ‌cell‌ ‌disease.‌ ‌What‌ ‌are‌ ‌the ‌priority‌ interventions‌ ‌for‌ ‌this‌ ‌client? ‌Select‌ ‌all‌ ‌that‌ ‌apply.‌ Minimizing end-organ damage. Correct Answer (Blank) Receiving annual influenza vaccination. Correct Answer (Blank) Preventing opioid abuse. Finding a bone marrow donor. Preventing sickle cell crisis. Correct Answer (Blank) Managing pain effectively. Correct Answer (Blank) Maintaining fluid intake of 2 to 3 liters per day. Correct Answer (Blank)

Rationale: Care for the client with sickle cell disease (SCD) focuses on prevention of crises and sequelae from the disease. Because an acute infection can precipitate a sickle cell crisis, the client should receive the influenza vaccine annually. Increased intake of fluids is recommended to reduce blood viscosity and maintain renal function. SCD is a genetic disease and currently incurable. SCD can be very painful, especially during a crisis. Clients will require increasing doses of analgesics in order to achieve pain control. Morphine and hydromorphone are the drugs of choice.

A client who has been newly diagnosed with carpal tunnel syndrome asks the nurse why they are having pain and tingling in their fingers. Which is the best response from the nurse? A "The pain and tingling is caused by uric acid crystals collecting in the small joints of your fingers." B "The pain and tingling is caused by compression of the median nerve in your wrist." Correct Answer (Blank) C "The pain and tingling is due to sclerotic plaques along the nerves in your hand." D "The pain and tingling is caused by the fluid build-up in the soft tissue of your fingers."

Rationale: Carpal tunnel syndrome (CTS) is a common, repetitive motion-related condition in the wrist. The carpal tunnel is a rigid canal lying between the carpal bones and a fibrous tissue sheet called the flexor retinaculum. A group of nine tendons, enveloped by synovium, share space with the median nerve in the carpal tunnel. When the synovium becomes swollen or thickened, the median nerve is compressed. This causes pain, numbness and painful tingling in the client's fingers and hand. CTS typically does not cause soft tissue fluid build-up. Uric acid crystals collecting in small joints is seen with gout. Sclerotic plaques along nerve fibers tend to occur with multiple sclerosis (MS). Therefore, the best response includes information about the median nerve in the wrist being compressed.

The nurse is providing discharge teaching for a client after cataract surgery of the left eye. Which statements by the client indicate an understanding of the teaching? Select all that apply. "I will call the surgeon if my eye is bloodshot." "I will not rub, press on or scratch my eye." Correct Answer (Blank) "I will follow the instructions for the eye drops." Correct Answer (Blank) "I will call the surgeon if the pain is intense." Correct Answer (Blank) "I will drive very carefully today."

Rationale: Cataract surgery is generally done in an outpatient environment, with the client being discharged within a few hours of the procedure. There is generally little to no pain, and what pain the client has can be relieved with mild analgesics. The client should not press on, scratch or rub the eye. They will likely be prescribed eye drops with varying instructions and should follow these directions. Intense pain may indicate an increase in intraocular pressure or hemorrhage and the surgeon should be notified immediately. Eyes that are bloodshot or bruised is an expected occurrence and should be resolved within a week. The client should not drive until instructed by the surgeon, generally for 48 hours.

The nurse is reviewing the urine culture results for a client with incontinence who has an indwelling urinary catheter in place. The results indicate the presence of Klebsiella. Which action should the nurse take? A Insert a new catheter B Change the catheter tubing and bag at the seal C Flush the catheter using the luer lock port D Obtain a prescription for a condom catheter Correct Answer (Blank)

Rationale: Catheter-associated urinary tract infections (CAUTIs) are the most frequently reported hospital-acquired infection. The Centers for Medicare and Medicaid Services named hospital-acquired UTI as one of the original "never events". CDC guidelines recommend external catheters as an alternative to indwelling catheters in clients without urinary retention or obstruction. If a CAUTI is diagnosed, the indwelling catheter should be discontinued, if possible. While it may be necessary to insert a new catheter, based on guidelines an alternative should be attempted. Catheters are not disconnected at the seal and flushing the catheter is not recommended as both actions increase the risk of bacterial contamination.

The nurse at the outpatient clinic is reviewing after-visit instructions with a client diagnosed with Staphylococcus aureus cellulitis to the right thigh area. Which statement by the client indicates an understanding of the instructions? A "This infection is a result of my diabetes and not preventable." B "The infection is contagious, and I need to make sure to cover it completely." C "I will apply an ice pack to the area to reduce the swelling." D "I will take all of the antibiotic pills until they are gone." Correct Answer (Blank)

Rationale: Cellulitis is an inflammation of the subcutaneous tissue. Staphylococcus aureus and group A beta-hemolytic streptococci are common organisms responsible for causing bacterial skin infections such as cellulitis. It is important to complete the entire course of the prescribed antibiotic to prevent recurrence or drug resistance. Cellulitis of this type is not typically contagious. Although being a diabetic predisposes the client for developing an infection in general, cellulitis tends to occur following a break in the skin that becomes infected. The client should apply moist heat, not cold, to the area.

A client who is receiving continuous oxygen therapy by nasal cannula for an acute respiratory problem is becoming increasingly confused. What should the nurse do first? A Raise the head of the bed B Obtain a pulse oximeter reading Correct Answer (Blank) C Notify the health care provider D Administer a bronchodilator

Rationale: Cerebral hypoxia can be a cause of confusion and is an indicator that the client requires more oxygen. By following the nursing process, the nurse should first determine the client's current oxygen saturation. The nurse should then notify the health care provider of the change in the client's condition and implement the other interventions if appropriate.

A client who has been hospitalized for pneumonia is ready to be discharged home. Which statement indicates that the client understood the discharge instructions given by the nurse? A "I will schedule an appointment for the influenza vaccine." B "I will call the doctor if I still feel tired after a week." C "I will need to use home oxygen therapy for one month." D "I will continue deep breathing and coughing exercises at home." Correct Answer (Blank)

Rationale: Clients should continue to cough and perform deep breathing exercises after discharge to promote alveolar expansion and clearing of the airways. Fatigue is expected for several weeks. Home oxygen therapy is not needed with successful treatment of pneumonia. The influenza vaccine does not protect against pneumonia.

The nurse is assessing an 8 month-old infant diagnosed with atonic cerebral palsy. Which statement from the parent supports this diagnosis? A "When I put my baby on the back to sleep, there is no change in position a few hours later." Correct Answer (Blank) B "When I put my finger in one of the hands, there is no grasp response." C "My baby doesn't seem to follow when I shake toys in front of the face." D "When it thundered loudly last night, the baby didn't even jump."

Rationale: Cerebral palsy is a condition whereby motor dysfunction occurs secondary to damage in the motor centers of the brain. It is most commonly associated with cerebral hypoxia during the birth process. Inability to roll over by eight months of age would illustrate motor dysfunction and a delay in the attainment of developmental milestones. Not following items could be a sign of a visual disturbance, not responding to loud noise could be a sign of hearing disturbance, and not grasping at eight months of age is normal as the grasp reflex begins to diminish after six months of age.

A postpartum client admits to frequent alcohol use throughout the pregnancy. Which newborn assessment finding does the nurse associate with fetal alcohol syndrome (FAS)? A High birth weight. B Craniofacial abnormalities. Correct Answer (Blank) C Organ dysfunction. D Poor muscle tone.

Rationale: Characteristic facial abnormalities are seen in the newborn with FAS, including small head circumference, smaller eye openings, flattened cheekbones and indistinct philtrum. Newborns often have a low birth weight (not high birth weight). Other irreversible effects of alcohol exposure during pregnancy include mental retardation and delayed development; heart defects and vision difficulties or hearing problems; learning disorders; and behavior problems. The newborn with FAS may be irritable. The newborn with FAS will have a low birth weight and have a below average height (length). The organs are not affected in FAS.

The home health nurse is assisting a client who is scheduled for a chemotherapy infusion the next day. The client's medical record indicates frequent episodes of nausea and vomiting after previous chemotherapy treatments. Which action would be most helpful in preventing nausea and vomiting in this client? A Reinforce teaching to the client about the side effects of chemotherapy. B Keep the client on only ice chips for 24 hours after the infusion. C Administrate metoclopramide prior to start of the infusion. Correct Answer (Blank) D Administrate ondansetron immediately after the infusion.

Rationale: Chemotherapy-induced nausea and vomiting (CINV) arises from a variety of local and central nervous system mechanisms. Most chemotherapy drugs are emetogenic to some degree. The most helpful way to prevent CINV is by premedicating the client with an antiemetic drug, such as metoclopramide, before the infusion. The client may need to take antiemetics for a couple days after the infusion, but the need for them can be decreased if the client has been premedicated with antiemetics. Although education is important, it will not prevent CINV.

The nurse is admitting a client to the emergency department (ED) who reports chest pain. Which of the intervention(s) does the nurse expect to be implemented within the first 10 minutes of the client's arrival in the ED? Select all that apply. Blood draw for cardiac troponin Correct Answer (Blank) Supplemental oxygen Problem-focused cardiovascular assessment Correct Answer (Blank) 12-lead ECG Correct Answer (Blank) Intravenous thrombolysis Intravenous access Correct Answer (Blank)

Rationale: Chest pain can be associated with a blockage in a coronary artery. All clients reporting chest pain should be treated as if the pain is cardiac and ischemic in nature. Treatment will depend on whether the chest pain is due to a myocardial infarction (MI) and the type of MI. IV (intravenous) thrombolysis should be used if an ST-elevated myocardial infarction (STEMI) is confirmed, and the client is unable to be transported to the cardiac catheterization lab within 90 minutes. Supplemental oxygen should only be used to maintain oxygen saturation greater than 90%. Supplemental oxygen may harm nonhypoxic clients with STEMI. Treatment in the emergency department (ED) begins with a problem-focused cardiovascular assessment due to assess history and risk factors. IV access should be established, and labs drawn for cardiac markers (i.e. troponin). A 12-lead ECG should be performed to help confirm if the chest pain is an MI.

The nurse is caring of a client with chronic obstructive pulmonary disease (COPD) who has a prescription for chest physiotherapy (CPT). Which action should the nurse take? A Question the healthcare provider's prescription B Prepare suctioning equipment C Perform the maneuver in short time segments Correct Answer (Blank) D Educate the client on coughing and deep breathing

Rationale: Chest physiotherapy (CPT) is used to loosen secretions in the airways. CPT can be performed via percussion, vibration, or postural drainage. Clients with chronic obstructive pulmonary disease (COPD) have decreased activity tolerance and may not tolerate a full session of CPT. The procedures should be done in short intervals followed by rest periods. CPT can be performed in clients with COPD if secretions are retained in the airways. Suctioning equipment should be prepared for all clients receiving CPT in case they are unable to remove their own secretions. Coughing and deep breathing should be taught to all clients to assist in clearing their own secretions.

A nurse is caring for a client who has a pleural chest tube in place to a closed chest drainage system. When assessing the water seal, which of the following findings indicate that the client's lung has re-expanded? A Continuous bubbling B Fluctuation during respiration C Absence of fluctuation or bubbling Correct Answer (Blank) D Intermittent bubbling

Rationale: Chest tubes and a closed drainage system are used to re-expand the involved lung and to remove excess air, fluid, and blood. Wet systems use a water seal to prevent air from moving back into the chest on inspiration. The traditional water seal system for chest drainage has three chambers: a collection chamber, a water seal chamber, and a wet suction control chamber. The water seal chamber has a one-way valve or water seal that prevents air from moving back into the chest when the patient inhales. There is an increase in the water level with inspiration and a return to the baseline level during exhalation; this is referred to as tidaling. Constant or intermittent bubbling in the water-seal chamber indicates leaks in the drainage system or that the lung has not fully expanded/healed. The absence of both tidaling and bubbling indicates that the lung has re-expanded.

The nurse in the clinic is caring for a client who is taking newly prescribed HARRT. Which diagnostic test should the nurse review to determine the effectiveness of this treatment? A Complete blood count B Viral load Correct Answer (Blank) C Total lymphocyte count D CD4 level

Rationale: Viral load measures the amount of HIV genetic material that is in the blood, A decrease in the viral load would show that HAART is effective. CD4 levels, total lymphocytes, and blood count will show the impact of HIV on the immune system but not the effectiveness of antiviral treatment.

The nurse is assisting the healthcare provider with a chest tube removal for a client who has a history of hepatitis B. Which priority intervention will the nurse perform post chest tube removal? A Apply sterile gloves before handling the chest tube B Dispose of the chest tube in the biohazard container Correct Answer (Blank) C Document the client's response to the chest tube removal D Assess the client's understanding of the chest tube removal

Rationale: Chest tubes contain a significant amount of blood product. Nurses should ensure proper disposal of medical waste to prevent the transmission of infectious diseases. Sterile gloves are not required to handle the chest tube. Clean gloves may be worn to dispose of the chest tube. Documenting the client's response is an important intervention. However, the nurse should ensure infection control prior to documenting the findings. Assessing the client's understanding of the procedure should be performed before and after removal. The nurse should first ensure infection control before utilizing teach-back methods.

The nurse is assessing a 2-year-old toddler with a possible diagnosis of congenital heart disease. Which of the following findings will the nurse most likely see with this diagnosis? A Several otitis media episodes in the last year B Takes frequent breaks while playing Correct Answer (Blank) C Changing food preferences and dislikes D Weight and height in the tenth percentile since birth

Rationale: Children with heart disease tend to have exercise intolerance. The child self-limits activity, which is consistent with manifestations of congenital heart disease in children.

The nurse is planning the care for a client who was admitted with complications related to chronic diabetes insipidus. Which interventions are a priority for this client? Select all that apply. Monitor fluid intake and output. Correct Answer (Blank) Measure blood glucose levels before meals. Restrict fluid intake to 1000 mL per day. Weigh the client every morning. Correct Answer (Blank) Monitor urinary specific gravity. Correct Answer (Blank) Evaluate the moisture level of mucous membranes. Correct Answer (Blank) Measure blood pressure and heart rate. Correct Answer (Blank)

Rationale: Chronic diabetes insipidus is a disorder associated with an inadequate level of antidiuretic hormone or the decreased ability for the renal tubules to respond to antidiuretic hormone. Clinical manifestations of this disorder are related to dehydration due to the large amounts of diluted urine being excreted. For clients with diabetes insipidus, priority interventions within the plan of care should focus on fluid balance, including blood pressure and heart rate, fluid intake and output, inspecting the mucous membranes, weighing the client daily and monitoring urinary specific gravity. Fluid restrictions are contraindicated for this client and could lead to severe dehydration. Additionally, diabetes insipidus is not associated with blood glucose imbalance.

Which discharge instruction should the nurse make sure to include for a client with chronic pancreatitis? A "Make sure to eat a low-fat, high-fiber diet." Correct Answer (Blank) B "Take the prescribed pancreatic enzymes on an empty stomach." C "Limit alcohol intake to one drink a day." D "Try to reduce smoking cigarettes to half a pack per day."

Rationale: Chronic pancreatitis is a progressive, destructive disease of the pancreas that has remissions and exacerbations (i.e., flare-ups). Inflammation and fibrosis of the tissue contribute to pancreatic insufficiency and diminished function of the organ. Acute episodes can be lessened by dietary management and lifestyle changes. These include eating bland, low-fat, frequent meals and avoiding rich, fatty foods. Alcohol consumption should be avoided completely as alcohol can precipitate an acute episode. The client should avoid nicotine. The pancreatic enzymes should be taken with food to replace enzymes lacking due to the pancreatitis and aid in digestion.

The nurse is reviewing discharge instructions with a client who has been prescribed ciprofloxacin following a minor burn injury. Which statement by the client requires additional teaching? A "After healing, I should have no scarring from this burn." B "I can take ibuprofen for the pain related to this burn." C "I will protect my skin from the sun with sunscreen and clothing." D "I will not take ciprofloxacin prior to sun exposure." Correct Answer (Blank)

Rationale: Ciprofloxacin is an antibiotic that is associated with causing photosensitivity. Clients should be instructed to protect their skin from sun exposure while taking this medication. Appropriate methods to protect the skin are to limit sun exposure and to wear sunscreen and protective clothing. For a superficial-thickness burn, no scarring will occur, and healing should take 3 to 6 days. The client may take a nonsteroidal anti-inflammatory drug (NSAID), such as ibuprofen, to alleviate the pain associated with the burn. It is inappropriate for the client to stop taking their antibiotic. However, if the client cannot avoid sun exposure, the nurse may contact the health care provider and request that the antibiotic be changed to one that does not cause photosensitivity.

The nurse is caring for a middle-aged client who sustained multiple fractures after a motor vehicle accident. The client refuses to participate in physical therapy and states, "All of those exercises cause me too much discomfort." Which action should the nurse take? A Educate the client on the importance of participating in physical therapy sessions B Collaborate with the physical therapist to modify exercises C Consult with the healthcare provider to decrease the frequency of physical therapy sessions D Administer prescribed pain medication prior to physical therapy sessions Correct Answer (Blank)

Rationale: Client participation in physical therapy sessions will promote recovery and prevent complications of immobility. The nurse should ensure adequate pain management, so the client is able to participate in the sessions. Modifying the exercises may not be conducive to the client's recovery. Decreasing the frequency of physical therapy sessions does not promote recovery and increases the risk of immobility. Educating the client on the benefits of physical therapy will not alleviate the physical discomfort.

The nurse is assessing a client with decreased cardiac output. Which of the following findings should the nurse expect to observe? A Oliguria Correct Answer (Blank) B Bradycardia C Bounding pulses D Metabolic alkalosis

Rationale: Client's with decreased cardiac output are at risk for developing organ dysfunction, particularly renal, due to decreased perfusion to the kidneys and other organs. Oliguria is a finding that may indicate reduced kidney function in clients with decreased cardiac output. Tachycardia is often seen as a compensatory mechanism due to hypotension from the decreased cardiac output. Weak, thready pulses are often seen due to reduced blood pressure and perfusion. Clients may have metabolic acidosis, not alkalosis, due to lactic acid buildup causing lactic acidosis.

The nurse is caring for a client who is experiencing a cardiac arrest due to ventricular fibrillation. The client is currently receiving high-quality cardiopulmonary resuscitation. Which of the following actions should the nurse take? A Administer Atropine, 6mg IV B Prepare the client for transcutaneous pacing. C Administer Epinephrine, 1mg IV. Correct Answer (Blank) D Perform carotid massage to stimulate a vagal response.

Rationale: Clients experiencing a cardiac arrest due to ventricular fibrillation should have high-quality CPR initiated as soon as possible, followed by rapid defibrillation. The client should have Epinephrine administered per CPR guidelines and possibly amiodarone if indicated. Atropine and transcutaneous pacing are indicated for clients who are experiencing symptomatic bradycardia or heart block, not ventricular fibrillation. Carotid massage is occasionally performed for clients experiencing supraventricular tachycardia in order to stimulate a vagal response; however, this practice is controversial.

The nurse is in the room with a client who is experiencing a seizure. The client has visible secretions in the oral cavity. Which action does the nurse take? A Place the client in a supine position B Open the client's mouth C Suction the visible secretions Correct Answer (Blank) D Insert a padded tongue blade

Rationale: Clients experiencing a seizure may experience autonomic manifestations including salivation, incontinence, and vomiting. Oral secretions may cause aspiration if not removed. The nurse should suction the visible secretions without attempting to open the client's mouth. Placing the client in a supine position will increase the risk of aspiration. Opening the client's mouth while there is stiffening of the muscles can lead to injury. Objects should never be inserted into the oral cavity of a client having a seizure. This can lead to injury and aspiration.

The charge nurse is observing a newly hired nurse caring for a client experiencing a seizure. Which of the following actions by the newly hired nurse requires intervention? A Turning the client's head to the side B Recording the time of seizure onset C Loosening clothing around the client's chest D Placing a tongue depressor in the client's mouth Correct Answer (Blank)

Rationale: Clients experiencing a seizure should be protected from injury, including loosening any restrictive clothing and turning the client's head to the side to prevent aspiration. The time of onset of seizure should always be noted by the nurse. Objects should never be inserted into the client's mouth during a seizure. Inserting a tongue depressor to prevent the client from biting the tongue during a seizure is contraindicated and this action requires intervention, as this can obstruct the airway and possibly break the client's teeth.

The nurse is teaching a client about asthma. What information is important for the nurse to include? Select all that apply. Action and purpose of medications Correct Answer (Blank) Use of peak flow monitoring Correct Answer (Blank) When to seek medical assistance Correct Answer (Blank) Avoiding triggers for asthma attacks Correct Answer (Blank) Eating fewer but larger meals Limiting fluid intake

Rationale: Clients must understand the use of medications including quick-relief (rescue) and long-acting (maintenance) therapies. Clients use the peak flow meter to assess effectiveness of medication or breathing status. An acute attack can be a medical emergency and knowing where and how to seek medical care is important. Certain conditions (triggers) can exacerbate an attack and should be avoided. Consumption of large meals can distend the abdomen, which can add to respiratory distress. Smaller, frequent meals are better tolerated. Clients should increase, not limit, fluid intake to help liquefy secretions for easier expectoration.

The nurse is caring for a client receiving chemotherapy for breast cancer. Which client statement indicates that additional teaching is required? A "I feel nauseous every day after receiving my medications." B "My neighbor is bringing me fresh flowers from her garden." Correct Answer (Blank) C "I've been careful to drink water in small sips throughout the day." D "I have been waking up throughout the night, feeling restless."

Rationale: Clients receiving chemotherapeutic treatment are at-risk for neutropenia and associated infections. Chemotherapy can suppress, or weaken, the immune system, otherwise known as immunosuppression. Fresh flowers and plants introduce the potential for the client to be exposed to fungi or bacteria, and thus should be avoided in immunosuppressed patients on chemotherapeutic agents. Nausea and impaired sleep are common side effects of chemotherapy, and although they should be addressed by the nurse, they are not a priority concern. Clients who drink water throughout the day are likely to stay hydrated despite potential nausea and vomiting, so this is also not a concerning statement.

A nurse is providing discharge education about newly prescribed levofloxacin to a client with a history of diabetes type II who has hospital-acquired pneumonia. Which of the statements by the client indicates an understanding of the teaching? A "I can take an antacid with this medication if it upsets my stomach" B "It is common to feel joint aches and soreness when on levofloxacin" C "I will increase my intake of plain yogurt while on this prescription" Correct Answer (Blank) D "This medication may increase my blood sugar"

Rationale: Clients should be instructed to avoid antacids containing magnesium or aluminum; any products containing iron, magnesium, calcium, or zinc at the same time or for 2 hours before or after a dose of the fluoroquinolone. These medications decrease the effectiveness of the antibiotic. The FDA has issued a black box warning for fluoroquinolones, alerting health professionals to the risk of tendinitis and tendon rupture. Therefore, joint tenderness should be reported immediately. Fluoroquinolones are known to have a potential risk of hypoglycemia sometimes resulting in coma, occurring more frequently in the elderly and those with diabetes taking an oral hypoglycemic medicine or insulin. Potent antibiotics increase the risk of superinfection so increasing the intake of probiotics is encouraged.

The nurse enters a client's room as the client begins to have a tonic-clonic seizure. What action should the nurse take? A Place the client on one side Correct Answer (Blank) B Hold the client's arms at the side C Elevate the head of the bed D Insert a padded tongue blade in client's mouth

Rationale: Clients should be positioned on their side. This position keeps the airway patent and allows saliva to drain from the mouth, which prevents aspiration. The nurse should also protect the client from injury by clearing furniture (if the client is on the floor). The client should not be restrained nor should anything be forced in the client's mouth.

An emergency room nurse is assigned to the triage area of a nearby mass casualty event. Which of these clients should the nurse tag as "Black" or "to be seen last"? A A 7-month-old infant with closed fractures to both lower legs who is crying loudly B A 14-year-old client with a small amount of bright red blood dripping from their nose C A 45-year-old client with second and third degree burns over 90% of their body Correct Answer (Blank) D An 83-year-old client with an open fracture of the left arm

Rationale: Clients that are deemed least likely to survive are tagged "black" or "to be seen last." This increases the ability to provide treatment to victims who have a greater chance of survival. Fractures are treatable with splinting and immobilization. It is a positive sign that the infant is alert and crying. The client with minor bleeding from the nose should be evaluated for head trauma, but appears stable at this time. A client with burns over 90% of their body will experience massive fluid loss and the burn injuries will most likely be fatal. Therefore, this client should receive a black tag or be seen last.

The nurse is obtaining the health history of a 71-year-old client who is being admitted for mitral valve replacement surgery related to mitral valve stenosis. During the health history, the nurse should ask if the client experienced which health issue as a child? A Hay fever B Encephalitis C Rheumatic fever Correct Answer (Blank) D Measles

Rationale: Clients that present with mitral valve stenosis often have a history of rheumatic fever or bacterial endocarditis as a child. These illnesses cause valvular damage due to the infection, which leads to thickening and calcification of the valve. These changes will affect the cardiac system by causing the left atrium to dilate, the left atrial pressure to increase, the pulmonary pressure to increase, and the right ventricle will hypertrophy. The client will experience shortness of breath on exertion, paroxysmal nocturnal dyspnea, palpitations, and dry cough. Eventually, the client will experience right-sided heart failure.

The nurse is caring for a client who is experiencing alcohol withdrawal. The nurse notes the client has tremors in the upper extremities, hyperactive deep tendon reflexes, and a change in mental status. For which of the following electrolyte imbalances would the nurse consider a priority for assessment? A Hypomagnesemia Correct Answer (Blank) B Hypokalemia C Hyponatremia D Hypocalcemia

Rationale: Clients who are experiencing alcohol withdrawal are at risk for developing hypomagnesemia. The client will exhibit tremors, muscle weakness, changes in mental status, and hyperactive deep tendon reflexes. The nurse should obtain a serum hypomagnesemia level. Clients with inadequate calcium intake can develop hypocalcemia. Clients with vomiting or diarrhea or taking prescribed diuretics can develop hypokalemia. Hyponatremia develops from an excessive gain of water.

The nurse is caring for a client who had a below-the-knee amputation (BKA) 4 weeks ago and is experiencing phantom limb pain. Which of the following instructions should the nurse provide to the client? A "Avoid massaging your stump for the next 4 weeks." B "Apply cold compresses to your stump 4 times daily." C "Avoid any physical activity until the pain subsides." D "Take your prescribed antiepileptic medication even if pain is not present." Correct Answer (Blank)

Rationale: Clients who are experiencing phantom limb pain are often prescribed antiepileptic medications to treat nerve pain. These medications should be taken regularly as prescribed to ensure maximum effectiveness and are not intended to be used for breakthrough pain on an as-needed basis. Additional interventions for clients experiencing phantom limb pain include exercise, massage, and heat therapy.

The nurse is caring for a client who had a transurethral resection of the prostate (TURP) 1 day ago and has continuous bladder irrigation infusing. Which of the following findings requires the nurse to notify the primary health care provider? A Pink-tinged urine in the urinary drainage bag B Client report of dizziness and nausea Correct Answer (Blank) C Small amounts of blood clots in the client's drainage bag D Client report of occasional bladder spasms

Rationale: Clients who are receiving continuous bladder irrigation following a TURP are at risk for developing the rare, but potentially life-threatening, complication of TURP syndrome. This occurs when irrigation fluid is over-absorbed into the body and stress is placed on the heart. Signs of TURP syndrome include headache, nausea, confusion, shortness of breath, dizziness, hypertension, and bradycardia. The physician should be notified about these client findings immediately. Clients may experience occasional bladder spasms, and these are not a concerning finding, however, severe spasms should be reported as this could indicate catheter obstruction. The nurse should adjust the flow of the continuous bladder irrigation to maintain clear urine output and remove clots. The nurse should adjust the flow with these findings; however, the physician does not need to be notified.

The nurse is caring for a client with an acute ischemic stroke who will be receiving prescribed intravenous fibrinolytic therapy. Which of the following instructions should the nurse provide to the client? A "Your neurological status will be closely monitored every shift." B "You will need an indwelling urethral catheter placed following this procedure." C "Your blood pressure will be closely managed during and after administration of this medication." Correct Answer (Blank) D "You should expect severe headaches during the administration of this medication."

Rationale: Clients who are receiving fibrinolytic therapy are at very high risk for intracerebral hemorrhage as a complication of the therapy. Blood pressure will be closely monitored and managed to reduce hypertensive episodes that may cause intracerebral hemorrhage. The client's neurological status will be monitored every 15 minutes during the infusion, and hourly thereafter per standard protocols. Any invasive lines should be placed prior to fibrinolytic therapy infusion and avoided for 24 hours after to reduce the risk of hemorrhage from the insertion site. The client should be closely monitored for severe headaches, nausea, or vomiting, which may indicate a complication of intracerebral hemorrhage.

The nurse is caring for a client with end-stage renal disease (ESRD). Which manifestations would the nurse expect to see with this client? Select all that apply. Conjunctivitis Correct Answer (Blank) Pruritus Correct Answer (Blank) HbA1c of 5.9% Blood pressure of 119/78 Frequent fractures Correct Answer (Blank)

Rationale: Clients with chronic kidney disease (CKD) and ESRD will present with calcium and phosphorous imbalance, low calcium levels and high phosphorous levels. Bone mineral loss as a result of low calcium levels can result in frequent fractures. Additionally, excessive phosphorous, called metastatic calcifications, can become deposited in various body tissues and systems, including the optic area, which can result in conjunctivitis. Pruritus is a common side effect of excessive serum phosphate. Both diabetes mellitus and hypertension are risk factors for CKD/ESRD, but a HbA1c of 5.9% shows that the diabetes is well-controlled, so does the blood pressure of 119/78 for hypertension.

The nurse in a rehabilitation facility is caring for a client who had a total left hip arthroplasty, using a posterior approach, three days ago. Which intervention should the nurse make sure to include in the client's plan of care? A Keep the client's affected hip bent at least 90 degrees. B Apply an abduction pillow while the client is in bed. Correct Answer (Blank) C Rest the client's heels flat on the bed, in line with the hip. D Instruct the client to cross their legs at their ankles only.

Rationale: Clients who have had a total hip arthroplasty (THA), i.e., hip replacement, are at risk for post-operative hip joint dislocation. An abduction pillow should be used to prevent the client from closing or crossing their legs while in bed, causing adduction beyond the midline of the body, which can lead to dislocation of the new joint. The client's heels should be elevated off the bed, not flat on the bed, to prevent pressure injury to the heels. The affected hip should not be flexed to 90 degrees. Even crossing the legs at the ankles should be discouraged and prevented with this type of hip surgery.

The nurse is caring for a client who had a cardiac catheterization via the left femoral artery 1 hour ago. Which of the following findings is a priority for the nurse to report to the primary health care provider? A Left dorsalis pedal pulse +2 B Scant amount of blood at the insertion site C Left upper extremity weakness Correct Answer (Blank) D Sinus tachycardia at a rate of 102

Rationale: Clients who have undergone a cardiac catheterization should be closely monitored for complications such as dysrhythmias, bleeding, hematoma formation, and thromboembolism, which can cause stroke, cardiac ischemia, or pulmonary embolism. A client who has left upper extremity weakness may be experiencing signs of stroke, and the nurse should notify the primary care provider immediately. The client's pedal pulse of +2 indicates adequate circulation to the affected extremity. Scant amounts of blood at the insertion site may be present following the procedure and should be marked and monitored closely for signs of hemorrhage. Mild sinus tachycardia does not warrant immediate notification of the primary health care provider; however, any dysrhythmias should be reported immediately.

The nurse is caring for a client who received fibrinolytic therapy for an acute ischemic stroke 6 hours ago. Which of the following findings requires the nurse to notify the primary health care provider? A Client has become drowsy and difficult to arouse Correct Answer (Blank) B Bilateral pupils remain 3 mm and sluggish C Client has continued left-sided weakness D Client's blood pressure is 155/70 mmHg

Rationale: Clients who receive fibrinolytic therapy should be closely monitored for any change in neurological status due to an increased risk of intracerebral hemorrhage. The first indication of neurological compromise is the client's level of consciousness, and the primary health care provider should be notified immediately if the client has become drowsy and difficult to arouse. Pupillary reflexes and weakness that have remained unchanged do not need to be reported, and the blood pressure should be maintained at a systolic of less than 160 mm Hg.

The nurse is providing discharge teaching to a client who has had a total hip arthroplasty performed. Which instruction should the nurse include? A Do not cross your legs at the ankles or knees. Correct Answer (Blank) B Avoid climbing stairs for 3 months. C Sleep only on your back and not on your side. D Ambulate using crutches only.

Rationale: Clients who underwent a hip arthroplasty or replacement are at risk for dislocating the new hip joint if certain precautions are not followed. The risk will vary, depending on the surgical approach (anterior vs. posterior). To prevent a post-surgical hip dislocation, the nurse should instruct the client to prevent hip flexion beyond 90 degrees or hip hyperextension. Furthermore, it is generally recommended to keep the legs slightly abducted and avoid adduction such as crossing the legs. The other instructions are not appropriate or required following a hip arthroplasty.

The nurse is developing the plan of care for a client with chronic obstructive pulmonary disease (COPD). Which of the following should the nurse include? A Encourage the client to consume a low calorie, low-protein diet B Assist the client to identify strategies for managing anxiety Correct Answer (Blank) C Restrict the client's fluid intake to less than 2 L/day D Encourage the client to perform most activities of daily living at 1 time in the morning

Rationale: Clients with COPD often experience anxiety during episodes of dyspnea. Anxiety can also cause dyspnea; therefore, clients should identify strategies to manage these episodes. The work of breathing with COPD raises calorie and protein needs, which can lead to protein-calorie malnutrition. Clients should be encouraged to eat a high-calorie, high-protein diet. Clients should be encouraged to drink more than 2 L of water per day to loosen any thick, tenacious secretions that occur with COPD. Clients should be encouraged to space out their activities of daily living throughout the day to conserve energy.

A nurse is caring for an 83-year-old client diagnosed with Parkinson's disease. Which findings should the nurse anticipate? A Muscle rigidity and a shuffling gait Correct Answer (Blank) B Muscle spasm and a bent over posture C Nonintention tremors and urgency with voiding D Voluntary tremor and jerky movement of the elbows

Rationale: Clients with Parkinson's disease have a very distinctive gait with quick short steps (shuffling) that may increase in speed so that they are unable to stop, as well as muscle rigidity. In the other options, only one of the two findings listed is associated with Parkinson's disease: clients may have nonintention tremors, but there is no urgency with voiding; their posture may be "bent over," but there are no muscle spasms; and while they may experience a cogwheel or jerky movement of the elbows, their tremors are not voluntary.

The nurse is reviewing discharge instructions with a client who is newly diagnosed with cardiomyopathy. The client asks the nurse "How will I know if I am overdoing an activity?" Which of the following is the best response by the nurse? A "Stop the activity if you begin to feel fatigue." Correct Answer (Blank) B "Avoid activities that you know will increase your fatigue." C "You will experience chest pain if the activity is too strenuous." D "The activity will need to be modified if you develop a cough."

Rationale: Clients with cardiomyopathy experience decreased cardiac output from the inability of the ventricular to adequately pump. Clients with decreased cardiac output should be encouraged to engage in activity but can develop activity intolerance. The nurse should instruct the client to stop the activity if they become fatigued. Developing a cough or chest pain indicates execration of the decreased cardiac output. The client should stop the activity before they develop symptoms of exacerbation.

The nurse has received report on a group of assigned clients who are receiving prescribed peritoneal dialysis. Which client should the nurse see first? A The client who is scheduled for peritoneal dialysis in 1 hour and has a serum potassium level of 5.5 mEq/L B The client who is receiving continuous ambulatory peritoneal dialysis (CAPD) who has become restless and has a respiratory rate of 36 Correct Answer (Blank) C The client who had a peritoneal dialysis catheter placed 1 day ago and is reporting pink-tinged dialysate outflow D The client who is reporting redness, warmth, and purulent drainage at the peritoneal dialysis catheter site

Rationale: Clients with chronic kidney disease are at high risk for complications related to fluid and electrolyte imbalances. The client in the key is exhibiting manifestations of acute pulmonary edema, which is life-threatening and should be assessed first. The client with hyperkalemia is scheduled for peritoneal dialysis, which will treat the client's laboratory abnormalities. Pink-tinged dialysate outflow is expected for the first week following the new placement of a peritoneal dialysis catheter. The client with redness and purulent drainage at the catheter site is exhibiting signs of possible peritoneal dialysis catheter infection but can be seen after the client in the key.

The nurse is caring for a client in renal failure who has a subclavian vascular access port for hemodialysis. Which finding requires immediate action by the nurse? A Pruritus B Fatigue C Anuria D Fever Correct Answer (Blank)

Rationale: Clients with chronic medical conditions and a central venous access device or line that is intermittently being accessed are at a high risk for developing an infection. A fever in this client could indicate a possible central line-associated blood stream infection. Therefore, a fever should be immediately reported to the client's health care provider. The other findings would be expected in a client with renal failure that requires hemodialysis.

The nurse is evaluating the understanding of disease management of a client with chronic obstructive pulmonary disease. Which statement by the client indicates an understanding of pursed-lip breathing? A "Pursed-lip breathing helps me control how fast I breathe in and out." B "I can reduce my risk of getting pneumonia with pursed-lip breathing." C "Pursed-lip breathing prevents my mouth from getting too dry." D "Pursed-lip breathing reduces carbon dioxide trapped in my lungs." Correct Answer (Blank)

Rationale: Clients with chronic obstructive pulmonary disease (COPD) have difficulty exhaling fully as a result of air trapping in the alveoli due to the disease process. Alveolar collapse can be avoided with the use of pursed-lip breathing allowing the client to exhale more effectively. This technique facilitates appropriate gas exchange as carbon dioxide-rich air that has been trapped in the lungs is blown off allowing oxygen-rich air to be inhaled. This is the primary reason to use pursed-lip breathing.

The nurse is caring for a client who has a closed-chest drainage system in place. Which of the following findings is a priority for the nurse to notify the primary health care provider? A No drainage from the tube Correct Answer (Blank) B Crepitus palpated around the insertion site C Bubbling in the water seal chamber with coughing D Tidaling in the water-seal chamber with inspiration and expiration

Rationale: Clients with closed-chest drainage systems should be closely monitored for complications that could lead to airway compromise, such as pneumothorax. No drainage from the tube may indicate a blockage and requires notification of the primary health care provider. If drainage is blocked, this can lead to a tension pneumothorax. A small amount of crepitus around the insertion site is usually not problematic and is monitored to ensure it does not extend further to the anterior chest and neck. Tidaling in the water-seal chamber is an expected finding. Bubbling in the water-seal chamber with forceful coughing or sneezing is expected, however, continuous bubbling in the water-seal chamber could indicate an air leak, in which the primary health care provider should be notified.

The nurse is teaching a client with heart failure strategies to manage activity intolerance. Which statement should the nurse include in the teaching? A "Complete all your activities in the beginning of your day." B "Finish one task before moving on to the next activity." C "Adapt tasks to allow for minimizing effort." Correct Answer (Blank) D "Focus on activities that require more energy first."

Rationale: Clients with decreased cardiac output should be taught strategies to manage activity intolerance that promote energy conservation. Clients should be taught to take frequent rest periods during activity. Clients should be encouraged to adapt tasks to minimize effort, such as sitting when preparing meals. Clients should perform tasks that require little energy first. Activities should be completed at peak periods of energy, which can be at different times of the day.

The nurse is providing education to a client with diabetes mellitus type 2 about diet. What should the nurse include in the teaching? A "Your carbohydrate intake should be 50 to 75% of your total daily calories." B "Include at least 14 grains of fiber for every 1,000 calories you ingest." Correct Answer (Blank) C "Proteins should make up 10% of your daily food intake." D "Limit your sodium intake to 2,500 mg per day."

Rationale: Clients with diabetes mellitus type 2 should follow a diet that is high in fiber and low in saturated fat. Fiber improves carbohydrate metabolism and lowers total cholesterol. Recommended fiber intake is 14 grains per 1,000 calories. Carbohydrates should be complex and make up 45 to 65% of the total daily caloric intake. Proteins such as meats, eggs, and fish should make up 15 to 20% of the total daily food intake. Sodium intake should be limited to 2,300 mg a day in clients with diabetes.

A nurse in the emergency department is caring for a client with left-sided heart failure who reports the sudden onset of dyspnea. The client's blood pressure is 108/79, the apical pulse is 112, and the nurse notes the client has blood-tinged, frothy sputum. After administering high-flow oxygen to the client, which intervention should the nurse do next? A Obtain a prescription for a chest x-ray. B Place the client in high-Fowler's position with legs dependent. Correct Answer (Blank) C Administer prescribed furosemide IV. D Initiate bedside cardiac monitoring.

Rationale: Clients with heart failure are at increased risk for pulmonary edema. Pulmonary edema develops when the left ventricular function is impaired resulting in a backup of pressure in the pulmonary vasculature system. After administering high flow oxygen, the next action is to place the client in high-Fowler's position with legs dependent. This position decreases preload, with decreases the volume of blood in the heart and pulmonary vasculature. Then, the nurse will administer prescribed furosemide. Cardiac monitoring should be initiated once the client has interventions to decrease preload.

The nurse is caring for a client who was recently diagnosed with hypopituitarism. Which client statements would indicate additional teaching is needed? Select all that apply. "I should feel less fatigued within the next few weeks." "I will need to take calcium and vitamin D supplements." "I should let my health care provider know if there are changes in my urinary patterns." "I should expect breast swelling or tenderness." Correct Answer (Blank) "I should expect to feel more thirsty throughout the day." Correct Answer (Blank)

Rationale: Clients with hypopituitarism are at-risk for diabetes insipidus, in addition to increased levels of prolactin and subsequent breast tenderness, swelling and leakage. Both of these findings are abnormal and should be evaluated by a health care provider (HCP). Thus, additional teaching is required for a client who states that breast swelling or tenderness is expected or that increased thirst is expected. For the same reason, changes in frequency of urination may indicate diabetes insipidus and should be reported to a HCP. Clients with hypopituitarism should expect to see an improvement in symptoms within a few weeks. Additionally, clients are at-risk for osteoporosis and may need calcium and vitamin D supplements.

The nurse is caring for a client recently diagnosed with hypothyroidism. Which client statement indicates that additional teaching is needed? A "Once I feel better, I won't need to take this medication every day." Correct Answer (Blank) B "Within the next few weeks, I should likely feel less fatigued." C "I will have to have blood tests done regularly." D "I might experience anxiety or problems with sleeping."

Rationale: Clients with hypothyroidism often need lifelong thyroid hormone replacement therapy (HRT). Therefore, a client who plans on stopping medications once symptoms have improved is likely to have a re-emergence of symptoms, demonstrating the need for additional teaching. Clients can expect symptoms of hypothyroidism to improve within a few weeks after starting HRT. HRT with such drugs as levothyroxine may cause such side effects as anxiety, nervousness or insomnia. The client should be taught that these are often temporary but to notify their health care provider (HCP) if they worsen. Clients on HRT will have their thyroid stimulating hormone (TSH) levels checked regularly to monitor the effectiveness of their medication therapy.

The nurse is caring for a client with portal hypertension and esophageal varices caused by liver failure. Which nursing problem is the priority? A Risk for impaired skin integrity B Risk for bleeding Correct Answer (Blank) C Risk for malnutrition D Risk for falls

Rationale: Clients with liver failure are at risk for developing portal hypertension secondary to the fibrous changes that occur with liver failure. Esophageal varices are dilated and tortuous vessels of the esophagus that are at risk for rupture if the portal circulation pressure rises. Bleeding from the varices could lead to shock and death; therefore, risk for bleeding is the priority nursing problem.

A client has received instructions for the management of osteoarthritis. Which statement by the client would indicate a need for additional teaching? A "Early surgical intervention is the preferred treatment." Correct Answer (Blank) B "Gradual weight loss may help my pain." C "I will avoid driving after I have taken cyclobenzaprine." D "It is important for me to balance my exercise and rest periods."

Rationale: Clients with osteoarthritis experience the erosion of cartilage in their joints, which leads to pain and swelling of the joints. Weight loss has been shown to decrease pressure on the joints, which can decrease pain. Balancing exercise and rest periods allow the client to be active to help decrease joint stiffness while decreasing the likelihood of more inflammation in the joint. Cyclobenzaprine is a muscle relaxant used to manage pain and muscle spasms in clients with osteoarthritis. Cyclobenzaprine can cause drowsiness, fatigue, and dizziness. For safety reasons, the client should not drive after taking cyclobenzaprine. Initial management of osteoarthritis includes physical therapy, medications, and weight loss. Surgical management is typically not considered until all medical interventions have failed

The home health nurse is discussing safety concerns with a client who has osteoporosis. Which interventions should the nurse recommend to the client? Select all that apply. Request a referral for physical therapy. Correct Answer (Blank) Increase intake of dairy products. Correct Answer (Blank) Provide assistive devices, if needed. Correct Answer (Blank) Go for a jog or run several times a week. Enroll in a smoking cessation program. Correct Answer (Blank)

Rationale: Clients with osteoporosis have fragile bones and are at risk for fractures. The nurse should encourage coordination with physical therapy to increase muscle strength, balance and decrease the likelihood of a fall. The nurse would also provide assistive devices if the client requires them. Not all clients with osteoporosis will need an assistive device. Due to the impact on joints that occurs with running, the nurse should not recommend jogging or running to a client with osteoporosis. Low-impact activities such as walking would be better. Since smoking decreases tissue perfusion in general and impacts bone development, the client should stop smoking. Dairy products are high in calcium and will help with strengthening bones.

The nurse is caring for a client with a diagnosis of pericarditis. The unlicensed assistive person reports to the nurse that the client's last set of vital signs were blood pressure of 84/40 mm Hg, respiratory rate of 28 breaths/minute, heart rate of 112 bpm, and the client seemed short of breath. The nurse examines the client and also notes the presence of jugular vein distention. What should the nurse do next? A Obtain a 12-lead electrocardiogram B Place the client on nothing by mouth status C Administer the prescribed metoprolol D Notify the health care provider Correct Answer (Blank)

Rationale: Clients with pericarditis are at risk for developing cardiac tamponade. Cardiac tamponade means fluid builds up within the pericardial sac, compressing the heart and making it difficult to pump, thus reducing cardiac output. The client is showing signs and symptoms of cardiac tamponade and reduced cardiac output: hypotension, tachycardia, tachypnea, shortness of breath, and jugular vein distention. The nurse should immediately notify the health care provider (HCP) because cardiac tamponade is a life-threatening emergency that requires immediate intervention. The nurse should not give the metoprolol, a beta-blocker, since the drug will further lower the client's blood pressure and may cause the client to go into shock. The other interventions should be performed after the HCP has been notified.

The home health nurse is visiting a client who has peripheral artery disease. It is winter time and cold outside. While observing the client getting dressed, which clothing choice by the client should the nurse question? A A polyester fleece inner layer B A fleece hat with ear protection C Two pairs of cotton socks Correct Answer (Blank) D Wind-protecting pants and jacket

Rationale: Clients with peripheral artery disease (PAD) are at risk for frostbite or hypothermia. It is therefore important for the nurse to ensure that the client understands how to prevent injury by dressing appropriately for cold weather. When cotton becomes damp or wet, it doesn't insulate well. Non-cotton materials are preferred. Additionally, a double layer of socks may become constricting and further decrease circulation. Instead, the client should carry an extra pair of socks if needed. The other clothing choices are appropriate.

The nurse is providing self-care instructions to a client with peripheral vascular disease. Which client statement indicates the need for additional teaching? A "I should not walk around my home without wearing shoes." B "I will trim my toenails every week to prevent ingrown toenails." Correct Answer (Blank) C "I will try different resources to help me stop smoking cigarettes." D "If pain occurs, I will limit my activity until the pain goes away."

Rationale: Clients with peripheral vascular disease (PVD) or diabetes should not cut their nails, corns, and calluses. They should have them trimmed by their provider, nurse, or another provider who specializes in foot care. Clients who have vascular disease have decreased circulation and often experience reduced sensation in one or both feet. They should wear cotton socks, and they should always wear shoes when out of bed. Pain is a defining characteristic of PVD. This pain, termed intermittent claudication, will occur during activity due to the oxygen perfusion abilities not meeting the oxygenation demands of the muscles at work. Clients should verbalize understanding that it is important to rest until the pain subsides. Smoking is a significant risk factor for PVD, and the nurse should inform the client of smoking cessation resources and advocate for a pharmacological smoking cessation treatment if indicated.

The nurse is caring for a client who has a serum sodium level of 117 mEq/L. Which of the following actions is a priority for the nurse to take? A Place the client on continuous cardiac monitoring. B Restrict the client's oral fluid intake. C Implement seizure precautions. Correct Answer (Blank) D Provide a high-sodium diet.

Rationale: Clients with severe hyponatremia are at high risk for developing seizures, which could result in coma and death. The priority action for this client is to implement seizure precautions to protect the client's safety. Cardiac dysrhythmias are common with other electrolyte abnormalities, and therefore, is not the priority intervention for a client with hyponatremia. The nurse should implement restricting the oral fluid intake and replacing dietary sodium; however, the response to these interventions is gradual, as sodium should not be replaced too rapidly, and therefore, is not the priority action over initiating seizure precautions.

The emergency department nurse is taking care of a client with suspected opioid toxicity. The client is difficult to arouse and has shallow breathing. Which intervention does the nurse expect to perform first? A Place the client on prescribed supplemental oxygen. Correct Answer (Blank) B Prepare to perform prescribed gastric lavage. C Assess bilateral pupils. D Administer prescribed naloxone.

Rationale: Clients with suspected opioid toxicity are at high risk for respiratory depression and death. Airway and breathing should be secured before performing other interventions. The nurse should administer supplemental oxygen. A gastric lavage is used for the ingestion of toxic agents. Clients with opioid toxicity will benefit from a reversal agent. Assessing bilateral pupils can help confirm the diagnosis but will not correct the problem. Naloxone is the reversal agent for opioids and should be administered to prevent further complications. However, the client's airway and breathing should be secured first.

The clinic nurse is reinforcing discharge instructions to an older adult client with type 2 diabetes mellitus. The nurse should teach the client to contact the health care provider immediately for which finding? A An open, reddened wound on the heel B Nausea with indigestion C Temperature of 102.2°F (39.0°C) with dysuria Correct Answer (Blank) D Insomnia with daytime fatigue

Rationale: Clients with type 2 diabetes are at risk for developing many systemic complications, including infections. A fever and painful urination (dysuria) can indicate a urinary tract infection (UTI). Because a UTI can quickly lead to urosepsis in high-risk client, these findings are the priority to report to the health care provider. An open wound is also important to report, but not before the fever and dysuria. Nausea and indigestion could indicate gastroparesis; which is common due to vagus nerve damage secondary to long-standing hyperglycemia. Insomnia with daytime fatigue may indicate obstructive sleep apnea. However, the complaints are not exclusive to type 2 diabetes or as urgent as the other findings.

The nurse is assessing a postoperative client with a Hemovac drain to the left knee. The client appears restless and has cold, clammy skin. The Hemovac drain is completely expanded filled with sanguineous drainage in the collection chamber. Which action does the nurse perform next? A Documents the amount of output. B Instructs the client to maintaining suction via compression. C Empties the Hemovac collection chamber. D Applies a pressure dressing to the site. Correct Answer (Blank)

Rationale: Closed drainage systems collect excess fluid or blood from surgical wounds. Hemovac drains will expand as blood fills the chamber. The client's symptoms are indicative of wound hemorrhaging. The nurse should apply a pressure dressing to control the bleeding. Documenting the amount of output is important; however, this action should be performed after the client is stabilized. Educating the client on maintaining suction to the collection chamber is important; however, the client is not stable, and this intervention is not indicated at this time. Emptying the collection chamber will not stop the bleeding.

The nurse has provided discharge teaching to a client with chronic kidney disease (CKD) who is starting peritoneal dialysis. Which of the following statements by the client indicates the need for further teaching? A "I should avoid the use of magnesium oxide antacids if I experience an upset stomach." B "I will monitor my peritoneal dialysis catheter site daily and report any signs of infection." C "I should expect my dialysate outflow to be cloudy or opaque in appearance." Correct Answer (Blank) D "I will avoid using my microwave to warm my dialysate."

Rationale: Dialysate outflow that is cloudy or opaque in appearance indicates possible infection and peritonitis, which needs to be reported. Clients with renal failure should avoid magnesium oxide antacids due to the potential for magnesium toxicity. It is the correct understanding of the teaching to monitor the peritoneal dialysis catheter site for infection and to avoid using the microwave to warm dialysate.

The nurse is observing an unlicensed assistive person (UAP) on the nursing unit. For which action by the UAP should the nurse intervene immediately? A The UAP uses hand sanitizer upon exiting the room of a client with Clostridium difficile. Correct Answer (Blank) B The UAP applies a surgical mask before entering the room of a client on droplet precautions. C The UAP dons gloves before emptying the urinary drainage bag of a client. D The UAP washes their hands after emptying the colostomy pouch of a client.

Rationale: Clostridium difficile (C. diff) is a gastrointestinal infection that is easily spread in health care settings. It is important for the client with C. diff to be placed on contact precautions, which include wearing a gown and gloves when entering the room. Although using an alcohol-based hand sanitizer is acceptable for many situations, hand sanitizer does not kill the C. diff pathogen. Therefore, the UAP must wash their hands with soap and water after being in contact with the client. The other actions are appropriate and do not require immediate interventions by the nurse.

The nurse is monitoring a client who had a surgical debridement of a pressure injury wound. Which finding observed by the nurse would indicate the wound is healing by secondary intention? A Closed with staples immediately following surgery with the absence of erythema and drainage B Open with a small amount of yellow slough and a moderate amount of white exudate C Edges are not approximated, and there is granulation tissue in the wound bed with a scant amount of serous drainage and no odor Correct Answer (Blank) D Closed a week after surgery using sutures with a small amount of erythema and no drainage present

Rationale: Closure of a "clean wound" is typically done by primary closure. The wound is usually closed using sterile techniques with sutures, staples, or synthetic adhesive closure materials. Secondary intention happens when a wound has a large amount of lost tissue, and the edges cannot be brought together, such as is the case with a pressure injury. Therefore, healing must occur from the bottom of the wound upwards. The wound healing process for secondary intention results in longer repair and healing time, more scarring, and an increased chance of infection. A tertiary intention, also called delayed or secondary closure, occurs when there is a need to delay closing a wound, such as when there is poor circulation in the wound area or infection. An example would be an abdominal wound that is kept open to allow drainage and then later closed.

The nurse is caring for a child who was diagnosed with coarctation of the aorta. Which finding should the nurse expect when assessing the child? A Diminished carotid pulses B Bounding pulses in the arms Correct Answer (Blank) C Normal femoral pulses D Strong pedal pulses

Rationale: Coarctation of the aorta, which is a narrowing or constriction of the descending aorta, causes increased blood flow to the upper extremities, resulting in a bounding pulse in the arms. Cardinal signs include resting systolic hypertension, absent or diminished femoral and pedal pulses and a widened pulse pressure.

The nurse is planning the discharge for a client diagnosed with a hip fracture who is homeless. The client has a prescription for outpatient physical therapy. Which action by the nurse would be most appropriate for this client? A Make a follow-up appointment for the client B Educate the client on the importance of attending the sessions C Collaborate with the social worker for transportation Correct Answer (Blank) D Teach mobility exercises that can be performed independently

Rationale: Collaboration with social workers is an important intervention for vulnerable clients. Arranging transportation for the homeless client will ensure they are able to attend their physical therapy sessions. Making a follow-up appointment does not guarantee the client will be able to attend the session. Educating the client on the importance of attending physical therapy is a valuable intervention. However, the client may not have the means to remain compliant. Teaching mobility exercises to the client does not fulfill the discharge prescription.

A client is seen at the primary care clinic for allergic rhinitis. Which clinical manifestations should the nurse expect with this diagnosis? Select all that apply Watery, itchy, reddened eyes Correct Answer (Blank) Alteration in sense of smell Correct Answer (Blank) Purulent, green nasal discharge Increase in serum eosinophil count Correct Answer (Blank) Worsening of symptoms during spring and fall season Correct Answer (Blank)

Rationale: Common symptoms of allergic rhinitis are due primarily to the release of immune mediators such as histamine, prostaglandins, eosinophils and cytokines. This leads to sneezing, runny nose with clear discharge, nasal congestion and an increased eosinophil counts. Symptoms may appear similar to a cold. Due to drainage, the client's sense of smell can be altered. However, purulent green nasal discharge is not consistent with this diagnosis and would be more expected in the case of a sinus infection. Exposure to mold, pets, dust and pollens, especially during spring and fall, can exacerbate allergic rhinitis.

The nurse is caring for a client with a femur fracture. Which assessment findings require the nurse's immediate action? Select all that apply. Absent pulse in affected extremity Correct Answer (Blank) Allergy to penicillin Blood pressure of 88/54 mm Hg Correct Answer (Blank) Palpable hard mass near fracture site Correct Answer (Blank) Shortness of breath Correct Answer (Blank) Pain level of 5 (0 to 10 scale) History of deep vein thrombosis

Rationale: Complications related to fractures, especially of the long bones such as the femur, can include fat embolism, compartment syndrome and hemorrhage. Findings seen with compartment syndrome will include worsening pain, paresthesia (numbness, tingling), pallor (coolness and loss of color) and weak, diminished or absent pulse. A fat embolism will typically travel to the pulmonary vasculature and cause respiratory symptoms. Hemorrhage near the fracture site will manifest with swelling, bruising/hematoma, hypotension and tachycardia. The other findings are important to note but are not life-threatening and should be addressed at a later time.

The nurse is speaking at a senior citizen community center on how to prevent constipation. What information should the nurse include? Select all that apply. Take a daily laxative. Eat foods high in fiber, such as fresh fruit and whole grains. Correct Answer (Blank) Increase your intake of dairy products. Exercise regularly such as walking for 30 minutes 3 to 5 times a week. Correct Answer (Blank) Drink 2 to 3 liters of fluids per day. Correct Answer (Blank) Take iron supplements regularly.

Rationale: Constipation is hardened, formed stool in the bowel and can cause the client to be uncomfortable and have difficulty passing stool. To prevent constipation, it is important to consume at least 2 liters of fluid a day, eat a diet high in fiber and whole grains, and be active, which will help with peristalsis. Taking a daily laxative will cause the body to rely on the laxative and will actually increase the likelihood of constipation. Taking iron supplements or increasing the intake of dairy products will not help prevent constipation.

The nurse is caring for a client with multiple fractures. After the client's physical therapy session, the client's caregiver verbalizes to the nurse, "I don't know if I can manage all of this at home." What is the nurse's response? A "I will speak with the case manager regarding outpatient options." Correct Answer (Blank) B "It is difficult, but the fractures will heal soon." C "Who else will be helping you at home?" D "Why do you think you will have trouble performing the therapy?"

Rationale: Continuity of care ensures that the client and their caregivers feel comfortable performing mobility exercises at home. Concerns regarding the ability to continue treatment should be referred to a case manager for follow-up. The client may benefit from outpatient therapy or home health services. Telling the caregiver that the fractures will heal soon does not address the concern. Asking who else will be helping the caregiver assumes that the client has additional social support and does not address the caregiver's concerns. "Why" questions have an accusatory tone and do not promote empathy towards the client or caregiver.

The nurse is caring for a patient who has acute kidney injury after ingesting a toxic substance. The client is receiving continuous renal replacement therapy. Which of the following interventions will be included in the plan of care? A Maintain the blood flow rate between 300 mL and 400 mL/minute B Utilize strict sterile technique when accessing the temporary dialysis catheter Correct Answer (Blank) C Calculate the client's net fluid loss each shift. D Change the filter when membrane pressures begin to drop

Rationale: Continuous renal replacement therapies (CRRTs) may be indicated for patients with acute kidney disease who are too clinically unstable for traditional hemodialysis. CRRT removes blood from the client at a slower rate than traditional hemodialysis. Usually, between 150 and 250 mL/min. CRRT is performed using a temporary hemodialysis catheter, which is considered a central line. Therefore, strict aseptic technique is required. The client's net fluid loss is calculated hourly and programmed into the machine. Filter changes are indicated when membrane pressures begin to climb.

The nurse is caring for a postoperative client following an appendectomy. The client has a nasogastric tube connected to low, continuous suction. Twice during the shift, the nurse irrigated the nasogastric tube with 50 mL of sterile water each time. At the end of the shift, the nurse empties a total of 450 mL from the drainage container. How much actual gastric drainage did the client have for the nurse's shift? A 350 Correct Answer (Blank) B 450 C 400 D 300

Rationale: Correct answer: 350 mL After surgery, a nasogastric (NG) tube is placed to decompress the stomach and prevent abdominal distension. The nurse should carefully monitor and record the amounts of gastric drainage from the NG tube. Drainage Container Total - Irrigation Volume Total = Shift TotalTotal drainage was 450 mLIrrigation was 50 mL x 2 occurrences = 100 mL

A nurse is inserting a nasogastric tube on a client with a small bowel obstruction. Which event indicates an adverse response to the procedure? A The client gags and vomits green-colored emesis. B The client begins to cough and have oxygen desaturations. Correct Answer (Blank) C Resistance is felt upon entering the nostril, and the client verbalizes pain. D Gastric contents are not aspirated after insertion and flushing of the tube.

Rationale: Coughing with oxygen desaturations indicates that the tube has likely entered the trachea. The nurse should stop the procedure, remove the tube, and assess the client's respiratory status. Gagging and vomiting is a common response to the insertion of a nasogastric tube. The nurse should allow the client to rest before restarting the procedure. Resistance in one or both nostrils is common. The nurse should inspect the nostril to ensure there are no blockages before continuing the procedure. The absence of gastric contents may indicate the nasogastric tube is above the stomach. The nurse should request an X-ray from the healthcare provider to confirm placement.

The nurse is providing education to a client with chronic cystitis on the prevention of recurring urinary tract infections. Which statement by the client indicates a need for further teaching? A "I will avoid scented hygiene products." B "I will drink at least 3 liters of water daily." C "I will empty my bladder every 4 hours." D "I will include cranberry juice in my diet." Correct Answer (Blank)

Rationale: Cranberry juice should be avoided in clients with chronic cystitis. Cranberry juice irritates the bladder and can lead to further inflammation. Scented hygiene products, such as tampons, pads, and toilet paper can irritate the urethra and increase bacterial growth. Clients should be instructed to drink at least 3 liters of fluid daily. Drinking at least 3 liters of water daily flushes out bacteria from the urinary tract. The bladder should be emptied at least every 4 hours to avoid stagnant urine and bacterial growth in the bladder.

A nurse is reviewing laboratory data for a client with diabetes mellitus type 2. Which finding indicates disease progression? A Serum glucose level of 110 mg/dL B Hemoglobin level of 11 g/dL C HbA1C level of 5.8% D Creatinine level of 2.1 mg/dL Correct Answer (Blank)

Rationale: Creatinine level measures kidney function. One of the complications of diabetes mellitus is diabetic nephropathy. The normal creatinine level is 0.5 to 1.0 mg/dL. Elevated levels indicate kidney damage. A serum glucose level of 110 mg/dL is slightly elevated but not uncommon for a client with a history of diabetes mellitus. The normal serum glucose level is 65 to 99 mg/dL. A hemoglobin level of 11 g/dL is below normal but is unrelated to diabetes mellitus. Hemoglobin levels evaluate the amount of oxygenated red blood cells. An HbA1C level of 5.8% is slightly above normal (5.7%). However, this does not indicate disease progression.

The nurse is assessing a toddler diagnosed with croup. Which of the following findings requires the nurse's immediate attention? A Coughing up copious secretions B Apical pulse of 130 C Lethargy for the past hour D Respiratory rate of 54 Correct Answer (Blank)

Rationale: Croup is a viral or bacterial invasion of the upper airway that extends throughout the larynx, trachea, and bronchi. This invasion leads to edema and obstruction of the airway. This condition is seen more commonly in children ages 6 months to 6 years. Characteristic manifestations of croup include a barking cough and hoarseness. Typically, clients with croup have copious secretions. It is a normal finding of the condition. Because croup is an infection, clients will typically present with malaise, fatigue and a low-grade fever. Findings of impending airway obstruction in croup include increased respiratory rate and pulse; substernal, suprasternal, and intercostal retractions; nostril flaring and increased restlessness or agitation. The normal respiration rate in a 1 to 3 year-old child is 24 to 40 breaths/min, the normal pulse rate is 90 to 150 bpms.

The nurse is caring for a client who experienced second-degree burns over 50% of their body. The nurse understands that which medication is used for the prevention of stress ulcers for this client? A Bumetanide 2 mg PO every six hours B Ibuprofen 400 mg PO every eight hours C Furosemide 40 mg IV daily D Pantoprazole 40 mg IV daily Correct Answer (Blank)

Rationale: Curling's ulcers are stress ulcers that occur in clients with severe burns. These ulcers occur within 24 hours of the injury due to the decreased blood flow to the gastrointestinal tract. This leads to a reduction in the protective layer of mucosa while a simultaneous increase in hydrogen ions occurs. Curling's ulcers generally manifest themselves as gastric bleeding and are prevented by administering proton-pump inhibitors, such as pantoprazole. Other factors that prevent these stress ulcers are early enteral feeding, H2 histamine blockers, and medications that protect the mucosa. Nonsteroidal anti-inflammatory drugs (NSAIDs), such as ibuprofen, should be used with caution due to the risk of further decreasing the protective gastric mucosa. In severe burn clients, the priority is fluid resuscitation and increasing cardiac output. Therefore, diuretics such as furosemide or bumetanide are contraindicated.

The nurse is caring for a client who has suspected Cushing's disease. The nurse should monitor for which potential symptoms? Select all that apply. Tachycardia and panic attacks Changes in visual acuity Polyuria and polydipsia History of pathologic fractures Correct Answer (Blank) Large fat pads on the back and shoulders Correct Answer (Blank)

Rationale: Cushing's disease occurs when there is an excess amount of cortisol. The nurse must understand that glucocorticoids, including cortisol, regulate metabolism and immune function, and play a role in the regulation and distribution of serum calcium levels. Therefore, deposition of fat pads on the back and shoulders, as well as fractures secondary to osteoporosis, are signs and symptoms of Cushing's disease that the nurse should be able to recognize. The nurse should understand that tachycardia and panic attacks, as well as to polyuria, are not often associated with Cushing's disease, but are instead associated with other endocrine conditions. Tachycardia and panic attacks are sometimes seen with adrenal tumors, such as pheochromocytoma. Polyuria and polydipsia may be associated with both diabetes insipidus and diabetes mellitus, and changes in vision can be associated with advanced diabetes mellitus.

A nurse suspects a client with a pulmonary embolism has developed complications. Which assessment finding is most indicative of a significant decrease in cardiac output? A A blood pressure of 98/55 mmHg B Client verbalization of lightheadedness C Cyanosis to the oral mucous membranes Correct Answer (Blank) D Urinary output of 35 mL/hr

Rationale: Cyanosis is the most indicative sign of decreased blood flow. Bluish discoloration of the oral mucous membranes indicates a significant decrease in cardiac output and oxygenation. A blood pressure of 98/55 mmHg is on the lower end of normal. Hypotension, which may indicate decreased cardiac output, is defined as a blood pressure lower than 90/60 mmHg. Lightheadedness is not the most indicative symptom of decreased cardiac output. Significant decrease in cardiac output results in syncope. A urine output of 35 mL/hr is on the lower end of normal. Urinary output should be at least 30 mL/hr.

The nurse is performing an assessment on an infant recently diagnosed with cystic fibrosis. Which of the following findings should the nurse anticipate? A Watery nose B Persistent cough Correct Answer (Blank) C Weight gain D Loose stools

Rationale: Cystic fibrosis (CF) is an inherited disease that causes the lungs and digestive system to be severely damaged. Respiratory symptoms include persistent cough with thick, sticky mucus, wheezing, breathlessness and frequent lung infections. The child's nasal passages could become inflamed leading to a stuffy nose. Digestive symptoms include foul-smelling greasy stools, poor weight gain, and intestinal blockage that could lead to severe constipation. Respiratory failure is the most dangerous consequence of CF. The nurse must perform a thorough respiratory assessment and notify the health care professional immediately if signs or symptoms of respiratory distress are observed.

The nurse in a pediatrician's office is completing the health history on an infant boy. The parent comments, "He tastes salty when I kiss him." Based on this comment, for which genetic disorder should the infant be screened? A Sickle cell anemia B Cystic fibrosis Correct Answer (Blank) C Congenital hypothyroidism D Phenylketonuria

Rationale: Cystic fibrosis (CF) is an inherited disorder where a gene mutation causes the cystic fibrosis transmembrane conductance regulator (CFTR) protein to not work properly causing thick mucus in vital organs (especially the lungs). This mutation also causes an increase in sodium and chloride in saliva and sweat making the skin of infants and children with CF taste salty when kissed. Congenital hypothyroidism, sickle cell anemia, and phenylketonuria do not cause an increase in sodium in saliva or sweat.

The nurse is providing care for a 9-year-old child with cerebral palsy who has recently been admitted for repeated episodes of aspiration pneumonia and weight loss. During a discussion with the child's caregivers, which statement by the nurse demonstrates client advocacy? A "Let's review some deep breathing and coughing exercises." B "I will show you how to do manual jaw control during feedings." C "It is possible that we may need to discuss inserting a feeding tube." Correct Answer (Blank) D "An orthotic device may help with positioning during feedings."

Rationale: Deep breathing and coughing exercises may be helpful, but they will not prevent aspiration. The nurse should reinforce manual jaw control and proper positioning during feeding. However, due to repeated episodes of aspiration, it is likely that the client is having significant difficulty controlling the muscles of the tongue/throat and jaw. The nurse needs to discuss the possibility of inserting a feeding tube to prevent future complications associated with repeated aspiration and weight loss.

The nurse is educating a client with a deep vein thrombosis to the right lower extremity about preventing complications. What should the nurse include in the teaching? A "Maintain bedrest throughout your therapy." B "Massage your right leg if you are feeling pain." C "Wear thigh-high compression stockings while in bed." Correct Answer (Blank) D "Apply ice packs to the extremity as needed."

Rationale: Deep vein thrombosis (DVT) is a blood clot that impairs circulation. Thigh-high compression stockings improve circulation and help pump blood back to the heart. Prolonged bed rest increases the risk of further clot formation. Clients should be encouraged to ambulate after initiation of anticoagulant therapy. Extremities with a DVT should never be massaged. Massaging the extremity can dislodge the blood clot and travel to smaller vessels throughout the body. Ice will constrict blood vessels and further impair circulation. Warm, moist compresses are recommended.

The nurse is reinforcing teaching about preventing episodes of agitation in a client with dementia. Which statement by the caregiver would indicate a correct understanding of the teaching? A "I will encourage her to drink coffee when she is tired." B "I will leave the TV on during the night to provide background noise." C "I will speak in a loud voice to make sure she hears me clearly." D "I will use a consistent routine for our activities each day." Correct Answer (Blank)

Rationale: Dementia is a syndrome that is characterized by a slow, progressive decline in cognitive function. Two important actions that are necessary for a client with dementia are preventing overstimulation and providing a structured and orderly environment to reduce and prevent episodes of anxiety and agitation. Environmental distractions and noise should be kept to a minimum. Raising one's voice should be avoided, since it may make the client more agitated. Clients with dementia may have sleep disturbances, thus caffeinated beverages or having a TV on at night should be avoided because those actions are likely to further disrupt the client's ability to sleep.

The nurse is caring for a client who presents with polyuria, polydipsia and a urine specific gravity of 1.002. The nurse suspects that the client is experiencing diabetes insipidus. Which risk factors would support this diagnosis? Select all that apply. Current use of lithium Correct Answer (Blank) History of radiation treatment Correct Answer (Blank) Recent neurologic injury Correct Answer (Blank) Recent neurologic injury Correct Answer (Blank) History of pulmonary disease

Rationale: Diabetes insipidus is a pathological condition caused by a deficient volume of antidiuretic hormone or an inability of the kidneys to respond to antidiuretic hormone. This results in the excretion of large volumes of dilute urine, accompanied by thirst to maintain homeostasis in the light of dehydration. The nurse should be able to identify that polydipsia and a low urine specific gravity may be indicative of diabetes insipidus, and should be able to assess for the presence of risk factors. A history of recent surgery, radiation treatment or neurologic injury may predispose an individual to poor antidiuretic hormone secretion if the posterior pituitary gland, where antidiuretic hormone is excreted from, was injured. Lithium use can be nephrotoxic and can result in the kidney's failure to respond to antidiuretic hormone. A client's history of pulmonary disease is a risk factor for the development of syndrome of inappropriate antidiuretic hormone (SIADH), a condition that results from excess antidiuretic hormone.

A school nurse is talking with a teen who comes into the health office complaining about not feeling well. The nurse notices that the teen is breathing rapidly, has fruity-smelling breath and is clenching their abdomen. The teen's medical records indicate that they have diabetes mellitus type 1. What is the most likely cause of the teen's symptoms? A Diabetic ketoacidosis Correct Answer (Blank) B Acute kidney failure C Acute pancreatitis D Respiratory failure

Rationale: Diabetic ketoacidosis (DKA) is a serious complication related to the deficiency of insulin in individuals with type 1 diabetes mellitus. The most common cause for DKA is poor adherence to insulin treatment or not taking insulin altogether. Manifestations include hyperglycemia, abdominal pain, nausea, vomiting, fruity-smelling breath (due to the build-up of ketones), frequent urination and deep, rapid respirations (Kussmaul's) due to the metabolic acidosis. Respiratory failure, kidney failure and pancreatitis would not cause DKA or the symptoms the teen is exhibiting.

The nurse in a walk-in care clinic is reviewing the medical record of a client who is being treated for frostbite on their toes. Which medical condition most likely placed the client at a higher risk for this type of injury? A Systemic lupus erythematosus B Diabetes mellitus Correct Answer (Blank) C Hyperthyroidism D Aortic stenosis

Rationale: Diabetic neuropathy is a complication of diabetes mellitus that is characterized by decreased sensation in the lower extremities. Clients with diabetic neuropathy are at risk for hypothermal tissue injuries (i.e., frostbite) because they may not feel the pain associated with cold exposure making them unaware of soft tissue injury until it is severe. Diabetes most likely contributed to this client's frostbite injury. There is no immediate increased risk for hypothermal tissue injuries with the other conditions.

The nurse is caring for a newly admitted 6 month-old infant diagnosed with nonorganic failure-to-thrive (NOFTT). What findings would the nurse expect to observe during the initial assessment? A Irritable and "colicky," making no attempts to turn or sit up B Alert, laughing, playing with a rattle and sitting with support C Pale skin, thin arms and legs and uninterested in surroundings Correct Answer (Blank) D Dusky in color with poor skin turgor over abdomen

Rationale: Diagnosis of NOFTT is weight consistently below the 3rd to 5th percentile for age and gender, progressive decrease in weight to below the 3rd to 5th percentile, or a decrease in the percentile rank of two major growth parameters in a short period of time. The nurse would expect to see a child who avoids eye contact, has pale skin, thin arms and legs, and is easily fatigued. NOFTT is due to psychosocial problems such as neglect, lack of knowledge about proper feeding or of the infant's needs. Many times the child engages in self-stimulatory behaviors (head banging or rocking) and is wary of close contact with people.

The nurse is teaching a client with chronic kidney disease (CKD) about home management for peritoneal dialysis (PD). Which of the following statements by the client indicates a need for further teaching? A "My dialysate outflow may be bloody or pink-tinged for the first week." Correct Answer (Blank) B "If my dialysate outflow is brown, it indicates that I am dehydrated." C "My dialysate outflow should typically be clear to light-yellow in appearance." D "If my dialysate outflow is cloudy in appearance, it could mean that I have an infection."

Rationale: Dialysate outflow should typically be clear to light-yellow in appearance. Dialysate that is brown in appearance occurs with a bowel perforation and should be reported immediately. It is common for dialysate outflow to be bloody or pink-tinged the first week after insertion of a PD catheter; however, pink-tinged urine is not expected after this time frame. Cloudy appearance of dialysate indicates possible infection and should be reported immediately to reduce the risk of peritonitis.

The nurse in the emergency room is admitting a client with acute coronary syndrome. Which intervention should the nurse implement first? A Insert a peripheral venous access device B Obtain a 12-lead ECG C Place on continuous ECG monitoring D Administer oxygen via nasal cannula Correct Answer (Blank

Rationale: It is extremely important to quickly diagnose and treat a patient with acute coronary syndrome (ACS) to preserve heart muscle. Initial management of the patient with chest pain most often occurs in the ER. The nurse should first administer supplemental oxygen to keep O2 saturation above 93% and help with myocardial tissue perfusion. A 12-lead ECG should be obtained and the client should be placed on continuous ECG monitoring. Next, the nurse should establish IV access.

The nurse is monitoring a client on a high-dose dopamine infusion post-resuscitation for cardiac arrest. Which finding indicates the client is having a positive response to treatment? A Blood pressure of 109/64 mmHg Correct Answer (Blank) B Pedal pulses +1 C Urine output of 30 mL/hr D Heart rate of 110 beats/min

Rationale: Dopamine is an adrenergic vasopressor used to increase blood pressure after cardiac arrest. The goal of therapy is to maintain the blood pressure within normal limits. A blood pressure of 109/64 mmHg is a positive finding for a client on dopamine. The normal pulse strength is +2. A strength of +1 indicates a weak, thready pulse and decreased circulation. Urinary output of 30 mL/hr is a normal finding. However, high doses of dopamine cause renal blood vessel constriction and are not intended to treat urinary output. A heart rate of 110 beats/min is not a positive response to treatment. Although vasoconstriction increases the heart rate, the intended goal is to maintain normal limits. The normal heart rate is 60 to 100 beats/min.

The nurse in the urgent care clinic is assessing an older male adult client who hit his head during a fall. Which finding should immediately be reported to the health care provider? A Chronic tremors B A 4 x 4 cm hematoma at the base of the skull C Clear drainage from the ears Correct Answer (Blank) D Severe headache

Rationale: Drainage from the nose and ear in a client with head trauma may be caused by leakage of cerebrospinal fluid (CSF) due to a basilar skull fracture. This finding should be immediately reported to the health care provider. The other findings are unrelated or expected after a fall with head injury and are not as urgent as the possible leakage of CSF fluid.

A client is admitted with a diagnosis of myocardial infarction (MI) and reports having chest pain. The nurse provides care based on the knowledge that pain associated with an MI is related to which of the following findings? A Insufficient oxygenation of the cardiac muscle Correct Answer (Blank) B Cardiac arrhythmia C An electrolyte imbalance D Fluid volume excess in the lungs

Rationale: Due to ischemia of the heart muscle, the client will experience pain. This happens because destroyed myocardial tissue can block or interfere with normal cardiac circulation.

The nurse is developing a plan of care for an older adult client who will be undergoing a total hip arthroplasty. To improve the client's postoperative recovery, which interventions should the nurse include? Select all that apply. The use of assistive devices for ambulation Correct Answer (Blank) Administration of subcutaneous warfarin Preoperative pain control with naproxen Application of sequential compression device Correct Answer (Blank) Instruction on plantar and dorsiflexion exercises Correct Answer (Blank)

Rationale: Due to the client's age and the surgical procedure, the client is at risk for a venous thromboembolism. The nurse should include the use of sequential compression devices to decrease venous stasis along with providing instruction on plantar and dorsiflexion exercises. Warfarin is administered orally; it does not come in an injectable form. The client will most likely need assistive devices initially for safe ambulation postoperatively. Preoperatively, the nurse should not use naproxen to control pain because it is a nonsteroidal anti-inflammatory drug (NSAID) and can increase the risk of bleeding during surgery.

A client is admitted for first and second degree burns on the face, neck, anterior chest and hands. Which action should be the nurse's priority? A Assess for dyspnea or stridor. Correct Answer (Blank) B Administer pain medication. C Cover the areas with dry sterile dressings. D Initiate intravenous therapy.

Rationale: Due to the location of the burns, the client is at risk for the development of upper airway edema and subsequent respiratory distress. The other options are correct, but the priority is to assess breathing and manage the airway. The client with any signs of airway injury may need be intubated.

The nurse is caring for a client when the client begins to have a seizure. Which is the priority action for the nurse to take? A Maintain the client's airway. Correct Answer (Blank) B Apply restraints to prevent injury. C Administer an antiepileptic. D Insert an oral airway device or bite block.

Rationale: During a seizure, the client may not be able to maintain an open airway and is at risk for aspirating their own secretions. In order to maintain an open airway, the nurse should turn the client to their side and have suction equipment available. During a seizure, restricting the client's movements and/or inserting an object into the client's mouth is contraindicated and can cause injury or further airway obstruction. After addressing the client's airway and breathing needs, the nurse should administer any prescribed antiepileptics.

The nurse is caring for a client when the client starts to have a tonic-clonic seizure. Which intervention should the nurse implement first? A Administer the prescribed lorazepam B Check the pulse C Prepare for suctioning D Turn the client on their side Correct Answer (Blank)

Rationale: During a seizure, the nurse should use the airway-breathing-circulation prioritization approach. A tonic-clonic seizure causes a person to lose consciousness and have violent muscle contractions. Clients can vomit during a seizure and, therefore, protecting and maintaining an open airway should be done first. This can be accomplished by turning the client on their side. This position assists in maintaining an open airway, draining secretions, and reduce the risk of aspiration, if vomiting occurs. After this intervention, the nurse can prepare for possible suction and administer medications as prescribed.

The nurse is caring for a client who has just been intubated in preparation for mechanical ventilation. Which action should the nurse do next? A Obtain a STAT chest x-ray. Correct Answer (Blank) B Collect arterial blood gas specimen. C Verify end-tidal CO2. D Auscultate bilateral breath sounds.

Rationale: During intubation, an endotracheal tube is inserted through the client's mouth, into the airway. Before the endotracheal tube can be connected to the mechanical ventilation, the tube placement must be confirmed. The nurse will auscultate bilateral breath sounds. A chest x-ray, arterial blood gas specimens, and verifying end-tidal CO2 are performed but after the placement has been confirmed.

The nursing care plan for a client in the diuresis stage of acute kidney injury (AKI) should include monitoring for which complication? A Urinary retention B Electrolyte imbalance Correct Answer (Blank) C Excess fluid volume D Acute pain

Rationale: During the diuresis stage of AKI, the client will be losing an excessive amount of urine (3 to 6 liters per day) and will be at risk for fluid volume deficit and electrolyte imbalances. Therefore, the nurse must monitor the client's electrolyte levels closely, especially potassium (hypokalemia). The other conditions are not complications typically seen during the diuretic phase of AKI.

The postpartum nurse is caring for a mother who vaginally delivered a full-term infant four hours ago. The mother is experiencing heavy vaginal bleeding. Which intervention should the nurse implement first? A Firmly massage the mother's uterine fundus Correct Answer (Blank) B Instruct the mother to remain on strict bedrest C Start an IV infusion of normal saline D Request a transabdominal ultrasound

Rationale: During the immediate postpartum period, the nurse should closely assess the mother for signs and symptoms of postpartum hemorrhage (PPH) (e.g., heavy vaginal bleeding, hypotension, tachycardia). PPH is most often caused by uterine atony. Uterine massage promotes uterine contractions and tone, which decreases bleeding. Uterine massage involves placing a hand on the woman's lower abdomen and stimulating the uterus by repetitive massaging or squeezing movements. The nurse should first provide uterine massage to see if the vaginal bleeding will decrease. It is premature to implement the other interventions at this time.

The nurse is planning care for a client with prostate cancer who has a high-dose brachytherapy implant. Which intervention should the nurse implement to prevent adverse effects of the brachytherapy? A Place the client in a negative pressure room. B Limit visitors to 10 to 30 minutes. Correct Answer (Blank) C Stand 3 feet away from the client. D Increase nursing interaction with the client.

Rationale: During the time the implant is in place, staff entering the room are exposed to gamma rays. Once the treatment is completed and the implant is removed, the patient is no longer radioactive and presents no hazard. The client should be in a private room, but negative pressure is unnecessary. Nurses should work quickly, but effectively and courteously. Minimize time in the room. Maintain the greatest distance possible from the patient consistent with effective care. Six feet is a safe distance for conversation, and visitors should be taught to keep this distance and limit exposure to 10 to 30 minutes.

The nurse is preparing a client for discharge following inpatient treatment for pulmonary tuberculosis. Which instruction should be given to the client? A Avoid contact with children, pregnant women or immunosuppressed persons. B Continue taking medications until symptoms are relieved. C Take medication with aluminum hydroxide if epigastric distress occurs. D Continue taking medications as prescribed. Correct Answer (Blank)

Rationale: Early cessation of treatment may lead to development of drug-resistant tuberculosis (TB). Active TB is usually treated with a combination of four different antibiotics (Isoniazid, rifampin, ethambutol and pyrazinamide) and can now take anywhere from 6-12 months to completely kill the bacteria. As with any antibiotics, clients should continue to take medications even after they begin to feel better. There is no reason to avoid contact with children, pregnant women or immunosuppressed persons once discharged from the hospital as long as the client is adhering to medication schedules. Isoniazid should be taken on an empty stomach; ethambutol can be taken with food to avoid stomach upset. If taken with TB medications, aluminum hydroxide will interfere with absorption of these medications.

A client is admitted with severe injuries resulting from an auto accident. The client's vital signs are BP 120/50 mmHg, pulse rate 110 bpm, and respiratory rate of 28 breaths per minute. Which action should the nurse complete first? A Administer oxygen as ordered. Correct Answer (Blank) B Initiate the ordered intravenous therapy. C Initiate continuous blood pressure monitoring. D Institute continuous cardiac monitoring.

Rationale: Early findings of shock are associated with hypoxia and manifested by a rapid heart rate and rapid respirations. The nurse should use the Airway-Breathing-Circulation approach to prioritize interventions. Therefore, maintaining adequate oxygenation is critical and oxygen should be administered first. The other interventions are secondary to oxygen therapy.

The nurse is caring for a client who has a surgical drain and is one day postoperative. Which of the following interventions should be included in the client's care for today? A Avoid the use of antiembolism stockings B Flush the surgical drain with normal saline C Remove the client's sutures D Assist the client with ambulation Correct Answer (Blank)

Rationale: Early mobility reduces the risk of many postoperative complications. The nurse should assist the client with ambulation to ensure safety (especially due to the presence of a surgical drain). Antiembolism stockings are encouraged to prevent thrombi formation, and sutures should not be removed the day following surgery; they are typically removed about one week after the procedure.

A client has been on antibiotics for 72 hours to treat cystitis. Which findings reported by the client require priority attention by the nurse? A Nausea and anorexia B Elevated temperature Correct Answer (Blank) C Smelly urine D Burning on urination

Rationale: Elevated temperature after 72 hours on an antibiotic indicates that the antibiotic has not been effective in eradicating the offending organism. The health care provider should be informed immediately so that an appropriate medication can be prescribed and complications, such as pyelonephritis, are prevented. The smelly urine and burning are expected with cystitis and during initial treatment. Gastrointestinal findings may be related to the antibiotics as a side effect and should also be reported. However, they are a lower priority and may resolve if the antibiotic is changed.

A 72-year-old client reports having discomfort immediately after a below-the-knee amputation. Which initial action by the nurse is most appropriate? A Wrap the stump snugly in an elastic bandage B Administer opioid narcotics as ordered C Ensure that the stump is elevated Correct Answer (Blank) D Conduct guided imagery or distraction

Rationale: Elevating the stump is the priority intervention for the first 24 hours after surgery. This will help prevent pressure due to postoperative swelling, which will minimize pain or discomfort. Without this action, a firm elastic bandage, opioid narcotics, or guided imagery will have little effect. Analgesics appropriate to the level of pain should be administered as needed in the postoperative period to promote client comfort. After the first day, the residual limb should be flat on the bed.

A child is admitted to the hospital with findings consistent with rheumatic fever. During the admission process, which statement made by a parent would the nurse associate with this disease? A "Our child had a sore throat a month ago, which I treated with an herbal remedy." Correct Answer (Blank) B "Our child is being tested for allergies and has reacted to some allergens." C "Both ears were infected when our child was 3-months-old." D "Last week both feet had a fungal skin infection."

Rationale: Evidence supports a strong relationship between group A streptococcal infections and subsequent rheumatic fever (usually within two to six weeks). Therefore, the history of sore throat may have been an undiagnosed strep A infection. Appropriate antibiotic treatment of strep throat is the most effective way to reduce the risk of developing rheumatic fever.

The nurse is admitting a client with a diagnosis of acute bacterial endocarditis. Which of the following findings would alert the nurse to a complication of this condition? A Macular rash B Pain and pallor in one foot Correct Answer (Blank) C Heart murmur D Hemorrhage

Rationale: Endocarditis is an inflammation of the endocardium layer of the heart secondary to an infection. Acute endocarditis occurs very quickly and the symptoms are severe. Vegetations grow on the heart valves in bacterial endocarditis. These vegetations may break off and travel through the bloodstream, lodge in small vessels, and result in necrosis of the tissue distal to the embolus. Although clients with endocarditis can manifest with lesions and petechiae, the presence of a rash is not commonly found with this condition. Pain and pallor are findings in an embolic arterial occlusion of an extremity. Other findings would include pulselessness, paresthesia, paralysis, and poikilothermia (coldness), known as the 6 Ps of ischemia. Hemorrhage or bleeding is not a typical manifestation of a complication of endocarditis. Heart murmurs are a common finding in endocarditis, and clients with murmurs caused by valve damage are at the highest risk of developing endocarditis.

The nurse is caring for a client who is taking leuprolide for endometriosis. The nurse should monitor the client for which side effects? Select all that apply. Increased fertility Anorexia Amenorrhea Correct Answer (Blank) Hot flashes Correct Answer (Blank) Emotional lability Correct Answer (Blank) Vaginal dryness Correct Answer (Blank)

Rationale: Endometriosis is a benign gynecologic condition in which endometrial tissue grows outside of the uterus. It can be controlled, but not cured, by drug therapy. Drugs commonly used include oral contraceptives and GnRH agonists such leuprolide and nafarelin. GnRH drugs result in amenorrhea (absence of menstruation) and other symptoms that mimic menopause such as hot flashes, vaginal dryness and emotional lability. Anorexia (lack of appetite) is not a side effect usually seen with leuprolide. Leuprolide does not increase fertility.

A nurse is counseling a client on hemodialysis who has chronic constipation about ways to promote normal bowel function. What should be included in client teaching? A "Glycerin suppositories are safe and can be used on a regular basis." B "Magnesium hydroxide can be used for occasional relief from constipation." C "Use bulk-forming fiber powder to increase regularity." Correct Answer (Blank) D "Increase your fluid intake to promote softer stools."

Rationale: Ensure proper dietary habits, such as eating high-residue and high-fiber foods (e.g., fruits, vegetables), adding bran daily (must be introduced gradually), and increasing fluid intake (unless contraindicated) to help prevent constipation. Avoid overuse or long-term use of laxatives. Daily dietary intake of 25 to 30 g/day of fiber (soluble and bulk-forming) is recommended, especially for the treatment of constipation. If laxative use is necessary, one of the following may be prescribed: bulk-forming agents (fiber laxatives), saline and osmotic agents, lubricants, stimulants, or fecal softeners. Magnesium is contraindicated in patients with renal insufficiency.

A male client at a public health clinic is diagnosed with epididymitis. Which additional information is most important for the nurse to obtain? A "Were you ever tested for a sexually transmitted infection?" Correct Answer (Blank) B "Did you know that a consequence of epididymitis is infertility?" C "What are you taking for pain and does it provide total relief?" D "Do you have any questions about your care?"

Rationale: Epididymitis is an inflammation of the epididymis which is a coiled tube at the back of the testicle which helps store sperm. Symptoms may include a swollen or reddened scrotum, testicular pain, penile discharge, and/or blood in the semen. This is most often caused by a bacterial infection, including sexually transmitted infections (STI) such as chlamydia or gonorrhea. Therefore, it is most important to inquire about testing for an STI so that the client can be treated and educated on prevention, as necessary.

The labor and delivery nurse is providing care to a client in active labor who just received epidural anesthesia. Which intervention should the nurse implement following this procedure? A Monitor the fetal heart rate for possible tachycardia B Monitor maternal pulse for possible bradycardia C Monitor maternal blood pressure for possible hypotension Correct Answer (Blank) D Reduce the intravenous fluid infusion to a keep vein open rate

Rationale: Epidural anesthesia can cause transient hypotension. Therefore, the nurse should frequently monitor the maternal blood pressure for signs of hypotension. After an epidural in the laboring client, IV fluids would be increased, not decreased, to prevent hypotension. The nurse would observe for signs of fetal bradycardia (not tachycardia) following an epidural and monitor for signs of maternal tachycardia, not bradycardia, secondary to a decrease in maternal blood pressure.

The home health care nurse is caring for a client who has epilepsy. While the nurse is providing care, the client has a seizure. Which intervention would be most appropriate to prevent an injury to the client? A Lowering the client to the ground Correct Answer (Blank) B Loosening clothing around the waist C Placing a pillow under the client's head D Asking the client to state where they are

Rationale: Epilepsy is a disorder that involves two or more unprovoked seizures. A seizure is an abnormal discharge of electrical activity in the brain which can cause alterations in motor function, sensation, consciousness, behavior and autonomic function. During a seizure, clients may suddenly lose consciousness and fall to the ground, increasing their risk of breaking a bone or suffering a head injury. The most appropriate intervention at this time is to prevent further injury by lowering the client to the ground and placing them in the recovery position to prevent aspiration. Clothing should be loosened around the neck, not the waist, to ensure a patent airway. Once the client is more awake, the nurse can reoriented them to their surroundings.

A nurse is preparing to remove staples from a client's surgical incision who had a total knee arthroplasty. Which of the following actions should the nurse take? A Instruct the client that this is a painless procedure B Squeeze the handle while simultaneously lifting the staple remover C Scrub the surgical site prior to removing the staples Remove every other staple first Correct Answer (Blank)

Rationale: Every other staple should be removed first to monitor the wound edges for approximation. Surgical sites with sutures and staples should not be scrubbed but should be cleansed prior to removal to allow for assessment. Removal of staples requires a staple extractor. Do not pull up while depressing the handle on the staple remover. The closed handle depresses the middle of the staple causing the two ends to bend outward and out of the top layer of skin. Removal of the metal skin staples after total knee arthroplasty is associated with moderate-to-severe pain, and therefore, the nurse needs to premedicate the client as prescribed.

A 16-year-old adolescent is admitted for Ewing's sarcoma of the tibia. In discussing the care with the parents, the nurse should understand that the initial treatment for this diagnosis usually includes which approach? A Bone marrow graft in the affected leg B Amputation of the affected leg C Chemotherapy with adjunctive radiation Correct Answer (Blank) D Surgical excision of the mass

Rationale: Ewing's sarcoma is a rare type of cancerous tumor that grows on a person's bones or cartilage or nerves that surround the bone. Though there are different types of Ewing's sarcoma, the pelvis is typically where the tumor forms and then progresses to the femur. If left untreated, the tumor can spread to other bones, bone marrow, and other vital organs such as the heart, lungs, and kidneys. The initial treatment of Ewing's sarcoma is chemotherapy which may be combined with radiation to reduce the size of the tumor. Once the tumor is reduced in size, the next step is surgical excision of the tumor or oftentimes amputation of the affected leg or arm.

A client has received instructions about the management of their chronic dermatitis. Which action by the client indicates an understanding of the instructions? A The client avoids use of antihistamines when a flare-up occurs. B The client requests to be prescribed oral corticosteroids. C The client applies warm compresses to relieve itching. D The client avoids itching and scratching the affected area. Correct Answer (Blank)

Rationale: Excessive itching can cause excoriation of the skin potentially resulting in inflammation and infection. Cool compresses may cause vasoconstriction and decrease itching. Heat will exacerbate itching. Oral antihistamines are sometimes recommended to provide relief from itching, although they may cause drowsiness. Topical corticosteroids are also sometimes prescribed because they may numb the itch receptors.

The nurse is providing education to a client with exertional angina about treatment options. What should the nurse include in the teaching? A "A thrombolytic agent will be added to your medication regimen." B "You will require a coronary angioplasty to correct this condition." C "It is important to find healthy coping mechanisms to handle stress." Correct Answer (Blank) D "You should perform high intensity exercises to maximize endurance."

Rationale: Exertional angina (stable angina) occurs after strenuous exercise or emotional stress. Exertional chest pain is relieved with rest or nitroglycerin. Increased stress with ineffective coping skills is a risk factor. Clients should be encouraged to utilize healthy coping mechanisms to manage emotional stress. A thrombolytic agent is prescribed when there is a blood clot present during a myocardial infarction. A coronary angioplasty is indicated for clients with atherosclerosis and worsening (unstable) angina. High-intensity exercises should be avoided to decrease the incidence of angina.

The nurse is providing education on illness management to a client with rheumatoid arthritis. What instructions should the nurse provide to the client? A "Take over the counter medications if you have a fever." B "Apply an ice pack to your painful joints." C "Take a cold shower in the mornings." D "Space out activities throughout the day." Correct Answer (Blank)

Rationale: Expected findings in a client with rheumatoid arthritis (RA) include fatigue. Clients should be encouraged to space out activities throughout the day to conserve energy. Fever is an early or late manifestation of a RA exacerbation and should be reported immediately to the healthcare provider. Pain in the joints benefits from heat, not cold. Morning stiffness of the joints benefits from a hot, not cold shower.

The nurse in a primary health care provider's office is talking to a 35-year-old female client about her new diagnosis of uterine fibroids. Which statement by the woman indicates that additional teaching is needed? A "Fibroids occur more frequently in women my age but no one knows what causes them." B "Uterine fibroids are noncancerous tumors that grow slowly." C "I sometimes experience pelvic pressure and pain, along with heavy menstrual bleeding." D "Even if the fibroids do not cause problems, they must still need to be taken out." Correct Answer (Blank)

Rationale: Fibroids that cause no findings may require only "watchful waiting". The client may just need pelvic exams or ultrasounds periodically to monitor the fibroid growth. Treatment for the symptoms of fibroids (e.g. painful menses and heavy periods) may include oral contraceptives, an intrauterine device (IUD), iron supplements to prevent or treat anemia (due to heavy periods), non-steroidal anti-inflammatory drugs (NSAIDs) for cramps or pain or even short-term hormonal therapy to help shrink the fibroids. Surgical removal using my lobectomy or hysterectomy is usually reserved as a final alternative after other treatment options have failed to provide adequate relief.

The nurse is providing discharge education to a client diagnosed with fibromyalgia syndrome (FMS). Which statement by the client indicates that additional teaching is needed? A "I will avoid caffeine, sugar, and alcohol before bedtime." B "I should take the duloxetine once a day, every day." C "If my pain stays the same, I will take an extra dose of the pregabalin." Correct Answer (Blank) D "I will take an exercise class - maybe I'll sign up for a yoga class."

Rationale: Fibromyalgia Syndrome (FMS) or fibromyalgia is a chronic pain syndrome, characterized by pain in the lower back, neck, or head that can be triggered by pressure, noxious stimuli, or stress. The client will often complain of sleep cycle disturbance and moderate to severe fatigue. Treatment for fibromyalgia is varied, but medications such as duloxetine and pregabalin are FDA-approved for the treatment of FMS. Pregabalin [Lyrica] reduces GABA neurotransmitter release, helping to relieve neuropathic pain seen with FMS. Pregabalin has the potential for abuse and physical dependence and is classified as a Schedule V controlled substance. The client should be instructed not to take additional doses if pain is not relieved and to notify their health care provider instead.

The school nurse is educating teachers that the number of children diagnosed with fifth disease has increased. Which clinical manifestation of fifth disease should the nurse emphasize to the teachers? A Koplik spots appear first followed by a rash that appears first on the face and spreads downward B Macule that rapidly progresses to papule and then vesicles C Bright red cheeks, with a "slapped face" appearance Correct Answer (Blank) D Discrete rose pink macules will appear first on the trunk and fade when pressure is applied

Rationale: Fifth disease is also referred to as parvovirus infection or erythema infectiosum. Some people may call it slapped-cheek disease because of the face rash that develops resembling slap marks. It is also commonly called fifth disease because it was fifth of a group of once-common childhood diseases that all have similar rashes. The other four diseases are measles, rubella, scarlet fever and Dukes' disease. People will not know that a child has parvovirus infection until the rash appears, and by that time the child is no longer contagious.

The client is diagnosed with infective endocarditis (IE) and has been receiving antibiotic therapy for four days. Which finding suggests that the antibiotic therapy has not been effective and must be reported to the health care provider immediately? A Muscle tenderness B Nausea with vomiting C Temperature of 103° F (39.5° C) Correct Answer (Blank) D Streaks of red under the nails

Rationale: Findings of IE include skin rash (petechiae) and small areas of bleeding (splinter hemorrhages) under the fingernails. Muscle or joint pain or weakness are also common symptoms of IE. Nausea and vomiting may be side effects of the treatment; these findings probably would have appeared shortly after beginning treatment. Prolonged fever after 72 hours of antibiotic therapy indicates the antibiotic regime is not effective against the strain of microorganism - the nurse must call the HCP about this finding. Surgical intervention may be indicated for persistent sepsis after 72 hours of appropriate antibiotic treatment.

The nurse is caring for a client receiving hemodialysis three times weekly. The client has a new arteriovenous fistula. Which of the following will be included in the plan of care? A The fistula will be assessed for thrill and bruit each shift Correct Answer (Blank) B The client will be encouraged to eat a low protein diet C Antihypertensive medications will be administered immediately before dialysis treatments D The new access will be allowed to mature for six months.

Rationale: Fistulas should be assessed for a thrill and bruit each shift to assess for stenosis and clotting. Hemodialysis patients are instructed to eat a moderate or high protein diet since dialysis removes protein from the blood. Some antihypertensive agents are cleared by HD, including beta-blockers so they are administered after the treatment. In addition, dialysis removes fluid, and a reduction in blood pressure may occur during the treatment if the medication is administered. AV fistulas are allowed to mature for 2 to 4 months.

The nurse is providing care to a client who is postoperative from an above-the-knee amputation. Which intervention should the nurse implement to prevent flexion contractures? A Elevate the residual limb on several pillows. B Have the client sit in a chair for 1 hour every shift. C Position the client prone several times a day for 20 minutes. Correct Answer (Blank) D Wrap the residual limb using an elastic bandage.

Rationale: Flexion contractions occur due to improper positioning after an amputation. A prone position will promote the extension of the hip and prevent flexion contractures. Elevating the residual limb on several pillows will place the hip in a prolonged flexed position and cause contractures. Having the client sit for a prolonged period of time will flex the hip joint and potentially cause contractures. Wrapping the residual limb using an elastic bandage will promote shrinkage of the residual limb but will not prevent contractures.

The nurse is caring for a client diagnosed with septic shock. The client has received fluid resuscitation and has been started on IV norepinephrine. Which of the following is the intended effect of the intervention? A Improved tissue perfusion Correct Answer (Blank) B Anticoagulation C Improved stress response D Target the invading organism

Rationale: Fluids and vasopressors (norepinephrine) are the first-line treatments of the hypotension and fluid shifts that occur in septic shock. Bacterial (endo)toxins are released into the bloodstream, which trigger vasodilation. Activation of the coagulation system occurs in sepsis and puts the client at risk of thrombus & eventually bleeding due to consumption of platelets. However, fluids and vasopressors are not the treatment of hypercoagulability. Cortisol is released as a physiologic response to stress from sepsis, but depletion is treated with corticosteroid therapy. Anti-infectives are used to kill the invading pathogen.

The nurse in the postanesthesia care unit is caring for a client who is recovering from a left lower lobectomy. The client has a chest tube in place. While repositioning the client during the first post-op check, the nurse notices 75 mL of a dark red fluid flowing into the collection chamber of the chest drainage system. What action should the nurse take? A Check if the client had a type and crossmatch done. B Turn the client back to the original position. C Notify the surgeon immediately. D Continue to monitor the rate of the drainage. Correct Answer (Blank)

Rationale: Following a lobectomy, it is not unusual for blood to collect in the chest and be released into the chest drainage system when the client changes positions. This is most common in the immediate, post-operative phase. The dark color of the blood indicates it is likely old blood and there is not active bleeding inside of the chest. Sanguineous drainage should be expected within the initial 24 hours post-op, progressing to serosanguineous, and then to a serous type. If the drainage exceeds approximately 100 mL in one hour, then the nurse should call the surgeon. In this case, the nurse should continue to monitor the rate of the drainage.

The nurse on the surgical unit is caring for a client who underwent a thyroidectomy eight hours ago. Which finding requires immediate action? A Respiratory rate of 24 B Voice hoarseness C Diaphoresis D Wheezing Correct Answer (Blank)

Rationale: Following a thyroidectomy, it is critical that the nurse monitors the client closely for hemorrhage or airway obstruction, such as tracheal compression, which can lead to respiratory arrest. Signs of hemorrhage or tracheal compression include wheezing, frequent swallowing or choking and bleeding noted from the incision site. The elevated respiratory rate is related to the tracheal compression/airway obstruction and should return to normal once the compression has been addressed. Voice hoarseness frequently occurs following a thyroidectomy due to edema at the site of incision and diaphoresis may be a symptom of hyperthyroidism. Both should be monitored, but do not require immediate action.

A school nurse is called to the playground for an episode of mouth trauma. The nurse finds that the front tooth of a 9-year-old child has been avulsed ("knocked out"). After recovering the tooth, which action should the nurse take? A Ask the child to replace the tooth even if the bleeding continues B Rinse the tooth in water before placing it into its socket Correct Answer (Blank) C Place the tooth in a clean plastic bag for transport to the dentist D Hold the tooth by the roots until reaching the emergency room

Rationale: Following avulsion of a permanent tooth, it is important to rinse the dirty tooth in water, saline solution or milk before re-implantation. If possible, replace the tooth into its socket within 30 minutes while avoiding contact with the root. The child should be taken to the dentist as soon as possible

In which situation would a child be treated by the use of enemas followed by an antitoxin? A A child who is diagnosed with botulism Correct Answer (Blank) B A child who has swallowed a handful of iron-fortified vitamins C A child who bit into a laundry detergent packet D A child who has eaten an undetermined number of ibuprofen tablets

Rationale: Foodborne botulism is treated by removing the contaminated food from the gastrointestinal tract by use of enemas (or inducing vomiting) and by administration of a botulinum antitoxin. Iron poisoning is treated with a strong laxative fluid; severe poisoning may require intravenous chelation therapy. Non-steroidal anti-inflammatory drugs, such as ibuprofen, are treated with activated charcoal; very large overdoses may require orogastric lavage. Since laundry detergents are alkaline agents, intravenous therapy to promote dilution is used; tracheal intubation with ventilation may also be required.

The nurse is caring for a client who is experiencing hypothermia following submersion in cold water. The client is unresponsive with a core body temperature of 25°C (82.5°F) and is prescribed active rewarming. The nurse should expect to use which rewarming method for this client? A Cardiopulmonary bypass Correct Answer (Blank) B Warming blankets C Heating lamps D Compression heat packs

Rationale: For a client with hypothermia, rewarming methods used are active and passive interventions. The client with severe hypothermia (less than 28°C to 32.2°C or 82.5°F to 90°F) will require active internal rewarming techniques such as cardiopulmonary bypass, infusion of warm IV fluids, and warmed peritoneal lavage. Clients with mild hypothermia will require passive external rewarming techniques, such as warming blankets, heat packs, and heating lamps.

The nurse is providing education to a client with asthma who is allergic to house-dust mites. Which information about the prevention of an asthma attack would be the most important for the nurse to include during teaching? A Choose 100% cotton linens with a low thread count. B Open the curtains to let the sunlight in each morning C Wash and rinse the bed linens in hot water Correct Answer (Blank) D Change the pillow covers every month

Rationale: For asthma clients who are allergic to house-dust mites, the mattresses and pillows should be encased in allergen-impermeable covers. All bed linens such as pillow cases, sheets and blankets should be washed and rinsed weekly in hot water at temperatures above 130 F (54.4 C), the temperature necessary to kill the dust mites.

The nurse is reviewing the laboratory data for a client who is receiving prescribed intravenous (IV) fluids to treat fluid volume deficit. Which result would indicate the fluid therapy has been effective? A Serum sodium 138 mEq/L Correct Answer (Blank) B Blood urea nitrogen (BUN) 26 mg/dL C Hematocrit (Hct) 56% D Urine specific gravity 1.038

Rationale: For clients who are receiving prescribed IV fluids to treat fluid volume deficits, laboratory data can be used to determine if the fluid therapy is effective. In fluid volume deficit, the client will have low sodium, increased BUN, increased hematocrit, and increased urine osmolarity. A serum sodium of 138mEq/L is within the normal range (135-145), indicating that the fluid therapy has been effective. Normal BUN is 6-20, normal hematocrit is 35%-47% for females and 39%-50% for males, and normal specific gravity is 1.010-1.025. The elevated BUN, Hct, and urine specific gravity indicate the client is still experiencing fluid volume deficit.

The hospital nurse is teaching a client who is being discharged home about their new diagnosis of type 2 diabetes mellitus. Which statement by the client would require clarification from the nurse? A "When I administer my insulin, I will rotate injection sites." B "I will make sure to have an eye exam every five years." Correct Answer (Blank) C "It is important to increase my physical activity gradually." D "At home, I should check my blood sugar before meals and at bedtime."

Rationale: For diabetic clients, it is imperative they protect and monitor the function of their eyes and kidneys due to the vascular damage associated with diabetes mellitus. Eye exams should be performed annually for diabetic clients due to the risk of diabetic retinopathy. Clients should increase physical activity slowly to prevent injury. Additionally, before meals and at bedtime is an appropriate time for checking blood sugar. Rotating injection sites helps prevent lipodystrophy and increase the absorption of insulin.

The nurse is monitoring a client with psoriasis who is receiving prescribed localized phototherapy. Which findings should the nurse expect to observe after phototherapy? A Redness and a burning sensation at the site Correct Answer (Blank) B Fever less than 101⁰F and chills C Headache, fatigue, and nausea D New growths or patches near the site

Rationale: For patients who do not respond well to topical treatments, phototherapy using narrow-band ultraviolet-B (UVB) therapy may be effective as a single-therapy modality. Maintenance of mild skin erythema is desired for optimal results; however, burning should be avoided. Signs and symptoms of burning post-phototherapy include moderate-to-severe redness, tenderness, pain, tightness, itching, and rarely, blistering of the skin. Phototherapy may trigger a recurrence of herpes simplex infections and increase the risk of skin cancer. Therefore, all new lesions need to be evaluated.

An older adult client, admitted after a fall at home, begins to seize and loses consciousness. What action should the nurse do next? A Stay with client and monitor the condition Correct Answer (Blank) B Collect pillows and pad the side rails of the bed C Announce a cardiac arrest and plan to assist with intubation D Place an oral airway in the mouth and suction

Rationale: For the client's safety, remain at the bedside and observe respirations, the movements of the extremities and level of consciousness. Prepare to clear the airway or suction if obstructed. If suction equipment is not at the bedside, request that someone else get it for you, rather than leaving the client. Do not place anything in the client's mouth. For safety, do not leave the client unattended. A cardiac arrest should only be announced if pulse or respirations are absent after the seizure.

During evening rounds, the nurse notices a foul smell in the room of a client diagnosed with pneumonia who was started on intravenous (IV) antibiotics 10 hours ago. Which statement by the client would best indicate a possible complication of this diagnosis? A "I have been coughing up foul-tasting, brown, thick sputum." Correct Answer (Blank) B "I feel hot on and off and have been sweating all day." C "I have been incontinent of urine and need to change my pad often." D I have a sharp pain in my chest when I take a breath."

Rationale: Foul smelling and tasting sputum signals the possible development of a lung abscess, a complication of pneumonia, particularly in aspiration pneumonia. This puts the client in grave danger because abscesses are often caused by anaerobic organisms. This client most likely would need a change of antibiotics. Sharp chest pain on inspiration called pleuritic pain is an expected finding with this type of pneumonia. The other options are expected in the initial 24 to 48 hours of therapy for any type of infection.

The nurse in the primary care office is reviewing after-visit instructions with a client who was recently diagnosed with gastroesophageal reflux disease (GERD). Which action should the client implement to decrease the symptoms associated with GERD? A Eliminate dairy products from the diet. B Increase oral fluid intake to 4 liters a day. C Avoid caffeinated and carbonated beverages. Correct Answer (Blank) D Limit foods high in fiber.

Rationale: GERD means the chronic backward flow (reflux) of stomach contents into the esophagus. This reflux produces symptoms (i.e., heartburn) by exposing the esophageal mucosa to the irritating effects of acidic gastric or duodenal contents, resulting in inflammation. The most common cause of GERD is excessive relaxation of the lower esophageal sphincter (LES). Because caffeinated beverages, such as coffee, tea and cola, and the carbonation in carbonated beverages will further lower the LES pressure, increasing the risk for gastric reflux and esophageal irritation, those drinks should be avoided. The other actions will not help with the symptoms of GERD.

The nurse in a public health clinic is caring for a female client diagnosed with genital herpes who reports dysuria, dyspareunia, leukorrhea, and lesions of the labia and perianal skin. Which interventions should the nurse include in the discharge instructions? Select all that apply. Soak in a tub of hot water Apply a wrapped ice pack to the sores Correct Answer (Blank) Use lubrication during sexual intercourse Increase fluids to dilute urine Correct Answer (Blank) Apply petroleum jelly to blistered areas Correct Answer (Blank) Avoid any tight clothing Correct Answer (Blank)

Rationale: Genital herpes is a sexually transmitted infection caused by the herpes simplex virus. Genital herpes can cause pain, itching, and sores/lesions/blisters to the genital and anal area. Other symptoms include painful urination, pain during sexual intercourse, and a whitish or yellowish discharge of mucus from the vagina. Interventions should include keeping the area clean to prevent blisters/ulcers from becoming infected. To reduce pain, the client should wrap an ice pack and apply to the sores, apply petroleum jelly to any blisters/sores to reduce pain during urination, drink sufficient amounts of fluids to help make passing urine less painful, and avoid tight clothing to reduce irritation to the blisters/sores. Clients with active lesions should not soak in hot water or have sexual intercourse until they are finished with their treatment and the lesions (blisters/ulcers) have disappeared.

The nurse is preparing to give a dose of prescribed gentamycin IV to a client who has been admitted with meningitis. Which lab results should be reported to the doctor before beginning the medication? A Creatinine level of 2.5 mg/dL Correct Answer (Blank) B Hematocrit of 45% C White blood cell count of 20,000 mm3 D Erythrocyte count of 4.7 million/mm3

Rationale: Gentamycin is used to treat meningitis but is nephrotoxic. The labs for kidney function need to be monitored along with the WBCs. Creatinine is high and would indicate some concerns related to the gentamycin.

The nurse is caring for a child diagnosed with seizures. While teaching the family and the child about the medication phenytoin, which concept should the nurse emphasize? A Maintain good oral hygiene and dental care Correct Answer (Blank) B A rash is normal with this medication C Omit the medication if the child is seizure-free D Serve a diet that is high in iron

Rationale: Gingival hyperplasia may occur with this medication. It is important that good oral hygiene is maintained. The medication should never be stopped, even if the child is seizure-free. A sudden discontinuation could result in status epilepticus. A diet high in iron interferes with phenytoin absorption and will reduce the effectiveness. A blister-like rash is not normal with this medication and could indicate medication-related Stevens-Johnson syndrome, which is a serious disorder of the skin and mucous membranes.

The nurse admits a 50-year-old client with a three-day history of swelling of the face, hands and feet; foamy brown urine; fever and malaise. Which information obtained in the admission interview alerts the nurse that these findings may reflect a diagnosis of acute glomerulonephritis? A Type 1 diabetes since age 15 B Travel to a foreign country C Sore throat two weeks ago Correct Answer (Blank) D History of mild hypertension

Rationale: Glomerulonephritis commonly presents with proteinuria (foamy urine) that is rusty or brownish in appearance and swelling due to the systemic protein loss. In the majority of cases of acute glomerulonephritis, there is a history of an untreated streptococcal throat infection preceding the onset of symptoms by two to three weeks. The other options are not directly related to the development of acute glomerulonephritis.

The nurse is caring for a client who is experiencing an acute gout attack. Which action should the nurse implement? A Administer indomethacin. Correct Answer (Blank) B Restrict sodium intake. C Provide a high-protein diet. D Monitor liver enzymes.

Rationale: Gout is a disease where uric acid crystals form and accumulate in joints and other tissues. During an acute gout attack, the client experiences pain and inflammation in the joints. The nurse should administer a non-steroidal anti-inflammatory medication such as indomethacin to help decrease pain and inflammation. Restricting sodium would not benefit the client and providing a high-protein diet may make the situation worse. There is no need to monitor liver enzymes with an acute gout attack.

The office nurse is teaching a client with gout how to manage the disease. Which actions should the nurse recommend to the client? Select all that apply. Limit the intake of shellfish and red meats. Correct Answer (Blank) Take the prescribed prednisone regularly. Implement stress reduction techniques. Correct Answer (Blank) Make sure to drink at least 2,000 mL of water daily. Correct Answer (Blank) Limit the consumption of alcohol. Correct Answer (Blank)

Rationale: Gout is a disease where uric acid crystals form and accumulate in joints and other tissues. Gout attacks may be brought on by excessive alcohol intake, increased stress and a diet high in purine. Clients should be encouraged to have a low-purine diet by limiting red meats and shellfish, along with drinking alcohol in moderation. The client should be encouraged to drink at least 2,000 mL of water daily to maintain hydration and prevent the buildup of uric acid. Stress management can decrease the likelihood of triggering an acute attack. Prednisone is used during an acute attack, but it does not prevent an attack from occurring.

The nurse is assessing a client who was admitted with suspected Guillain-Barré syndrome. Which assessment findings should the nurse expect? Select all that apply. Weakness Correct Answer (Blank) Seizures Diarrhea Paresthesia Correct Answer (Blank) Hyporeflexia Correct Answer (Blank) Hypotonia Correct Answer (Blank)

Rationale: Guillain-Barré syndrome (GBS) is an autoimmune process that occurs after a viral or bacterial infection, causing acute inflammatory demyelinating polyneuropathy. Transmission of nerve impulses is stopped or slowed. This leads to flaccid paralysis with muscle denervation and atrophy. The main features of GBS include acute, ascending, rapidly progressive, symmetric weakness of the limbs. The first symptoms are weakness, paresthesia (numbness and tingling), and hypotonia (reduced muscle tone) of the limbs. Reflexes in the affected limbs are weak or absent. Diarrhea and seizures are not typically associated with GBS.

The nurse is caring for a client admitted to the acute care setting with a diagnosis of Guillain-Barré syndrome. While reviewing the client's chart, which of the following orders would the nurse question? A Physical therapy and occupational therapy consults B Schedule surgery for a tracheostomy C Administer pyridostigmine Correct Answer (Blank) D Obtain vital signs prior to plasmapheresis

Rationale: Guillain-Barré syndrome is an autoimmune condition where the immune system attacks the peripheral nervous system and cranial nerves. More specifically, the immune system attacks the myelin sheath of the nerves. As the myelin sheath starts to break down, nerve transmission slows down. Manifestations of this syndrome include paraesthesias, paralysis, loss of reflexes, and loss of muscle tone. The syndrome is temporary, and most clients typically make a full recovery. During the acute phase of the condition, the client may be totally paralyzed and may need to be placed on a mechanical ventilator. Pyridostigmine is a cholinesterase inhibitor medication that is used to treat myasthenia gravis, not Guillain-Barre syndrome. Once recovery begins, physical and occupational therapy is ordered. Plasmapheresis is a blood purification procedure used to treat autoimmune conditions; it reduces the severity and duration of the Guillain-Barre episode. The procedure can cause hypotension and arrhythmias.

The nurse is receiving a report from a skilled nursing facility (SNF) for a client being admitted to an inpatient unit. The client has a worsening wound infection that requires treatment. Which statement from the SNF nurse will require further follow-up by the nurse? A "The client is able to ambulate 50 feet independently." B "The last oral temperature was 37.5°C (99.6°F)." C "The latest white blood cell count (WBC) was 7,500/mm³." D "There is a stage 3 pressure ulcer to the coccyx." Correct Answer (Blank)

Rationale: Handoff report should follow the situation-background-assessment-recommendation (SBAR) strategy. A detailed report should include a focused physical assessment and a current treatment plan. The nurse reporting a stage 3 ulcer should include accompanying symptoms, drainage, and treatment. The statement regarding the client's independent ambulation does not require further details. The latest temperature and laboratory values are expected findings for someone with an infection.

The home health nurse is developing a teaching plan for a client with Class III left-sided heart failure. Which intervention is the priority? A Rest in an armchair instead of lying in bed B Record and monitor weights daily Correct Answer (Blank) C Engage in moderate exercise 2 to 3 times a week D Limit the intake of foods high in sodium

Rationale: Heart failure (HF) is a condition that is characterized by fluid volume excess or overload. The best way for the client to monitor their fluid balance is by weighing themselves daily. An increase in their weight above a couple of pounds over 1 to 2 days can indicate worsening of their HF and the client should be instructed to notify their health care provider right away.

The nurse is educating a client with heart failure on therapeutic nutrition. What should the nurse include in the teaching? A "Avoid adding extra condiments to your food." Correct Answer (Blank) B "Drink plenty of fluids to stay hydrated." C "Decrease the percentage of protein in your diet." D "Eat large, high-calorie meals to increase your energy."

Rationale: Heart failure is the inability of the heart to effectively pump blood throughout the body. Inadequate blood flow leads to fluid retention and edema. Condiments contain large amounts of sodium. Sodium attracts fluid, increasing retention and edema throughout the body. Clients should be advised to monitor their fluid intake and possibly restrict it to 2 liters/day. Excess fluid will cause the workload of the heart to increase. Protein is necessary for muscle building. Clients with heart failure should increase their protein intake to 1.12 g/kg. Large, high-calorie meals can lead to obesity, which increases the risk of complications from heart disease. Clients should be encouraged to eat smaller, low-calorie foods.

The nurse is teaching a client with liver cirrhosis about the management of hepatic encephalopathy. Which statement by the client indicates that additional teaching is needed? A "I will eat enough protein and calories to stay healthy." B "I will stop taking ibuprofen for my knee and back pain." C "I will stop taking my lactulose when I have more than one loose stool." Correct Answer (Blank) D "I will brush my teeth with a soft toothbrush to avoid bleeding gums."

Rationale: Hepatic encephalopathy is a life-threatening complication of liver failure. The functions of the liver include generating proteins for clotting, preventing bleeding, metabolizing waste products such as ammonia and producing albumin to maintain oncotic pressure. Lactulose is a common medication prescribed to a client with hepatic encephalopathy. Lactulose will bind to ammonia and is excreted from the body by stool. Without this medication, the ammonia level will build because the damaged liver is unable to metabolize it, increasing the severity of the disease. While taking lactulose the client may have loose bowel movements, but this should not stop them from taking the medication. The client needs to ensure they are eating enough protein to meet their body's energy demands. With a higher risk of bleeding, clients should use a soft toothbrush and avoid any type of NSAID medications.

The nurse is reviewing the laboratory data for a client with hepatitis who is taking prescribed lactulose. Which finding indicates the treatment is effective? A Ammonia 18 mcg/dL Correct Answer (Blank) B AST 12 U/L C Bilirubin 2 mg/dL D ALT 10 U/L

Rationale: Hepatitis causes a build-up of ammonia in the circulation. Lactulose decreases the pH in the colon which causes ammonia to leave the circulatory system and move into the colon to be expelled. This causes ammonia blood levels to return to normal.

A client who was admitted with viral hepatitis is being discharged home. Which of the following statements by the client indicate an understanding of the discharge instructions? Select all that apply. "I will call my doctor if my skin turns yellow." Correct Answer (Blank) "I will avoid drinking alcoholic beverages." Correct Answer (Blank) "I will take acetaminophen for pain." "I will call my doctor if my belly gets bigger." Correct Answer (Blank) "I will avoid too much salt in my diet." Correct Answer (Blank)

Rationale: Hepatitis is an inflammation in the liver that leads to liver cell damage. Clients with hepatitis should avoid drinking alcohol and taking drugs metabolized in the liver, such as acetaminophen. A yellow discoloration of the sclera of the eyes and the skin on the rest of the body is called jaundice. Jaundice is an indication of worsening hepatic function. Clients with impaired hepatic function should also avoid high levels of salt in their diets to prevent fluid retention. Abdominal distention (i.e., the client's belly getting bigger) could be ascites, a condition which occurs as a result of low serum albumin levels that can lead to fluid accumulation in the abdominal cavity. Clients should report these findings to their health care provider.

The nurse is caring for a client receiving mechanical ventilation when the device signals a high-pressure alarm. The nurse should include what assessments in addressing this alarm? Select all that apply. Assess client for partial or total extubation Assess for obstructing secretions Correct Answer (Blank) Assess tubing to ensure it is not kinked Correct Answer (Blank) Assess client for signs of bronchospasm Correct Answer (Blank)

Rationale: High pressure alarms are usually caused by something preventing or blocking air from being delivered by the ventilator to the lungs. Common causes for this include kinked tubing, secretions and/or bronchospasms, or the client fighting the tube. Low pressure alarms are usually caused by air escaping the closed unit. A total or partial extubation would cause a low-pressure alarm.

The nurse is caring for an older adult client with heatstroke. Upon admission, the client appears confused and has a temperature of 104°F. Which is the priority action for the nurse to take first? A Apply cooling blanket Correct Answer (Blank) B Initiate seizure precautions C Position a fan towards the client D Administer prescribed IV fluids

Rationale: Homeostatic regulatory mechanisms fail to function during an episode of heatstroke and result in a critical elevation in body temperature usually greater than 104°F. If the emergent condition is not treated quickly, or there is a lack of a positive response to acute treatment, multiorgan system failure and death can potentially result. Priority actions include cooling down the client with cooling blankets, ice, and providing adequate hydration with cooled intravenous saline. Once the cooling blanket is applied, the nurse will then position a fan towards the client to decrease any dissipating heat.

The nurse is transporting a client with an indwelling urinary catheter to the radiology department. What intervention should be implemented to reduce the risk of catheter-associated urinary tract infection (CAUTI)? A Empty the drainage bag and keep it below the level of the bladder during transport. Correct Answer (Blank) B Break the red seal and disconnect the drainage bag for transport. C Remove the catheter and reinsert when the client returns to the room. D Remove the catheter and reinsert when the client returns to the room.

Rationale: Hospital-acquired infections (HAI) can often be linked to invasive devices, such as a urinary catheter. For example, CAUTIs are the most common type of HAIs. Infection control measures include adherence to recommended best practices or bundles. The CAUTI bundle includes core prevention strategies, such as using aseptic technique and sterile equipment during insertions, maintaining a closed drainage system, and maintaining unobstructed urine flow. Therefore, it is important to keep the tubing and bag below the level of the bladder. Changing indwelling catheters or drainage bags is not recommended unless based on clinical indications such as infection, obstruction, or when the closed system is compromised.

The nurse in a pediatrician's office is assessing an infant with a ventriculoperitoneal shunt to treat hydrocephalus. Which finding might indicate a possible shunt malfunction? A Lethargy Correct Answer (Blank) B Loose stools C Dry mucous membranes D Depressed fontanels

Rationale: Hydrocephalus is an increased accumulation of cerebral spinal fluid in the brain resulting in increased intracranial pressure (ICP) and is commonly treated with insertion of a ventriculoperitoneal (VP) shunt. The major complications of VP shunts are infection and malfunction. All shunts are subject to mechanical difficulties, such as kinking, plugging, or separation and migration of tubing. Malfunction is most often caused by mechanical obstruction either within the ventricles from particulate matter (tissue or exudate) or at the distal end from thrombosis. A non- or malfunctioning VP shunt will lead to an increased ICP. Changes in the level of consciousnessm such as lethargy, are an early sign. The other findings are not typically associated with an increase in ICP.

The nurse is reviewing the serum laboratory findings for a client who has hypertension and is prescribed hydrochlorothiazide. Which finding should the nurse report to the health care provider? A Sodium 134 mEq/L B Potassium 2.3 mEq/L Correct Answer (Blank) C Phosphorus 4.3 mg/dL D Calcium 8.6 mg/dL

Rationale: Hydrochlorothiazide is a diuretic that can cause an increased loss of potassium from the kidneys, leading to hypokalemia. Normal potassium levels are 3.5 to 5.0 mEq/L. The nurse should report a potassium level of 2.3 to the provider. The other lab values are within normal range or just barely out of range that would not require notifying the provider.

The nurse is caring for a client with chronic obstructive pulmonary disease who is receiving prescribed oxygen. The client reports an increase in drowsiness, and the nurse notes that the client is alert and oriented to name and time. Which intervention should the nurse implement? A Obtain a prescription for an arterial blood gas (ABG) Correct Answer (Blank) B Obtain a prescription for a chest X-ray C Encourage incentive spirometry D Increase the oxygen flow rate to achieve an increased oxygen saturation

Rationale: Hypercapnia is a common complication of COPD. Signs of hypercapnia include confusion and drowsiness. Increasing the oxygen flow rate can worsen the retention of carbon dioxide. The partial pressures of both oxygen and carbon dioxide are measured through ABGs. A chest radiograph can assist with the diagnosis of COPD and pneumonia but does not provide information on gas exchange. Incentive spirometry can worsen CO2 retention in clients with COPD.

The nurse is admitting a client with a history of end-stage renal disease who was found unresponsive at home. The client has a history of chronic hemodialysis. The EKG is displaying bradycardia and peaked t-waves. Which of the following immediate interventions should be anticipated? A 50% dextrose and regular insulin administered intravenously Correct Answer (Blank) B Insertion of central venous line C Oral administration of kayexalate D Initiation of a hemodialysis treatment

Rationale: Hyperkalemia is the most life-threatening of the fluid and electrolyte changes that occur in patients with kidney disorders. Therefore, the patient is monitored for potassium values greater than 5.0 mEq/L (5 mmol/L), ECG changes (tall, tented, or peaked T waves), and changes in clinical status. If the patient is hemodynamically unstable (low blood pressure, changes in mental status, or dysrhythmia), IV dextrose 50%, insulin, and calcium replacement may be given to shift potassium back into the cells. The shift of potassium into the intracellular space is temporary, so arrangements for dialysis will then need to be made.

The nurse is planning care for a client newly diagnosed with essential hypertension. Which interventions should the nurse include in the client's plan of care? Select all that apply. Evaluate the client's understanding of a low-sodium diet. Correct Answer (Blank) Evaluate the client's ability to take their own blood pressure. Correct Answer (Blank) Encourage the client to limit smoking to one pack of cigarettes per day. Instruct the client to abstain from drinking any alcohol. Encourage the client to take daily, 30-minute walks. Correct Answer (Blank) Explain the negative effects of hypertension on the body. Correct Answer (Blank)

Rationale: Hypertension can occur when the resistance to blood pumping through the arteries increases. Risk factors for the development of hypertension include older age, smoking, salt intake, family history and ethnicity. The client should be advised on the negative effects of hypertension on the body, such as kidney failure and cardiovascular disease. The client should know how to correctly check their blood pressure at home. Clients with hypertension should not only limit smoking, but should not smoke at all due to the vasoconstrictive effects of nicotine. Clients should understand what foods are appropriate for a reduced- or low-sodium diet and how engaging in regular physical activity can help manage their disease. Complete abstinence from alcohol is not required.

A nurse is assessing a client who has a calcium level of 8.0 mg/dL. Which finding should the nurse expect to observe? A Depressed reflexes B Polyuria C Muscle tremors Correct Answer (Blank) D Nausea

Rationale: Hypocalcemia, Ca <9.0 mg/dL, decreases the threshold for activating the sodium channels, which is required for muscle contraction. This causes nerve excitability and sustained muscle contraction resulting in muscle tremors. A client with hypercalcemia will experience depressed reflexes, nausea, and polyuria.

The nurse is assessing a client who has a potassium level of 3.2 mEq/L. Which finding should the nurse expect to observe? A Shallow respirations Correct Answer (Blank) B Hypoglycemia C Tetany D Abdominal pain

Rationale: Hypokalemia, or a decreased level of potassium, occurs with gastrointestinal or renal losses. For clients at risk for developing hypokalemia, the nurse should monitor for signs such as shallow respirations, constipation, hyperglycemia, and irregular pulse. Assessment findings associated with hyperkalemia include irritability, muscle weakness, paresthesia, abdominal pain, diarrhea, vomiting, and confusion.

The nurse is caring for a client who has received intravenous magnesium sulfate for a serum magnesium level of 1.2 mEq/L. Which of the following client findings indicates that the treatment has been effective? A Prolonged QT intervals on the client's electrocardiogram B Hyperactive deep tendon reflexes C Reduction in the client's abdominal distention Correct Answer (Blank) D Numbness and tingling in the toes

Rationale: Hypomagnesemia can cause cardiovascular, neuromuscular, and intestinal changes in clients. A reduction in the client's abdominal distention indicates improvement in the client's condition since hypomagnesemia can cause reduced motility, constipation, and possibly paralytic ileus. Numbness and tingling in the extremities, hyperactive deep tendon reflexes, and prolonged QT intervals are all symptoms of hypomagnesemia, which indicates that the treatment has not been effective.

The nurse is caring for a client admitted with a phosphorus level of 1.5 mg/dL. Which statement by the client should alert the nurse to collect further data about possible causes for the phosphate imbalance? Select all that apply. "I take a calcium supplement with every meal." Correct Answer (Blank) "I have a history of decreased kidney function." "I had my parathyroid gland surgically removed." "I snack on nuts in between meals." "I do not eat any meat or dairy products." Correct Answer (Blank)

Rationale: Hypophosphatemia refers to a below-normal concentration of phosphorus in the ECF (serum phosphate less than 2.5 mg/dL). The most common causes of hypophosphatemia are depletion of phosphorus because of insufficient intestinal absorption, hypercalcemia, transcompartmental shifts, and increased renal losses. Lack of parathyroid hormone (PTH) after removal of the parathyroid gland leads to decreased blood levels of calcium (hypocalcemia) and increased levels of blood phosphorus (hyperphosphatemia), not hypophosphatemia. Nuts are high in organic phosphorus and are considered good snack foods. Meats and dairy products are high in phosphate, and a lack of intake of those foods can lead to a low serum phosphorus level.

The client is admitted with the diagnosis of chronic obstructive pulmonary disease (COPD). Which findings would require the nurse's immediate attention? A Nausea and vomiting B Frequent productive cough with brownish sputum C Restlessness and confusion Correct Answer (Blank) D Low-grade fever and cough

Rationale: Hypoxia and respiratory failure in COPD may be signaled by excessive somnolence, restless, aggressiveness, confusion, central cyanosis and shortness of breath. When these findings occur, the oxygen saturation and arterial blood gases (ABGs) should be assessed and oxygen should be rapidly titrated upward to correct the hypoxia. Signs of respiratory distress or failure may necessitate the use of ventilatory assistance BIPAP or emergent intubation and mechanical ventilation. Cough, discolored sputum, and fever may indicate a respiratory infection such as pneumonia, but this is a less urgent situation.

The nurse is caring for a client receiving a continuous infusion of isotonic fluid. When assessing the site, the nurse notes that the skin is cool and pale and there is edema. The IV therapy is discontinued. Which action would be appropriate for the nurse to make? A Elevate the extremity Correct Answer (Blank) B Massage the extremity C Prepare to administer phentolamine D Prepare to administer hyaluronidase

Rationale: Iatrogenic complications of IVT include infection, phlebitis, infiltration, extravasation, soft-tissue necrosis, and compartment syndrome. Infiltration refers to leakage of nonvesicant (nonirritating) solution into the extravascular space while extravasation involves a vesicant (irritating) solution. Maneuvers to reduce edema, such as limb elevation, should be instituted, but massage is not recommended. Phentolamine is the antidote to vasoactive medications. Hyaluronidase is the antidote to many other hypertonic extravasations.

The nurse in the primary health care provider's office is reviewing the medical record of a client with idiopathic pulmonary arterial hypertension. The nurse should expect which potential clinical manifestations with this disease? Select all that apply. Abnormal heart sounds Correct Answer (Blank) Elevated serum creatinine level Cor pulmonale Correct Answer (Blank) Dyspnea on exertion Correct Answer (Blank) Exertional chest pain Correct Answer (Blank)

Rationale: Idiopathic pulmonary arterial hypertension (IPAH) has no apparent cause and is characterized by an elevated pressure in the pulmonary arterial circulation. The disease is incurable but drug therapy will greatly help. Classic symptoms include: exertional dyspnea and chest pain, fatigue, right-sided heart failure (cor pulmonale) due to the increased workload of the right ventricle and abnormal heart sounds, such as an S3. An elevated creatinine level would indicate renal dysfunction, not pulmonary hypertension.

The nurse is caring for a client who is in the post-anesthesia care unit 30 minutes after a major surgical procedure. The client states that they are experiencing nausea. Which of the following actions by the nurse is appropriate to minimize additional gastrointestinal upset? A Provide the client with a regular diet B Administer an antiemetic medication Correct Answer (Blank) C Tell the client to walk to the bathroom D Increase intravenous fluid intake

Rationale: If a client complains of nausea, they should be given an antiemetic medication to avoid worsening of nausea or vomiting. The client should not be given a regular diet until they have been able to tolerate clear liquids. While a client might be physically able to walk to the bathroom 30 minutes after a surgical procedure, this is most likely a safety risk. Increasing intravenous fluids does not minimize gastrointestinal discomfort.

The nurse is caring for a client with a pressure ulcer on their heel that is covered with hard, dry black tissue. Which action should the nurse take? A Apply a wet-to-dry dressing B Remove the eschar C Leave the pressure ulcer as is Correct Answer (Blank) D Apply a hydrocolloid dressing

Rationale: If the black tissue (eschar) is dry and intact, no treatment is necessary; the stable eschar serves as a natural cover. Mechanically removing (i.e., debriding) the eschar is typically performed by the health care provider, not the nurse. The other actions are not appropriate or indicated for this stage of pressure ulcer.

The home health nurse is caring for a 6-year-old client with cerebral palsy. The client's parent reports to the nurse that the child's older sibling was just diagnosed with impetigo. What priority intervention should the nurse add to the client's plan of care? A Contact the client's pediatrician for a prescription for an oral antibiotic as a preventative measure B Start applying a topical anti-inflammatory cream to the client's skin to prevent the client from becoming infected C Instruct the client's parent to keep the client isolated in their room until the sibling's infection has resolved D Instruct the parent to provide the infected child with washcloths and towels separate from the client's Correct Answer (Blank)

Rationale: Impetigo is a common and highly contagious, bacterial skin infection that mainly affects infants and children. Impetigo usually appears as red sores on the face, especially around a child's nose and mouth and on hands and feet. The sores burst and develop honey-colored crusts. The priority nursing functions related to bacterial skin infections are to prevent the spread of infection and to prevent complications. Impetigo can easily spread, and hand washing is mandatory before and after contact with an affected child. The infected child should be provided with washcloths and towels separate from those of other family members, and the infected child's clothes should be changed daily and washed in hot water. A topical bactericidal ointment can be used if the client becomes infected too. An anti-inflammatory cream, such as hydrocortisone, will not prevent the client from becoming infected. Antibiotics should not be prescribed for prevention in this situation. Oral or parenteral antibiotics (penicillin) are reserved for severe cases of an actual infection. Keeping the client isolated is not necessary or appropriate in a home setting.

The nurse is caring for a client who has hearing loss. Which actions should the nurse implement to facilitate communication with this client? Select all that apply. Speak at a slower rate than usual. Speak to the client at eye level. Correct Answer (Blank) Use short sentences. Correct Answer (Blank) Know basic sign language techniques. Speak in a loud voice toward the client's ear.

Rationale: Important actions to improve communication between the nurse and a client with hearing loss is for the nurse to be sure they speak to the client at eye level and use short sentences. It is not necessary to speak at a slower rate or speak loudly. Not all clients with hearing loss understand sign language and nurses are not required to learn it.

The nurse receives a client who was transported to the emergency department for severe hypertension. Which finding requires immediate action by the nurse? A Jugular vein distension B Cough with frothy, pink sputum C Crackles in the lung bases D Weakness in the left arm Correct Answer (Blank)

Rationale: In a client who has uncontrolled hypertension, weakness in the extremities is a sign of cerebral involvement. Cerebral infarctions account for approximately 80% of the strokes in clients with hypertension. The remaining choices indicate fluid overload, which may be associated with heart failure related to the uncontrolled hypertension. While concerning, these are not medical emergencies. Jugular vein distension (JVD) is due to the elevated central venous pressure (CVP). Crackles in the bases of the lungs and a cough with frothy, pink sputum indicate pulmonary congestion. Crackles in all lung fields accompanied by dyspnea and orthopnea would indicate acute pulmonary edema, which would also be considered a medical emergency.

A nurse arrives at a child daycare center that was the site of an explosion. Which child should be tagged "green," or needing minimal treatment? A An infant with bulging of the anterior fontanel while crying Correct Answer (Blank) B A preschooler with a lower leg fracture on one side and an upper leg fracture on the other C A toddler with severe and deep abrasions over 98% of the body D A school-age child with singed eyebrows and hair

Rationale: In a disaster or mass casualty situation, the color-coded triage system is used to identify victims based on the severity of injuries. The "black" category is for victims who are already deceased or have such extensive injuries that they would not be expected to survive. The "red" category is for clients needing immediate treatment for survival, such as those with chest injuries or open fractures. The "yellow" category means the client does have a chance of survival with medical intervention. The "green" category means that the victim needs minimal treatment and will be expected to survive the injuries. A bulging fontanel with crying is to be expected in an infant; therefore, this client should receive a green tag.

The off-duty nurse is helping to administer first aid following a mass casualty incident in the community. Emergency medical personnel at the scene have started to triage victims, using a common, color-tagging system. Which tag color usually indicates the highest priority for a victim to receive care? A Green B Red Correct Answer (Blank) C Yellow D Black

Rationale: In a mass casualty incident (MCI), first responders often use a color-tagging system to facilitate rapid triage of victims. Generally speaking, a green tag would indicate minor injuries, a yellow tag would indicate more significant but not expected to be life-threatening injuries, a red tag would indicate life-threatening injuries, and a black tag would identify a victim who has died, is near death or has the lowest chance for survival. Victims assigned a red tag are the highest priority for care and transport to the nearest hospital.

A client, admitted with palpitations and dyspnea, is diagnosed with atrial fibrillation (AF). Normal sinus rhythm is later restored using pharmacologic interventions. In addition to controlling cardiac rate and rhythm, the nurse understands that treatment for AF should include which additional intervention? A Cardioversion B Coronary artery bypass graft surgery C Catheter ablation D Anticoagulation Correct Answer (Blank)

Rationale: In addition to rate and rhythm control, acute management of AF includes anticoagulation. Effective anticoagulation in clients with AF significantly reduces the risk of stroke and other thromboembolic events. When a client does not respond to pharmacologic interventions to restore sinus rhythm, cardioversion is used. Catheter ablation is used to destroy the triggers in the atrium for AF, but is not the first line of treatment. Bypass surgery is not used to treat AF.

The nurse is caring for a client who has recently undergone an above the knee amputation. Which of the following questions by the nurse is appropriate to assess the client's adaptation to this physical change? A "Are you currently having any pain in your leg?" B "Have you been taking your prescribed medications?" C "What modifications have you made to your daily activities?" Correct Answer (Blank) D "Do you plan to go to outpatient physical therapy?"

Rationale: In order to assess the client's adaptation, the nurse should ask questions about feelings, behavioral changes, or the effect that the condition will have on the client's life. In this case, asking about modifications to daily activities allows the nurse to assess how they are adapting to this drastic physical change. Asking if the client has any pain, taking prescribed medications, and participating in physical therapy assesses the client's management of the amputation but does not assess adaption.

The nurse is caring for a client who has recently been admitted to a skilled nursing facility. Which question by the nurse is appropriate to assess how the client is adapting to their new environment? A "Has your family come to visit you?" B "Do you think you'll be discharged soon?" C "How do you feel about staying here?" Correct Answer (Blank) D "Are the nursing staff members treating you well?"

Rationale: In order to assess the client's adaptation, the nurse should ask questions about feelings, behavioral changes, or the effect that the illness will have on the client's life. In this case, asking how the client feels about their admission to the skilled nursing facility allows the nurse to assess how they are adapting to the new environment. Asking the client how the staff is treating the client, when the client will be discharged, and if family has visited assesses the environment, not how the client has adapted to the change.

The nurse is reviewing the medical record of a client with diabetes who was admitted for a surgical site infection. Which findings should the nurse report to the health care provider? Select all that apply. Complete metabolic panel: Glucose, serum: 220 mg/dL BUN: 16 mg/dL Creatinine: 1.06 mg/dL Potassium, serum 4.4g/dL AST (SGOT) 31 IU/L ALT (SGPT) 30 IU/L Hemoglobin A1C 8% Hemoglobin A1C of 8% Correct Answer (Blank) ALT 30 IU/L Urine pH 7.3 Serum glucose level of 220 mg/dL Correct Answer (Blank) Negative protein in the urine Positive glucose in the urine Correct Answer (Blank) BUN of 16 mg/dL

Rationale: In reviewing the lab values, the nurse should notify the HCP of the positive glucose in urine (normally, glucose is not seen in urine), A1C of 8% (desired range for a client with diabetes is 7% or less), and the serum glucose level of 220 mg/dL, which is higher than the normal range of 70 to 110 mg/dL. These abnormal lab results indicate that the client's diabetes is not managed well and most likely contributed to the client developing an infection. The BUN, ALT and urine pH listed are considered within normal limits.

The home health nurse is reviewing the plan of care for a client with advanced heart failure who reports activity intolerance. Which nursing intervention should the nurse add to the care plan? A Encouragement of adequate fluid intake B Use of incentive spirometry C Utilization of compression stockings D Use of oxygen as needed Correct Answer (Blank)

Rationale: In the advanced stages of heart failure (HF), cardiac output and tissue perfusion, i.e., oxygenation, will be decreased. Client reports of exertional dyspnea with even the slightest activity are common. Therefore, addressing the low oxygenation by using supplemental oxygen via a nasal cannula will help with the activity intolerance and should be added to the client's plan of care. The other interventions do not address the problem of activity intolerance caused by low tissue perfusion and are actually contraindicated with HF (e.g., encouraging fluid intake).

A 67-year-old client is admitted to the telemetry unit with substernal chest pressure that radiates to the jaw. The client's diagnosis is an acute myocardial infarction. To monitor the client, the nurse should give priority to which assessment? A Assess the client's activity tolerance. B Assess the client's level of anxiety. C Assess the client's pain level. D Assess the client's cardiac output. Correct Answer (Blank)

Rationale: In the immediate post- myocardial infarction (MI) period, altered cardiac output is a potential problem. An area of myocardial tissue has been damaged by a lack of blood flow and oxygenation, increasing the risk for decreased cardiac output, dysrhythmias and heart failure. Findings would include low blood pressure, tachycardia, low urine output, unrelieved or worsening chest pain, and shortness of breath. Nursing assessments and interventions should be directed toward promoting myocardial tissue perfusion and oxygenation. The other assessments are also relevant, but monitoring cardiac output is the priority.

A nurse is performing chest compressions on an adult client in cardiac arrest. Another nurse enters the room in response to the call for help. What should happen next? A The second nurse will maintain an open airway and perform ventilations while the first nurse continues chest compressions. Correct Answer (Blank) B The second nurse will leave to get personal protective devices (PPD) and oxygen. C The first nurse will now perform 15 chest compressions and then wait while the second nurse performs 2 ventilations. D The second nurse will take 10 seconds to check the pulse and "look, listen and feel" for breathing.

Rationale: In two-rescuer CPR, the first rescuer performs chest compressions and the second rescuer performs (bag-mask) ventilation. Rescuers should switch roles after five cycles (about every two minutes). The compressing rescuer should give (30) continuous chest compressions at a rate of at least 100 per minute without pauses for ventilation; the rescuer will provide a breath every six to eight seconds. Rescuers should all have quick access to personal protective devices (PPD) prior to any intervention.

The nurse has received reports on a group of assigned clients with neurological disorders. Which client should the nurse see first? A The client who had a lumbar laminectomy, equal strength in bilateral lower extremities, lower back pain 4/10, and denies numbness/tingling B The client who had an ischemic stroke, received thrombolytic therapy in the ER, a BP of 160/90, and a stable neurological assessment C The client who reports uncontrolled headaches, was diagnosed with glioblastoma, and is vomiting for the second time in 3 hours Correct Answer (Blank) D The client who has kyphoplasty of a spinal compression fracture, back pain 3/10, and persistent bilateral tingling and weakness in the lower extremities

Rationale: Increased intracranial pressure (ICP) occurs in patients with acute neurological conditions including head injury, brain tumor, hemorrhage, and anoxic injury. Signs and symptoms of increased ICP include changes in mental status, restlessness, pupillary changes, unilateral weakness, headaches, and new-onset vomiting. The lumbar laminectomy patient has expected post-op assessment findings. Maintaining adequate cerebral blood flow is essential for cerebral oxygenation. The target BP post-ischemic stroke with thrombolytic treatment is <185/110 for the first 24 hours after admission. The post-op patient reporting paresthesia and bilateral weakness requires an assessment and potential intervention as this is not an expected finding. However, the nurse's priority is still the patient with increased ICP because there is an urgent need to intervene to prevent morbidity and/or mortality.

A client is admitted to the hospital with endocarditis. The nurse understands that which risk factors can lead to the development of endocarditis? Select all that apply. Treatment of substance use disorder with methadone Placement of a central venous access device Correct Answer (Blank) Atrial fibrillation with use of warfarin Oral abscess with tooth extraction Correct Answer (Blank) Placement of an arteriovenous fistula for hemodialysis Correct Answer (Blank) History of aortic valve replacement Correct Answer (Blank)

Rationale: Infective endocarditis (IE) is an infection of the inner layer of the heart wall and muscle. The infection is commonly caused by bacteria such as Staphylococcus aureus. The endocardium can become infected when bacteria are carried through the heart by blood flow. IE can occur in individuals with existing cardiac disease or with no cardiac disease. Clients who have poor oral hygiene and a history of dental procedures (i.e., tooth extraction) are at risk for developing IE. Invasive devices (i.e., indwelling catheters, venous access devices, pacemakers and arteriovenous fistulas) and heart valve replacement place clients at risks for IE. One complication of endocarditis includes arrhythmias, but arrhythmias (i.e., atrial fibrillation) do not place the client at risk for developing IE. Individuals who engage in intravenous drug use are at risk for IE. However, individuals who take methadone by mouth for a substance use disorder are not at risk for IE.

The nurse is assessing a 1-month-old infant. Which finding should the nurse report immediately? A Abdominal respirations B Irregular breathing rate C Inspiratory grunt Correct Answer (Blank) D Increased heart rate with crying

Rationale: Inspiratory grunt is an abnormal finding and indicates respiratory distress in infants. Other signs of respiratory distress in this age group are nasal flaring, often the initial finding, as well as sternal and intracostal retractions. Abdominal breathing is a normal expected breathing process for infants. The other findings are also normal in infants.

The nurse has removed the sutures from a client with an abdominal wound and applied steri-strips. What education should be provided to the client after suture removal? A Instruct patient to take showers rather than bathe. Correct Answer (Blank) B Tell the client to remove the steri-strips in one week. C Inform the client that the skin is fully healed. D Advise the client that clear drainage is expected.

Rationale: Instruct patient to take showers rather than bathe to reduce the risk of infection. Instruct patients not to pull off the Steri-Strips and to allow them to fall off naturally and gradually (usually takes one to three weeks). Wound healing time can be diverse, but the maturation phase begins at day 21 and may take up to a year or more to heal completely. Sutures are usually removed around day 7 to 10 after insertion. Therefore, the skin would not be fully healed. Once a wound is beyond the inflammatory phase, serous drainage can indicate infection and should be reported.

The nurse is providing care to a client post arterial revascularization surgery. Which intervention will the nurse implement to promote circulation in the client? A Provide the client with an ice pack for the extremities. B Instruct the client to limit bending of the hip. Correct Answer (Blank) C Tell the client to elevate swollen legs above the level of the heart. D Initiate a turning schedule while the client is in bed.

Rationale: Instructing the client to limit bending of the hip prevents clot formation after revascularization surgery and promotes adequate blood flow. The client should limit bending at the hip and knee. Providing the client with an ice pack will cause vasoconstriction, the opposite intended effect of the procedure. Telling the client to elevate swollen legs above the level of the heart slows arterial blood flow to the lower extremities. Legs should be elevated no higher than the level of the heart. Initiating a turning schedule maintains skin integrity for clients at risk for pressure ulcers. This activity does not directly promote circulation of the extremities.

The nurse is caring for a client with severe iron deficiency anemia. Which interventions should the nurse include in the client's plan of care? Select all that apply. Monitor the client's stool for color, consistency, and frequency Correct Answer (Blank) Review the client's medical record for NSAID use Correct Answer (Blank) Encourage the client to eat more green leafy vegetables and beans Correct Answer (Blank) Administer the client's prescribed iron supplements with milk Instruct assistive personnel to allow the client to rest during care activities Correct Answer (Blank) Prepare the client for a packed red blood cells transfusion Monitor the client for palpitations and orthostatic hypotension Correct Answer (Blank)

Rationale: Iron deficiency anemia is the most common type of anemia. Lower levels of iron in the body fail to produce red blood cells (RBCs). Without sufficient RBCs, the client's body and tissues will not receive enough oxygen. Risk factors for iron deficiency anemia include pregnancy, gastric bypass, blood loss, gastric ulcer, gastrectomy, and menstruation. Because iron deficiency anemia can be caused by gastrointestinal (GI) bleeding, it is recommended to check the color of the client's stool. Dark, tarry stools could indicate upper GI bleeding. Manifestations of anemia include palpitations, tachycardia, hypotension, pallor, and shortness of breath. Clients should take iron on an empty stomach for better absorption. Taking iron with milk will block the medication's absorption. Clients should be encouraged to eat foods that contain iron, such as green leafy vegetables, beans, legumes, and muscle meats. NSAID use has the potential to cause gastric ulcer development and GI bleeding. Clients with anemia will be fatigued and tire easily and should be allowed to rest during care activities. The scenario does not indicate that the client needs a blood transfusion at this time.

The nurse is caring for a 10-month-old infant diagnosed with iron-deficiency anemia. Based on this diagnosis, which of these findings should the nurse anticipate? A Pale mucosa of the eyelids and lips Correct Answer (Blank) B Heart rate of 120 bpm C Increased appetite D Hemoglobin level of 12 g/dL

Rationale: Iron-deficiency anemia commonly occurs in infants 9-24 months old. Although infants are born with iron stores available, because they grow very rapidly, they need to absorb iron every day. Breast milk or formula provides enough daily iron for infants. However, in the event that an infant does not absorb enough iron, they may become iron-deficient anemic. Common manifestations of anemia include irritability, fatigue, brittle nails, and cyanosis. An infant with iron-deficiency anemia would suffer from a poor appetite and not an increased appetite. A hemoglobin level of 12 g/dL is considered normal in an infant. The normal hemoglobin range for an infant is 11 to 13 g/dL. A heart rate of 120 bpm is considered normal in an infant. The normal heart rate range for an infant is 120 to 180 bpm. Pale mucosa of the eyelids and lips would be the anticipated finding in this client.

The nurse is providing education to a client diagnosed with iron-deficiency anemia who reports fatigue and dizziness. The nurse explains the cause of the client's symptoms are from which condition? A Reduced oxygen saturation B Destruction of red blood cells (RBCs) C Decreased cardiac output D Tissue hypoxia Correct Answer (Blank)

Rationale: Iron-deficiency anemia is when the RBCs are unusually small and pale (due to their low hemoglobin content.) Tissue hypoxia is the result of not having enough functioning hemoglobin in the blood to oxygenate the tissues; tissue hypoxia is responsible for symptoms such as fatigue, leg cramps and chewing ice. It's possible the client could have a normal SpO2 because the small amount of hemoglobin that's in the blood may be well saturated with oxygen. Anemia does not decrease cardiac output. Destruction of RBCs is associated with hemolytic anemia, not iron-deficiency anemia.

The nurse in an assisted living facility comes upon an 85-year-old client lying on the bathroom floor. The nurse observes a deformity in the left leg and the client is unable to move the leg. The client is alert and oriented but in severe pain. Which action should the nurse take first? A Administer pain medication. B Elevate the extremity above heart level. C Apply an ice pack to the site. D Immobilize the fracture with a splint. Correct Answer (Blank)

Rationale: It appears that the client suffered a bone fracture in the left leg. After confirming that the client's respiratory and neurologic status is stable, the nurse should immobilize the fracture with a splinting device. This will prevent movement of the extremity by the client and further pain or bleeding along the fracture into the surrounding tissues. Next, the nurse should notify the health care provider or call emergency medical services to transport the client to the nearest emergency room.

The nurse is teaching a client about genital herpes. Which statement by the client indicates that the teaching was effective? A "My infection must be reported to the health department." B "My infection will be cured after I take all antibiotics." C "I can only get the disease from someone with visible lesions." D "I need to inform my sexual partner of my infection." Correct Answer (Blank)

Rationale: It is critical for the nurse to make sure the client understands that informing their partners of the disease is important to help stop the spread of the infection. Genital herpes is transmittable even when lesions are not visible. Genital herpes is an incurable, life-long infection. Although not a cure, antiviral medications are often used to shorten healing time of lesions and reduce the frequency of outbreaks. Herpes simplex virus (HSV) is typically not required to be reported. However, gonorrhea and syphilis are required to be reported to public health authorities throughout the U.S. In some states, chlamydia infections must also be reported.

The nurse is conducting teaching with a family whose newborn infant was diagnosed with hypothyroidism. Which point is important for the nurse to emphasize during the teaching? A Hormone replacement therapy will prevent complications. Correct Answer (Blank) B Expect the child to be developmentally delayed. C This rare problem is always hereditary. D Physical growth will be stunted.

Rationale: It is important to emphasize that early identification (ideally before 13 days old) and continued treatment with levothyroxine thyroid hormone replacement will correct hypothyroidism in newborns and prevent future problems. If undetected and untreated, hypothyroidism can result in poor growth, weight gain, slow heart rate, low blood pressure and babies who are unusually quiet. An untreated child will be at risk for permanent brain damage and intellectual disabilities. Approximately one in every 4,000 babies is born with hypothyroidism. Congenital hypothyroidism can be caused by a variety of factors, only some of which are genetic.

The nurse is caring for a client with a chronic ankle wound. There is limited hair growth on the leg, the skin is pale, non-edematous, and the patient reports pain with ambulation. Which prescription would require collaboration with the healthcare provider? A Obtain an ankle brachial index B Apply a compression bandage Correct Answer (Blank) C Schedule a doppler ultrasound D Perform neurovascular checks

Rationale: Knowing the etiology of a wound directly impacts decisions related to assessment and treatment of the wound. The client is demonstrating signs and symptoms of arterial disease. Compression therapy is used in the treatment of venous insufficiency and would not be used in the management of arterial disease. All other options are appropriate for the diagnosis and management of peripheral artery disease.

The nurse is caring for a client with late-stage liver cirrhosis. The nurse should monitor the client for which clinical manifestations? Select all that apply. Fluid volume deficit Spider angiomas Correct Answer (Blank) Ascites Correct Answer (Blank) Encephalopathy Correct Answer (Blank) Pulmonary hypertension Splenomegaly Correct Answer (Blank) Increased albumin level

Rationale: Late clinical manifestations related to cirrhosis are the result of portal hypertension and the inability of the liver to maintain normal functions such as detoxification, blood clotting, bile production, blood filtration and carbohydrate, protein and fat metabolism. Common findings include low serum albumin levels, leading to edema and fluid volume excess, spider angiomas (i.e., telangiectasia or spider nevi), encephalopathy and increased venous pressure in the portal circulation, leading to ascites and an enlarged spleen. Pulmonary hypertension is not a typical finding with cirrhosis of the liver.

The nurse is assessing a client with left-sided heart failure. Which finding requires the nurse's immediate attention? A Nocturia B Pulse oximeter 85% Correct Answer (Blank) C Diaphoresis D Pulse 100 bpm

Rationale: Left-sided heart failure is characterized by a low cardiac output. As a result, there is a decrease in the amount of oxygen and nutrients that is delivered to other body systems. The inability of the left ventricle to empty itself can lead to pulmonary congestion and pulmonary edema. This would cause poor gas exchange, hypoxia as evidenced by cyanosis or general weakness. An oxygen saturation of 88% or less indicates hypoxemia and would require the nurse's immediate attention. The hyoxemia is related to low cardiac output with low oxygen distribution and pulmonary congestion. Tachycardia, diaphoresis and nocturia can also be common occurrences in clients with heart failure or left-sided heart failure. However, these findings are not high priority. The pulse rate is high due to a compensatory mechanism from low cardiac output and hypoxia. Diaphoresis can occur secondary to a stress response from the heart failure.

The nurse in the neurology office is reviewing information about levetiracetam with a 30-year-old female client with a history of seizures. Which instruction about the medication should the nurse make sure to include? A "You should avoid becoming pregnant while taking this medication." B "Call the office immediately if you feel like hurting or killing yourself." Correct Answer (Blank) C "You should stay away from large crowds and sick children." D "You might experience irregular menses and intermittent bleeding."

Rationale: Levetiracetam is an anti-convulsant medication used to prevent seizures. One of the significant side effects is behavioral changes and suicidal ideations. It is important to notify the provider office immediately if the client experiences these thoughts. The other instructions do not apply to this particular medication.

The nurse understands that the prescribed levothyroxine is effective when the client with hypothyroidism makes which statement? A "I still feel lethargic and fatigued." B "I was reprimanded at work after becoming angry with my boss." C "I have been having daily, formed bowel movements." Correct Answer (Blank) D "I have to change my sheets in the morning because I sweat a lot at night."

Rationale: Levothyroxine sodium is utilized to treat hypothyroidism. The nurse must first understand signs and symptoms of hypothyroidism, such as fatigue, lethargy, constipation, hypotension, anorexia and weight gain. In doing so, the nurse can identify that reports of having regular bowel movements is indicative of the levothyroxine working as intended, as constipation is a common symptom of hypothyroidism. Additionally, the nurse must also recognize symptoms of hyperthyroidism, as some clients on levothyroxine sodium may be receiving too high a dose, resulting in excess thyroid hormone and symptoms of hyperthyroidism, such as diaphoresis, irritability, heart palpitations, weight loss and diarrhea. The nurse should recognize that a client who sweats through the night or who is irritable and angry at work may be displaying symptoms of hyperthyroidism.

The nurse is teaching a 27-year-old client with asthma about the therapeutic regimen. Which statement by the client indicates further teaching is needed? A "I should monitor my peak flow every day." B "I should contact the clinic if I am using my medication more often." C "I should learn stress reduction and relaxation techniques." D "I need to limit my exercise, especially activities such as walking and running." Correct Answer (Blank)

Rationale: Limiting physical activity in an otherwise healthy, young client should not be necessary. If exercise intolerance exists, the asthma management plan should include specific medications to treat the problem such as using an inhaled beta-agonist five minutes before exercise. The goal is always to return to a normal lifestyle. It is appropriate for the client to learn to relax. The asthmatic client should monitor their peak flow every day and medications may need to be adjusted if the reading is in the yellow or red-zone. If the client is using their rescue inhaler more often, a medication adjustment may be needed.

The nurse is assessing a 6 year-old child for the first time in the clinic and finds that the child has deformities of the joints, limbs and fingers; a thinned upper lip; and small teeth with poor enamel. The mother states: "My child struggles with counting and color recognition." Based on this data, the nurse suspects that the child is most likely displaying the effects of which problem? A Lead poisoning B Congenital abnormalities C Chronic toxoplasmosis D Fetal alcohol syndrome (FAS) Correct Answer (Blank)

Rationale: Major features of fetal alcohol syndrome (FAS) are facial and other malformed physical features, such as small head circumference and brain size (microcephaly), small eyelid openings, a sunken nasal bridge, an exceptionally thin upper lip, a short, upturned nose and a smooth skin surface between the nose and upper lip. Vision difficulties include nearsightedness (myopia). Other findings are mental retardation, delayed development, abnormal behavior such as short attention span, hyperactivity, poor impulse control, extreme nervousness and anxiety. Behavioral problems, cognitive impairment and psychosocial deficits are also associated with this syndrome.

The nurse is assessing a client who sustained multiple fractures, contusions, and lacerations in a motor vehicle accident three days ago. The client suddenly becomes confused. Which findings would support the nurse's concern that the client has developed a fat embolism? Select all that apply. Elevated temperature Correct Answer (Blank) Low oxygen saturation Correct Answer (Blank) Petechiae on the upper anterior chest Correct Answer (Blank) Dyspnea Correct Answer (Blank) Hypertension

Rationale: Manifestations of acute confusion, hypoxia, fever, and hypotension may indicate fat embolism in a client who has sustained multiple fractures, particularly fractures of the long bones. The occlusion of dermal capillaries by fat with increased friability of the capillaries can result in skin petechiae. This is most common on the chest, neck, upper arm, axilla, shoulder, oral mucous membranes, and conjunctiva.

The nurse is caring for the client who had a cesarean birth 6 hours ago. The client has silent bowel sounds and has not passed flatus. Which of the following interventions is appropriate? A Assist the client with ambulation. Correct Answer (Blank) B Provide clear liquids C Encourage a bland diet D Obtain a prescription for a laxative

Rationale: Manipulation of the abdominal organs during surgery may produce a loss of normal peristalsis for 24 to 48 hours, depending on the type and extent of surgery. Even though nothing is given by mouth, swallowed air and GI tract secretions enter the stomach and intestines producing distention and causing the patient to complain of fullness or pain in the abdomen. Distention may be avoided by having the patient turn frequently, exercise, and ambulate as early as possible. Chewing gum has also been shown to improve peristalsis and reduce the risk of paralytic ileus. Food and liquid should not be encouraged until bowel sounds are auscultated or flatus is passed. Laxatives are for the treatment of constipation.

A nurse is monitoring a client with a head injury who is receiving prescribed mannitol. Which finding would indicate that the treatment is effective? A Client is more responsive. Correct Answer (Blank) B Pupils are 8mm and non-reactive. C Urine output decreases. D Systolic BP remains around 140 mmHg.

Rationale: Mannitol is an osmotic diuretic that raises plasma and extracellular osmolality, which moves fluid out of the brain and into the circulating blood volume. This leads to an increase in stroke volume, cardiac output, urine output, and increased blood pressure.

A nurse is interviewing the parents of a child who was recently diagnosed with asthma. During data collection, which question is a priority for the nurse to ask? A "Have you recently purchased new furniture?" B "Do you have plants in the home?" C "Do any pets live in the home?" Correct Answer (Blank) D "Did you paint your home recently?"

Rationale: Many cases of childhood asthma are associated with environmental triggers, such as animal dander. These triggers stimulate the inflammatory response and constriction of the terminal bronchioles. Animal dander is a very common allergen affecting children and adults. Other triggers include pollens, carpeting, cigarette smoke, and household dust.

During a yearly health screening, an older female client reports having perimenopausal symptoms including irregular menstrual cycles, mood swings and hot flashes. She requests a more natural approach to manage the symptoms. Which non-pharmacological interventions should the nurse include? Select all that apply. "1-2 glasses of red wine with dinner can help you manage stress." "Use deep breathing exercises when you start having a hot flash." Correct Answer (Blank) "You should drink at least 8-10 glasses of water a day." Correct Answer (Blank) "Incorporate more vegetables and legumes in your diet." Correct Answer (Blank) "Try exercising just before bedtime to help you sleep more soundly." "Yoga may help you manage stress and relieve symptoms." Correct Answer (Blank)

Rationale: Measures that have been found to be effective in helping manage symptoms of hot flashes include exercise, stress reduction and getting enough sleep at night. Reducing the temperature in the room at night and taking a warm bath or shower before bedtime can help clients get a better night's sleep. Slow abdominal breathing (6-8 breaths per minute) at the onset of hot flashes can help. Other measures that can lessen the number of and severity of hot flashes include yoga, as well as avoiding alcohol, spicy foods and caffeine. Eating a more plant-based diet can also help.

A client presents with a small, elevated, and ulcerated skin lesion on the upper back. The lesion has irregular edges that vary in color. The nurse knows that this finding could be associated with which type of skin cancer? A Actinic keratosis B Basal cell carcinoma C Melanoma Correct Answer (Blank) D Squamous cell carcinoma

Rationale: Melanomas tend to appear as lesions with irregular edges, which are small in size, flat or elevated, and eroded or ulcerated. They can be black, brown, gray, or white in color. The most common sites of melanoma include the back, chest, or legs. Squamous cell carcinoma is more commonly found in sun-exposed areas, such as the face and hands. Basal cell carcinoma lesions tend to look like sharply defined, pearly, and flat or barely elevated plaques.

The nurse on an inpatient medical unit is caring for a client who is in the advanced stage of multiple myeloma. Which intervention should the nurse include in the plan of care? A Monitor the client for hypokalemia. B Administer diuretics as ordered. C Place the client in protective isolation. D Use careful repositioning techniques. Correct Answer (Blank)

Rationale: Multiple myeloma occurs when abnormal plasma cells (myeloma cells) collect in several bones. This disease may also harm other tissues and organs, especially the kidneys. This type of cancer causes hypercalcemia, renal failure, anemia, and bone damage. Because multiple myeloma can cause erosion of bone mass and fractures, extra care should be taken when moving or positioning a client due to the risk of pathological fractures.

A male client who is diagnosed with gonococcal urethritis tells the nurse he had recent sexual contact with a woman who did not appear to have any disease. What is the best response by the nurse? A "Women might not have the disease but can be a carrier and infect others." B "Women might not realize that they have gonorrhea because they are often asymptomatic." Correct Answer (Blank) C "Gonorrhea in women only affects the ovaries and not the genital organs." D "Men are at a much greater risk than women for acquiring gonorrhea."

Rationale: Men and women who are sexually active are equally at risk for contracting gonorrhea. Many women with gonorrhea are asymptomatic or have minor symptoms that are easily overlooked. The disease may affect both the genitals and the other reproductive organs and cause complications such as pelvic inflammatory disease. Only persons with an active infection of gonorrhea can transmit the disease. A person does not become a carrier of gonorrhea.

The nurse is caring for a client with meningitis. Which observation by the nurse would indicate that the client's condition is worsening? A Complaints of a headache B Decreased level of consciousness Correct Answer (Blank) C A temperature of 101.3° F. (38.5° C) D Complaints of light sensitivity

Rationale: Meningitis is an inflammation of the arachnoid and pia matter of the brain and spinal cord. Expected findings for a client with meningitis include photophobia, headache, fever and chills. The most important nursing intervention for clients with meningitis is the accurate monitoring and recording of their neurologic status, vital signs and vascular assessment. Increased Intracranial Pressure (ICP) is a life-threatening complication of meningitis. A decrease in the client's level of consciousness is typically the first sign of an increasing ICP.

The nurse is educating a client with metabolic syndrome about disease management. Which client statement indicates the need for further teaching? A "I should eat less saturated fat to keep my triglyceride levels below 150 mg/dL." B "I need to control my sodium intake to keep my systolic blood pressure below 130 mmHg." C "I will eat fewer simple carbohydrates, so my fasting blood glucose is below 100 mg/dL." D "I will consume more fatty fish to lower my high-density lipoprotein below 40 mg/dL." Correct Answer (Blank)

Rationale: Metabolic syndrome is characterized by hyperlipidemia, hypertension, and hyperglycemia. High-density lipoprotein is considered good cholesterol and levels should be above 40 mg/dL for males and 50 mg/dL for females. Triglyceride levels below 150 mg/dL are the desired range. Sodium increases fluid retention and raises blood pressure. The ideal value is systolic blood pressure below 130 mmHg. Simple carbohydrates raise blood sugar quicker than complex carbohydrates. A fasting blood sugar level below 100 mg/dL is the desired value.

The nurse working in a medical office answers a phone call from a client. The client asks how to improve symptoms of their migraine headache. The nurse should advise the client to use which interventions? Select all that apply. Apply a cold cloth to their forehead. Correct Answer (Blank) Rest in bed and watch television. Wear sunglasses while indoors. Correct Answer (Blank) Lie down in a darkened room. Correct Answer (Blank) Drink an 8 oz. glass of red wine. Take 600 mg ibuprofen as prescribed. Correct Answer (Blank)

Rationale: Migraine headache is an episodic familial disorder manifested by unilateral, frontotemporal, throbbing pain in the head, which is often worse behind one eye or ear. It is often accompanied by sensitive scalp, anorexia, photophobia, phonophobia and nausea, with or without vomiting. The priority for interdisciplinary care of the client experiencing a migraine is pain management, which may be achieved by abortive and preventative drug therapy, as well as non-drug measures. The client may be able to alleviate pain by lying down in a dark room. The client may want both eyes covered and a cool cloth on their forehead. If the client falls asleep, they should remain undisturbed until awakening. Sunglasses may eliminate symptoms of photophobia. Bright lights and noise should be avoided (watching television). Alcohol is a migraine trigger and should be avoided.

The nurse is assessing a client who is 18 weeks pregnant. The client reports mild abdominal cramping and moderate vaginal bleeding. Further assessment by the healthcare provider reveals a dilated cervix. The nurse suspects which complication? A Ectopic pregnancy B Placenta previa C Hydatidiform mole D Spontaneous abortion Correct Answer (Blank)

Rationale: Mild abdominal cramping, moderate vaginal bleeding, and a dilated cervix are characteristic signs of an inevitable abortion. A spontaneous abortion is imminent and cannot be prevented. An ectopic pregnancy occurs when there is abnormal implantation of the fertilized ovum outside of the uterus. A dilated cervix is not an expected finding. Placenta previa occurs when the placenta does not attach to the fundus and implants in the lower segment of the uterus or over the cervical is. Placenta previa is characterized by painless vaginal bleeding. A hydatidiform mole is a benign proliferative growth of placental cells. Molar pregnancies are characterized by severe nausea and vomiting.

The nurse is assisting with an endoscopy. The client is receiving moderate sedation. What finding during an endoscopy indicates an expected response? A Responds to verbal commands with depressed level of consciousness. Airway protection and ventilation require no intervention. Correct Answer (Blank) B Requires repeated verbal stimuli for purposeful response. Airway protection and ventilation require increased monitoring. C No response to repeated verbal or painful stimuli. Airway protection and ventilation support are required. D Responds to verbal commands with minimally reduced consciousness. Airway protection and ventilation require no intervention.

Rationale: Moderate conscious sedation (MCS) is a medication-induced depression of consciousness used to prevent or minimize the patient's anxiety and discomfort while maintaining protective reflexes and the ability to respond appropriately to verbal and tactile stimuli. MCS is used to produce sedation, analgesia, and retrograde amnesia during invasive treatments and diagnostic procedures (such as endoscopy). Under some circumstances, MCS is also used during patient care in critical care settings. MCS is considerably safer than general anesthesia and allows patients to return to baseline physical and neurologic status rapidly The desired outcome of caring for and monitoring the adult patient receiving MCS is that the procedure will be performed safely and efficiently and cause minimal or no complications or discomfort to the patient as evidenced by stable vital signs and EKG rhythm, adequate ventilation, and other clinical findings (e.g., skin color, respiratory effort, response to verbal commands).

The nurse is reviewing the plan of care for a 30-year-old client newly diagnosed with multiple sclerosis. Which interventions should the nurse include for this client? Select all that apply. Instruct the client on how to self-catheterize as needed. Encourage participation in vocational rehabilitation. Correct Answer (Blank) Encourage participation in physical and occupational therapy. Correct Answer (Blank) Encourage independence in personal care and bathing. Correct Answer (Blank) Review methods to prevent and treat constipation. Correct Answer (Blank)

Rationale: Multiple sclerosis (MS) is a debilitating disorder affecting the myelin sheaths of the nervous system. Symptoms will vary depending on the extent and area of damage. Clients with MS are encouraged to maintain as high a level as possible of independence. They are encouraged to participate in physical and occupational therapy (PT/OT), as well as exercise therapies. They should be taught about methods to prevent and treat constipation. They should be taught how to manage cases of bladder incontinence, but it is not necessary to self-catheterize. The client should be encouraged to maintain independence in personal care. Depending on the level and severity of the disease, the client may need to participate in vocational rehabilitation (VR). VR is a federal-state program that helps people who have physical disabilities maintain or return to employment.

The nurse is caring for a client diagnosed with multiple sclerosis who plans to begin an exercise program. Which of the following information should the nurse be sure to emphasize when discussing this topic with the client? A Dress warmly B Avoid dehydration Correct Answer (Blank) C Avoid aerobic exercise D Focus on strength training

Rationale: Multiple sclerosis (MS) is an autoimmune disease in which the body's immune system attacks and damages the myelin sheath, the insulating material that surrounds the nerve fibers of the brain and spinal cord. When the myelin sheath becomes damaged, nerve impulses to and from the brain are interrupted. As a result, clients with MS experience muscle weakness, poor balance and coordination, muscle spasticity, and paralysis that may be temporary or permanent. Clients with MS who participate in regular aerobic exercise have better cardiovascular fitness, greater strength, better bowel and bladder function, and less fatigue. The client must take in adequate fluids before and during exercise periods to prevent dehydration. It is recommended that clients with MS exercise when it is colder and perform exercise earlier in the day to avoid fatigue.

The nurse is collecting data about a client admitted with cardiomyopathy. How should the nurse document the low pitch, blowing sound the nurse heard when auscultating near the apex of the heart? A Stridor B Murmur Correct Answer (Blank) C Wheeze D Pulsus paradoxus

Rationale: Murmurs reflect turbulent blood flow through normal or abnormal heart valves. They are classified according to their timing in the cardiac cycle: systolic murmurs (e.g., aortic stenosis and mitral regurgitation) occur between S1 and S2, whereas diastolic murmurs (e.g., mitral stenosis and aortic regurgitation) occur between S2 and S1. They are also graded by the provider according to their intensity. The quality of murmurs can be further characterized as harsh, blowing, whistling, rumbling or squeaking. They are also described by pitch, which is usually high or low. An auscultatable murmur near the apex of the heart or the left fifth intercostal space is most likely related to mitral valve problems that could have led to cardiomyopathy. Stridor and wheeze are abnormal sounds found with auscultation of the lungs and are airway-related. Pulsus paradoxus is obtained during a blood pressure measurement and is indicative of cardiac tamponade.

The nurse is taking care of a client with muscle aches, headache, and a rash with a "bull's-eye" appearance on the upper back. Which statement made by the client is consistent with the physical findings? A "I scraped myself with a piece of metal while working in my garage yesterday." B "I just returned from a weekend trip to the beach." C "My family and I went camping last week in the woods." Correct Answer (Blank) D "I ate potatoes that I canned at home last month."

Rationale: Muscle aches, headache, and a distinctive rash with a bull's-eye pattern are indicative of Lyme disease. Lyme disease is transmitted via ticks. Ticks live in wooded, grassy areas. The incubation period is 3-30 days. An injury from a piece of scrap metal can potentially lead to tetanus. The symptoms are not consistent with a tetanus infection. Lyme disease is caused by ticks. A sand and water environment is not conducive to ticks. Home-canning vegetables or food can lead to botulism. The symptoms presented are not indicative of botulism.

A client is admitted to the nursing unit for respiratory distress due to a myasthenia gravis crisis. Which recent treatment may have been a contributing factor to the client's condition? A The administration of intravenous immunoglobulin for immune support B The administration of morphine sulfate for acute pain management Correct Answer (Blank) C The administration of corticosteroids for immunosuppression D The plasmapheresis exchange procedure performed for antibody removal

Rationale: Myasthenia gravis is a chronic autoimmune disease characterized by fatigue and weakness, primarily in muscles innervated by the cranial nerves, as well as in skeletal and respiratory muscles. As a result, nerve impulses are not transmitted to the skeletal muscle at the neuromuscular junction. The priority for nursing management of the client in myasthenic crisis is maintaining adequate respiratory function. Since this is an autoimmune disease, corticosteroids may be used for immunosuppression as well as intravenous immunoglobulin (IVIg) therapy. Plasmapheresis exchange is performed to remove antibodies from the blood. Drugs containing magnesium and morphine should be avoided because they may increase the client's weakness and respiratory suppression.

The nurse is assessing a client with myasthenia gravis who has a dose of pyridostigmine ordered for 7 am. Prior to giving the medication, the nurse observes and notes diplopia, dysphagia, and a weak cough. The client has ordered breakfast for 8 am. Which action is the priority? A Assess for lower extremity weakness B Give the client edrophonium chloride C Administer the pryidostigmine as soon as possible Correct Answer (Blank) D Hold the medication and notify the health care provider

Rationale: Myasthenia gravis is a chronic, progressive autoimmune disorder that is characterized by periods of remission and exacerbation. Auto-antibodies attack the neuromuscular junction of skeletal muscles, thus leading to weakness of skeletal muscle groups. The condition worsens with activity and improves with rest and progresses to more severe weakness over weeks to months. Edrophonium chloride is typically administered for diagnostic purposes, not for treatment. The findings indicate that the priority is for pyridostigmine to be administered promptly to decrease symptoms of muscle weakness and facilitate the client's ability to eat breakfast. Lower extremity weakness is expected in this diagnosis and is not relevant to the situation. Holding the medication would only be an option if a cholinergic crisis is suspected.

The nurse in the emergency department is admitting a client with a reduced level of consciousness due to severe hypothyroidism. Which intervention should the nurse implement first? A Orient the patient to person, time and place. B Administer propranolol as prescribed. C Implement warming blankets as indicated. D Provide supplemental oxygen. Correct Answer (Blank)

Rationale: Myxedema coma is a complication of poorly treated hypothyroidism, and occurs when levels of thyroid hormone are critically low. Low thyroid levels can result in a reduction in metabolism and significant cardiac dysfunction. This can result in decreased cardiac output, poor oxygenation to tissues and organs, and ultimately tissue and organ failure. For those with myxedema coma, appropriate interventions include administration of propranolol, utilization of warming blankets to prevent hypothermia and reorientation to person, time and place. However, the priority is to maintain respiratory functioning and to provide airway support. Therefore, monitoring oxygen saturation levels and providing supplemental oxygen should be done first.

A nurse is providing care to a client with negative pressure wound therapy to the left lower extremity. The nurse notes an alarm on the device indicating a leak has been detected. Which action will the nurse perform next? A Change the wound dressing on the incision B Reposition the client's lower extremity C Replace the negative pressure wound therapy device D Ensure the dressing is completely sealed Correct Answer (Blank)

Rationale: Negative pressure wound therapy requires an airtight seal around the wound dressing. Leaks around the drape will break the seal and allow air to enter. The nurse should ensure there are no leaks around the wound dressing or tubing. The wound dressing should be replaced only if troubleshooting any leaks is unsuccessful. Repositioning the client's extremity will not correct a leak around the wound dressing or tubing. The dressing must maintain an airtight seal. The device should be replaced only after troubleshooting for leaks or changing the wound dressing are unsuccessful.

The nurse is caring for a child diagnosed with nephrotic syndrome. Which clinical manifestation should the nurse expect to find? A Periorbital edema Correct Answer (Blank) B Bradycardia C Weight loss D Hyperactivity

Rationale: Nephrotic syndrome in children causes excess excretion of protein and retention of fluid causing edema (around the eyes, feet, and ankles) and weight gain. In this type of kidney disease, large amounts of protein are lost in the urine (proteinuria). Children may be more tired and irritable than usual. The other manifestations are not typically seen with nephrotic syndrome.

A nurse at a pediatric clinic examines a toddler with a possible diagnosis of neuroblastoma. Which findings are consistent with the client's diagnosis? A Hearing loss and ataxia B Lymphedema and nerve palsy C Abdominal mass and weakness Correct Answer (Blank) D Headaches and vomiting

Rationale: Neuroblastoma is a type of cancer that commonly arises in and around the adrenal glands. This cancer may also develop in other areas of the abdomen in which the client will complain of abdominal pain or feel a mass underneath the skin. The client may also have changes in bowel habits such as constipation or diarrhea and unexplained weight loss. Client's will complain of weakness due to the symptoms of the cancer.

The nurse is caring for a client with neurogenic shock. The client's temp is 97.9°F, blood pressure is 85/54, heart rate is 53 beats per minute, and skin is warm and dry. What intervention will be implemented? A Apply warming blankets B Prepare for transcutaneous pacing Correct Answer (Blank) C Remove the stimuli that is causing the client's symptoms D Administer intravenous fluid resuscitation

Rationale: Neurogenic shock is a form of distributive shock that leads to hypotension. However, the overriding parasympathetic stimulation that occurs with neurogenic shock causes bradycardia unlike other forms of shock. The first-line intervention for distributive shock is fluid resuscitation. The client's heart rate is slow but does not require transcutaneous pacing at this point because the skin is warm. Warming the client may result in worsening hypotension, so this should be avoided unless the client is hypothermic.

The nurse is performing a respiratory assessment on a newborn. Which assessment finding would require intervention by the nurse? A Rapid, shallow respirations B Short periods of apnea (<10 seconds) C Nasal flaring Correct Answer (Blank) D Symmetric chest movement

Rationale: Newborn respirations are often rapid, shallow and irregular with short periods of apnea (<15 seconds). The respiratory rate of a newborn is dependent on their activity level but ranges from 30 to 60 breaths per minute and chest movement should be symmetrical. While rapid, the respirations should not be labored. Nasal flaring, cyanosis, sternal retractions and expiratory grunting are signs of respiratory distress and should be further evaluated.

A client has been diagnosed with Zollinger-Ellison syndrome. Which information is most important for the nurse to reinforce? A It is a condition in which one or more tumors called gastrinomas form in the pancreas or in the upper part of the small intestine (duodenum) B Treatment consists of medications to reduce acid and heal any peptic ulcers and, if possible, surgery to remove any tumors C With the average age of diagnosis at 50 years, the peptic ulcers may occur at unusual areas of the stomach or intestine D It is critical to promptly report any signs of abdominal pain or gastric bleeding to your health care provider Correct Answer (Blank)

Rationale: Night-time awakening with burning, cramp-like abdominal pain, vomiting (even hematemesis), and change in appetite are some of the findings of peptic ulcers. Abdominal pain, rigidity and tenderness can signal perforation of the ulcer and should be reported to the provider immediately. Zollinger-Ellison syndrome can occur in both children and adults. All of the other options are correct information about this syndrome but are less important to reinforce when teaching the client.

A client diagnosed with an acute anterior myocardial infarction is receiving nitroglycerin and heparin intravenously. The client still reports chest pain. Which action should the nurse take? A Review and compare serial ECG strips. B Auscultate heart and lung sounds. C Administer intravenous morphine sulfate as ordered. Correct Answer (Blank) D Administer antidysrhythmic drugs as indicated.

Rationale: Nitrates are useful for pain control due to their coronary vasodilator effects. The nurse will titrate the intravenous nitroglycerin infusion for chest pain according to standing orders, but if chest pain is unrelieved by the nitroglycerin infusion, the nurse can administer morphine intravenously (IM injections are avoided because they can alter the CPK). Morphine not only relieves pain and reduces anxiety, but also dilates the blood vessels. After giving the pain reliever, the nurse can do a more in-depth assessment of the client (auscultate heart and lung sounds, review ECGs, vital signs and labs).

The nurse is assessing a client who had a closed-chest drainage system placed 6 hours ago for treatment of a hemothorax. The nurse should notify the primary health care provider (PHCP) of which of the following findings? A Drainage from the chest tube has stopped Correct Answer (Blank) B Fluid in the water seal chamber rises with inspiration C Bubbling of water in the water seal chamber with coughing D Crepitus palpated in the area surrounding the insertion site

Rationale: No drainage from a chest tube in the first 24 hours after insertion requires immediate notification to the surgeon; if drainage stops, it can lead to a tension pneumothorax. Bubbling in the water seal chamber is normal during forceful expiration or coughing because the air in the chest is being expelled. Crepitus around the insertion site indicates subcutaneous emphysema, which, unless extensive and extending to the neck, is not a serious complication. Fluid in the water seal chamber should rise with inspiration and fall with expiration, which is called tidaling.

The nurse is counseling a 6-year-old child who was recently diagnosed with nocturnal enuresis. Which of the following must the nurse understand about the pathophysiology of this disorder prior to counseling the child? A Enuresis has a definite genetic link. B Enuresis may be associated with sleep phobia. C Enuresis is a sign of willful misbehavior. D Enuresis often has no clear etiology. Correct Answer (Blank)

Rationale: Nocturnal enuresis (NE) occurs in a child over the age of five who does not have any other physical or mental conditions contributing to the problem. A urinalysis (UA) is indicated to rule out other reasons (i.e. medical and mental disorders) for the nighttime bed-wetting. An individualized treatment plan is indicated for the client. Although there are many predisposing factors associated with NE, no clear etiology has been determined for the condition. NE is not the fault of the child nor is it caused by willful misbehavior. Often children are asleep when the bed-wetting occurs. It is not done on purpose. There is no evidence that supports that NE is associated with sleep phobia. Although NE is commonly associated with a family history of the condition, no genetic link has been definitely confirmed.

The nurse is conducting medication administration with the client. When given an intravenous (IV) medication, the client says to the nurse, "I usually take pills. Why does this medication have to be given in my arm?" Which nursing response is appropriate? A "The medication will cause fewer adverse effects if given intravenously." B "The intravenous medication will be absorbed into the bodies' tissues more slowly." C "The medication will begin to work sooner when given intravenously." Correct Answer (Blank) D "There is a lower chance of allergic reaction when medications are given intravenously."

Rationale: Numerous factors affect the rate and extent of drug absorption, including dosage form, route of administration, blood flow to the site of administration, gastrointestinal function, the presence of food or other drugs, etc. For rapid drug action and response, the IV route is most effective because the drug is injected directly into the bloodstream. The risk for adverse effects is potentially increased with the IV route. The risk of allergic reaction remains the same.

The nurse in a long-term care facility is reviewing the medical record of a newly admitted client. Which of the following factors put the client at an increased risk for developing a pressure ulcer? Select all that apply. The client has a body mass index (BMI) of 30. Correct Answer (Blank) The client has diabetes mellitus. Correct Answer (Blank) The client is receiving an immunosuppressant drug for rheumatoid arthritis. Correct Answer (Blank) The client is alert and oriented to person, place, time, and situation. The client has a history of exercise-induced asthma.

Rationale: Obesity or low body weight are risk factors for pressure ulcer injury. A BMI of 30 puts the client in the obese range, causing increased pressure while sitting or lying in bed. Diabetes mellitus may cause sensory altercations, which also is a risk factor. Immunosuppressant drugs may suppress or reduce the strength of the body's immune system. Exercise-induced asthma is not a direct risk factor, and there is no indication the client is in respiratory distress. Clients who are confused may not report or sense pain or discomfort, which could decrease their ability to protect skin integrity, relieve pressure, maintain hygiene, or report discomfort.

The nurse is caring for a 78-year-old client with influenza who reports dyspnea on exertion. The client's respirations are shallow and 32 breaths per minute. Which action should the nurse perform first? A Auscultate lung fields for breath sounds. Correct Answer (Blank) B Notify respiratory therapy to administer a nebulizer treatment. C Apply a non-rebreather mask at 100% oxygen. D Evaluate the client's 24-hour intake and output.

Rationale: Older adult clients with influenza are at risk for developing respiratory complications such as pneumonia. Shallow respirations and tachypnea can indicate impending acute respiratory failure. Using the nursing process to prioritize what to do first, the nurse should further assess the client to collect more data. Based on the assessment findings, the nurse will then decide which action would be most appropriate to take next.

The nurse is caring for an 80-year-old client in an assisted living facility, who has a temperature of 100.6°F (38.1°C). This is a sudden change from the client's usual temperature. Which of the following assessments should the nurse perform first? A Lung sounds B Urine output C Level of alertness Correct Answer (Blank) D Appetite

Rationale: Older adults have atypical signs and symptoms of infection. This may make it challenging to identify changes in an older client's condition. Anorexia is considered a symptom of infection. However, it is a vague finding that could be applicable to anything. It is not the most important finding. Confusion and decreased level of consciousness are commonly seen in older adults with an infection. They are often the first sign of infection even in the absence of fever. If the client is alert and responds to questions appropriately, then the temperature should be rechecked. Assessing the client's level of consciousness will help the nurse determine the severity of the temperature elevation and the possibility that this represents an infection. The urine and lungs should be assessed for findings of infection because urinary tract infections and pneumonia are common causes of fever in older adults. However, the client's level of consciousness should be assessed first.

The nurse is providing education on self-care to a client with diabetes type 2. Which statement made by the client indicates an understanding of the teaching? A "I should eat a carbohydrate snack after intense exercise." B "I need to test the temperature of the water before I wash my feet." Correct Answer (Blank) C "I will buy a soft toothbrush to brush my teeth." D "I should avoid taking my hypoglycemic medication if I'm sick."

Rationale: One of the complications of diabetes is diabetic neuropathy. Damage to the sensory nerve fibers causes numbness, tingling, and pain in the lower extremities. Clients with diabetes are at a higher risk for injury due to a lack of sensation on the feet. Water should be tested prior to use to avoid burns. Carbohydrates should be eaten prior to exercising to avoid hypoglycemia. A soft toothbrush is indicated for clients on anticoagulants to prevent bleeding gums. Clients should continue taking their medication even if ill to prevent hyperglycemia and complications from diabetes.

A 3-year-old child presents with exam findings that may suggest a neuroblastoma. The nurse is collecting information from the child's parents. Which statement by the parent is suggestive of neuroblastoma and requires follow-up by the health care provider? A "He seems to be getting weaker and weaker and is sometimes unsteady on his feet." B "We keep having to buy him larger size pants because he's growing so big around the waist." Correct Answer (Blank) C "Our child has been quieter than normal lately and has lost weight." D "He doesn't seem to be going to the bathroom as much and his urine is dark yellow in color."

Rationale: One of the most common signs of neuroblastoma is increased abdominal girth due to the mass or tumor in the abdomen. The mass can cause pain and/or a feeling of fullness and the pressure may affect the child's bladder or bowel. Although the child with a neuroblastoma may not want to eat (which can lead to weight loss), this finding could have many causes. A more significant finding would be if the parents reported that child keeps outgrowing clothing or that clothing is tight around the abdomen.

A nurse is providing education on lifestyle modifications to a client with multiple sclerosis. What will the nurse include in the teaching? A "Restrict the amount of fluid intake to prevent volume overload." B "Perform high-intensity exercises to maintain muscle strength." C "Group activities throughout the day to allow for rest periods." Correct Answer (Blank) D "Ask family members to help you communicate your needs."

Rationale: One of the primary symptoms of multiple sclerosis is fatigue. The nurse should encourage the client to group activities and allow for rest periods throughout the day to conserve energy. Fluid volume overload is not a characteristic sign of multiple sclerosis. Clients should be encouraged to increase fluid intake to prevent urinary tract infections associated with bladder dysfunction. High-intensity exercises lead to overexertion and fatigue. Clients with multiple sclerosis benefit from exercises such as range of motion and stretching. Asking family members to communicate needs does not promote independence. The nurse should suggest the use of communication boards or speech therapy to help with dysarthria.

A nurse is providing care to a client with hemorrhagic shock. The client is hypotensive and tachycardic. Which priority action does the nurse perform to maintain organ perfusion? A Applies pressure to the bleeding site B Elevates the client's lower extremities Correct Answer (Blank) C Initiates a peripheral intravenous line D Monitors the client's blood pressure

Rationale: Organ perfusion is a priority for a client experiencing shock. Elevating the extremities helps to shunt blood flow to vital organs and maintain perfusion. Applying pressure to the bleeding site will prevent further hypovolemia and blood loss. However, this does not directly maintain organ perfusion. Initiating a peripheral intravenous line is required for the administration of fluids and medications. However, this action alone does not directly maintain organ perfusion. Monitoring the client's blood pressure provides an assessment of adequate cardiac output. However, this intervention does not directly maintain organ perfusion.

A client who has osteoarthritis, affecting both knees, is reporting constant pain at a level of 4 on a 0 to 10 scale. Which nonpharmacological intervention should the nurse implement for this client to help alleviate the pain? A Collaborate with physical therapy for paraffin dips to the knees. B Position the client with the knee joints in a flexed position. C Place the client on strict bedrest with bathroom privileges only. D Provide opportunity for the client to participate in hydrotherapy Correct Answer (Blank)

Rationale: Osteoarthritis (OA) means the degeneration of cartilage in the joints, primarily the weight-bearing joints. These degenerative changes lead to swelling and pain in the joint. To prevent joint stiffness, it is important to encourage the client to balance activity and rest. Strict bedrest would only increase joint stiffness and further decrease in joint mobility. Paraffin (a type of wax) dips are helpful for clients with OA in the hands, but are not usually used for OA in the knees. The joints should be placed in a neutral, not flexed, position to prevent contractures. Soaking in a hot bathtub or doing hydrotherapy with physical therapy provides warmth that will decrease pain. The buoyancy of the client's body in water decreases weight on the joints, which will also decrease pain

The nurse should provide which dietary instruction to a client with osteoporosis? A "Decrease your intake of foods that contain vitamin D." B "Eat more bananas to increase your potassium intake." C "Decrease your intake of nuts and seeds." D "Eat more dairy products to increase your calcium intake." Correct Answer (Blank)

Rationale: Osteoporosis causes a reduction in skeletal bone mass, leading to porotic and brittle bones. To offset this reduction, the nurse should advise the client to increase calcium intake by consuming more dairy products, which provide about 75% of the calcium in the average diet. Decreasing vitamin D intake is incorrect as vitamin D helps facilitate calcium utilization. None of the other options would stop osteoporosis from worsening.

The nurse is reviewing the chart of a client with suspected osteoporosis. Which diagnostic test to confirm the diagnosis should the nurse plan for? A Computerized axial tomography scan B Dual-energy X-ray absorptiometry scan Correct Answer (Blank) C Magnetic resonance imaging scan D Positron-emission tomography scan

Rationale: Osteoporosis is a metabolic disease in which bone mineralization results in decreased bone density. A dual-energy X-ray absorptiometry (DEXA) scan is a painless scan that measures bone mineral density (BMD) in the hip, wrist or vertebral column. It is the recommended test for the diagnosis of osteoporosis. Magnetic resonance imaging (MRI), computerized axial tomography (CAT) and positron-emission tomography (PET) scans are imaging tests used for evaluating a range of musculoskeletal diseases, but they are not typically used to diagnose osteoporosis.

The nurse is reviewing the medical record of a client who has been diagnosed with osteoporosis. The nurse identifies which risk factors for this condition? Select all that apply. The client weighs 200 lbs. (90.7 kg) with a height of 5 feet 2 inches (157 cm). The client has a 30 pack per year smoking history. Correct Answer (Blank) The client takes 10 mg of prednisone daily. Correct Answer (Blank) The client is a 75-year-old Caucasian female. Correct Answer (Blank) The client performs weight-bearing exercises six days a week.

Rationale: Osteoporosis is the loss of bone density that leads to weakness of the bone. Risk factors for osteoporosis include being a postmenopausal woman (lack of estrogen), smoking, thin stature, steroid use, lack of weight-bearing exercise, such as prolonged immobility or a sedentary lifestyle, and ethnicity. Steroid use is associated with osteoporosis because it impacts the body's ability to rebuild new bone. Smoking is also associated with osteoporosis. Performing weight-bearing exercise increases bone strength and promotes bone development. A client who is 5 feet 2 inches (157 cm) in height and weighs 200 lbs. (90.7 kg) is considered obese and obesity is associated with osteoarthritis, not osteoporosis.

The nurse is admitting a client diagnosed with uncontrolled hypertension. Which of the following questions is a priority for the nurse to ask? A "Describe your family's cardiovascular history." B "Tell me about your usual diet for one day." C "Describe your usual exercise and activity patterns." D "What over-the-counter medications do you take?" Correct Answer (Blank)

Rationale: Over-the-counter (OTC) medications, especially those that treat cold symptoms, can increase blood pressure. Clients diagnosed with hypertension should be educated to avoid OTC medications that contain phenylephrine and look for OTC cold medication specifically design for people with hypertension. The other options are essential parts of this client's medical history. However, they do not pose the greatest risk to the client.

The nurse is initiating prescribed oxygen therapy at 5 L/min via nasal cannula for a client with pneumonia. After verifying the flow rate, which action should the nurse take next? A Educate the client on safe oxygen delivery. B Connect a humidifier to the regulator. Correct Answer (Blank) C Change the delivery method to a simple mask. D Document the client's response to oxygen therapy.

Rationale: Oxygen delivered via a nasal cannula at rates above 4 L/min can dry out mucous membranes. A humidifier prevents drying of the membranes. Providing education to the client regarding oxygen delivery is an important intervention. However, the nurse should ensure client comfort before educating. Nasal cannulas can deliver up to 6L/min. The delivery method does not need to be changed. Documentation of the client's response should be completed until actions that promote safety and comfort are performed.

The nurse is caring for a client in the emergency room with a fractured lower right leg, who receives morphine IV for the pain. One hour later, the client reports that "the pain is getting worse." The nurse should recognize that the client may be developing which of the following complications? A Fatty embolism B Thromboembolic complications C Osteomyelitis D Acute compartment syndrome Correct Answer (Blank)

Rationale: Pain is one of the most common complaints of a client who suffers a fracture. It is not uncommon for clients to receive intravenous (IV) pain medications in the initial setting after a fracture. Thromboembolic complications, such as deep vein thrombosis and pulmonary embolism, are not characterized by increased pain at the site of injury. Increasing pain that is not relieved by narcotic analgesics is a possible sign of compartment syndrome. This condition occurs when the perfusion in the leg decreases due to ongoing swelling at the site. It requires immediate action by the nurse to prevent permanent muscle damage. A fat embolism is associated with sudden changes in respiratory status, petechial hemorrhages, and chest pain. This condition does not increase pain at the site of injury. Osteomyelitis is a bone infection that could occur sometime after the initial injury typically around 48 to 72 hours later.

The nurse is developing a plan of care for a client with acute rheumatoid arthritis. Which priority interventions should the nurse include? Select all that apply. Managing stress Establishing a weight loss goal Preserving joint function Correct Answer (Blank) Relieving pain Correct Answer (Blank) Preventing joint deformity Correct Answer (Blank)

Rationale: Pain relief is a high priority during the acute phase of RA because the pain is typically severe and interferes with the client's ability to function. Preserving joint function and preventing joint deformity are high priorities during the acute phase to promote an optimal level of functioning and reduce the risk of contractures. Managing stress and establishing a goal for a healthy weight are also important, but can wait to be addressed until the acute episode has resolved.

The nurse is planning the discharge for a client with a poor prognosis. The client requires pain management and assistance with activities of daily living. Which action does the nurse take to ensure continuity of care? A Educate the client on the importance of mobility B Collaborate with the case manager for palliative care Correct Answer (Blank) C Request a prescription for home health services D Provide the client with a list of support groups

Rationale: Palliative care is a resource that aggressively treats the symptoms of a chronic illness and provides social, emotional, and practical support. The nurse should ensure the client receives follow-up care prior to discharge. Educating the client on the importance of mobility is important. However, it does not address pain management. Requesting a prescription for home health services is not indicated at this time. Home health services provide intermittent care for an acute illness. The client has a poor prognosis and requires specialized care. Providing the client with a list of support groups does not address the physiological needs.

The nurse is caring for a client with acute pancreatitis. After pain management, which intervention should be included in the plan of care? A Institute seizure precautions. B Encourage the client to cough and deep breathe every two hours. Correct Answer (Blank) C Provide a diet high in protein. D Place the client in contact isolation.

Rationale: Pancreatitis is inflammation of the pancreas in which the digestive enzymes digest the pancreas. One of the complications of pancreatitis is respiratory infections due to the fluid in the retro-peritoneum pushing up against the diaphragm, causing shallow respirations. Coughing and deep breathing every two hours will reduce the chances of respiratory infection. The client will be NPO so a diet high in protein is inappropriate. There is no need for contact isolation or seizure precautions for pancreatitis.

A nurse is assessing a client admitted for diarrhea. The client verbalizes numbness to the lips and muscle twitching to the extremities. Which action does the nurse perform next? A Contact the healthcare provider B Dim the lightning in the client's room C Place the client on seizure precautions D Review the client's latest serum calcium level Correct Answer (Blank)

Rationale: Paresthesia of the lips and tetany (muscle twitching and cramping) are common signs and symptoms of hypocalcemia. The nurse should review the client's latest calcium level to confirm hypocalcemia. The healthcare provider should be contacted after the nurse obtains all objective and subjective data. Soft lighting is recommended to avoid overstimulation. However, the nurse should first confirm the client's symptoms are due to hypocalcemia. Placing the client on seizure precautions is an appropriate intervention after the nurse reviews the calcium level to confirm hypocalcemia.

The nurse in the long-term care facility is reviewing the plan of care for a client with Parkinson's disease. Which interventions should the nurse make sure to include for this client? Select all that apply. Set-up a bladder training program for the client. Correct Answer (Blank) Encourage participation in speech therapy. Correct Answer (Blank) Use cognitive strategies to enhance the client's memory. Correct Answer (Blank) Promote independence by letting client wander throughout the facility. Provide assistance with ambulation. Correct Answer (Blank)

Rationale: Parkinson's disease is a neurological disease that primarily affects movement and can impact cognitive function such as memory. All of the interventions are appropriate for this client with the exception of allowing the client to wander throughout the facility. Clients with Parkinson's disease often suffer from postural instability and are at great risk for falls and injury. The client should have someone with them if they wish to walk throughout the facility. There are other ways to promote the client's independence and the client's safety must come first.

The nurse is reviewing the chart of a client with Parkinson's disease. Which manifestations would the nurse expect to find? Select all that apply. Postural instability Correct Answer (Blank) Rigidity Correct Answer (Blank) Anuria Epistaxis Akinesia Correct Answer (Blank) Tremors Correct Answer (Blank) Jaundice

Rationale: Parkinson's disease is a result of an imbalance in dopamine and acetylcholine (AcH). In Parkinson's disease, dopamine levels are low and the client loses the ability to refine voluntary movement. AcH secreting neurons remain active, creating an imbalance between excitatory and inhibitory neuronal activity. The resulting excessive excitation of neurons prevents a person from controlling or initiating voluntary movement. Parkinson's disease is characterized by four cardinal symptoms/clinical manifestations: tremors, rigidity, akinesia and postural instability. Jaundice, epistaxis (nosebleed) and anuria (low or no urine output) are not expected manifestations of a client with Parkinson's disease.

The nurse is participating in a disaster simulation that involves a school bus accident. The nurse is assigned to care for the following four clients in a rural hospital's emergency department. Which client should the nurse see first? A The client with a penetrating abdominal wound Correct Answer (Blank) B The client with multiple facial abrasions C The client with a third degree burn to the arm D The client with an open humerus fracture

Rationale: Part of a nurse's role is being a part of disaster management and assisting in client care throughout all aspects of health care delivery. To better prepare nurses for disaster situations, simulation is a method used to evaluate preparedness. The nurse needs to be able to respond to disasters in the community and keep clients safe. Answering this specific scenario requires the application of survival potential priority setting frameworks. A client with a penetrating abdominal wound should be seen first because a penetrating injury usually causes internal injuries, such as bleeding, which can quickly lead to death.

The emergency room nurse is caring for a client who has 25% total body surface area burns and is in the initial burn stage. Which intervention should be the priority for the nurse to implement? A Administer a unit of packed red blood cells B Administer potassium chloride IVPB C Administer a broad-spectrum antibiotic D Administer IV crystalloids Correct Answer (Blank)

Rationale: Pathophysiologic changes resulting from major burns during the initial burn-shock period include massive fluid losses. Addressing these losses is a major priority in the initial phase of treatment. Antibiotics and PRBCs are not normally given during this phase. Potassium chloride would exacerbate the client's hyperkalemia, which is common due to electrolyte shifts outside of the cells.

The nurse is reviewing the medical record for a client who is receiving prescribed phototherapy for psoriasis. Which information noted by the nurse would require further intervention? A The client started taking ciprofloxacin 5 days ago. Correct Answer (Blank) B No skin care products were applied to the site prior to arrival. C It has been 48 hours since the last session. D The client reports using an emollient cream after showering yesterday.

Rationale: Patients must bring a list of all current medications, including herbs and supplements, to the first appointment for review with the phototherapy nurse and be taught to report any medication changes as they arise. Fluoroquinolone antibiotics, such as ciprofloxacin, increase photosensitivity and may result in a canceled treatment or reduced dosing. It is important for clients to moisturize the skin at least twice daily while receiving phototherapy. The best time to moisturize is after bathing. Skincare products should not be applied on the morning of the treatment but will be applied afterward. Emollients may be prescribed for application immediately prior to the treatment to help promote the effectiveness of the UVB light. There should be a minimum of 24 hours between treatments.

A client with a history of asthma is admitted for a minor surgical procedure. Preoperatively, the peak flow is measured at 480 liters/minute. Postoperatively, the client reports chest tightness and the peak flow is now 200 liters/minute. What should the nurse do first? A Notify both the surgeon and primary care provider B Apply oxygen at two liters per nasal cannula C Administer the PRN dose of albuterol Correct Answer (Blank) D Repeat the peak flow reading in 30 minutes

Rationale: Peak flow monitoring during exacerbations of asthma is recommended for clients with moderate-to-severe persistent asthma to determine the severity of the exacerbation and to guide the treatment. A peak flow reading of less than 50% of the client's baseline reading is a medical alert condition and a short-acting beta agonist must be taken immediately. Notifying the health care provider is important, but that is not what would be done first. First, the client needs assistance. Oxygen administration will not be effective if the airway constriction is not relieved with the albuterol. Leaving the client and returning in 30 minutes will do nothing to help a client in acute distress.

The nurse is caring for a client at the community clinic who requires treatment for recurrent pelvic inflammatory disease (PID). The nurse knows that this condition most frequently follows which type of infection? A Trichomoniasis B Syphillis C Chlamydia Correct Answer (Blank) D Herpes simplex 2

Rationale: Pelvic Inflammatory Disease (PID) is an infection of the female upper reproductive tract. Treatment is broad-spectrum antibiotics. Chlamydia and gonorrhea infections are the most frequent cause of PID. These sexually transmitted infections often have subtle findings; therefore, they are often not diagnosed early in their course before more widespread infection and complications occur. This also prevents appropriate detection and treatment before transmission to others during sexual activity. A complication of recurrent infection is the obstruction and scarring of the fallopian tubes resulting in infertility.

A client is being admitted to the hospital who reports experiencing bloody stools for several days. Which interventions should the nurse expect to be prescribed for this client? Select all that apply. Surgical consult for a bowel resection Collection of a stool sample for occult blood testing Correct Answer (Blank) Administration of enoxaparin Discontinuation of all NSAID medications Correct Answer (Blank) Administration of pantoprazole Correct Answer (Blank)

Rationale: Peptic ulcer disease (PUD) can affect the gastric, duodenal or esophageal areas of the gastrointestinal (GI) tract. Peptic ulcers cause erosions in the lining of the GI tract and can causing bleeding. PUD may be caused by increase gastric acid, intake of NSAID medications or other irritating agents. The nurse should anticipate the client being started on a proton-pump inhibitor, such as pantoprazole, to decrease the gastric acid levels. All NSAID medications should be discontinued to prevent further disruption of the GI mucosa. To validate the client's claim of bloody stools, the nurse would expect to obtain a stool sample. The nurse should not expect to administer enoxaparin, an anticoagulane often prescribed for venous thromboembolism (VTE) prophylaxis, since that would aggrevate the bleeding in the GI tract. A surgical consult would be premature at this time.

The nurse is developing the plan of care for a client who was admitted with newly diagnosed pericarditis. Which intervention should the nurse include in the plan of care? A Strict bed rest Correct Answer (Blank) B Fluid restriction C Supine position D Decrease stimuli

Rationale: Pericarditis is inflammation of the pericardial sac around the heart. The nurse should implement bed rest to decrease the activity demand on the heart. Supine position increases pain, so clients should be positioned side-lying. The client should be encouraged to drink fluids or may require IV fluid infusion. Decreasing stimuli does not impact pericarditis.

The nurse is caring for a 75-year-old client with peripheral artery disease of the lower extremities. Which intervention should be included in the plan of care to reduce leg pain? A Apply cold compresses. B Follow activity restrictions. C Elevate the legs above the heart. D Support smoking cessation efforts. Correct Answer (Blank)

Rationale: Peripheral artery disease (PAD) is caused by atherosclerosis to the lower extremities. The client with PAD has intermittent claudication resulting in pain, heaviness, or numbness in the extremity when walking. The primary management is through lifestyle changes, such as quitting smoking. The nurse should support any smoking cessation efforts. Another lifestyle change to improve pain in PAD is regular exercise, so a restriction is not necessary. Cold compresses or limb elevation have not proven to improve pain in this situation.

The nurse is caring for a client with a medical history of peripheral artery disease, hypertension and smoking. The client reports severe pain in the right lower leg that started very suddenly and did not get better after receiving an analgesic. What action should the nurse take first? A Notify the health care provider. B Administer an additional dose of the analgesic. C Check the client's pedal pulse. Correct Answer (Blank) D Offer the client an ice pack for the pain.

Rationale: Peripheral artery disease (PAD) refers to excessive plaque buildup in the arterial walls. Excessive plaque buildup, due to atherosclerosis, can have an impact on perfusion to limbs. The client is exhibiting symptoms of an acute arterial occlusion. This occlusion usually causes severe pain, loss of pulses and skin color changes. The nurse should follow the nursing process and first perform an assessment, i.e., check the pulse in the affected extremity. Based on the findings (e.g., an absent pulse), the nurse should notify the health care provider right away because this would signal a medical, possibly surgical, emergency. Ice would be contraindicated as that would further reduce tissue perfusion to the leg.

The nurse is caring for a client with chronic renal failure who is undergoing peritoneal dialysis. The nurse notes that the dialysate solution is instilling very slowly. Which of the following actions would be appropriate for the nurse to implement? Select all that apply. Check tubing and catheter for kinks Correct Answer (Blank) Assess for headache and hypertension Assess for bruit or vibration Reposition the client Correct Answer (Blank)

Rationale: Peritoneal dialysis instills fluid into the peritoneal cavity via an access device. One of the causes for the solution to not be instilling at the prescribed rate would be a kink in the catheter, thus checking for kinks in the catheter or tubing is an appropriate action. Repositioning the client may facilitate the flow of the dialysis solution. A bruit or vibration would be appropriate for the client receiving hemodialysis through an AV shunt or graft. A headache or hypertension may occur in the client receiving hemodialysis.

The nurse is caring for a client who is undergoing an initial peritoneal dialysis treatment. Which finding should the nurse report immediately to the healthcare provider? A Client reports discomfort during dialysate inflow. B The dialysate outflow is blood tinged. C There is leakage of dialysate during inflow. D The dialysate outflow is cloudy. Correct Answer (Blank)

Rationale: Peritoneal dialysis is a type of dialysis that uses a catheter through the peritoneal cavity to instill dialysate to remove toxins. When caring for a client who is undergoing initial peritoneal dialysis treatment, the nurse should expect to observe blood-tinged outflow and client report of discomfort which occurs with initial treatment. Leakage can occur if the catheter is not secure. Cloudy dialysate outflow could indicate an infection and should be reported to the healthcare provider.

The home health nurse is caring for a client who underwent a partial gastrectomy due to gastric cancer several months ago. Which finding would indicate that the client is suffering from pernicious anemia? Select all that apply. The client is exhibiting alopecia. The client is experiencing urinary retention. The client's sclerae are icteric. Correct Answer (Blank) The client's tongue is shiny and beefy-red. Correct Answer (Blank) The client reports numbness and tingling in the feet. Correct Answer (Blank)

Rationale: Pernicious anemia or B12 deficiency are expected in this client due to the removal of a portion of the stomach. With the absence of intrinsic factor, B12 absorption cannot occur and, if left untreated, will lead to pernicious anemia. Typical symptoms include a smooth, beefy-red tongue (glossitis), fatigue, weight loss and jaundice (yellowing of the skin and sclerae). B12 also plays a key role in nerve function and, when absent, can cause paresthesia in the hands and feet. Urinary retention and alopecia (hair loss) are not usually seen with pernicious anemia.

The nurse in a community clinic is completing the health history of a young child who presents with a low-grade fever, runny nose, and paroxysmal, violent coughing spells for one week. The nurse should review the medical record for evidence of which vaccine administration? A Hib B HBV C MMR D Tdap Correct Answer (Blank)

Rationale: Pertussis (whooping cough) is a highly contagious respiratory tract infection caused by the bacterium Bordetella pertussis. Clinical manifestations of pertussis include rhinorrhea (runny nose), fever, and a cough that has a high-pitched sound and tends to be sudden and violent (paroxysmal) causing vomiting and fatigue. Pertussis is prevented by administration of the tetanus, diphtheria, and pertussis (Tdap) vaccine; therefore, the nurse should look for evidence of administration because the child's presenting symptoms suggest pertussis. The MMR vaccine protects against measles, mumps, and rubella. The Hib vaccine protects against infections caused by H. influenza type b. The HBV vaccine protects against the hepatitis B virus.

The nurse is monitoring a neonate with jaundice who is receiving prescribed phototherapy. What finding would indicate to the nurse the treatment is effective? A Urine specific gravity level is decreasing. B Hemoglobin level is decreasing. C Albumin level is decreasing. D Bilirubin level is decreasing. Correct Answer (Blank)

Rationale: Phototherapy reduces bilirubin levels in the blood by breaking down unconjugated bilirubin to be excreted in the bile and stool. Phototherapy aims to prevent kernicterus, a condition in which unconjugated bilirubin enters the brain. If untreated, kernicterus can lead to brain damage and death. Side effects of phototherapy include frequent loose stools and increased insensible water loss. Therefore, when caring for newborns receiving phototherapy for jaundice, nurses should monitor fluid and electrolyte balance, weight, urine output, and urine specific gravity. However, changes to specific gravity do not indicate treatment effect. Serum protein is unaffected by phototherapy. Hemoglobin levels may decline due to hemolysis or hydration status but do not indicate treatment effectiveness.

During the morning rounds, the nurse observes that a client diagnosed with heart failure has developed sudden anxiety, diaphoresis and dyspnea. The nurse auscultates crackles bilaterally. Which nursing intervention should be performed first? A Sit the patient on the edge of the bed Correct Answer (Blank) B Take the client's vital signs C Administer the PRN IV morphine D Contact the health care provider

Rationale: Place the client in a sitting position with legs dangling to pool the blood in the legs. This helps to diminish venous return to the heart and minimize the pulmonary edema and helping the client breathe more easily. The next actions would be to contact the heath care provider, then take the vital signs and then administer the IV morphine. Intravenous diuretics will also be indicated to reduce the fluid volume excess.

The nurse is admitting a client through the emergency department who is 28-weeks pregnant and has a tentative diagnosis of placenta abruptio. Which of the following actions should the nurse take first? A Administer oxygen by mask at 100% Correct Answer (Blank) B Check fetal heart rate every 15 minutes C Insert urethral catheter and monitor urine output hourly D Start a second IV with an 18 gauge cannula

Rationale: Placenta abruptio is the separation of the placenta from its attachment to the uterus wall. Administration of oxygen in this situation is the priority; it will increase the oxygen in the mother's and the fetus' circulation. This action will minimize complications of hypoxia. Additional treatment includes IV fluids and possibly a blood transfusion, which is why starting a second line with a large gauge needle is required. The fetus will be monitored for signs of distress. Additional treatment depends on the severity of the separation and the location of the separation.

A nurse is performing an assessment on a client with pneumococcal pneumonia. Which finding should the nurse anticipate? A Bronchial breath sounds in the outer lung fields. Correct Answer (Blank) B Hacking, nonproductive cough. C Decreased tactile fremitus. D Hyper-resonance of areas of consolidation.

Rationale: Pneumococcal pneumonia is a bacterial infection in the lungs. This can lead to consolidated lung tissue containing the exudate from the infection. This consolidation transmits the bronchial breath sounds to the outer lung sounds. The client will have a productive cough, increased tactile fremitus and the nurse will percuss a dull sound over areas of consolidation.

The nurse is planning care for a client with pneumonia. Which of the following interventions would be the most effective by the nurse, in promoting the clearance of respiratory secretions? A Increase fluid intake throughout the day Correct Answer (Blank) B Maintain bed rest with bathroom privileges C Administer pain medications as needed D Administration of cough suppressants

Rationale: Pneumonia is an infection of the lower respiratory system that affects the lungs and alevoli. The lung fields and sacs can become inflamed and filled with fluid. Pneumonia can be caused by a bacteria, virus or fungus. Manifestations of pneumonia include cough, pleuritic chest pain, fever and shortness of breath. In clients who have pleuritic chest pain, coughing will often exacerbate the client's chest discomfort. Pain medication may be indicated to help the client cough more effectively and remove sputum. Clients need to be out of bed as frequently as tolerated. Lying in bed causes pulmonary secretions to be stagnant in the lung fields. The client should be encouraged to drink adequate fluids (i.e. 2-3 L) throughout the day. Secretion removal is enhanced with adequate hydration, which thins and liquefies secretions, making them easier to cough out. The client should not be instructed to use cough suppressants. Suppressants don't allow clients to cough, and remove secretions and sputum.

The psychiatric nurse is caring for a client admitted with psychogenic polydipsia. Which finding indicates that the client might be experiencing a complication of the condition? A Muscle spasms B Lethargy Correct Answer (Blank) C Urinary retention D Polyuria

Rationale: Polydipsia is excessive or abnormal thirst accompanied by the intake of excessive quantities of water or fluid. Psychogenic polydipsia (PPD), or primary polydipsia, is characterized by excessive volitional water intake and is often seen in patients with severe mental illness and/or developmental disability. There may be no physical effects, but hyponatremia can occur. Neuropsychiatric manifestations of hyponatremia include headache, nausea, cramping, hyporeflexia, dysarthric speech, lethargy, confusion, seizures, and delirium. Coma and even sudden death can ensue as sodium status worsens. It is critical that the nurse monitors the client closely for symptoms of hyponatremia. The other findings are either expected with polydipsia (polyuria) or unrelated to hyponatremia.

The nurse is assessing a client with portal hypertension. Which of the following assessment findings is consistent with the client's diagnosis? A Blurred vision B Dilated pupils C Expiratory wheezes D Abdominal distension Correct Answer (Blank)

Rationale: Portal hypertension can occur in a client with right-sided heart failure, cirrhosis of the liver, or cancer. Portal hypertension can lead to ascites causing increased abdominal distension, pain, and difficulty breathing due to the buildup of fluid. Fluid builds up in the peritoneal cavity due to increased portal pressure and decreased colloid osmotic pressure. Decreased osmotic pressure is due to low serum albumin which causes fluids to leave the intravascular space and leak into the interstitial space in the body. Other manifestations include gastrointestinal bleeding, encephalopathy or confusion, and reduced levels of platelets.

When planning care for a client on a mechanical ventilator, the nurse understands that the application of positive end-expiratory pressure (PEEP) to the ventilator settings has which therapeutic effect? A Prevention of ventilator-associated pneumonia B Prevention of alveolar collapse during expiration Correct Answer (Blank) C Increased inflation of the lungs D Decreased need for suctioning

Rationale: Positive end-expiratory pressure (PEEP) is positive pressure that is applied to the airway during exhalation. This positive pressure prevents the alveoli from collapsing, improving oxygenation and enabling a reduced fraction of inspired oxygen (FIO2) requirement. The other answers are incorrect.

The nurse is assessing a client on positive-pressure ventilation for a flail chest. Which finding indicates the client is responsive to treatment? A Respiratory rate of 20 breaths/min B Symmetrical chest expansion C Oxygen saturation of 94% Correct Answer (Blank) D Absence of pain

Rationale: Positive-pressure ventilation (PPV) forces air into the lungs to assist with oxygenation and perfusion. A flail chest results from fractured, detached ribs that prevent the lung from expanding properly. The expected response is adequate oxygenation and perfusion as indicated by a normal oxygen saturation (>92%). Symmetrical chest expansion would not be an expected finding on a client with a flail chest. A respiratory rate of 20 breaths/min is normal. However, the respiratory rate is not a direct reflection of the effectiveness of PPV. The absence of pain does not evaluate adequate oxygenation.

The nurse is caring for a client who is postoperative and is unable to void 6 hours after removal of the indwelling urinary catheter. Which of the following actions is appropriate? A Obtain a prescription to reinsert the indwelling catheter. B Perform a bladder scan. Correct Answer (Blank) C Monitor the client for discomfort. D Provide the bedpan more frequently.

Rationale: Postoperative urinary retention is the inability to urinate after a surgical procedure despite having a full bladder. It is a relatively common complication. Anesthesia, medications, and pain can all affect the client's ability to void. Measures to encourage normal voiding include providing privacy and assisting the patient with the use of the bathroom or bedside commode, rather than a bedpan, to provide a more natural setting. When the patient cannot void, bladder scanning is used to assess for distension, then straight catheterization (as prescribed) is used to prevent overdistention of the bladder. Continuing to monitor the client is not ideal as retained urine increase the risk for infection.

The nurse is preparing to implement prescribed postural drainage (PD) for a client with cystic fibrosis. The nurse notes that the client has an accumulation of mucous secretions in the left lower lateral bronchus. The nurse should position the client in which position? A High-Fowler's B Low-Fowler's C Left Sims' D Right Trendelenburg Correct Answer (Blank)

Rationale: Postural drainage (PD) requires client positioning that promotes drainage of secretions into the trachea for easier removal. Secretions in the left lower lateral bronchus require the client to be on their right side with the head of the bed lowered to facilitate the movement of secretions via gravity. High- and low-Fowler's positions will cause further accumulation of secretions at the bases of the lungs. Left Sims' will not promote movement of the secretions into the trachea. The movement needs to flow opposite the area of concern.

The client who was admitted with exacerbation of ulcerative colitis has developed hyperglycemia. Which medication that the client was prescribed most likely caused this adverse drug effect? A Diphenoxylate/atropine B Dicyclomine C Prednisone Correct Answer (Blank) D Acetaminophen

Rationale: Prednisone is a corticosteroid, specifically a glucocorticoid. Corticosteroid therapy may be prescribed during exacerbations of ulcerative colitis to decrease inflammation. Common adverse effects include hyperglycemia, osteoporosis, peptic ulcer disease and an increased risk for infection. The nurse should monitor clients who are receiving prednisone for hyperglycemia. Dicyclomine hydrochloride and diphenoxylate with atropine are cholinergic blocking drugs prescribed for gas (flatus) and diarrhea, commonly seen with ulcerative colitis. Acetaminophen is a non-narcotic analgesic given for mild-to-moderate pain. None of those drugs are known to cause an elevated blood sugar.

The nurse suspects cardiac tamponade in a client who has acute pericarditis. How should the nurse determine the presence of pulsus paradoxus? A Subtract the diastolic blood pressure from the systolic blood pressure. B Listen for a pericardial friction rub when the client is instructed to hold their breath. C Check the electrocardiogram for dysrhythmias during the respiratory cycle. D Note when Korotkoff sounds are auscultated during inspiration and expiration. Correct Answer (Blank)

Rationale: Pulsus paradoxus is a decrease in systolic blood pressure (SBP) during inspiration that is exaggerated in cardiac tamponade. Pulsus paradoxus exists when there is a difference greater than 10 mm Hg between when Korotkoff sounds are heard during expiration and when they are heard throughout the respiratory cycle. The other methods described would not be useful in determining the presence of pulsus paradoxus.

The nurse in the prenatal clinic is developing a plan of care for a client with preeclampsia. Which interventions should the nurse include? Select all that apply. Count and record fetal movement daily Correct Answer (Blank) Avoid all sexual activity Limit sodium intake Correct Answer (Blank) Self-monitor blood pressure daily Correct Answer (Blank) Maintain complete bedrest Use acetaminophen for headache Correct Answer (Blank)

Rationale: Preeclampsia is defined as an elevated BP (≥140/90 mm Hg) after the 20th week of pregnancy on more than one occasion, and proteinuria. Recommended interventions include accurate, regular monitoring of BP and education on when to report readings to the health care provider, fetal movement counts and lowering or limiting sodium intake to avoid fluid retention. Headaches are common with preeclampsia and using acetaminophen for pain management would be appropriate. Maintaining complete bedrest or abstaining from all sexual activity is no longer recommended or indicated at this time.

The nurse is caring for a preterm newborn who develops nasal flaring, cyanosis and diminished unilateral breath sounds. The client is diagnosed with a spontaneous pneumothorax. Which of the following procedures should the nurse prepare the client for first? A Cardiopulmonary resuscitation B Insertion of a chest tube Correct Answer (Blank) C Assisted ventilation D Humidified oxygen using face mask

Rationale: Premature infants, infants on mechanical ventilators or who have lung diseases (such as respiratory distress syndrome) are at risk for pneumothorax. If a newborn's breathing is labored or if the level of oxygen in the blood declines, the air must be removed rapidly from the chest cavity by inserting a chest tube. Asymptomatic pneumothorax can be treated by placing the infant in a 100% oxygen hood; oxygen will enter the stomach when a face mask is used.

The nurse is reviewing the cardiac rhythm strip of a client with a history of chronic obstructive pulmonary disease. The nurse should understand the client is experiencing which abnormal cardiac rhythm? A Premature ventricular contraction Correct Answer (Blank) B AV nodal reentry C Atrial flutter D Sinus bradycardia

Rationale: Premature ventricular contraction (PVC) is an electrical impulse conducted from an ectopic area of the ventricles. A PVC will be a wide complex that occurs before a normal complex on a cardiac rhythm strip. Atrial fibrillation will have a fast, irregular rate with no identifiable P waves. Supraventricular tachycardia, which is an AV nodal reentry, will have a regular rhythm with a rate over 150 and no identifiable P waves on the rhythm strip. Sinus bradycardia will have a slow regular rate, but identifiable P waves.

The nurse is reviewing the cardiac telemetry for a client admitted with exacerbation of left-sided heart failure. Which action should the nurse take? A Assess the client's pulse oximetry. Correct Answer (Blank) B Obtain serum cardiac enzymes. C Request a prescription for IV fluids. D Administer prescribed lisinopril.

Rationale: Premature ventricular contractions (PVCs) are early beats, generated from the ventricles. PVCs indicate irritability of the ventricles, which can be caused by hypoxia, electrolyte imbalance, or disease, such as cardiomyopathy. Cardiac enzymes are obtained when a client is experiencing myocardial infarction, which would be indicated with an elevated ST segment. A prescription for IV fluids would be appropriate when a client is experiencing hypotension. Lisinopril, an ACE inhibitor, is indicated to decrease blood pressure.

The home health nurse is teaching a client with reduced mobility after a stroke about how to prevent pressure injuries or ulcers. Which statement from the client indicates that additional teaching is needed? A "I should monitor my skin for redness or warmth." B "I should shift my body weight frequently throughout the day." C "I should walk with my walker when I can." D "I should massage areas of my skin that are red." Correct Answer (Blank)

Rationale: Pressure ulcers occur in areas of soft tissue when pressure applied over time exceeds normal capillary closure pressure resulting in tissue necrosis. For this reason, a critical intervention is the relief of pressure. Thus, clients should understand that it is important to shift their body weight and reposition throughout the day and to maximize mobility by ambulating when they are physically able and with assistive devices. Additionally, clients should monitor for early signs of pressure-related tissue damage, such as skin redness or warmth. The client's statement about massaging any reddened areas is incorrect and requires additional teaching. Clients should avoid massaging areas of tissue damage, as this can lead to capillary damage and deep tissue injury.

The nurse is caring for a client who is septic shock. Which finding would indicate to the nurse the client is experiencing acute organ dysfunction? A Serum creatinine level of 1.5 B Heart rate of 110 on intravenous adrenergic agonists C Drop in blood pressure of >40mmHg from baseline Correct Answer (Blank) D Serum lactate level of <4mmol/L

Rationale: Prevention of the progression of sepsis to acute organ dysfunction requires prompt assessment and interventions. The sepsis bundle includes assessment of lactate levels, rapid fluid resuscitation for hypotension and mean arterial pressure <65mmHg, and initiation of vasoactive medications for MAP <65 mmHg after infusion of fluids. Persistent hypotension is an indication of worsening sepsis and progression to organ dysfunction. Serum lactate of <4 indicates the client's condition is not deteriorating and is possibly improving. An exaggerated response to vasoactive medications indicates that the client is responding to the medication.

The nurse is assessing a client who has an oral endotracheal tube with mechanical ventilation. Which finding requires immediate action by the nurse? A Visible mist in the ventilator circuit B Pulse oximetry reading of 86% Correct Answer (Blank) C Client is unable to speak D Diminished breath sounds bilaterally

Rationale: Pulse oximetry should not be lower than 90% saturation; therefore a pulse oximetry reading of 86% requires immediate action. Breath sounds are diminished but heard bilaterally so the placement of an endotracheal tube ET is most likely in the proper position. A client with an ET tube in place will not be able to speak when the ET tube balloon Is inflated. Due to the need for humidification with mechanical ventilation, it would be expected to have a fine mist visible.

The nurse is caring for a client who was in a motor vehicle accident. Which finding should be the highest priority if newly identified by the nurse? A Reduced sensory responses B Pupils fixed and dilated Correct Answer (Blank) C Diminished spinal reflexes D Flaccid paralysis

Rationale: Pupils that are fixed and dilated indicate overwhelming injury and intrinsic damage to the upper brain stem, and would be the highest priority as it is a poor prognostic sign. The other findings are more consistent with partial dysfunction of the brain or spinal cord.

The nurse in an urgent care clinic is discharging a client diagnosed with pyelonephritis due to a urinary tract infection. Which instruction is most important to include? A "Increase your fluid intake to 3 to 4 liters a day." B "Schedule an appointment for a repeat urinalysis in one week." C "Monitor your blood pressure at home daily." D "Notify your health care provider if you do not start feeling better in a few days." Correct Answer (Blank)

Rationale: Pyelonephritis is an inflammation of the kidney, generally caused by a bacterial infection. Usually, initial treatment consists of a broad-spectrum antibiotic and symptoms should start to improve once the client has started the antibiotic. However, if the symptoms do not improve, the client may require a different antibiotic. Therefore, it is most important for the client to notify their health care provider if their symptoms do not improve in a few days. The client should also increase their fluid intake and may require a repeat urinalysis, but those actions are not as important. At home monitoring of the blood pressure is not required in this situation.

A nurse admits a premature infant who has been diagnosed with respiratory distress syndrome (RDS). In planning care for the infant, the nurse understands that the pathophysiology of this disorder affects the infant's ability to do what? A Stabilize thermoregulation B Regulate intrapulmonary airway pressures C Adequately clear thick, sticky mucus from the lungs D Maintain alveolar surface tension Correct Answer (Blank)

Rationale: RDS is primarily a disease related to a developmental delay in lung maturation. Although many factors may lead to the development of the disorder, the central factor is the lack of a normally functioning surfactant system in the alveolar sac from immaturity in lung development because the infant is premature. A lack of surfactant production results in the collapse of the alveolar sacs.

The nurse is planning care for a client who is receiving radiation therapy for breast cancer. The client has a nursing diagnosis of risk for impaired skin integrity. Which of the following interventions should the nurse include in the client's plan of care? A Wear a swimsuit cover-up when going to the pool B Apply ice packs to the affected area to reduce itching C Use a mild soap and tepid water to clean the affected area Correct Answer (Blank) D Wear a supportive, tight-fitting bra during therapy

Rationale: Radiation can lead to skin changes or skin reactions in the treatment area. Skin changes are commonly seen between the gluteal folds, perineum, collar line, and breast. The goal of skincare is to prevent skin breakdown and infection. Clients should be instructed to avoid wearing tight-fitting bras or belts over the treatment areas. During treatment, clients should avoid exposing their treatment areas to direct sunlight and should also avoid swimming in salt water and chlorinated pools. Clients should also avoid exposing their treatment area to extremes in temperature (hot or cold). To keep the affected area clean, use a mild soap and tepid water.

The nurse is providing education on radiation therapy to a client receiving treatment for cancer. Which of the following statements indicates that teaching has been effective? A "I can use petroleum jelly if my skin becomes reddened." B "I need to apply an SPF 15 sunscreen when I am outside for the day." C "I can apply an ice pack to the site if it becomes sore." D "I should not wash off the markings." Correct Answer (Blank)

Rationale: Radiation causes damage to both normal and cancerous cells in a localized area. The area to be treated is marked carefully to minimize damage to normal cells. Therefore, it is important for the client to avoid soaping the area. Clients should apply unscented aqueous creams or moisturizers that are not petroleum-based. Avoid the use of heat or ice packs at the site. During the treatment, and for 8 weeks after, the skin will be photosensitive. Protecting the skin with a high-SPF (30 or above) sunscreen is advised.

A client with cancer is scheduled for radiation therapy. The nurse is providing teaching about generalized adverse effects. Which adverse effect should the nurse prepare the client to expect? A Fatigue Correct Answer (Blank) B Vomiting C Stomatitis D Alopecia

Rationale: Radiation may cause generalized effects such as fatigue and anorexia. Hair loss, vomiting, and stomatitis are related to the systemic effects of chemotherapy toxicity.

The nurse is caring for a client who is receiving radiation therapy in the head and neck region. Which of the following questions by the nurse would be appropriate to assess for symptoms of adverse effects? A "Have you experienced any incontinence?" B "How long have you been receiving radiation?" C "Have you noticed a reduction in your mobility?" D "Are you finding that your mouth is dry?" Correct Answer (Blank)

Rationale: Radiation of the head and neck region may produce xerostomia (dryness of the mouth), stomatitis (inflammation of the oral tissues), loss of taste, etc. Asking the client about these symptoms is necessary to determine if treatment needs to be modified. The length of treatment does not directly affect the likelihood of these adverse effects. Incontinence and mobility are not directly affected by head and neck radiation therapy.

The nurse is caring for a client who has type I diabetes mellitus. Upon entering the room, the nurse notes the client has rapid, deep respirations, and is lethargic and difficult to arouse. What should the nurse do first? A Check the client's blood sugar. Correct Answer (Blank) B Administer glucagon per protocol. C Review when the last dose of insulin was given. D Review the client's insulin pump settings.

Rationale: Rapid, deep respirations and a decreased level of consciousness are signs that the client may be in diabetic ketoacidosis (DKA). Onset of DKA can be rapid with precipitating factors associated with infection, surgery or stress. Due to the acidotic state within the body, the respiratory system attempts to "blow off" CO2 to correct the metabolic acidosis, hence the rapid, deep respirations. Glucagon is given in cases of hypoglycemia and should not be given until the blood sugar level has been verified. While reviewing the last dose of insulin and verifying that the insulin pump is working correctly are important, it is essential to check the blood sugar first. Assessment is the first step in the nursing process and will help the nurse decide what to do next.

The emergency department nurse is receiving a report on a client with insulin-dependent diabetes mellitus being brought in by ambulance. The client reports extreme thirst, vomiting, and frequent urination. Ketones are present in the urine. Based on these findings, which of the following prescriptions will be implemented? A Dextrose 50% IV push B Regular insulin via continuous infusion Correct Answer (Blank) C Humulin N subcutaneous injection on a sliding scale D Dextrose 5% with lactated Ringer's (D5LR) infusion at 50 mL/hour

Rationale: The client is likely experiencing diabetic ketoacidosis, which is treated with isotonic fluid resuscitation, electrolyte replacement, and a continuous infusion of regular insulin via protocol. Administering dextrose will worsen hyperglycemia. Lactated Ringer's will worsen the acidosis.

The nurse in the outpatient clinic is reviewing the medical record of a client diagnosed with Raynaud's disease. What information from the client's health history would support this diagnosis? Select all that apply. The client smokes two packs of cigarettes per day. Correct Answer (Blank) The client reports brittle fingernails that break easily. Correct Answer (Blank) Fingers become cyanotic when exposed to cold objects. Correct Answer (Blank) The client works in an office setting as a typist. Correct Answer (Blank) Warfarin is listed on the medication reconciliation form.

Rationale: Raynaud's disease is considered a vasospastic disorder that affects the small arteries of the fingers and toes. Raynaud's occurs due to an imbalance between vasodilation and vasoconstriction. Cases of Raynaud's are considered either idiopathic or pathologic. Risk factors include occupation-related factors, such as repetitive hand motions (typing, industrial equipment), or hyperhomocysteinemia. Clients with Raynaud's disease may complain of their fingers or toes becoming cyanotic or pale when they come in contact with cold objects or a cold environment. When this occurs, clients may complain of numbness or tingling in the affected digits. Exposure to tobacco, emotions, and caffeine have been known to trigger a vasospastic event. Medications that can be used for Raynaud's include statins, calcium channel blockers, and vasodilators. Anticoagulants are not indicated for the treatment or prevention of Raynaud's disease. A construction worker who operates a jackhammer would be at risk for Raynaud's due to the vibrating equipment. Clients who suffer from frequent vasospastic attacks can develop brittle nails.

The nurse should monitor which clients who may be at-risk for the development of acute kidney injury? Select all that apply. A client with a history of syndrome of inappropriate diuretic hormone A client with a history of cirrhosis A client admitted with an acute myocardial infarction Correct Answer (Blank) A client who received multiple blood transfusions Correct Answer (Blank) A client recovering from septic shock Correct Answer (Blank)

Rationale: Reduced renal perfusion is a risk factor for acute kidney injury (AKI). A client with significant blood or fluid loss, such as one who was recently in a motor vehicle accident would be at-risk for the development of AKI due to poor perfusion. Myocardial infarction is another risk factor for AKI due to reduced cardiac output and subsequent hypoperfusion. Finally, septic shock is associated with significant hypotension, which can cause poor blood flow and hypoperfusion to the kidneys. All of these factors predispose a client to developing AKI. Clients with a history of syndrome of inappropriate antidiuretic hormone secretion (SIADH) or cirrhosis, on the other hand, may present with volume overload, and thus are not at increased risk for the development of AKI.

A nurse is providing education on illness management to a client diagnosed with asthma. Which client statement indicates further teaching is required? A "It is essential to develop stress management techniques." B "I should avoid exercises that elevate my heart rate." Correct Answer (Blank) C "It is important to prevent environmental temperature changes." D "I will make sure to receive the influenza vaccine."

Rationale: Regular exercise is encouraged for clients with asthma to help promote ventilation, perfusion, and cardiac health. The nurse should instruct the client to premedicate if needed. Stress and emotional distress are triggering factors for asthma. The nurse should encourage the client to practice stress management techniques. Environmental factors, such as changes in temperature, can exacerbate asthma. Clients with asthma should be encouraged to receive influenza and pneumonia vaccines to prevent respiratory illnesses that may worsen the symptoms of asthma.

The nurse in a pediatric hospital is admitting an infant with respiratory syncytial virus (RSV) infection. Which interventions should the nurse include in the infant's plan of care? Select all that apply. Initiate contact precautions Correct Answer (Blank) Suction as needed Correct Answer (Blank) Administer antibiotics Give acetaminophen for fever Correct Answer (Blank) Administer IV fluids Correct Answer (Blank) Provide chest physiotherapy Use humidified oxygen Correct Answer (Blank)

Rationale: Respiratory syncytial virus (RSV) is a respiratory illness that causes cold-like symptoms but can be a much more serious illness in infants. Humidified oxygen should be administered at a concentration that maintains an oxygen saturation at or above 90%. Infants with thick nasal secretions benefit from suctioning. Contact precautions are initiated to decrease the spread of the virus. IV fluids should be administered until the acute phase of the illness is over or the infant is able to effectively breastfeed or bottle-feed. Acetaminophen is used to reduce fever. Chest physical or physiotherapy (CPT) is not recommended for management of RSV. Antibiotics are not used for a viral infection.

A nurse is caring for a child diagnosed with Reye's syndrome. Which action should be given the highest priority by the nurse? A Assist with range of motion B Assess level of consciousness Correct Answer (Blank) C Provide good skin care D Monitor intake and output

Rationale: Reye's syndrome is a rare disorder that causes liver and brain damage. This syndrome may happen at any age; however, it is more prevalent with children. Reye's syndrome occurs after a child has had a recent viral infection, like the flu or chickenpox. Signs and symptoms may include confusion, seizures, and loss of consciousness that would require emergency treatment. The highest priority for the nurse would be to assess the child's level of consciousness. The other interventions may occur after if there are no complications.

The nurse prepares to care for a 4-year-old child newly admitted with rhabdomyosarcoma. Which area and function of the child's body should the nurse alert the staff to pay attention to? A The kidneys B The leg bones C The cerebellum D The muscles Correct Answer (Blank)

Rationale: Rhabdomyosarcoma (RMS) is a malignant tumor that develops from normal skeletal muscle cells. Because skeletal muscle cells are found in every site of the body, RMS can develop in any part of the body. RMS is the most common children's soft tissue sarcoma. It originates in striated (skeletal) muscles and can be found anywhere in the body. The clue is in the middle of the word "myo," which typically means muscle. The most common locations for these tumors are in the structures of the head and neck, the male or female genitourinary tract (e.g. bladder, prostate, etc.), and the extremities. Although the tumors originate in the muscle, they can metastasize to the bone. Kidney and brain tissue are not composed of striated skeletal muscle; therefore, tumors would not develop in these locations. Clients would complain of a lump or swelling in the body, which may be painful.

The nurse is providing discharge education to a client hospitalized for an acute exacerbation of rheumatoid arthritis. The nurse includes information focusing on conserving energy. Which statements by the client demonstrate the teaching has been effective? Select all that apply. "I will set priorities and complete the important tasks first." Correct Answer (Blank) "If possible, I will delegate some things to my friends or family." Correct Answer (Blank) "I will schedule activities throughout the day instead of trying to complete everything in the morning." Correct Answer (Blank) "I will sit on a tall kitchen stool instead of standing when I am preparing meals." Correct Answer (Blank) "On days of increased pain, I will stay in bed and relax instead of being active."

Rationale: Rheumatoid arthritis is a chronic, systemic, progressive autoimmune disease involving the joints. As the disease progresses, the client's joints will become increasingly inflamed and painful. An important topic to teach these clients focuses on maintaining consistent activity and energy conservation. Regular exercise is essential for clients with arthritis to prevent complications. The client should be as active as possible and not be immobile on days of increased pain. Regarding energy conservation, important concepts include pacing, prioritizing, and delegating activity. The client should rest when possible.

The nurse is caring for a client who is receiving treatment for neurogenic shock. The client's mean arterial pressure (MAP) has fallen to 55 mm Hg. When assessing the impact on the client's kidney function, which data would indicate potential kidney injury? A Glomerular filtration rate of 90 B Blood Urea Nitrogen 10 C Urine output of 0.6 ml/kg/hr D Creatinine 1.9 Correct Answer (Blank)

Rationale: Rising creatinine levels are indicative of potential kidney injury. BUN is a less reliable indicator because it is influenced by fluid volume. The client's urine output is above 0.5 ml/kg/hr, which is considered normal. Glomerular filtration less than 90 would be an indication of worsening kidney function. It is important to monitor trends in all labs to identify deterioration or improvement.

The post-anesthesia care nurse is monitoring a client who had an open reduction internal fixation of the femur. Which finding would require immediate follow-up by the nurse? A Impaired airway protection Correct Answer (Blank) B Unstable cardiovascular status C Decreased oxygenation D Diminished neurological status

Rationale: Routine post-anesthetic assessment is a requirement for the recognition of clinical deterioration in post-operative patients. The ABCDE approach should be used in postoperative assessment in both the post-anesthesia care unit as well as in the admitting floor unit. The mnemonic "ABCDE" stands for Airway, Breathing, Circulation, Disability, and Exposure. A = airway, B = breathing, C=circulation, D=disability (neurological status), E = exposure (skin). In this case, the client's immediate priority is the airway, which is likely compromised by a reduced level of consciousness. The reduced O2 saturation will likely improve with a stable airway. Heart rate and blood pressure are in the expected range after skeletal surgery.

The nurse is teaching the partner of a client with a newly placed tracheostomy tube about home care. Which statement by the client's partner indicates understanding of tracheostomy care? A "I will make sure my partner is lying flat while I perform the care." B "A cotton filled gauze dressing should be applied around the tube." C "I will throw away left over saline solution at the end of the day." Correct Answer (Blank) D "This procedure must always be performed in a sterile environment at home."

Rationale: Saline solution can harbor bacteria and enter the respiratory system if used. Caregiver education should include instructions on infection control. The client should be placed in a semi-Fowler's position as tolerated. This position facilitates breathing and coughing. Gauze dressings should not be filled with cotton as they can cause the aspiration of foreign bodies into the lungs. Sterile technique is not necessary in a home setting. Clean technique may be used.

The nurse is assessing a client diagnosed with salmonella. The client reports dizziness and dry mouth for over 48 hours. Which intervention should the nurse implement? A Provide disposable oral swabs B Educate on proper hand washing C Infuse intravenous fluids Correct Answer (Blank) D Administer an enema

Rationale: Salmonella is a food-borne illness that is transmitted via contaminated food or animals. Symptoms associated with salmonella include diarrhea, fever, and stomach cramps. Prolonged diarrhea can lead to dehydration and cause dizziness and dry mouth. The nurse should expect to administer intravenous (IV) fluids to rehydrate the client. Oral swabs will improve the client's dry mouth but will not correct the issue of dehydration. Salmonella can be prevented by washing hands with soap and water. However, this will not correct the client's problem. Administering an enema will worsen the client's dehydration.

The medical-surgical nurse is reviewing the postoperative orders for an adolescent following spinal fusion surgery to correct scoliosis. Which prescribed intervention should the nurse question? A Continue patient-controlled analgesia B Change dressing if it becomes saturated Correct Answer (Blank) C Assess sensation to lower extremities frequently D Log-roll when changing position

Rationale: Scoliosis is a spinal deformity that sometimes is treated with spinal fusion surgery. Spinal fusion surgery is a major surgery requiring close monitoring to prevent postoperative complications. There is a risk of cerebral spinal fluid (CSF) leak postoperatively. A saturated dressing that was applied in surgery should not be removed or changed by the nurse. Instead, the nurse should notify the surgeon immediately to come and evaluate the client and dressing. The other orders are appropriate. Clients should be log-rolled when changing position in order to prevent flexion of the spine. Frequent neurovascular assessments to the lower extremities should be performed to monitor for any neurosensory impairment. Considerable pain is common after surgery and is most often managed with opioids via patient-controlled analgesia.

The school nurse in an elementary school identifies an outbreak of head lice (pediculosis). Which interventions should the nurse implement to prevent the spread of the infestation? Select all that apply. Reassure students that itching of the scalp is a common symptom Correct Answer (Blank) Notify the local health department of the outbreak Provide individual headsets or ear buds for each student Correct Answer (Blank) Instruct school parents, teachers and volunteers on how to detect lice and nits Correct Answer (Blank) Do not permit children to share bike helmets Correct Answer (Blank)

Rationale: Sharing items that touch the head, such as helmets, headsets, hats, combs, towels, etc., is a primary source of spreading pediculosis. Itching is a common and early sign of infestation and should be investigated immediately. Instructing parents and school personnel how to detect lice and nits will foster early recognition and treatment. Head lice is not a reportable disease.

The nurse is reviewing discharge instructions with a client with a new diagnosis of a seizure disorder. Which statements indicate that the client understood the instructions? Select all that apply. "I will make sure to wear a medical alert bracelet." Correct Answer (Blank) "My family will hold me down tightly during a seizure." "My family has developed a plan for when I have a seizure." Correct Answer (Blank) "I will withdraw from my college classes just in case." "I will keep a diary of any seizure activity." Correct Answer (Blank) "I will not miss a dose of my seizure medication." Correct Answer (Blank)

Rationale: Seizures may involve sudden jerky movements accompanied by a loss of consciousness. Once a seizure has occurred it is important for a client and their family members to be prepared and try to prevent recurrence. It is important to take anticonvulsant medication as ordered. Diaries of seizure activity help to determine if there is a trigger. Wearing a medical alert bracelet and having a plan for the family, such as when to call an ambulance, is a proactive plan to manage the disease. The client does not have to stop attending college and should continue with their normal life and activities. A client experiencing a seizure should not be restrained or held down during the seizure as that can cause injury to the client or family member.

The nurse is assisting with an emergency endotracheal intubation who has a prescription for soft, bilateral wrist restraints. Which of the following is an appropriate action for the nurse to take? A Secure the restraint to the bed using a quick release knot. Correct Answer (Blank) B Ensure four fingers fit under the restraint cuff. C Time periods without restraints are attempted. D Assess circulation every 4 hours.

Rationale: Self-extubating poses a serious risk to a client with respiratory failure. Restraints are implemented to protect the patient from harm, but also pose their own risks. Two fingers should fit between the cuff and the client's wrist. Restraints should be tied to the bed frame using a quick-release knot (i.e., slipknot). Assess the patient at least every hour or according to facility policy. Assessment should include the placement of the restraint, neurovascular assessment of the affected extremity, and skin integrity. In addition, assess for signs of sensory deprivation, such as increased sleeping, daydreaming, anxiety, panic, and hallucinations. Monitor the patient's vital signs. Restraints should be removed to assess the skin and then replaced. Trialing a restraint-free period is not performed.

The nurse is removing staples from a client's abdominal incision. Upon removal of a few staples, the nurse notes the incision begins to separate. Which intervention does the nurse perform next? A Clean the incision with saline solution and continue the procedure. B Remove the remaining staples and apply an adhesive wound-closure strips. C Apply adhesive wound-closure strips along the incision after staples are removed. D Cover the area with moistened sterile towels and notify the healthcare provider. Correct Answer (Blank)

Rationale: Separation of wound edges is known as dehiscence. Dehiscence can lead to evisceration if not assessed and treated. The nurse should cover the incision with sterile towels moistened with saline and notify the healthcare provider. Continuing the procedure can lead to evisceration despite cleaning the wound. Removing the remaining staples can lead to evisceration. A pressure dressing secured with an abdominal binder can increase pressure on the area of dehiscence and further separate the wound. Adhesive wound-closure strips are not enough to keep the wound from complete dehiscence.

The nurse is assessing a client with an abdominal wound who had surgery 5 days ago. Which finding would the nurse report immediately to the healthcare provider? A Serosanguinous fluid draining from the incision. Correct Answer (Blank) B Tissue swelling along the incisional line. C Tenderness on the incision upon palpation. D Dryness noted around the skin of the incision.

Rationale: Serosanguinous fluid draining from an incision that is 5 days post-operatively is an indication of impending dehiscence. The nurse should report this drainage to the healthcare provider promptly. Tissue swelling along the incisional line can be indicative of collagen synthesis, a normal response to wound healing. Tenderness upon palpation of the incision is an expected finding within the post-operative timeframe. Dryness noted around the skin of the incision requires education to the client. The wound and surrounding skin should be kept well hydrated to promote healing.

The nurse is taking care of a client with severe Alzheimer's. The nurse expects to perform which action to promote safety? A Raising the head of the bed Correct Answer (Blank) B Placing seizure pads on the client's bed C Activating the bed alarm D Assembling an overhead trapeze on the bed

Rationale: Severe Alzheimer's is characterized by severe cognitive decline. Clients may experience the inability to swallow or clear their own secretions. Raising the head of the bed will prevent aspiration. Placing seizure pads on the client's bed is not indicated for a client with Alzheimer's. Seizures are not expected findings of the disease. Activating the bed alarm is useful for earlier stages of the disease. Late stages result in ataxia, or the inability to move extremities. Assembling an overhead trapeze will not promote safety related to cognitive decline.

The nurse is caring for a client receiving external beam radiation for the treatment of prostate cancer. The client has asked for additional information on the reproductive effects of radiation therapy. What statement would be included in the teaching? A "Sexual function should not be affected by the treatments" B "Sperm banking is an option if you are considering starting a family" Correct Answer (Blank) C "Libido is usually unaffected by radiation therapy" D "Erectile dysfunction will likely resolve after completing treatments"

Rationale: Sexual dysfunction occurs in 20~80% of patients who undergo radiation therapy. Radiation therapy nearly always impairs fertility. Radiated prostate cells and seminal vesicles produce semen that cannot transport the sperm well. Therefore, the nurse should counsel the client on sperm banking. Sexual dysfunction is common in cancer. Prostate cancer by itself reduces sexual desire and the frequency of sexual intercourse. The chance of maintaining erectile function after radiation therapy has been reported to be approximately 50%, which is higher than that of radical prostatectomy. Erectile dysfunction does not resolve after treatment.

A nurse is assessing a client with a deep vein thrombosis. Which finding should the nurse anticipate? A Swelling of one lower extremity Correct Answer (Blank) B Rapid respirations C Chest pain D Bilateral ankle edema

Rationale: The most common signs of deep vein thrombosis are pain in the region of the thrombus and unilateral swelling of the extremity distal to the site. Bilateral pedal edema is usually associated with fluid overload in right heart failure or venous insufficiency. Chest pain and tachypnea that occurs suddenly in the client with DVT are signs of possible pulmonary embolism, a serious complication of DVT.

A nurse is assessing a client who requires frequent repositioning in bed and is unable to assist with mobility. The nurse notes skin tears to the client's elbows and heels. The wounds are likely due to which factor? A Shear Correct Answer (Blank) B Friction C Pressure D Ischemia

Rationale: Shearing occurs when tissue layers slide against each other and injure the underlying blood vessels and capillaries. Skin tears are likely to result when clients are pulled during repositioning, particularly if they are unable to assist. Friction is due to pressure when two surfaces rub against each other. A client who assists in repositioning while applying pressure to the joints is likely to cause a friction injury. Pressure compresses blood vessels and is evident primarily by erythema (stage 1 pressure injury) before progressing to open skin. Ischemia is a deficiency in blood flow that causes hypoxia and necrosis to the affected areas.

The home health nurse is developing a plan of care for an older adult client diagnosed with shingles (herpes zoster) lesions to the face and left eye. What is the priority nursing problem? A Pain related to nerve root inflammation and skin lesions Correct Answer (Blank) B Knowledge deficit related to disease process C Risk for impaired skin integrity related to skin lesions D Risk for social isolation due to pain and location of rash

Rationale: Shingles is a reactivation of the herpes zoster virus responsible for chickenpox. It is characterized by a vesicular rash in a unilateral dermatomal distribution. The first symptom of shingles is usually pain, tingling, or burning before the blisters form. The pain and burning may be severe, and can lead to long-term residual pain, known as postherpetic neuralgia. Using Maslow's hierarchy of needs and considering acute vs. at risk for problems, pain is the priority nursing problem for this client.

The nurse is assessing an infant with developmental dysplasia of the hip. Which finding should a nurse anticipate? A Unequal leg length Correct Answer (Blank) B Diminished femoral pulses C Symmetrical gluteal folds D Unlimited hip abduction

Rationale: Shortening of the affected leg is a sign of developmental dysplasia of the hip. Other signs of hip dysplasia in an older infant include limited hip abduction and asymmetric gluteal skin folds. An ultrasound examination is typically used to confirm developmental dysplasia of the hip in the young infant; x-rays are used when the infant is older than 3 months.

The nurse is providing postoperative care for a client following a laparoscopic cholecystectomy. Which assessment finding should be of highest concern? A Client reports shoulder discomfort. B Client has absent bowel sounds. C Client reports right upper quadrant pain. Correct Answer (Blank) D Client is drowsy.

Rationale: Shoulder pain or discomfort is a common complaint following laparoscopic surgery due to the effects of carbon dioxide gas used during the procedure. Postoperative drowsiness is expected. The absence of bowel sounds immediately after surgery is not a cause for alarm. Right upper quadrant pain could be from a retained gallstone or bile duct injury; therefore postoperative pain in the right upper quadrant should be of highest concern.

The medical-surgical nurse is developing a plan of care for a client with sickle cell disease who was admitted for an acute sickle cell crisis. Which intervention is the priority? A Ambulate in hallway four times a day B Encourage increased caloric intake C Administer prescribed analgesics Correct Answer (Blank) D Increase fluid intake to 3 to 4 liters a day

Rationale: Sickle cell crisis is pain that begins suddenly and may last several hours or even days. This crisis beings when sickled red blood cells block small blood vessels that carry blood to a person's bones. This can cause a person to have severe pain in their back, knees, legs, arms, and chest. The pain can be described as throbbing, sharp, dull, or stabbing. Although all of interventions are appropriate for the client in sickle cell crisis, the priority intervention is effective pain management and administering prescribed analgesics.

The nurse is reviewing the laboratory data for a client who is postoperative abdominal surgery with an incision that is edematous and erythemic. Which data indicates to the nurse the client is experiencing a complication? A White blood count (WBC): 3,500 cells/mcL Correct Answer (Blank) B Hemoglobin (HgB): 14.5 grams/dL C Platelets: 140,000/mcL D White blood count (WBC): 10,500 cells/mcL

Rationale: Signs and symptoms of infection include localized swelling, redness, pain or tenderness, loss of function in the affected area, palpable heat. White blood count (WBC) increasing identifies the body's ability to fight off infection. Low: below 4,500, Normal: 4,500- 11,000, High: Greater than 11,000. Significantly low WBC can indicate a risk for infection particularly with the older adult population. An infection may be present without any change in WBC count.

The nurse is caring for a 16-year-old client who had surgical repair of a fractured femur 14 hours ago. Assessment findings include tachycardia, increased shortness of breath, a temperature of 100.2 F (37.8 C), feelings of anxiety, and an oxygen saturation level of 88%. The nurse immediately notifies the health care provider, recognizing that the client is at risk for which complication? A Atelectasis B Sepsis C Fat embolism Correct Answer (Blank) D Compartment syndrome

Rationale: Since the client recently had an orthopedic surgery, these symptoms are cardinal signs of fat embolism. A fat embolism is a piece of intravascular fat that lodges within a blood vessel and causes obstruction of blood flow. While fat emboli can generally resolve on their own, this complication can lead to fat embolism syndrome which can cause inflammation, multi-organ failure, and neurological changes that may be fatal. Early-onset of sepsis wouldn't appear until at least day 2 or 3, not within 14 hours of the procedure. Compartment syndrome does not cause increased shortness of breath or anxiety. Atelectasis occurs when ventilation is decreased and secretions accumulate.

A client is admitted to the telemetry unit with syncope due to sinus bradycardia. Which intervention should the nurse include in the client's plan of care? A Maintain the client on bedrest. B Implement seizure precautions. C Discuss the client's wishes for organ donation. D Administer a stool softener daily. Correct Answer (Blank)

Rationale: Sinus bradycardia is defined as a heart rate of less than 60 beats per minutes with a regular rhythm that originates from the sinoatrial (SA) node. Typically, clients who develop sinus bradycardia are asymptomatic. If a client develops symptomatic bradycardia, they can present with hypotension, shortness of breath, chest pain, syncope or syncopal episodes and altered mentation. To avoid a vasovagal response (i.e., the slowing of the heart rate caused by bearing down when trying to defecate) and the risk for another syncopal episode, it is important to ensure that the client's bowel movements are soft and easily expelled. The client should also be instructed to avoid holding their breath or bearing down (Valsalva maneuver). The other interventions are not appropriate or required for this client.

A nurse is providing education on disease management to a client with chronic obstructive pulmonary disease (COPD). The client has a history of hypertension, smoking, and asthma. The nurse will teach the client which activity to best prevent complications of COPD? A Smoking cessation Correct Answer (Blank) B Blood pressure control C Prevention of allergen exposure D Prescribed oxygen use

Rationale: Smoking cessation is a primary teaching concept for a client with chronic obstructive pulmonary disease (COPD). Smoking is the primary factor that causes the development and progression of COPD. Maintaining stable blood pressure is important for cardiovascular health. However, it does not specifically prevent complications of COPD. Environmental allergens can exacerbate asthma. However, environmental factors are not the primary factor for the development and progression of COPD. Prescribed oxygen and medications help to control COPD but do not reverse its effects or prevent complications.

A client with eczematous dermatitis (eczema) of the hands asks the nurse how to treat the excoriation and scaling of the palmar surface of both hands. What is the best response by the nurse? A "You should take OTC diphenhydramine orally three times a day." B "You should soak both hands in lukewarm water twice a day." Correct Answer (Blank) C "You should wear disposable gloves during the day." D "You should apply antibacterial cream to both hands."

Rationale: Soaking the hands in lukewarm water, ideally with colloidal oatmeal added to it, is the best response. Soaking the hands will debride crust and scales and soften the skin. Diphenhydramine is an antihistamine that can reduce itching but will not help with scaly, excoriated skin. In addition, the sedative effects of the medication can be dangerous and the drug should be taken only as needed, preferably at bedtime. Wearing gloves is not appropriate because it will trap moisture and warmth, most likely aggravating eczema. An antibacterial cream is not indicated at this time unless the client develops a localized infection.

The nurse is reviewing the laboratory data for a client who is taking newly prescribed lovastatin. Which results would indicate to the nurse that the medication has had a therapeutic effect? A Lactic acid levels are elevated B Total cholesterol is normal Correct Answer (Blank) C WBC is normal D LDL levels are elevated

Rationale: Statin medications reduce the formation of cholesterol precursors which causes LDL to lower, HDL to increase, and total cholesterol to decrease.

The nurse is preparing to perform nasotracheal suctioning for a client with secretions. Which action should the nurse take first? A Set the suction pressure to 160 mmHg. B Apply clean gloves. C Raise the head of the bed to 30 degrees. D Preoxygenate the client. Correct Answer (Blank)

Rationale: Suctioning of the airway can irritate the mucosa and remove oxygen from the airways. The nurse should administer supplemental oxygen before the procedure to avoid hypoxemia. The suction pressure should be set to no more than 150 mmHg for adults. The tube used to suction the airway can introduce organisms into the lungs. The procedure should be sterile. The head of the bed should be elevated to at least 45 degrees to promote coughing and lung expansion.

A nurse is providing care to a client with superficial partial-thickness burns to the upper extremities. The client is in excruciating pain and is requesting analgesics frequently. The nurse understands that the client's response is due to which factor? A Absence of blood flow in the subcutaneous layer B Destruction of epithelial cells in the epidermis C Injury to bone below the muscle tissue D Presence of nerve endings in the dermis Correct Answer (Blank)

Rationale: Superficial partial-thickness burns extend to the dermal layer of the skin. The dermis contains nerve tissue that is responsible for eliciting pain. A superficial partial-thickness burn does not extend into the subcutaneous layer. Blood flow is still present. The client's pain is due to nerve-ending involvement. The epidermis primarily contains epithelial cells. Bone and muscle tissue extend beyond the subcutaneous layer. These structures are not affected in superficial partial-thickness burns.

The nurse is reviewing the cardiac rhythm strip of a client who reports the sudden onset of palpitations. The nurse should understand the client is experiencing which abnormal cardiac rhythm?

Rationale: Supraventricular tachycardia (SVT) is an abnormal cardiac rhythm that occurs when the cardiac impulse reenters from the AV node, resulting in a fast, regular rate. SVT will have a narrow QRS and no identifiable P waves on the rhythm strip. Sinus tachycardia will have a fast regular rate but identifiable P waves. Atrial fibrillation will have a fast, irregular rate with no identifiable P waves. Ventricular fibrillation will have no rate, no P waves, and no QRS.

A client is admitted to the cardiology unit for treatment for recurrent supraventricular tachycardia. Which observation by the nurse would best indicate that the client's condition can be considered hemodynamically stable? A The client's blood pressure is 88/40 mm Hg. B The client denies any chest pain and capillary refill is less than three seconds. Correct Answer (Blank) C The client's pulse oximeter reads 91% on three liters nasal cannula. D The client's cardiac monitor shows a heart rate of 170 beats per minute.

Rationale: Supraventricular tachycardia (SVT) is an arrhythmia that originates above the atrioventricular (AV) node. Clients with SVT can have a heart rate between 160-250 beats per minute (BPM). Causes of SVT include electrolyte imbalances, cardiac disease, hypoxia and medications. Clients with SVT typically present with palpitations, chest pain and shortness of breath. They can also develop cardiogenic shock if the rhythm goes untreated. Treatment for SVT includes vagal maneuvers, medications (i.e., adenosine) and synchronized cardioversion. Based on the client's blood pressure and heart rate, the client is hemodynamically unstable. Although the client's oxygenation status is within acceptable limits, it is not a good indicator of hemodynamic status. The absence of chest pain and good capillary refill indicate that the client is maintaining an adequate cardiac output and are, therefore, the best indicators that the client is hemodynamically stable.

The charge nurse is observing a newly hired nurse perform a sterile dressing change. Which of the following actions by the newly hired nurse requires intervention? A Opening the dressing supplies and dropping them into the center of the sterile field B Reaching across the sterile package to open the outer flap Correct Answer (Blank) C Using clean gloves to remove the old dressing D Cleansing the wound from the center toward the edges

Rationale: The charge nurse should intervene if the newly hired nurse is observed reaching across the sterile package to open the outer flap. To maintain a sterile field, the outer flap of the sterile package should be opened by reaching around the package and opening the flap away from the nurse. It is the correct sterile technique to drop the dressing supplies into the center of the sterile field, use clean gloves to remove the old dressing, and cleanse the wound from the center outwards (cleanest to dirtiest).

The nurse is caring for a 30-year-old female client scheduled for a hypophysectomy due to a pituitary tumor. The client asks how removal of the pituitary gland will affect her Which is the best response by the nurse? A "Because of the surgery, you will be at an increased risk for seizures." B "You will be immunocompromised and need to stay away from large crowds." C "You will be required to monitor your blood sugar levels and take insulin." D "After the procedure, you might have difficulties getting pregnant." Correct Answer (Blank)

Rationale: Surgical removal of the pituitary gland, or hypophysectomy, is a common treatment strategy for patients with pituitary tumors. Unfortunately, because the pituitary gland is responsible for fertility-related hormones such as luteinizing hormone (LH) and follicle-stimulating hormone (FSH), complications of this surgery include problems with fertility and the cessation of menses in women of childbearing age. Removal of the pituitary gland will not affect glucose metabolism and the immune system or create a risk for seizures.

Due to a recent rheumatic fever outbreak in the community, the school nurse is speaking to a group of parents and elementary school teachers. Which information is important for the nurse to emphasize? A Clumsiness and behavior changes should be reported. Correct Answer (Blank) B Most play activities will be restricted indefinitely. C Children may remain strep carriers for years. D Home schooling is preferred to classroom instruction.

Rationale: Sydenham chorea is a major sign of acute rheumatic fever; it may be the only sign of rheumatic fever in some clients. Symptoms include jerky, uncontrollable, and purposeless movements that look like twitches (these disappear during sleep), loss of fine motor control (causing changes in handwriting), and loss of emotional control (as evidenced by inappropriate crying or laughing). Sydenham chorea usually clears up in a few months, and no complications are expected.

The nurse is caring for a client admitted with suspected urosepsis who has tachycardia and hypotension. The client had a central venous catheter placed for fluid resuscitation and monitoring of treatment effectiveness. What assessment data indicate that the treatment was effective? A Decreased central venous pressure (CVP), increased heart rate (HR), and elevated mean arterial pressure (MAP) B Elevated central venous pressure (CVP), decreased heart rate (HR), and elevated mean arterial pressure (MAP) C Elevated central venous pressure (CVP), decreased heart rate (HR), and normal mean arterial pressure (MAP) Correct Answer (Blank) D Decreased central venous pressure (CVP), decreased heart rate (HR), and decreased mean arterial pressure (MAP)

Rationale: Symptoms of sepsis include fever or hypothermia, tachycardia (e.g., heart rate >90 BPM), hypotension, chills/shaking, confusion, tachypnea (e.g., >20 breaths/minute), decreased urine output, increased cardiac output, and signs and symptoms that reflect the primary site of infection dysuria in urinary tract infections. Improved survival from septic shock has been demonstrated when early goal-directed therapy (e.g., maintaining adequate urine output, MAP at ≥65 mm Hg, and central venous pressure [CVP] at 8-12 mm Hg) is provided. Therefore, a return to normal CVP, normocardia, and a MAP >65 all indicate successful fluid resuscitation. Elevated MAP is an indication of increased cardiac workload and possibly fluid volume excess. Clients with increased HR may be demonstrating signs of inadequate fluid resuscitation or heart failure from fluid overload.

The nurse is caring for a client who is undergoing synchronized cardioversion for stable ventricular tachycardia. Which of the following instructions should the nurse provide to the client? A "Pacing electrodes will be applied to your chest and you will feel muscle contractions with each heartbeat." B "You will receive a transdermal analgesic patch to ensure comfort during the procedure." C "The procedure will start with the lowest energy and may be increased as needed to restore a normal cardiac rhythm." Correct Answer (Blank) D "This procedure will eliminate the possibility of future dysrhythmias from occurring."

Rationale: Synchronized cardioversion for ventricular tachydysrhythmias involves a synchronized energy delivery to the client on the R wave of the QRS complex on the ECG. This energy is typically started at the lowest energy of 50 - 100 Joules and increased as needed with further energy delivery if a normal sinus rhythm is not restored. Clients are typically given intravenous sedation for comfort, and all transdermal patches should be removed prior to the procedure. The procedure may temporarily restore a normal sinus rhythm, however, future and recurrent dysrhythmias may occur, requiring a permanent implantable cardioverter/defibrillator.

The nurse is caring for a client who was admitted to the hospital for syndrome of inappropriate antidiuretic hormone. Which interventions are appropriate for this client's plan of care? Select all that apply. Increase the client's fluid intake to 5,000 milliliters per day. Monitor the client for pulmonary edema and orthopnea. Correct Answer (Blank) Document the client's weight daily. Correct Answer (Blank) Monitor the client's serum sodium level. Correct Answer (Blank) Administer five units of vasopressin every six hours, intramuscularly. Keep a padded tongue blade at the bedside. Document changes in the client's neurologic status. Correct Answer (Blank)

Rationale: Syndrome of inappropriate antidiuretic hormone (SIADH) is caused by excessive antidiuretic hormone leading to fluid volume overload and dilutional hyponatremia. Associated clinical manifestations are related to fluid overload (pulmonary edema, orthopnea and crackles/rales in the lungs) and dilutional hyponatremia (confusion, headache, decreased muscle strength and decreased neuromuscular excitability). Appropriate interventions include monitoring the response to treatment, preventing complications, maintaining a safe environment and reinforcing education regarding fluid restriction. Vasopressin, an antidiuretic analogue, is contraindicated in clients with SIADH. While this client is at increased risk for seizures, padded tongue blades are not appropriate seizure precautions. Monitoring and documenting sodium levels, daily weight, pulmonary status and neurologic status are appropriate for this client.

The nurse is caring for a client who has been diagnosed with syndrome of inappropriate antidiuretic hormone (SIADH). Which interventions are appropriate for this client? Select all that apply. Administration of a loop diuretic Correct Answer (Blank) Implementation of a fluid restriction Correct Answer (Blank) Administration of vasopressin Implementation of a low-sodium diet Monitoring of intake and output Correct Answer (Blank)

Rationale: Syndrome of inappropriate antidiuretic hormone (SIADH) is the result of excess antidiuretic hormone secretion, leading to fluid retention, fluid volume overload and dilutional hyponatremia. Appropriate interventions include closely monitoring intake and output and restricting fluids. Furthermore, administering a diuretic will promote diuresis and help get rid of excess fluid. Vasopressin is an analog of antidiuretic hormone (ADH) and would worsen the client's condition. Vasopressin is used with diabetes insipidus, not SIADH. The client should be eating foods high in sodium, not low, to help with the hyponatremia.

A client has received education from the nurse about their new diagnosis of systemic lupus erythematosus. Which statement by the client indicates that additional teaching is needed? A "I will avoid foods that contain high levels of vitamin K." Correct Answer (Blank) B "I may feel more tired and fatigued than I used to." C "I will monitor my body temperature carefully." D "I will protect my skin from the sun when I'm outside."

Rationale: Systemic lupus erythematosus (SLE) is a chronic, progressive, inflammatory connective tissue disorder that can cause major body organs and systems to fail. Clients with SLE should avoid prolonged sun exposure. The nurse should instruct clients to wear long sleeves and a large brimmed hat when outdoors. They should use sun blocking agents with a sun protection factor (SPF) of 30 or higher on exposed skin surfaces. It is expected for clients with SLE to experience fatigue, so they should allow time to rest when needed. Clients with SLE should monitor their body temperature carefully because this is typically the first sign of an exacerbation, during which the client can become critically ill. There is no established diet recommendation for clients with SLE, except to eat a well-balanced diet. Avoiding foods that contain vitamin K is not necessary, so this statement should be followed up on.

The nurse is reviewing the medical record of a client who has been diagnosed with systemic lupus erythematosus. The nurse would expect which findings associated with this disease? Select all that apply. A recent ten pound weight gain A red, raised rash on the face Correct Answer (Blank) Polydipsia for the last month Reports of pain in the hands and knees Correct Answer (Blank) A temperature of 100.6° F (38° C) Correct Answer (Blank) Generalized weakness Correct Answer (Blank)

Rationale: Systemic lupus erythematosus (SLE) is an autoimmune, inflammatory disorder of the connective tissue. It can affect multiple organs. This disorder has remission periods and flare-ups. A client who was recently diagnosed often presents during an exacerbation. Common assessment findings during exacerbation include a red, raised, rash on the face, commonly known as the "butterfly rash" and generalized weakness that can be associated with the fever and joint inflammation that are also present. SLE most frequently affects small joints (such as the hands) and the knees. Clients tend to experience anorexia which often leads to reports of weight loss, not weight gain. Polydipsia (excessive thirst) is not associated with SLE.

A 68-year-old, postmenopausal female client has been prescribed tamoxifen for breast cancer with bone metastases. The nurse should teach the client about which potential adverse drug effect? A Insomnia B Symptoms of hypocalcemia C Stroke-like symptoms Correct Answer (Blank) D Seizures

Rationale: Tamoxifen is an antineoplastic drug, commonly prescribed for clients with breast cancer or for clients who are at high risk for developing breast cancer. The most common adverse drug effects (ADEs) are hot flashes, fluid retention, vaginal discharge, nausea, vomiting and menstrual irregularities. In women with bone metastases, tamoxifen may cause transient hypercalcemia. Because of its estrogen agonist actions, tamoxifen poses a small risk of thromboembolic events, including deep vein thrombosis, pulmonary embolism and stroke. Insomnia and seizures are not known ADEs of tamoxifen.

A client with benign prostatic hypertrophy has been prescribed tamsulosin. Which statement by the nurse correctly describes how this medication works? A "This drug will eliminate your nocturia." B "Your libido will increase with this medication." C "This medication will shrink your enlarged prostate gland." D "This medication will improve the flow of urine." Correct Answer (Blank)

Rationale: Tamsulosin is an alpha-adrenergic blocker that is prescribed to promote bladder and prostate gland relaxation for clients with benign prostatic hypertropy or hyperplasia (BPH). Common clinical manifestations of BPH include urine obstruction, urinary retention, decrease urine flow, hesitancy and nocturia. Tamsulosin will relax the smooth muscle of the bladder neck and prostate, allowing urine to flow more easily and decreasing bladder neck contractions that can cause hesitancy. Tamsulosin does not shrink the prostate, nor does it increase libido or sexual desire. Finasteride, an androgen inhibitor also commonly prescribed for BPH, reduces the prostate size, thus helping to alleviate the urinary symptoms of BPH. Although tamsulosin may reduce episodes of having to void during the night (nocturia), it might not eliminate them.

The nurse is caring for a client whose cardiac monitoring has been complicated by frequent artifact. While at the nurses' station, the central monitor alarms indicate that the heart rate is outside of the alarm parameters. What is the correct action by the nurse? A Call the telemetry technician to verify the rhythm. B Go to the client's room to collect additional assessment data. Correct Answer (Blank) C Ask a second nurse to review the rhythm on the monitor. D Silence the central monitor alarm.

Rationale: Telemetry monitoring comes with various alarms, some signaling critical events that require clinical intervention. Telemetry can also trigger nuisance alarms, such as low battery, artifact, and improperly set limits, contributing to alarm fatigue in nurses. Alarm fatigue increases the risk of adverse patient outcomes. Alarms should be set appropriately, and artifacts mitigated to protect patient safety. The patient should be assessed first to determine if there is an actual cardiac event, then additional interventions can be implemented.

The nurse on the telemetry unit is caring for a client who has a history of uncontrolled atrial fibrillation. The nurse notes the client's heart rate has suddenly increased from 90 to 150 on the central monitor. What action should the nurse take? A Monitor the client at the bedside with the code cart defibrillator Correct Answer (Blank) B Have the telemetry technician notify the nurse of any changes C Prepare to transfer the client to the intensive care unit D Review the monitor history for the rhythm change

Rationale: Telemetry monitoring should be used for clients at high risk for life-threatening dysrhythmias. However, when a change is detected, the client should be transitioned to a bedside monitor. The code cart defibrillator provides monitoring and intervention as needed. Monitoring of the client should not be delegated in an unstable client situation. The client has a history of atrial fibrillation and may not require transfer to a higher level of care. Even so, other interventions take priority. Reviewing the cardiac history in the monitor is not the priority for this patient.

The pediatric emergency room nurse is triaging several children. Which of the following children is at highest risk for an adverse respiratory event? A A child with a congenital heart defect B A child found submerged under water Correct Answer (Blank) C A child with a fractured leg D A child with an acute febrile illness

Rationale: The child who was found submerged under water is at highest risk for an adverse respiratory event such as acute respiratory failure and respiratory arrest due to the likely aspiration of water into the lungs.

The nurse is evaluating comprehension of a client newly diagnosed with testicular cancer. Which statement by the client indicate an understanding of this type of cancer? A "I will probably never be able to have children after receiving chemotherapy." B "After surgery, I can have a prosthesis placed inside my scrotum." Correct Answer (Blank) C "If they find lymph node involvement, I am pretty much dead, aren't I?" D "I should have been better about using a condom during sexual intercourse."

Rationale: Testicular cancer is a rare cancer that most often affects men between 20 and 35 years of age. With early detection and treatment, testicular cancer has a 95% cure rate. It can occur in one testicle or both. Surgery is the main treatment for testicular cancer. For stage 0 or 1 (localized disease), a unilateral orchiectomy is usually performed. A gel-filled silicone prosthesis may be surgically implanted into the scrotum at the time of the orchiectomy or later if the client desires. If there are concerns about sterility, the client has the option of sperm storage. Sexual intercourse, with or without a condom, does not cause testicular cancer.

The nurse is teaching the parents of a 2-week-old infant with tetralogy of Fallot. Which finding should the nurse instruct the parents to immediately report to the health care provider? A Feeding problems B Changes in level of consciousness Correct Answer (Blank) C Poor weight gain D Fatigue with crying

Rationale: Tetralogy of Fallot (TOF) is a congenital heart defect that is characterized by four structural abnormalities: right ventricular hypertrophy, aortic displacement, pulmonary stenosis, and a ventricular septal defect. While parents should report any of these findings, they should immediately notify the health care provider or call 911 if the level of consciousness (LOC) decreases, or the infant becomes unresponsive. A decreased LOC indicates brain anoxia, which may lead to death, and is a medical emergency. The other findings (e.g., feeding problems, poor weight gain, and fatigue with crying) can indicate the development of heart failure in an infant.

The nurse is reassessing a client after resuscitative efforts. Which finding indicates the disability component of the ABCDE approach is intact? A Client has a GCS score of 15 Correct Answer (Blank) B Lung sounds are clear bilaterally C Capillary refill is 2 seconds D Client is responsive to pain

Rationale: The ABCDE approach is a rapid assessment of emergency conditions. The disability component represents the "D" in ABCDE. Disability determines the client's level of consciousness. The Glasgow Coma Scale (GCS) is based on eye-opening, verbal, and motor responses. The normal finding is a score of 15. Clear lung sounds evaluate breathing, the "B" in the ABCDE approach. Capillary refill of 2 seconds is a normal finding for circulation, the "C" in the ABCDE approach. A client who is responsive to pain does not have an intact level of consciousness.

The automated external defibrillator (AED) has been applied to a client receiving cardiopulmonary resuscitation. Indicate how the nurse will proceed by placing the following actions in the correct order. 1. Allow time for the AED to administer a shock. 2. Press the shock button on the AED. 3. Wait for the AED to analyze the client's heart rhythm. 4. Immediately resume CPR. 5. Press the analyze button when the AED prompts the nurse to do so. 6. Call out "stand clear" when the AED prompts the nurse to administer a shock. A 5, 3, 6, 2, 1, 4 Correct Answer (Blank) B 3, 5, 2, 6, 1, 4 C 6, 5, 3, 2, 1, 4 D 3, 5, 2, 6, 1, 4

Rationale: The American Heath Association (AHA) guidelines for CPR recommend rapid CPR implementation including the use of an AED. AEDs provide early interpretation of the client's cardiac rhythm. It provides step-by-step instructions on how to proceed with defibrillation if indicated.

The nurse is monitoring the level of consciousness for a client who experienced a head injury. During the last assessment, the client scored a 15 on the Glasgow Coma Scale (GCS). Now, the client opens eyes to verbal command (GCS 3), has purposeful movement to painful stimulus (GCS 5) and is using inappropriate words (GCS 3). Which intervention by the nurse should be implemented first? A Raise the head of the bed B Call the rapid response team and health care provider Correct Answer (Blank) C Continue to monitor level of consciousness D Increase the flow of oxygen

Rationale: The GCS measures the client's highest motor response, verbal response, and eye response with scores ranging from 3 to 15. The GCS can be used to monitor progress and predict a client's outcome or prognosis. In the last assessment, this client was scored a 15 on the GCS, which indicates the baseline. Upon reassessment, the client's responses have decreased indicating a worsened neurological state. This requires urgent intervention and the rapid response team and health care provider should be notified. If the nurse continues to monitor the level of consciousness without notifying the HCP and the rapid response team, the client's condition could worsen. It is possible the change is related to increased intracranial pressure (ICP), but this needs to be determined before the other actions are taken.

The nurse is assessing a client diagnosed with chronic obstructive pulmonary disease (COPD). The client is on oxygen for low PaO2 levels. Which assessment is a nursing priority to evaluate the outcome of this therapy? A Frequently assess coughing and sputum characteristics B Frequently observe for skin color changes C Frequently assess lung sounds D Frequently evaluate oxygen saturation (SaO2) levels Correct Answer (Blank)

Rationale: The best method to evaluate a client's oxygenation is to evaluate the SaO2. The oxygen saturation should be around 88% to 91% for someone with COPD. This method is equally as effective as an arterial blood gas reading to evaluate oxygenation status and is less traumatic and expensive. Assessment of lung sounds, coughing and sputum, and color should also be components of the respiratory assessment for a client with COPD but are less precise indicators of the response to oxygen therapy than the oxygen saturation level.

The charge nurse is observing a newly hired nurse irrigate a client's stage 4 pressure injury. Which of the following actions by the newly hired nurse requires intervention? A Applying clean gloves and discarding the old dressing B Cleansing the wound from the outer edges toward the center Correct Answer (Blank) C Irrigating the wound until the solution becomes clear D Inserting the irrigation catheter into the wound until resistance is met

Rationale: The charge nurse should follow up if the newly hired nurse is cleansing the wound from the outer edge towards the center. Wounds should be cleansed from the cleanest area towards the least clean. If the wound is circular, this would be from the center of the wound outward. It indicates the correct technique for irrigating a wound if the newly hired nurse applies clean gloves while discarding the old dressing, irrigates the wound until the solution runs clear and is free from exudate, and only inserts the irrigating catheter into a wound until resistance is met to avoid tissue damage.

The nurse is admitting a 3-year-old child with manifestations of sudden onset of irritability, croaking on inspiration, and skin temperature that's hot to the touch. The child is currently leaning forward with suprasternal retractions, a protruded tongue, and excessive drooling. What should the nurse do first? A Collect a sputum specimen for culture B Prepare the child for an x-ray of the upper airways C Notify the health care provider of the child's status Correct Answer (Blank) D Examine the child's throat for redness

Rationale: The child's manifestations suggest epiglottitis, which is the inflammation of the epiglottis. Although rare, the condition is more commonly found in children and is usually caused by Haemophilus influenza B. Epiglottitis can lead to airway obstruction and thus is considered a medical emergency. Manifestations of epiglottitis include dysphagia, difficulty talking, apprehension, retractions, stridor on inspiration, and an elevated temperature. The nurse would first want to notify the health care provider. The child's condition is worsening as indicated by them leaning forward and having suprasternal retractions. Although a sputum specimen may be warranted in this situation, it is not the first action that should be implemented. In a client with epiglottitis, one should never insert anything into the client's mouth or throat, as this could lead to a spasm of the airway. Radiographic films would not be indicated in the diagnosis of epiglottitis.

The nurse is assessing a client who was admitted to the hospital with a diagnosis of right-sided heart failure. Which assessment findings should the nurse expect? Select all that apply. Dependent edema Correct Answer (Blank) Anorexia Correct Answer (Blank) Orthopnea Cough Polyuria Ascites Correct Answer (Blank)

Rationale: The classic findings of right-sided heart failure arise from blood backing up into the portal and systemic circulation, resulting in abdominal organ engorgement, ascites, loss of appetite (anorexia), and dependent edema. Orthopnea and cough are more commonly seen with left-sided heart failure. Polyuria is not a manifestation of right-sided heart failure.

The nurse responds to the room of a client who experienced a clonic seizure. The client is observed on the floor and is awake. What is the nurse's priority action to ensure the safety of the client? A Take the client's blood pressure. B Assess the client's pupils. Correct Answer (Blank) C Call the client's healthcare provider. D Assist the client back into bed.

Rationale: The client fell from a standing position before the seizure activity. Falls to the ground can result in head injuries. The nurse should assess the client's neurological status before performing other interventions. Taking the blood pressure is an important assessment. However, a fall can result in head injuries that must be assessed first. Calling the healthcare provider is an appropriate intervention. However, the nurse should obtain an accurate assessment before reporting the findings. Assisting the client back into bed should occur after the nurse ensures the client is stable.

The nurse has received report on a group of assigned clients who are receiving prescribed peritoneal dialysis. Which client should the nurse see first? A The client who is reporting abdominal pain and has a rigid, board-like abdomen Correct Answer (Blank) B The client who has a serum creatinine level of 1.8 mg/dL and is reporting the inability to void C The client who had a peritoneal dialysis catheter placed 1 day ago and is reporting pink-tinged dialysate outflow D The client who has a peritoneal dialysis catheter infection and has a temperature of 102.1°F

Rationale: The client in the key is exhibiting signs and symptoms of peritonitis, which is a life-threatening complication in the client undergoing peritoneal dialysis and should be assessed first. The client with a creatinine level of 1.8 mg/dL is exhibiting expected manifestations of chronic kidney disease. The client who has pink-tinged dialysate outflow is exhibiting expected findings following the placement of a catheter 1 day ago. Clients can expect pink-tinged and bloody dialysate outflow for 1 week following catheter placement. The client with a peritoneal dialysis catheter infection is exhibiting expected fever and can be seen after the client in the key.

A client is transported to the emergency department following a boating accident and submersion in cold water. The client is conscious, shivering and confused. What interventions should the nurse implement? Select all that apply. Provide warmed blankets Correct Answer (Blank) Massage extremities Remove wet clothing Correct Answer (Blank) Administer warmed IV fluids as ordered Correct Answer (Blank)

Rationale: The client is at risk for hypothermia. The nurse should remove wet clothing carefully. External rewarming, such as warmed blankets or heat packs, which are placed under the arms and on the neck, chest and groin. The client may also be ordered to receive warmed IV fluids and humidified oxygen to help stabilize the core temperature. Monitoring should include vital signs, level of consciousness, cardiac rhythm and core body temperature. The client should not receive any oral fluids until their condition is stabilized and extremities should not be massaged.

A postoperative client following a thyroidectomy suddenly develops difficulty breathing, stridor and an increase in swelling of the anterior neck area. What should the nurse do first? A Activate the hospital's emergency or rapid response system. Correct Answer (Blank) B Ask the charge nurse to come see the client immediately. C Place a heart monitor on the client and observe for dysrhythmias. D Check the client's blood pressure and heart rate.

Rationale: The client is demonstrating clinical manifestations of an airway obstruction related to bleeding and/or swelling following the thyroidectomy. This is a life-threatening, medical emergency and the nurse's first action should be to activate the hospital's emergency or rapid response system. It is possible that the client will need an emergency surgical airway intervention, such as a tracheostomy, to maintain a patent airway.

The nurse is caring for a client who is recovering from a right total hip arthroplasty. The client reports a sudden onset of chest pain and difficulty breathing. What action should the nurse take first? A Elevate the head of the bed Correct Answer (Blank) B Obtain the client's vital signs C Auscultate the client's lung fields D Notify the health care provider

Rationale: The client is exhibiting clinical manifestations of a pulmonary embolism (PE). A PE is a medical emergency, which requires immediate action from the nurse. Deep vein thromboses (DVTs) can occur after a total hip arthroplasty due to immobilization during and after surgery. By elevating the head of the bed, the nurse will decrease dyspnea associated with the PE. The nurse should then assess the client and report the clinical manifestations to the HCP.

A client with a spinal cord injury at the T-2 level reports having a "pounding" headache. Further assessment by the nurse reveals excessive sweating, rash, piloerection, facial flushing, congested nasal passages and a heart rate of 50 bpm. What is the priority action? A Transfer the client to the bed and administer the ordered PRN analgesic. B Measure the client's respirations, blood pressure, temperature and pupillary response. C Assist the client with relaxation techniques. D Check the client for bladder distention or kinking of the urinary catheter. Correct Answer (Blank)

Rationale: The client is exhibiting manifestations of autonomic dysreflexia, also called hyperreflexia, seen with a spinal cord injury (SCI), typically above the T6 level. It is most often caused by a noxious stimulus below the level of the injury such as a full bladder, an enema or bowel movement, fecal impaction, changing of an indwelling catheter and a vaginal or rectal examination. The stimulus creates an exaggerated response of the sympathetic nervous system that can be a life-threatening event. Therefore, the priority action is to identify and relieve the cause of the response.

The nurse is caring for a client who had abdominal surgery 2 days ago and develops a temperature of 102.1°F and purulent drainage from the surgical incision. Which of the following prescriptions from the primary healthcare provider (PCHP) should the nurse implement first? A Infuse Cefazolin, 1 G, I.V. every 6 hours. B Obtain wound culture and sensitivity (C&S). Correct Answer (Blank) C Irrigate wound with normal saline every 8 hours. D Administer acetaminophen 650mg P.O. every 6 hours for temperature greater than 100.4° F.

Rationale: The client is exhibiting signs of a post-surgical wound infection which requires prompt treatment with appropriate antimicrobial therapy. The first nursing action should be to obtain the wound culture prior to initiation of the prescribed broad-spectrum antimicrobial therapy to isolate the microorganism for further targeted antimicrobial therapy. After obtaining the wound culture, the nurse can then administer the prescribed Cefazolin, followed by antipyretics, then cleansing the wound.

The nurse is talking with a client during a home health visit. The client states, "my right arm and right leg are beginning to feel heavy." The nurse notices the client is having trouble speaking and has stopped moving the right side of their face. What action the nurse should take first? A Call 911. Correct Answer (Blank) B Ask the client if they have a headache. C Document the onset of symptoms in the medical record. D Take the client's vital signs.

Rationale: The client is exhibiting signs of an acute stroke. A stroke is caused by a disruption in the normal blood supply to the brain. A stroke is a medical emergency. The nurse in the home health setting should call 911 first. While waiting for emergency medical help to arrive, the nurse should gather additional data by obtaining vital signs and evaluating the client's neurological status. The data should be recorded in the medical record.

A nurse working in a nursing home is caring for an 80-year-old client with diabetes mellitus type 2. The nurse notes that the client is exhibiting new confusion, polyuria, an elevated temperature and hypotension. What is the most likely cause of the client's symptoms? A Insulin shock B Hyperosmolar hyperglycemic state Correct Answer (Blank) C Acute kidney failure D Stroke

Rationale: The client is exhibiting signs of hyperglycemic-hyperosmolar state (HHS). HHS is a hyperosmolar (increased blood osmolarity) state caused by hyperglycemia. HHS occurs most often in older clients with type 2 diabetes mellitus (DM). HHS results from a sustained osmotic diuresis. As serum concentrations of glucose exceed the renal threshold, the kidney's capacity to reabsorb glucose is exceeded, resulting in polyuria. The osmotic diuresis/polyuria will lead to decreased blood volume, dehydration, hypotension, shock and death if not caught early and treated aggressively. The other answers are incorrect.

A nurse is assessing a client with a traumatic brain injury who is unconscious. The nurse notes pinpoint, nonreactive pupils and flaccid extremities. Which action does the nurse perform next? A Administers a prescribed hypotonic solution B Elevates the head of the bed Correct Answer (Blank) C Prepares the client for surgery D Places the client on seizure precautions

Rationale: The client is exhibiting signs of increased intracranial pressure (ICP). Elevating the head of the bed helps decrease ICP. Clients with ICP will be administered hypertonic, not hypotonic, solutions to decrease fluid and pressure. The client may require surgery to alleviate cranial pressure. However, the first intervention is to prevent further increase in pressure by elevating the head of the bed. All clients with traumatic head injuries should be on seizure precautions for safety. Placing the client on seizure precautions will not decrease ICP.

The nurse on a telemetry unit is assessing orthostatic vital signs on a client with cardiomyopathy. The client's systolic blood pressure decreased from 145 to 110 mmHg between the supine and upright positions. The client's heart rate increased from 72 to 96 bpm during that time. The client reports feeling lightheaded when standing up. Which action should the nurse implement? A Increase the client's PO fluid intake for the next 2 hours. Correct Answer (Blank) B Instruct the client to empty their bladder and reassess their BP. C Instruct the client to follow a high protein diet. D Restrict the client's PO fluids for the next 4-6 hours.

Rationale: The client is experiencing postural hypotension. Postural hypotension is a decrease in systolic blood pressure of at least 15 mmHg, accompanied by an increase in heart rate of 15 to 20 beats above the baseline with a change of position from supine to upright. This is often accompanied by lightheadedness. Fluid replacement is appropriate, and must be instituted very cautiously. The client with cardiomyopathy will also be sensitive to changes in fluid status and fluid overload may develop rapidly with aggressive rehydration. After the client increases fluid intake for one to two hours, the client should be reassessed for resolution of the postural hypotension.

The nurse is caring for a client who experienced blunt chest trauma and has been prescribed oxygen and frequent assessments. Upon assessment, there is a new tracheal shift to the right, absence of lung sounds on the left side, dullness on percussion of the left chest, and HR 135 and BP 72/40. The nurse suspects the patient has developed which of the following complications? A Cardiac tamponade B Massive hemothorax Correct Answer (Blank) C Flail chest D Pleural effusion

Rationale: The client is likely experiencing a hemothorax. Tracheal deviation to the right, away from the side where the lung sounds are absent, indicates that something is filling the space (i.e. blood). The lack of breath sounds does not help distinguish between hemothorax and pneumothorax as both would result in absent breath sounds. Dullness on percussion indicates fluid in the space. Pleural effusions, which is a collection of fluid in the pleural space, result in muffled breath sounds. Flail chest occurs when sections of ribs are fractured, which causes a paradoxical chest movement.

A nurse prepares to insert an 18-gauge intravenous catheter into a client with hypovolemic shock. The client asks the nurse why this is necessary. What is the nurse's best response? A "We need to administer vasopressors immediately." B "You require antibiotics to treat your condition." C "Emergency medications cannot be administered through a smaller catheter." D "Your fluid volume needs to be replaced." Correct Answer (Blank)

Rationale: The client requires an intravenous catheter for fluid replacement. Hypovolemic shock is caused by a 15 to 30% decrease in intravascular volume. The first line of treatment is to replace fluid volume. Vasopressors should be administered after fluid volume is replaced and only if the blood pressure is not maintained. Antibiotics are administered for septic, not hypovolemic, shock. Although a large gauge is preferred, emergency medications can be administered through a smaller gauge if needed.

The visiting nurse is evaluating an ambulatory client who reports weight gain and increased swelling in their feet and ankles during the day that disappears while they sleep. The client has a history of emphysema. Which follow-up question would be most appropriate? A "Did you start a new exercise regimen recently?" B "Do you use any tobacco products?" C "Have you made any significant changes to your usual diet?" D "Do you have any shortness of breath with activities?" Correct Answer (Blank)

Rationale: The client seems to be exhibiting signs and symptoms of heart failure (HF); in particular, right-sided HF. To gather further information, the most appropriate follow-up question from the nurse would be to inquire about any shortness of breath on exertion. Other signs and symptoms of HF include fatigue, weakness, swelling of the feet and ankles, ascites, bounding pulses and rapid or irregular heartbeat. The other questions do not relate to the symptoms the client is reporting.

The nurse is caring for a client who has increased intracranial pressure. Which of the following medications should the nurse question? A Propofol 20 mcg/kg/min B Dobutamine 10 mcg/kg/min C 0.5% sodium chloride at 125 mL/hour Correct Answer (Blank) D Mannitol 135 grams over 60 minutes

Rationale: The client should question 0.5% sodium chloride, as this would increase fluid volume and ultimately increase intracranial pressure. Propofol can be used for sedation for the client with increased ICP, dobutamine is used to improve cardiac output, and mannitol is used to decrease extracellular fluid volume.

A nurse is caring for a client after a spinal fusion to treat scoliosis. Which nursing intervention is appropriate in the immediate postoperative period? Select all that apply. Encourage passive leg and ankle exercises Correct Answer (Blank) Perform neurovascular checks every 8 hours Encourage use of patient-controlled analgesia Correct Answer (Blank) Maintain bedrest with the head of the bed elevated at least 30 degrees Assist the client to stand and walk to the bathroom as needed Position the client flat in bed and logroll every 2 to 4 hours Correct Answer (Blank)

Rationale: The client should remain flat in bed for at least 6 hours and turn from side to side every 2 to 4 hours. The day after surgery, the client can sit up in bed a few times; the client will get out of bed to sit in a chair on the second or third day after surgery. Clients should be encouraged to perform isometric exercises right after surgery. Neuro checks will be performed every 2 hours for the first 24 hours.

A client with possible Hepatitis C discusses his health history with the nurse. The nurse should recognize which statement by the client as the most important in supporting this diagnosis? A "I ate the best raw oysters last week." B "I have had unprotected sexual contact with at least one person." C "I had a blood transfusion in 1990." Correct Answer (Blank) D "I got back from Africa a few weeks ago."

Rationale: The client who received a blood transfusion prior to screening for Hepatitis C (prior to July 1992) may show findings many years later due to Hepatitis C being asymptomatic in the early stages. Other risk factors for Hepatitis C include those who have been on long-term hemodialysis and have regular contact with blood at work. Contracting Hepatitis C from having unprotected sex with a person who has Hepatitis C is rare. However, unprotected sex with multiple partners does increase the risk. Eating raw oysters or drinking contaminated water would increase the risk of Hepatitis A. Travel to Africa would increase the risk of exposure to malaria from mosquitos carrying the disease as well as HIV if the person were exposed to blood carrying the infection or had unprotected sex with someone who was HIV positive.

A nurse is providing information to a client who is newly diagnosed with tuberculosis (TB). The nurse should be sure to include which statement when teaching the client about managing this disease? A "Follow up with your primary care provider in three months." B "Continue to take your medications even when you are feeling fine." Correct Answer (Blank) C "Isolate yourself from others until you are finished taking your medication." D "Continue to get yearly tuberculin skin tests."

Rationale: The client with TB needs is to understand the importance of medication compliance, even when the client is no longer having any symptoms. TB treatment usually requires a combination of medications with treatment for at least six months. Stopping treatment or skipping doses can lead to a drug-resistant form of TB. Clients are most infectious early in the course of therapy but the numbers of acid-fast bacilli are greatly reduced as soon as two weeks after therapy begins. Once clients no longer have a productive cough, they are not considered contagious.

The nurse in the outpatient clinic is following up on a client with a fractured arm. The client's arm was placed in a cast four hours ago. The client states, "my fingers are tingling and feel cold." Which action should the nurse take first? A Notify the health care provider B Elevate the client's arm above the level of the heart C Apply an ice pack to the cast to reduce swelling D Check the capillary refill in the client's fingers Correct Answer (Blank)

Rationale: The client with a cast on an extremity is at risk for the development of compartment syndrome. Compartment syndrome occurs when the swelling underneath the cast becomes so great that it will decrease circulation and tissue perfusion to the extremity distal to the cast. This is a medical emergency. Using the nursing process, the nurse should first collect more data by checking the client's capillary refill, which can support the possibility of compartment syndrome. After obtaining the additional information, the nurse can make the best decision about what to do next.

The nurse is admitting a 73-year-old client who has a fractured right hip. Which interventions should the nurse include in the client's plan of care? Select all that apply. Place the client on continuous pulse oximetry. Correct Answer (Blank) Palpate the client's bilateral pedal pulses every four hours. Correct Answer (Blank) Perform daily circulation, motion and sensation checks on the client's right leg. Ask about the client's pain level with every set of vital signs. Correct Answer (Blank) Reposition the client every hour to prevent skin breakdown.

Rationale: The client with a hip fracture is at risk for impaired perfusion to the affected extremity. Monitoring bilateral pedal pulses allows the nurse to compare the pulse strength in the injured site with that in the non-injured site. A decrease in the injured leg could signal a decrease in circulation that would require immediate intervention. A fat embolism is also a risk with a hip fracture and continuous pulse oximetry would allow the nurse to identify hypoxia quickly which could be associated with a fat embolism. Clients with a hip fracture usually experience great pain and assessing pain with each set of vital signs is key to effective pain management. Circulation, motion and sensation checks should be completed at least every four hours, not daily. Repositioning the client every hour is unnecessary and will only increase the client's pain level even more

The school nurse is performing an assessment on a 15-year-old client who sustained a mild traumatic brain injury without loss of consciousness during a football game, one week earlier. The nurse suspects post-concussion syndrome. Which findings would support this diagnosis? Select all that apply. Delayed pupillary response Short-term memory loss Correct Answer (Blank) Nausea and vomiting Insomnia Correct Answer (Blank) Learning difficulties Correct Answer (Blank) Positive Romberg sign

Rationale: The client with a traumatic brain injury (TBI) may have a variety of signs and symptoms depending on the severity of injury and the resulting increase in intracranial pressure (ICP). A mild TBI is characterized by a blow to the head with transient confusion or feeling dazed or disoriented and loss of memory of events immediately before or after the accident. Symptoms of a mild brain injury, often called a concussion, can include disturbances in sleep; affect, enjoyment of daily activities, work performance, mood, memory, and ability to learn new material; and can cause changes in personality. Nausea & vomiting, a slowed pupillary response and positive Romberg sign are not typically seen with a mild TBI, i.e., concussion; they tend to be seen with cerebellar involvement or a more severe TBI that involves an increased ICP.

A nurse is teaching a client with hypertension about safely choosing over-the-counter (OTC) medication. Which statement by the client indicates that the education has been effective? A "I should never take any over-the-counter (OTC) medicine without first calling and checking with the doctor's office." B "I should always ask the pharmacist about how to take an over-the-counter (OTC) medicine. " C "I will read all the directions on the label and ask the pharmacist if the directions are not clear. " Correct Answer (Blank) D "Medicines that are available over-the-counter (OTC) are really safe or they would be prescription medicines. "

Rationale: The client with hypertension should be taught to always read the labels on all over-the-counter medications. Clients should look for warnings to those with high blood pressure (BP) and to those who take blood pressure medications. Many OTC drugs and supplements can raise BP or interact with BP medications. Be aware of over-the-counter cold and flu preparations that contain decongestants, such as oxymetazoline, phenylephrine, and pseudoephedrine as they will raise the BP. Other OTC drugs and substances that can raise blood pressure include alcohol, caffeine, and non-steroidal anti-inflammatory drugs ( i.e., ibuprofen and naproxen sodium). Many herbs and foods can interact with BP medications as well.

The nurse is working with a client who is diagnosed with multiple sclerosis on how to reduce muscle spasticity. Which statement by the client indicates the need for further teaching? A "If I exercise daily, it can help with relieving spasticity." B "Taking a long hot bath may relieve the muscle spasms." Correct Answer (Blank) C "My stretching routine can help with the muscle spasms." D "Taking the prescribed muscle relaxant will relieve painful spasms."

Rationale: The client with multiple sclerosis (MS) should not use hot water for a bath because of the risk of thermal injury (e.g., burns) due to sensory deficits with MS. Instead, warm compresses may be used to relieve muscle spasms. The other actions can help with muscle spasms or spasticity and are appropriate for the client to use.

The nurse is evaluating a client admitted for exacerbation of chronic obstructive pulmonary disease (COPD). The client is receiving 2 liters of oxygen per nasal cannula and reports persistent dyspnea. Arterial blood gas results show a PaO2 65, pH 7.38, PaCO2 50, HCO3 28. Which action should the nurse take next? A Administer a bronchodilator B Increase the oxygen flow rate Correct Answer (Blank) C Encourage the use of incentive spirometry D Prepare the client for intubation

Rationale: The client's ABG results show hypoxemia and an expected respiratory acidosis. The likelihood of decreasing the respiratory drive from higher oxygen administration in clients with COPD is low and does not outweigh the potentially serious consequences of untreated hypoxemia. Therefore, the nurse should next increase the oxygen flow rate until an acceptable oxygen saturation level is reached. The other actions are premature at this time or do not address the issue of the hypoxemia.

A client in the emergency department reports severe abdominal pain, referred shoulder pain, and vaginal bleeding. An ultrasound confirms an ectopic pregnancy. Which intervention does the nurse expect to perform next? A Prepare the client for surgery. Correct Answer (Blank) B Educate the client on maintaining bedrest. C Provide emotional care and support. D Administer prescribed analgesic.

Rationale: The client's signs and symptoms are indicative of ectopic pregnancy with tubal rupture. Medical management includes surgery to remove the fallopian tube and control the bleeding. The nurse should prepare the client for surgery. Educating the client on maintaining bedrest is not indicated at this time. Providing emotional care and support is an important intervention after the client has been stabilized. Administering an analgesic will relieve the pain but is not a priority intervention at this time.

The nurse is evaluating self-management of a client who has type 1 diabetes. Which statement made by the client should be of highest concern? A "I had a penny in my shoe all day last week, and I didn't even realize it until I took my shoes off!" Correct Answer (Blank) B "Here are my glucose test readings that I wrote on my calendar. I check my blood sugar twice a day." C "I count the number of carbohydrates I eat. I eat several servings of fresh fruit per day." D "I give my insulin to myself in my thighs and belly. I make sure to alternate the sites."

Rationale: The client's statement about having a penny in their shoe without realizing it, indicates this client may have peripheral neuropathy. Peripheral neuropathy can lead to lack of sensation in the lower extremities. When clients cannot feel potential tissue injuries (something in their shoe), they are at high risk for impaired skin integrity such as diabetic foot ulcers. The other statements indicate that the client is managing their diabetes appropriately.

The nurse in the outpatient clinic is admitting a client scheduled for a prostatectomy this morning. Which statement by the client should be of greatest concern to the nurse? A "I have had an allergic reaction to an antibiotic before." B "I have not had to urinate since yesterday evening." Correct Answer (Blank) C "I have not had anything to eat since 9:00 pm last night." D "I am feeling nervous about the procedure."

Rationale: The client's statement about not having urinated in over 12 hours should be the greatest concern to the nurse. Urinary retention is a complication of an enlarged prostate gland and the nurse will need to further evaluate the client. The client may require a bladder ultrasound (bladder scan) to determine the amount of urine retained and the insertion of a catheter to drain the bladder may be indicated. The health care provider should also be notified. The nurse will then follow up on the client's other statements.

The nurse is teaching a client about prevention of recurrent kidney stones. Which statement by the client would indicate that additional teaching is needed? A "I will make sure I drink plenty of water throughout the day." B "I will contact my health care provider if I am having difficulty urinating." C "I will monitor the color of my urine." D "I will follow a low-calcium diet and avoid dairy products." Correct Answer (Blank)

Rationale: The client's statement regarding a low-calcium diet and avoiding dairy products would require further teaching from the nurse. Low-calcium diets are not generally recommended as this can lead to osteoporosis. Clients should be drinking fluids, preferably water, at least every 1 to 2 hours throughout the day. This can flush the system and prevent the occurrence of kidney stones. Clinical manifestations of kidney stones include pain, infection and difficulty with urination. Clients should notify their health care providers at the first sign of a urinary tract infection, as this can be caused by a kidney stone obstructing the flow of urine.

The nurse is caring for a client who suddenly develops slurred speech and a facial droop. What diagnostic test would the nurse expect to be performed first? A Computerized tomography scan Correct Answer (Blank) B Chest X-ray C Echocardiogram D Arterial blood gas

Rationale: The client's symptoms are indicative of an acute stroke. The nurse would anticipate that a non-contrast computerized tomography (CT) of the head will be done first because time is of the essence with an acute stroke. The other tests may or may not be indicated for this client.

The nurse is caring for a client with a history of drug abuse. The client informs the nurse their last drug use was 6 weeks ago and are now experiencing abdominal pain and clay-colored stools. Which statement would be appropriate for the nurse to make? A "Have you noticed dark urine since you took the drugs?" Correct Answer (Blank) B "Did you experience any headaches after you took the drugs?" C "What kind of medications are you taking for your abdominal pain?" D "Do you take any medications that change the color of your stool?"

Rationale: The client's symptoms are indicative of viral hepatitis infection. Drug use is a primary risk factor for hepatitis C. Hepatitis C is transmitted via injection-drug use and has an incubation period of 2-12 weeks. Dark urine is a symptom of a newly acquired hepatitis C infection. Headache is a non-specific symptom unrelated to an acute hepatitis C infection. Asking what medications are being taken for the abdominal pain does not help identify the diagnosis. Obtaining information regarding medications that change stool color is not enough to identify the client's problem. The nurse should focus on the timeframe of drug use and possible viral exposure.

The nurse is assigned to care for a client with end-stage liver failure and portal hypertension. Which clinical manifestations would the nurse expect to see with these conditions? Select all that apply. Shortness of breath Correct Answer (Blank) Diminished pedal pulses Increased abdominal girth Correct Answer (Blank) Elevated serum albumin level Increased weight gain Correct Answer (Blank)

Rationale: The clinical manifestations, common with end-stage liver disease, include yellowing of the skin (jaundice), ascites, dependent edema, bleeding and loss of appetite. The client will have an increased abdominal girth due to the ascites and weight gain related to fluid retention. With ascites, the client may experience shortness of breath. Having diminished pedal pulses is not a common with liver failure. Clients with liver failure will have decreased, not elevated, serum albumin levels.

The nurse is caring for a client on peritoneal dialysis. While performing a dialysate exchange, which finding(s) would alert the nurse that the client has developed an acute complication? A Respiration rate of 30 with crackles throughout the lung fields Correct Answer (Blank) B Catheter dressing saturated with clear fluid C Client sleeps throughout fluid exchange D Pulse 86 and blood pressure 112/74

Rationale: The development of an increased respiratory rate with crackles bilaterally indicates fluid overload, which is an acute complication of peritoneal dialysis. The vital signs are normal. Sleeping throughout the fluid exchange is normal and indicates the client is comfortable. Clear fluid on the dressing around the catheter indicates leakage of the dialysate fluid and can be controlled by instilling less fluid with each exchange.

A client reports the experience of a sudden deep and throbbing pain in one leg. What is the appropriate action to be taken by the nurse? A Ambulate for several minutes B Maintain the client on bed rest Correct Answer (Blank) C Suggest isometric exercises D Apply ice to the extremity

Rationale: The finding suggests deep vein thrombosis (DVT). The client must be maintained on bed rest, and the health care provider should be notified urgently. Deep vein thrombosis can lead to pulmonary embolism, which is a medical emergency that can cause severe problems with gas exchange and cardiac output and can even cause cardiac arrest. Anticoagulants are used to treat DVT, initially being administered by IV (heparin drip) or subcutaneous injection (low-molecular-weight heparin). This is then followed by long-term oral anticoagulation with warfarin.

The nurse is caring for a client following a right lower lung lobectomy. During the assessment of the chest drainage unit, the nurse notices bubbling in the water-seal chamber. What is the first action the nurse should take? A Position the client in a supine position. B Call the primary health care provider immediately. C Check for any increase in the amount of drainage. D Assess the chest tube dressing, tubing and drainage system. Correct Answer (Blank)

Rationale: The first action the nurse should take is to thoroughly check the dressing, tubing and drainage system. Intermittent bubbling in the water seal chamber right after surgery usually indicates an air leak from the pleural space. This is a common finding and should resolve as the lung re-expands. Continuous bubbling usually means a leak in the chest drainage unit such as a loose connection or a leak around the insertion site. Other nursing actions will include assessing the color and amount of the drainage and auscultating the lungs. After the initial post-operative period, the nurse will assist the client to change positions, cough and deep breathe to help re-expand the lung and promote fluid drainage.

The nurse walks into a client's room and finds the client lying on the floor. What should the nurse do next? A Call for help and activate the code team B Assess if the client's airway is patent C Determine if anyone witnessed the client fall D Establish if the client is unresponsive Correct Answer (Blank)

Rationale: The first step in cardiopulmonary resuscitation (CPR) is to establish the client's responsiveness. The nurse would then call for help, activate the code team, and check the client's pulse. A pulse check should occur for at least five seconds but no longer than ten seconds. If the client has no pulse, the nurse should immediately start chest compressions. Once the first thirty chest compressions have been completed, the nurse should then open the client's airway and perform two rescue breaths.

The nurse is reviewing the electronic medical record of a client diagnosed with endometriosis. The nurse should expect which findings with this diagnosis? Select all that apply. Dyspareunia Correct Answer (Blank) Amenorrhea Infertility Correct Answer (Blank) Urinary tract infection Dysmenorrhea Correct Answer (Blank)

Rationale: The following findings that would indicate the client has endometriosis are pain with menstruation (dysmenorrhea), pain with intercourse (dyspareunia), excessive bleeding, and infertility. The client may also complain of pelvic and/or back pain, along with pain during bowel movements. The endometrial tissue that implants outside the uterus may cause mild to severe pain, fluctuations in menstrual cycles and fibroids that can cause infertility. Endometriosis often times is mistaken for pelvic inflammatory disease (PID), which causes inflammation of the pelvis, irritable bowel syndrome (IBS) or ovarian cysts. A urinary tract infection and amenorrhea (absence of menstruation) are not usually seen with endometriosis.

The nurse is admitting a male client who is newly diagnosed with a frontal lobe brain tumor. Which statement by the client's spouse would support this diagnosis? A "His breathing rate is usually below 12." B "I find the mood swings hard to deal with." Correct Answer (Blank) C "It seems that he has to urinate more frequently." D "He has a hard time reading small print."

Rationale: The frontal lobe of the brain controls affect, judgment and emotions. Dysfunction in this area results in symptoms such as emotional lability, changes in personality, inattentiveness, flat affect and inappropriate behavior. The other statements do not pertain to symptoms or changes in behavior typically seen with frontal lobe problems.

The nurse is teaching the parents of a child with sickle cell disease about ways to prevent complications and crises. What information would be a priority for the nurse to emphasize to the family? A The child can maintain normal activity with some restrictions B The child should be cautious of being exposed to people with a cold or fever C The child may not be able to follow routine immunization schedules D The child should avoid becoming overheated or dehydrated during physical activity and exercise Correct Answer (Blank)

Rationale: The goal of sickle cell treatment is to manage and control symptoms and to prevent sickle cell crisis. Fluid loss caused by overheating and dehydration can trigger a sickle cell crisis. People with sickle cell anemia need to keep their immunizations up-to-date, treat infections quickly and avoid too much sun exposure.

24 hours after a dilation and curettage (D&C), an adult female client has a hemoglobin level of 14 g/dL and a hematocrit of 42%. Which finding should the nurse expect when assessing the client? A Respiration rate of 36 breaths per minute B Pale mucous membranes C Complaints of fatigue with ambulation D Capillary refill of less than three seconds Correct Answer (Blank)

Rationale: The hemoglobin and hematocrit results are within normal limits for an adult female (hematocrit 37-48%; hemoglobin 12.0-15.5 g/dL). This capillary refill time is normal. The other assessment findings would be seen if the hemoglobin and hematocrit levels were low (anemia).

The nurse is caring for a client with asthma. During the initial assessment the nurse notes low-pitched wheezes in the final half of exhalation. One hour later, the nurse notes the wheezes are higher-pitched and last throughout exhalation. What should this change in assessment indicate to the nurse? A The client is hyperventilating. B The client requires more oxygen. C The client needs to be suctioned. D The airway obstruction has increased. Correct Answer (Blank)

Rationale: The higher pitched a sound is, the more narrow the airway. Therefore, the airway obstruction has increased or worsened, necessitating urgent intervention to promote bronchodilation or a change in interventions that are not be effective. Other signs of worsening asthma and the development of respiratory distress would include restlessness, anxiety, increased pulse and blood pressure and increased respiratory rate with the use of accessory muscles of breathing. Although secretions may be thick and tenacious during an acute asthmatic attack, there is no data to support the need for suctioning in this question. There is no data provided to support adding oxygen or hyperventilating the client.

The nurse is eating in the hospital cafeteria when a toddler at a nearby table begins to choke on a piece of food and turns slightly blue. What initial action should the nurse take? A Give the child water to help with swallowing B Perform abdominal thrusts Correct Answer (Blank) C Begin mouth to mouth resuscitation D Call for the emergency response team

Rationale: The initial response should be to perform abdominal thrusts. Since this child is actively choking, it would be essential to begin this step in order to dislodge the foreign object. Once this action is being done, the nurse should yell for help or for someone to call the rapid response team. If the child stops breathing and/or is unconscious the nurse should open the mouth and look for the object, if no object is seen perform mouth to mouth resuscitation using a barrier device, continue to do the Heimlich remover, and call for help.

A nurse is teaching a client and their caregiver how to perform tracheostomy care at home. Which statement by the caregiver indicates further education is needed? A "I will clean the inner cannula with isopropyl alcohol." Correct Answer (Blank) B "Reusable supplies should be cleaned with warm, soapy water." C "I will disinfect reusable supplies in a water and vinegar mixture." D "The stoma should be cleaned using circular motions moving outward."

Rationale: The inner cannula of a tracheostomy should be cleaned with hydrogen peroxide or saline solution at home. Isopropyl alcohol can cause potential damage to tissues. Warm, soapy water is recommended to clean reusable supplies to prevent the growth of microorganisms. A water and vinegar mixture is recommended to disinfect reusable supplies. The supplies should be soaked in the mixture for 30 minutes. Circular motions moving away from the stoma prevents recontamination of the area.

The nurse in the preoperative clinic is caring for a client with breast cancer and providing preoperative teaching a week before surgery. The client expresses an interest in complementary and integrative health therapies. Which of the following statements is the most appropriate response by the nurse? A "I will set up a time before the surgery is scheduled for you to talk to your health care providers about complementary and integrative health therapies." Correct Answer (Blank) B "You may want to do some Internet research on the different nutritional therapies that are recommended for your type of cancer." C "You may use any complementary or alternative therapies that you wish to after you recover from surgery." D "Complementary and integrative health therapies are not recommended and you should focus on the upcoming surgery."

Rationale: The intent of complementary and integrative therapies is to help the body to heal itself. It is important for the nurse to be nonjudgmental and provide the client the opportunity to explore appropriate complementary therapies. The nurse should not recommend a specific therapy. While the client may want to explore nutritional options, this response avoids the question the client asked.

The nurse in the emergency department is caring for a client who is confused and disorientated. The client's partner states the client was working outside and was sweating profusely. Which laboratory finding should the nurse expect to observe with this client? A Serum potassium level of 3.5 mEq/L B Serum magnesium level of 1.9 mg/dL C Serum calcium level of 9.5 mg/dL D Serum sodium level of 125 mEq/L Correct Answer (Blank)

Rationale: The laboratory results show that the client has hyponatremia. Common symptoms associated with hyponatremia are confusion, disorientation, and poor appetite. Hyponatremia can occur in extreme temperatures with excessive exercise or working outside. The other laboratory findings are within normal limits, even though potassium should be closely monitored.

The nurse is developing a plan of care for a client admitted with an acute asthma attack. The client has audible wheezes and a pulse oximeter reading of 88%. Which nursing problem is the priority? A Impaired gas exchange Correct Answer (Blank) B Risk for infection C Anxiety D Spiritual distress

Rationale: The low pulse oximetry reading indicates poor gas exchange. This is the result of bronchoconstriction and mucosal edema that occurs during an acute asthma attack. Maintaining an open airway and correcting the impaired gas exchange is therefore the priority. Treatment includes administration of oxygen, administration of bronchodilators via nebulizer and the use of corticosteroids. While the other problems may also apply to the client, they are not the priority at this time.

The nurse is responding to a low-pressure alarm for a client who has acute respiratory distress syndrome and is on mechanical ventilation. Which action should the nurse take? A Check if there is a disconnection in the system Correct Answer (Blank) B Suction the endotracheal tube C Evaluate if the client is biting the tube D Prepare the client for chest tube insertion

Rationale: The low-pressure alarm on the mechanical ventilator indicates that there is a decrease in pressure or loss in volume. When responding to a low-pressure alarm, the nurse should check for disconnection in the system causing a leak. A high-pressure alarm indicates a peak in airway pressure, often causes by kinked tubing from biting, increased secretions blocking the tube, and a pneumothorax.

The nurse is caring for an 8-month-old infant who has prenatally acquired human immunodeficiency virus (HIV) infection. Which clinical manifestations should the nurse monitor the infant for? Select all that apply. Kaposi sarcoma Correct Answer (Blank) Recurrent diarrhea Correct Answer (Blank) Autism Failure to thrive Correct Answer (Blank) Developmental delays Hepatomegaly Correct Answer (Blank)

Rationale: The majority of infants with perinatally acquired HIV infection are clinically normal at birth. Common clinical manifestations of HIV infection in children vary and include such signs as lymphadenopathy, hepatosplenomegaly, and unexplained diarrhea. Diarrhea may be the result of pathogens or of HIV itself, due to malabsorption of carbohydrates, protein, and fat. HIV-infected children often do not grow normally. They may be proportionally smaller in both length and weight for their age. Kaposi sarcoma, one of the hallmarks of adult-acquired immunodeficiency syndrome (AIDS), is found in less than 1% of affected children. Autism or developmental delays are not conditions associated with HIV or AIDS.

A nurse is administering the influenza vaccine in an occupational health clinic. Within 10 minutes of giving the vaccine to a middle-aged adult male, the man reports having itchy and watery eyes, feeling anxious and short of breath. What should the nurse do first? A Administer SQ epinephrine. Correct Answer (Blank) B Apply oxygen. C Maintain the airway. D Take the client's vital signs.

Rationale: The man is exhibiting signs and symptoms of an anaphylactic reaction. Although all of the interventions are correct, the nurse should first administer epinephrine to stop the bronchial constriction and airway obstruction that is occurring.

The nurse is conducting a teaching session for a group of new nurses about types of oxygen delivery systems. Which system provides the most accurate delivery of oxygen? A A partial non-rebreather mask B A simple face mask C A Venturi mask Correct Answer (Blank) D A nasal cannula

Rationale: The most accurate way to deliver oxygen to a client is through a Venturi or Venti mask. The Venti mask is a high flow device that traps room air into a reservoir device on the mask and mixes the room air with 100% oxygen. The size of the opening to the reservoir determines the concentration of oxygen. The client's respiratory rate and respiratory pattern do not affect the concentration of oxygen delivered with this system. The maximum amount of oxygen that can be delivered by a Venti mask is approx. 55%.

The nurse in a long-term care facility is caring for an 89-year-old client with atrial fibrillation and a history of multiple falls. The client's medications include amiodarone, atorvastatin, baby aspirin and metoprolol. Which new finding should be of greatest concern to the nurse? A Right-sided facial droop Correct Answer (Blank) B Bibasilar crackles C SpO2 of 89% on room air D Heart rate of 106

Rationale: The most concerning finding would be the development of a right-sided facial droop. The client with atrial fibrillation is at increased risk of stroke, and this client's listed medications do not include an anticoagulant, typically prescribed to prevent a stroke. Given the finding of frequent falls, it is possible that the client is not on a stronger anticoagulant, such as warfarin, due to an increased risk of intracranial hemorrhage after a fall. A SpO2 of 89% on room air, a heart rate of 106 and crackles on auscultation are all concerning findings, but the possibility of a stroke should be of the greatest concern to the nurse.

A home health nurse is teaching the parents of a pediatric client with acute spasmodic croup. Which interventions are most important to include? A Antihistamines to decrease allergic responses B Antibiotic therapy for 10 to 14 days C Sedation as needed to prevent exhaustion D Humidified air with an increase in oral fluids Correct Answer (Blank)

Rationale: The most important aspects of home care for a child diagnosed with acute spasmodic croup are humidified air and increased oral fluids. Humidified air helps reduce vocal cord swelling. Taking the child out into the cool night air for 10 to 15 minutes can also reduce nighttime symptoms. Adequate systemic hydration aids mucociliary clearance by keeping secretions thin and easy to remove with minimal coughing effort.

The nurse is assessing a client admitted for acute exacerbation of chronic obstructive pulmonary disease. Which assessment finding would support this diagnosis? A Inspiratory laryngeal stridor B Crackles in the lung bases C An S3 heart sound D Audible expiratory wheezing Correct Answer (Blank)

Rationale: The nurse must be able to identify and differentiate assessment findings, such as adventitious lung sounds. Wheezing is associated with a narrowed, smaller airway. In an acute episode of obstructive airway disease, breathing is likely to be characterized by wheezing on expiration. This sound can often be heard without the use of a stethoscope. The other assessment findings are not typically seen with this diagnosis.

A nurse is caring for a client who is being evaluated for a possible myocardial infarction. The nurse notes what appears to be ventricular tachycardia on the cardiac monitor. Which action is a priority for the nurse? A Assess airway, breathing and circulation Correct Answer (Blank) B Begin cardiopulmonary resuscitation C Prepare for immediate defibrillation D Notify the rapid response team and the health care provider

Rationale: The nurse must treat the client, not the cardiac monitor. Always assess the client to determine the next step. This focused assessment includes checking the client's airway, breathing, and circulation (ABCs) and for signs of low cardiac output. Signs of low cardiac output include chest pain, dyspnea, hypotension and an altered level of consciousness. These clinical manifestations would indicate a need for cardioversion and other emergency interventions. The other options would be appropriate after the nurse has assessed the client.

The nurse is caring for a client who has end-stage renal disease and is scheduled for hemodialysis later today. The client has an arteriovenous fistula. Which interventions should the nurse implement to help prepare the client for dialysis? Select all that apply. Weigh the client Correct Answer (Blank) Assess the patency of the fistula Correct Answer (Blank) Hold all oral medications Ensure the client eats a high fiber, high protein breakfast

Rationale: The nurse should administer medications as prescribed, such as vitamin D and sevelamer (a phosphate binder). These medications may be prescribed to help control both serum calcium and phosphate levels. Some medications that are dialyzable or could lower blood pressure are held until after the procedure. The client should eat a meal that is easily digestible at least 2 hours before the procedure begins. A meal high in fiber and protein is not easily digested. The nurse should assess the client's weight as a baseline prior to the procedure and measure vital signs. The access site should be assessed including palpating a thrill, auscultating a bruit and palpating pulses and circulation distal to the site.

The nurse is completing a head-to-toe assessment on a client. The nurse notes a pulsating mass in the client's periumbilical area. Which assessment is appropriate for the nurse to perform? A Measure the length B Percuss C Palpate D Auscultate Correct Answer (Blank)

Rationale: The nurse should auscultate the mass. If the finding of a bruit is present, this could confirm the presence of an abdominal aortic aneurysm. The mass should not be palpated or percussed because of the risk of rupture.

The nurse is discussing health promotion activities with a group of new parents. One parent expresses concern about Reye's syndrome and asks about prevention methods. How should the nurse respond? A "Avoid the use of aspirin for viral infections in children." Correct Answer (Blank) B "Seek medical attention for serious injuries." C "Report exposure to this illness as soon as possible." D "Immunize your child against this disease."

Rationale: The nurse should educate the parents about reading drug labels for over-the-counter (OTC) medication and following the directions closely. To answer the parent's question, the nurse should explain that salicylates, such as aspirin, are contraindicated for children with viral infections such as chickenpox or influenza due to an increased risk of Reye's syndrome. Since viral infections can be common in children, salicylates should be generally avoided in children under the age of 19.

A client is admitted to the orthopedic nursing unit with a fractured right tibia. The client is reporting pain. Which action should the nurse take first? A Place an ice pack on the fracture site to reduce edema. B Contact the health care provider. C Check the pulse and capillary refill in the right foot. Correct Answer (Blank) D Administer acetaminophen 650 mg PO as ordered.

Rationale: The nurse should first collect more data about the client's pain. Compartment syndrome is a potential complication with an acute fracture and the nurse should evaluate tissue perfusion in the affected extremity to make sure that the pain is solely related to the acute fracture. Signs of compartment syndrome include worsening pain, weak peripheral pulses, edema, slow capillary refill and paresthesia (i.e., numbness, tingling). If the nurse suspects that compartment syndrome is occurring, the health care provider (HCP) must be notified immediately. After ruling out compartment syndrome, the nurse can proceed with administering an analgesic and applying ice.

A client comes to the community health clinic with symptoms of gonorrhea. Which intervention should the nurse implement first? A Instruct the client to notify past sexual partners. B Discuss the risk of infertility with the client. C Obtain information about the client's recent sexual encounters. Correct Answer (Blank) D Collect a urethral swab from the client.

Rationale: The nurse should first obtain information from the client about their recent sexual encounters. This will provide further insight and assist other health care providers who are coordinating care. The nurse should prepare for a urethral swab, but this should not be the first intervention. The nurse must provide sensitive care because some people are reluctant to seek health care when problems first arise. Sexually transmitted infections can cause emotional distress and may progress without symptoms. It would be important for the client to speak to any sexual partners that may have a risk of contracting the infection, but this should not be the first thing. Discussing the risk of infertility would not be appropriate at this time.

An older adult client admitted with hypoglycemia has an order for a continuous IV infusion of 1000 mL of dextrose 5% in water (D5W) at 83 mL per hour. When the nurse assesses the client, she discovers that the client mistakenly received 800 mL over the past two hours. What is the priority nursing action at this time? A Check the vital signs and compare to baseline readings. B Place the client in a mid- to high-Fowler's position. C Auscultate the lungs bilaterally at the bases. Correct Answer (Blank) D Obtain a blood glucose level.

Rationale: The nurse should follow the A-B-C prioritization approach. Clients can quickly develop hypervolemia following rapid infusion of isotonic IV fluids. Findings of fluid overload include increased blood pressure, tachycardia, shortness of breath, crackles heard in the lungs and distended neck veins. Therefore, the priority nursing action at this time is to auscultate the lungs to assess for pulmonary congestion.

The nurse is caring for a client who fell two hours ago while alone in their room. The client appears tired and disoriented. What should the nurse do first? A Use a gait belt to assist the client to their bed for a nap. B Ensure fall precautions are included in the client's plan of care. C Reorient the client to their surroundings, date and time. D Gather data about the client's baseline neurologic status. Correct Answer (Blank)

Rationale: The nurse should follow the nursing process and collect data first. Based on the data collected, the nurse will then be able to identify and make a clinical decision about which interventions are the priority to implement. The client in this scenario suffered an unwitnessed fall and may have hit their head. A decrease or change in the level of consciousness is typically the first sign of deterioration in neurologic status. By establishing the client's baseline data, the nurse can detect subtle changes in the client's neurologic status. This enables the health care team to prevent or treat the potentially life-threatening complications of a head injury. Reorientation, fall precautions and use of a gait belt would be beneficial for a client with a history of falling, but obtaining a neurological baseline is the priority in this scenario.

The nurse is performing suctioning of a client's endotracheal (ET) tube. Which of the following actions is appropriate for the nurse to make? A Apply suctioning for at least 30 seconds B Hyperoxygenate the client prior to inserting the suction catheter Correct Answer (Blank) C Apply suction while inserting and removing the suction catheter D Instill 2-3 drops of saline into the ET tube prior to suctioning

Rationale: The nurse should hyperoxygenate the client with 100% oxygen for 30 seconds to 3 minutes prior to suctioning the endotracheal tube to prevent hypoxemia. The nurse should suction no longer than 15 seconds at a time. Suctioning should only be applied while removing the catheter, never during insertion. Instillation of normal saline during suctioning is not supported by evidence; this action may impair gas exchange and increase the risk of pulmonary infection.

A nurse is providing care to a client admitted with an infected abdominal wound. Which clinical finding will the nurse immediately report to the healthcare provider? A Lactate level of 4 mmol/L Correct Answer (Blank) B White blood cell count of 11,000/mm³ C Temperature of 101°F (38.3°C) D Heart rate of 110 beats/min

Rationale: The nurse should immediately report a lactate level of 4 mmol/L. An elevated lactate level indicates tissue hypoxia and is indicative of septic shock. An elevated white blood cell count is expected for a client with an infected abdominal wound. A temperature of 101°F (38.3°C) is indicative of infection. However, this value is not critical. Although an elevated heart rate may indicate an infectious process, 110 beats/min is not a critical value.

A nurse is assisting a healthcare provider with a bronchoscopy on an older adult client. Which pre-procedure observation prompts the nurse to intervene? A The client's dentures have been removed. B The client is in the prone position. Correct Answer (Blank) C The client is connected to continuous pulse oximetry. D The client has a decreased cough reflex.

Rationale: The nurse should intervene if the client is in the prone position. The client should be placed in a sitting or supine position prior to beginning the procedure. The client's dentures should be removed prior to the procedure to ensure they are not damaged during insertion of the scope. Older adults have a higher risk of respiratory distress during the procedure. Continuous pulse oximetry is indicated. A decreased cough reflex is not a contraindication for a bronchoscopy. The nurse will monitor the client closely after the procedure to ensure adequate airway clearance.

The nurse is performing a dressing change for a client who had abdominal surgery 48 hours ago. Which of the following client findings would be a priority for the nurse to notify the surgeon? A Bruising around the surgical wound that is extending to the client's right flank Correct Answer (Blank) B Moderate amount of serosanguinous drainage in the client's surgical drain C Crusting underneath the staples and around the surgical incision D Small amount of purulent drainage from the client's surgical incision

Rationale: The nurse should notify the surgeon immediately of bruising that is extending into the client's flank, which indicates a hematoma and internal hemorrhage. Hemorrhage following surgery is a life-threatening complication and requires immediate intervention. Clients may have a moderate amount of serosanguinous drainage within the first few days following surgery. Crusting around the incision site and underneath the staples is an expected finding and indicates wound healing. Purulent drainage from the surgical incision may indicate the development of infection; however, it is not the priority over possible hemorrhage.

The nurse is observing an unlicensed assistive personnel (UAP) perform oral suctioning for a client. Which of the following actions by the UAP would require the nurse to intervene? A Raising the head of bed to 30 degrees prior to suctioning B Applying continuous suction down the side of the client's cheek towards the back of the throat Correct Answer (Blank) C Hyperextending the neck as tolerated prior to inserting the suction catheter D Adjusting the suction pressure to the level of 120 mm Hg

Rationale: The nurse should only apply suction once the catheter has reached the back of the oral cavity. Applying suction prior may cause oral mucosal damage and invoke gagging in the client. The nurse should position the client in a semi-Fowlers position at 30 degrees and hyperextend the neck as tolerated to promote comfort, prevent aspiration and reduce the incidence of gagging in the client. Suction dials should be set at 120 mm Hg for oral suctioning.

A client diagnosed with renal calculi is admitted to the medical surgical unit. Which intervention should the nurse implement first? A Monitor the client's urinary output. B Assess the client's pain. Correct Answer (Blank) C Review appropriate diet choices with client. D Increase the client's oral fluid intake.

Rationale: The nurse should use the nursing process to prioritize and plan which intervention to implement first. The first step in the process is assessment/data collection and should be taken before formulating a plan of care and implementing interventions. All of the interventions in the scenario are appropriate for a client with renal calculi, but asking the client about their pain level should be done first. Based on the data obtained (i.e., the client's pain level) the nurse should then decide how to proceed.

The nurse is assessing a client who is experiencing decreased cardiac output due to a left-sided heart failure exacerbation. Which of the following findings should the nurse expect to observe? A Jugular venous distention B Serum hematocrit of 55% C Dependent edema in the legs D Serum creatinine level of 2.1 mg/dL Correct Answer (Blank)

Rationale: The nurse who is caring for a client with decreased cardiac output due to a left-sided heart failure exacerbation should expect possible organ dysfunction, particularly renal, due to decreased blood flow. A serum creatinine level of 2.1 mg/dL indicates possible acute kidney injury due to reduced blood flow to the kidneys. Jugular venous distention and dependent edema in the legs would be observed in a right-sided heart failure exacerbation. A serum hematocrit of 55% is elevated, which is not an expected finding.

A client presents to the emergency department with a prolonged asthma attack that did not resolve after the client used a metered-dosed inhaler at home. Which medication should the nurse plan to administer first for this client? A Intravenous azithromycin B Nebulized albuterol Correct Answer (Blank) C Oral prednisone D Fluticasone inhaler

Rationale: The nurse would anticipate that nebulized albuterol would be given first in this situation to address the acute asthma attack through bronchodilation. While oral prednisone may be used in the treatment of this client, it would be given after administration of an inhaled B2-adrenergic agonist like albuterol. There is no information provided that would indicate antibiotic therapy is needed for the client. A fluticasone inhaler may be part of long-term asthma management for this client, but is not recommended as a rescue treatment for acute asthma attacks

The nurse is caring for a 68-year-old client who had a total hip replacement three days ago. Which client statement requires the nurse's immediate attention? A "It seems that the pain medication is not working as well today." B "I have to use the bedpan to pass my water at least every hour." C "I have bad muscle spasms in my lower leg, below the incision." D "I seem to have trouble breathing when I am resting in bed." Correct Answer (Blank)

Rationale: The nurse would be concerned about all of these comments, however the most life threatening is the respiratory focus (think ABCs). Clients who have had hip or knee surgery are at risk for developing pulmonary embolism. Sudden dyspnea, tachycardia and a feeling of impending doom are classic findings of pulmonary embolism. Muscle spasms do not require immediate attention. Frequent urination may indicate a urinary tract infection, particularly since the client likely had an indwelling urinary catheter during surgery. Although the thought that medication is not effective requires further investigation, it is not life-threatening.

An off-duty nurse witnesses a person collapse in a grocery store, and the individual is now unresponsive. Multiple bystanders are present. What should the nurse do first? A Deliver two rescue breaths. B Begin chest compressions. C Run to get the store's automated external defibrillator. D Check for a carotid pulse and instruct a bystander to call 911. Correct Answer (Blank)

Rationale: The off-duty nurse's first action when encountering this unresponsive individual who just collapsed should be to check for a pulse and to ensure the activation of 911 emergency response. While chest compressions may very well be needed, the nurse should first check for a carotid pulse. If a carotid pulse cannot be palpated in this unresponsive individual, cardiopulmonary resuscitation (CPR) chest compressions should be initiated. An automated external defibrillator (AED) should be incorporated into the response once it is available. However, the nurse should stay with the victim, begin CPR and assign the task of obtaining the AED to someone else at the scene.

The nurse on a critical care unit is admitting a client who is experiencing a hypertensive urgency or crisis. Which assessment is the priority? A Heart rate B Orientation Correct Answer (Blank) C Pedal pulses D Lung sounds

Rationale: The organ most susceptible to damage in hypertensive crisis is the brain, due to the high risk for rupture of cerebral blood vessels leading to a stroke or hemorrhage. Therefore, a neurologic assessment that should include orientation and level of consciousness is the priority for this client.

The nurse is caring for a client recovering from a cardiac catheterization. The introducer sheath has been removed from the right femoral artery and a pressure dressing is in place. Which of the following findings indicates a serious complication may be occurring? A Creatinine level is increasing Correct Answer (Blank) B Ecchymosis is present at the insertion site C Discomfort at the insertion site D Extremities are warm bilaterally with trace edema

Rationale: The risk of contrast-induced kidney injury is increased in clients with underlying moderate to severe renal disease, people with diabetes, the elderly, females, clients on diuretics, ACEI, and metformin. Adequate pre-hydration, use of iso-osmolar agents, and techniques to minimize the amount of dye used will help prevent this complication. Renal emboli can also cause renal failure. Ecchymosis and discomfort at the site are expected, but the nurse should monitor for hematomas and retroperitoneal bleeding. The nurse would monitor for signs of neurovascular impairment in the lower extremities, especially the right leg. Warmth is an expected finding, and trace edema is not a priority.

The nurse is planning care for a client with a myocardial infarction. The client has a nursing problem of pain related to cardiac ischemia. Which of the following interventions would be essential for the nurse to include in the client's plan of care? A Administer anti-platelet therapy as soon as possible Correct Answer (Blank) B Obtain a chest x-ray as soon as possible C Administer a stool softener to prevent constipation D Monitor the client's temperature every four hours

Rationale: The pain that occurs with a myocardial infarction (MI) is related to ischemia of the heart muscle. The majority of clients who suffer an MI develop a thrombus inside a coronary artery. It is these blockages that cause ischemia, which in turn lead to chest pain. Although the use of stool softeners is recommended in this scenario, it is not the highest priority intervention to be added to the client's plan of care. After an MI, clients may develop a low-grade fever due to an inflammatory response. Although assessing a client's temperature is appropriate in this scenario, it is not the highest priority intervention to be added to the client's plan of care. Administering an anti-platelet agent as soon as possible would be essential in this scenario. Evidence suggests that aspirin reduces the platelet aggregation of a thrombus. Obtaining a chest radiography after an MI is not the highest priority intervention to be added to this client's plan of care. A chest radiography is not a specific test performed to evaluate the occurrence of an MI.

The nurse in a well-baby clinic is speaking with the mother of a 3-week-old newborn. Which statement by the mother should be of highest concern to the nurse? A "I have not been able to empty my breasts completely with each feeding." B "No matter how hard I try, I don't feel any love for my baby." Correct Answer (Blank) C "I have not been doing my Kegel exercises as much as I should." D "I think my baby is not gaining as much weight as it should."

Rationale: The postpartum period is a time of great adjustment and change for the new parent. The nurse's role should focus on supportive care and monitoring for postpartum complications such as postpartum depression (PPD). Up to 20% of new mothers experience PPD. Symptoms of PPD include intense sadness lasting longer than two weeks, severe and labile mood swings, feelings of fear, anxiety or anger and worries about being an incompetent parent or not loving the child. Although all of the mother's statements should be followed-up by the nurse, the mother's statement about trying but not being able to feel love for their child should be of highest concern.

A nurse is observing a new nurse perform nasopharyngeal suctioning on an adult client. The precepting nurse will intervene if the new nurse performs which action? A Sets the suction pressure to 190 mmHg Correct Answer (Blank) B Applies sterile gloves to both hands C Suctions the client for 15 seconds D Withdraws the catheter while rotating it

Rationale: The precepting nurse will intervene if the suction pressure is set to 190 mmHg. Suction should be set between 100 and 150 mmHg for adults. Pressures exceeding 180 mmHg can cause hypoxia and damage to tissues. Sterile gloves may be applied to both hands. Alternatively, a sterile glove may be applied to the dominant hand and a nonsterile glove to the non-dominant hand. Intermittent suction should be applied for no more than 10-15 seconds at a time. Rotating the catheter while withdrawing is proper technique when suctioning the airway.

After placement of a ventriculoperitoneal (VP) shunt as a treatment for hydrocephalus, the parents of an infant ask the nurse: "Why is there a small incision on the abdomen?" Which response by the nurse is most appropriate for explaining the purpose of the incision? A "That's what is used for insertion of the catheter into the stomach." B "It's used to visualize the abdominal organs for correct catheter placement." C "It's there so the tubing can be inserted into the urinary bladder." D "It's used to pass the catheter into the abdominal cavity." Correct Answer (Blank)

Rationale: The preferred procedure in the surgical treatment of hydrocephalus is the placement of a ventriculoperitoneal shunt. This shunt procedure provides primary drainage of the cerebrospinal fluid from the ventricles to an extracranial compartment, which is commonly the peritoneum. A small incision is made in the upper quadrant of the abdomen so the shunt tip can be guided into the peritoneal cavity.

The nurse is evaluating an older adult client who had a generalized, tonic-clonic seizure. The client is drowsy, but moves all extremities. Vital signs are stable and there is vomit on the client's clothes and face. Which complication is the priority to monitor the client for? A Pneumonia Correct Answer (Blank) B Increased intracranial pressure C Dehydration D Urinary incontinence

Rationale: The presence of vomitus indicates that the client vomited during the seizure and the likelihood of aspiration is high. Therefore, the priority is to monitor the client for development of pneumonia. Aspiration pneumonia results from the abnormal entry of material from the mouth or stomach into the trachea and lungs. Conditions that increase the risk for aspiration include decreased level of consciousness (e.g., seizure). The aspirated material (food, water, vomitus, oropharyngeal secretions) triggers an inflammatory response. The most common form of aspiration pneumonia is a primary bacterial infection. The other complications are not typically associated with a seizure.

The nurse is performing suctioning of a client's tracheostomy. Which of the following actions is appropriate for the nurse to make? A Apply suction with the pressure dial at 80-120 mm Hg. Correct Answer (Blank) B Instill 2-3 drops of saline into the tracheostomy prior to suctioning. C Apply suction for less than 30 seconds. D Hyperoxygenate the client for 15 seconds after suctioning.

Rationale: The pressure dial for suctioning should never exceed 120 mm Hg to prevent hypoxemia and trauma to the respiratory mucosa. Instillation of normal saline during suctioning is not supported by evidence; this action may impair gas exchange and increase the risk of pulmonary infection. The nurse should suction no longer than 15 seconds at a time. Hyperoxygenation after suctioning should be performed for 1 to 5 minutes after suctioning or until the client's baseline heart rate and oxygen saturation are within normal limits.

The nurse is teaching a client with a new colostomy about self-care management. Which statement by the client would indicate to the nurse teaching was effective? A "I should expect my stool to look watery." B "I will eat dark leafy vegetables daily to decrease the odor." C "The stoma should look beefy red and moist." Correct Answer (Blank) D "I should empty the bag when it is two-thirds full."

Rationale: The stoma should appear beefy red or pink and be well hydrated. A discolored stoma can be an indication of anemia, and a dark purple or black stoma is a sign of impaired circulation. A colostomy is created in the area of the large intestine. The majority of the water has been absorbed. Therefore, the stool should be formed. Dark leafy vegetables increase the odor of the stool. The colostomy bag should be emptied when it is one-third to one-half full to prevent it from being dislodged from the appliance.

A 2-year-old child has been diagnosed with cystic fibrosis. The child's parent asks the nurse what is most concerning about the disease. Which is the appropriate response from the nurse? A "Thick, sticky secretions from the lungs are a constant challenge." Correct Answer (Blank) B "Cystic fibrosis results in nutritional concerns that can be dealt with." C "There is a high probability of life-long complications." D "You will work with a team of experts and have access to a support group."

Rationale: The primary factor, and the one responsible for many of the clinical manifestations of cystic fibrosis, is mechanical obstruction caused by the increased viscosity of mucous gland secretions.Because of the increased viscosity of bronchial mucus, there is greater resistance to ciliary action (probably secondary to infection and ciliary destruction), a slower flow rate of mucus and incomplete expectoration, which also contributes to the mucus obstruction. This retained mucus serves as an excellent medium for bacterial growth. Reduced oxygen-carbon dioxide exchange causes variable degrees of hypoxia, hypercapnia and acidosis.In severe cases, progressive lung involvement, compression of pulmonary blood vessels and progressive lung dysfunction frequently lead to pulmonary hypertension, cor pulmonale, respiratory failure and death. Pulmonary complications are present in almost all children with cystic fibrosis, but the onset and extent of involvement are variable.

A nurse is caring for a client diagnosed with an obstructing renal calculus. Which focus of the health care provider's orders would the nurse prioritize? A Push oral fluids B Start intravenous antibiotics C Apply warm compress over flank area D Morphine sulfate for pain control Correct Answer (Blank)

Rationale: The priority action for an obstructing renal calculus (kidney stone) is to provide prompt relief for the severe pain. Oral hydration or intravenous fluids will help move the stone though the urinary system, but would be prioritized after pain management. Applying a warm compress over the flank may help pain, but would be prioritized after narcotic analgesics for this diagnosis. A kidney stone is not an infection and does not indicate the need for intravenous antibiotics.

The nurse is caring for a client who had abdominal surgery 1 day ago and has a hemovac in place. Which of the following actions is most important for the nurse to take? A Monitor the client's hemoglobin and hematocrit. Correct Answer (Blank) B Evaluate the color and consistency of drainage once per shift. C Ensure that the reservoir is compressed to provide suction. D Secure the container to the client's gown.

Rationale: The priority action for the nurse is to closely monitor the client's hemoglobin and hematocrit levels while surgical drains are in place. Clients are at risk for developing anemia and associated complications. Evaluating the color and consistency of drainage each shift, ensuring the reservoir is compressed, and securing the container to the client's gown are all correct actions and are standard care; however, the priority is to monitor for complications of blood loss.

The nurse is caring for a client who was admitted with a diagnosis of renal calculi. The client reports moderate-to-severe flank pain and nausea. The client's oral temperature is 100.8°F (38.2°C). Which goal is the priority for this client? A Maintain fluid balance B Manage pain Correct Answer (Blank) C Control nausea D Prevent infection

Rationale: The priority nursing goal for a client diagnosed with renal calculi, i.e., kidney stones, is typically focused on managing the client's pain because the pain tends to be quite severe. The other goals are also important, but since the client does not appear to currently experience a fluid imbalance, alleviating the acute pain is the priority for this client. If nausea remains unresolved after the pain has been relieved, then the nurse should consider administering an antiemetic.

A client arrives at the clinic verbalizing fatigue, chills, and a headache after attending a large gathering 3 days ago. The nurse suspects influenza. Which stage of infection is consistent with the client's condition? A Incubation B Prodromal Correct Answer (Blank) C Chronic D Convalescent

Rationale: The prodromal stage of infection is the second stage and is characterized by vague and nonspecific symptoms. Influenza has an incubation period of 1-2 days before progressing into the prodromal stage. The client's time frame of exposure is consistent with the prodromal stage. Influenza symptoms are not yet present in the incubation stage. Chronic is not a stage of infection. The convalescent stage of infection is characterized by the recovery from the illness. Signs and symptoms are no longer present in the convalescent stage.

The nurse is assessing a client who is 16 hours post CABG. The nurse notes that the client's pulmonary artery wedge pressure is elevated at 14 mmHg. Which of the following would be the best action for the nurse to take? A Auscultate lung sounds Correct Answer (Blank) B Increase the prescribed intravenous fluids (IVF) C Position the client in modified Trendelenburg D Administer prescribed dopamine

Rationale: The pulmonary artery wedge pressure assesses left ventricular function. Normal wedge pressures are between 4-12 mmHg. A pulmonary wedge pressure of 14 mm/Hg indicates a high pressure in the left ventricular. High pressures usually occur with increased volume in the left ventricular, associated with left ventricular failure. The nurse should assess for the development of left ventricular failure and fluid volume overload by auscultating lung sounds. The nurse would increase IVF, administer prescribed dopamine, and position in modified Trendelenburg if the client had a low pulmonary artery wedge pressure, which would indicate decrease volume.

A client has had a mitral valve replacement. Postoperative orders include monitoring pulmonary artery pressure and pulmonary capillary wedge pressure with a pulmonary artery catheter. What is the purpose of these actions by a nurse? A Establish coronary artery stability B Determine change in acid-base balance C Assess the left ventricular end-diastolic pressure Correct Answer (Blank) D Compare the right ventricular pressures

Rationale: The pulmonary capillary wedge pressure is reflective of left ventricular end-diastolic pressure. Pulmonary artery pressures are an assessment tool used to determine the ability of the heart to receive and pump blood effectively.

The nurse is caring for a client with Legionnaire's disease. Which finding would require the nurse's immediate attention? A Decreased chest wall expansion Correct Answer (Blank) B Pleuritic pain on inspiration C A decrease in respiratory rate from 34 to 24 D Dry mucous membranes in the mouth

Rationale: The respiratory status of a client with this acute bacterial pneumonia known as Legionnaires' disease is critical. Note that all of these findings would be of concern, but a decrease in chest wall expansion is the priority because it reflects a possible decrease in the depth and effort of respirations. Further findings of restlessness, including low oxygen saturation, would indicate hypoxemia. The client may need to have oxygen titrated to maintain adequate O2 saturation. Mechanical ventilation may be needed for signs of respiratory failure.

The charge nurse is observing a newly hired nurse who is caring for a client with a nasogastric (NG) tube to continuous suction. Which of the following actions by the newly hired nurse requires the charge nurse to intervene? A Adjusting the suction pressure dial between 150 - 200 mm Hg Correct Answer (Blank) B Maintaining the air vent of the NG tube above the level of the stomach C Instilling 50 mL of irrigating solution into the NG tube to remove a blockage D Maintaining the client in a semi-Fowlers position

Rationale: The suction pressure dial should never be set above 120 mm Hg for gastrointestinal suctioning to reduce trauma to the gastrointestinal mucosa. The air vent of an NG tube should be maintained above the level of the stomach to prevent reflux of gastric contents into the air lumen of the tube. NG tubes may be irrigated per the healthcare provider's orders to remove blockages and ensure patency of the tube. Maintaining the client in a semi-Fowlers position prevents the likelihood that the NG tube lies against the wall of the stomach, which ensures adequate suctioning and prevents reflux of gastric contents that could lead to aspiration.

The nurse is admitting a client with a diagnosis of testicular cancer. The nurse should expect the client to have which of the following findings? A Heaviness in the affected testicle Correct Answer (Blank) B Scrotal discoloration C Sustained painful erection D Inability to achieve erection

Rationale: The testicles are located in the scrotum. Testicular cancer can occur in one testicle; however, 2-3% of tumors can occur in both testicles. The first sign of testicular cancer often is a firm, smooth, painless mass which is accompanied by a feeling of heaviness in the affected testicle. Other symptoms of testicular cancer include a feeling of swelling in the scrotum, discomfort in the scrotum, ache in the lower back, and pelvis area. In advanced stages symptoms include abdominal mass, coughing, weight loss, fatigue, pallor and lethargy. Scrotal discoloration is not a finding of testicular cancer. Sexual performance (e.g., ability to have an erection and a painful erection) is not affected by testicular cancer.

A community health clinic nurse is interviewing a client who is experiencing lightheadedness. The client reports a history of arthritis and is taking ibuprofen for the pain. The client is pale with a blood pressure of 88/40 mmHg. The client's pulse is 114 bpm, the respiratory rate is 22 breaths per minute, and the oral temperature is 98.2°F (36.7°C). Which additional information should the nurse solicit from the client? Select all that apply. Color of bowel movements Correct Answer (Blank) Presence of photophobia Presence of tingling or numbness in the extremities Presence of bruises Correct Answer (Blank) Frequency and amount of ibuprofen used Correct Answer (Blank)

Rationale: The therapeutic effects of nonsteroidal anti-inflammatory drugs (NSAIDs), such as ibuprofen, are due to the ability of the medications to inhibit the synthesis of prostaglandins by blocking COX-1 and COX-2 enzymes. The COX-1 enzymes have many roles in the body, such as blood clotting, stomach lining protection, and maintaining sodium and water balance in the kidneys. When taking this medication, inhibition of the COX-1 enzyme causes an inability of COX-1 to protect the gastrointestinal (GI) lining. These actions put clients at risk for a loss of GI lining integrity and GI bleeds. The nurse should recognize the client's skin color and changes in vital signs as indicating possible anemia related to GI bleeding. The nurse should investigate the risk by asking the client how often and how much ibuprofen is taken per day. The nurse should also question the color of bowel movements and any abnormal bruising. Black, tarry stools could indicate a GI bleed. These risks are also increased in clients taking acetylsalicylic acid (i.e., aspirin). Tingling, numbness, and photophobia are not side effects seen with ibuprofen use or overuse.

A client is admitted with low T3 and T4 levels and an elevated thyroid stimulating hormone (TSH) level. On initial assessment, the nurse should anticipate which of these findings? A Tachycardia B Lethargy Correct Answer (Blank) C Diarrhea D Heat intolerance

Rationale: The thyroid gland produces two hormones, triiodothyronine (T3) and thyroxine (T4). These hormones help regulate metabolism, stimulate the sympathetic nervous system, regulate body temperature, heart rate, and brain development. If these levels are low, the client will suffer from an underactive thyroid gland known as hypothyroidism. With low T3 and T4, the nurse can expect the client to show symptoms of weight gain, constipation, feeling tired, having an intolerance to cold, bradycardia, and/or memory loss. If T3 and T4 are low, the pituitary gland will attempt to stimulate the thyroid by stimulating TSH, which will be elevated.

The nurse is performing tracheostomy care on an unconscious client. The nurse is preparing to change the tracheostomy collar. Which action will the nurse perform prior to performing this step? A Raise the head of the bed to a Fowler's position. B Apply sterile gloves to both hands. C Ensure the oxygen source is applied. D Call another nurse for assistance. Correct Answer (Blank)

Rationale: The tracheostomy collar is used to prevent the tracheostomy tube from becoming dislodged. As the collar is being replaced, another nurse or staff member should hold the tube in place to prevent dislodgement. An unconscious client should be positioned laterally facing the nurse. Changing the tracheostomy holder is not a sterile procedure. Clean gloves may be used. The oxygen source should be applied until after the procedure is done.

The nurse is caring for a client with rheumatoid arthritis experiencing epigastric pain and dyspepsia. Which question will provide the nurse with further details about the client's symptoms? A "How long have you had rheumatoid arthritis?" B "Do you exercise regularly?" C "Which medication do you take for the inflammation?" Correct Answer (Blank) D "How much fiber do you consume daily?"

Rationale: The treatment for rheumatoid arthritis (RA) begins with nonsteroidal anti-inflammatory drugs (NSAIDs). NSAIDs reduce the inflammation associated with rheumatoid arthritis. NSAIDs can damage the gastric mucosa and cause ulcerations, heartburn, and indigestion. Asking the client how long they have had RA will not provide specific information related to the epigastric pain and dyspepsia. Asking the client if they exercise regularly is unrelated to the client's concern. The amount of fiber consumed daily is unrelated to the client's medical history.

The nurse is reviewing the medical record of a client diagnosed with breast cancer. The nurse notes M1 as part of the TNM staging system. How should the nurse interpret this information? A The tumor is 1 mm in size. B The cancer has spread to other parts of the body. Correct Answer (Blank) C The primary tumor cannot be evaluated. D The cancer has affected 1 axillary lymph node.

Rationale: The tumor-node-metastasis (TNM) system is used to stage cancer. The "M" signifies evidence of cancer spreading to other parts of the body. M1 represents distant metastasis. The tumor size and evaluation are characterized by the letter "T" in the TNM system. Lymph node involvement is characterized by the letter "N" in the TNM system.

The nurse in the outpatient clinic is reviewing discharge instructions with a client being treated for recurring sinusitis. Which statement by the client indicates that additional teaching is needed? Select all that apply. Use probiotics daily to reduce recurrence. Correct Answer (Blank) Sleep in a flat position to decrease postnasal drip Control asthma symptoms with prescribed medications. Correct Answer (Blank) Use nasal decongestant sprays several times daily. Wash hands and change clothing after outdoor activities. Correct Answer (Blank) Restrict water intake to reduce copious nasal drainage. Reduce the use of smokeless tobacco and cigarettes.

Rationale: The use of nasal decongestant sprays for more than three days often leads to increased rebound congestion when use is discontinued. Instead, the client may opt for saline nasal sprays, which relieve congestion without rebound effects. Probiotics have shown to be effective in reducing acute sinusitis and other upper respiratory tract infections. Sinusitis is much more common in clients with asthma, with up to half of those with moderate-to-severe asthma experiencing chronic sinus inflammation. Allergies often precipitate sinusitis, and handwashing and removing allergens from clothing can reduce allergy symptoms and the risk of sinusitis. Smoking is a risk factor for sinusitis and should be entirely avoided. The client should drink 6 to 8 glasses of water daily to thin secretions and reduce the risk of infection. The client should sleep with the head upright, not flat, in order to allow sinus drainage.

A woman who is 5 days postpartum and has a history of pregnancy-induced hypertension, calls the hospital triage nurse hotline to ask for advice. She states, "I have had the worst pounding headache for the past two days. Since this afternoon, everything I look at appears blurred. Nothing I have taken helps." What action should the nurse take? A Advise the client to have someone bring her to the obstetrician's as soon as possible. B Ask the client to explain what exactly she has taken for the headache and how often. C Explain to the client that changes in her hormones may be the problem. D Instruct the client to call 911 to be brought to the nearest emergency room. Correct Answer (Blank)

Rationale: The woman is describing symptoms related to pregnancy-induced hypertension (PIH) that appears to be progressing to preeclampsia/eclampsia. PIH may progress to preeclampsia and eclampsia prior to, during, or up to 10 days after delivery. This places the woman at risk for seizure activity which is a medical emergency. The client should call 911 to be brought immediately to the closest emergency room (ER).

A client presents with a burn that is painful, pale, and waxy with large flat blisters. The client asks the nurse about the severity of the burn. What is the best response by the nurse? A "The burn is similar to a sunburn." B "The burn is a partial-thickness burn." Correct Answer (Blank) C "The burn is a first-degree burn." D "The burn is a full-thickness burn."

Rationale: The wound described is a deep partial-thickness burn. A superficial, i.e., first-degree, burn or sunburn is bright red and moist and might appear glistening with blister formation. A full-thickness burn involves all layers of the skin and may extend into the underlying tissue and is usually not painful.

An emergency department nurse is preparing discharge instructions for a child who experienced a seizure at school. The parent reports that this is the first seizure occurrence and denies a family history of seizures. What information should the nurse include? A "Do not worry. Seizure disorders can be treated with medications." B "Long-term treatment will prevent future seizures." C "This seizure may or may not mean your child has a seizure disorder. Further evaluation is needed." Correct Answer (Blank) D "Since this was the first seizure, it may not happen again."

Rationale: There are many possible causes for a childhood seizure. Some causes are transient, and others require long-term treatment to prevent further seizures. Causes of seizure in childhood include fever, central nervous system conditions, trauma, metabolic alterations, and idiopathic (unknown) etiologies. EEG, an electroencephalogram, is a test commonly used to evaluate seizure disorders.

A nurse is providing education to a client with an ileostomy about self-care management. Which statement made by the client indicates to the nurse further teaching is required? A "I will make sure to leave enough space between the appliance and the stoma." Correct Answer (Blank) B "I will drink plenty of water to avoid dehydration." C "Eating crackers and toast can decrease the amount of gas." D "The stoma should appear round, pink, and hydrated."

Rationale: There should be none-to-minimal skin visible between the stoma and the appliance. Exposed skin increases the risk of leakage and can disrupt skin integrity. Ileostomies can have a large output due to the watery consistency of the stool. Clients should be encouraged to drink plenty of water. Foods with high fiber content will reduce the amount of intestinal gas produced. A stoma should be pink and moist. Variations in color can signal decreased oxygen or circulation.

A client who is two days post abdominal surgery has the following vital signs: blood pressure of 120/70 mm Hg, heart rate of 110 bpm, respiratory rate of 26 breaths per minute and a temperature of 100.4°F (38°C). The client suddenly develops severe shortness of breath, cyanosis and pallor. Which assessment is the priority? A Auscultate the lungs for diminished breath sounds. Correct Answer (Blank) B Palpate the pulses for bounding and irregularity. C Check for orthostatic hypotension. D Assess the pupils for unequal responses to light.

Rationale: This client could be experiencing a complication from surgery, such as a pulmonary embolism (PE). A PE occurs as a result of a piece of a clot from the veins in the leg that has broken off and traveled to the lungs. The breath sounds will most likely be diminished or absent in the lung where the embolus lodged, thus a respiratory assessment is the priority.

An adult client who has been experiencing a seizure for approximately 15 minutes is brought to the emergency department by private vehicle. Which intervention should the nurse implement first? A Obtain a STAT electroencephalogram. B Administer levetiracetam intravenously. C Administer lorazepam intravenously. Correct Answer (Blank) D Obtain a STAT 12-lead electrocardiogram.

Rationale: This client is experiencing status epilepticus and is in immediate need of medication to stop the seizure. Of the provided options, the highest priority would be to administer the intravenous (IV) lorazepam to stop the seizure. While levetiracetam, an anticonvulsant, may be indicated for the client, lorazepam, a benzodiazepine, would be administered first in an attempt to stop the seizure quickly. An electroencephalogram (EEG) is an important test when evaluating for seizures, but it would not be highest priority at this time. A 12-lead electrocardiogram (ECG) may be part of a more general diagnostic work-up for many clients, but it would be a lower priority than stopping the seizures.

The nurse is performing a surgical dressing change on a client who had a laparotomy five days ago. The nurse notices the incision edges are separated and there is a visible bulge of organ tissue protruding from the wound opening. Which is the best way for the nurse to dress the incision before leaving the room to call the surgeon? A Place iodine-soaked gauze over the wound and then cover it with an abdominal pad. B Approximate the wound edges as much as possible with wound-closure strips. C Apply antibiotic ointment to the wound and cover it with a non-adherent dressing. D Cover the wound with sterile gauze moistened with sterile 0.9% saline. Correct Answer (Blank)

Rationale: This client likely has a wound evisceration, a complication of surgery. An evisceration is when a surgical wound opens and has protrusion of internal organs. This is considered a surgical emergency. The nurse should notify the surgeon of this finding immediately. When evisceration occurs, the best way to dress the wound is to cover it with sterile gauze dampened with sterile 0.9% saline using sterile technique.

A client with a history of asthma and kidney stones is admitted with a diagnosis of recurrent renal calculi. The client experiences shortness of breath following a lithotripsy. The nurse auscultates the client's lungs and finds decreased air movement with no wheezing. The arterial blood gas (ABG) results are pH 7.31, PaO2 53 mm Hg, PaCO2 50 mm Hg, and O2 sat 82%. Which of the following actions are appropriate for the nurse to take? Select all that apply. Call respiratory therapy Correct Answer (Blank) Start oxygen via nasal cannula Prepare for possible intubation Correct Answer (Blank) Administer a short-acting bronchodilator via nebulizer Correct Answer (Blank) Contact the health care provider Correct Answer (Blank) Increase IV fluids Start high flow oxygen via face mask Correct Answer (Blank)

Rationale: This client needs emergency treatment to open the airways and improve gas exchange. The absence of lung sounds without wheezing indicates a severe narrowing of the airways in asthma with minimal air movement. Emergent intervention to open the closed airway including possible intubation is indicated. The high PaCO2 and low pH indicate respiratory acidosis due to inadequate gas exchange. The low oxygen saturation and PaCO2 indicate severe hypoxemia requiring high flow oxygen via a mask.

A parent brings a 3 month-old infant into the clinic, reporting that the child seems to be spitting up all the time and has a lot of gas. The nurse expects which findings on the initial history and physical assessment? A Increased sleeping and fatigue B Diarrhea and poor skin turgor C Restlessness and irritability Correct Answer (Blank) D Increased temperature and lethargy

Rationale: This infant could be experiencing gastroesophageal reflux or perhaps an allergic response to the formula. Restlessness, irritability and increased mucus production can develop if an allergy is present. Soy-based formula may be recommended when allergies to the proteins in cow's milk formulas are suspected. Protein hydrolysate formulas are available when babies have a milk or soy allergy. Reflux would be treated with an acid-reducing medication such as ranitidine and positioning with the head elevated after feeding and while sleeping to reduce symptoms causing esophageal irritation.

The nurse is working on a medical-surgical floor and is making initial safety rounds on the clients. Which statement made by a client would require immediate action by the nurse? A "When I take in a deep breath, it stabs like a knife." B "The pain came on after dinner. That soup seemed very spicy." C "I feel pressure in the middle of my chest like an elephant is sitting on my chest." Correct Answer (Blank) D "When I turn in bed to reach the remote for the TV, my chest hurts."

Rationale: This is a classic description of chest pain in men caused by myocardial ischemia, requiring immediate assessment and intervention to prevent possible damage to the heart muscle. Pain after spicy food is often the result of irritation and gastric indigestion. The pain with a deep breath is typically from an inflammation of the pleural covering of the lung, called pleurisy. Pain with movement of the chest, such as turning in bed, is typically caused by costochondritis, which is inflammation of the cartilage between the ribs and the sternum, and can be reproduced by palpation of the painful area.

The L&D nurse is caring for a pregnant client at 40-weeks gestation who was admitted with new onset contractions at 8 am. At 10 am, the client is ready to give birth. Based on this abnormal labor pattern, which potential complication should the nurse monitor the client for? A Placenta previa B Eclampsia C Cesarean delivery D Fetal hypoxia Correct Answer (Blank)

Rationale: This labor pattern is considered "precipitous," which is defined as active labor lasting less than three hours. Because the contractions are coming rapidly, with little time in between contractions, there is a risk of fetal hypoxia. The other complications are not associated with precipitous labor.

The nurse is assessing a client who just returned to the medical surgical unit after a segmental lung resection surgery. During the assessment, the client is coughing and clearing their throat. What is the first action the nurse should take? A Apply the pulse oximeter and monitor oxygen saturation. B Administer the PRN pain medication. C Assist the client to turn, deep breathe and cough. D Suction excessive tracheobronchial secretions. Correct Answer (Blank)

Rationale: This type of surgery involves removing a bronco-vascular segment of a lung lobe. It is typically used to remove small, peripheral lung tumors. Surgical manipulation during this procedure, along with anesthesia, and increased mucus production can lead to airway obstruction, which is why the nurse may need to suction the client if there are excessive secretions. The first action the nurse should take is to suction the excessive secretions. Since this client just returned from surgery, it is not the time to ask the client to turn, cough and deep breathe. Vital signs and oxygen saturation are important data to gather, but clearing the client's airway by suctioning needs to be done first.

The client is admitted with a pressure ulcer that's two inches in diameter with no tunneling. It is a shallow open ulcer with loss of dermis and a red/pink wound bed. The nurse observes some serous drainage. What intervention does the nurse anticipate will be ordered to treat this wound? A Alternating pressure pad overlay for the bed B Alginate dressing with silver added C Hydrogel dressing Correct Answer (Blank) D Whirlpool treatment and debridement

Rationale: This ulcer is a partial thickness wound. These types of wounds heal by tissue regeneration, which is why the nurse would expect a gel dressing to be ordered. This dressing will keep the wound moist, provide protection from infection and promote healing; also, the cool sensation provided by the gel offers pain relief. Pink/red wound edges are considered normal in the inflammatory stage of healing; the wound does not require debridement. There is nothing to indicate that there's an infection, which is why the alginate with silver is not needed; also, alginate dressings are better for wounds with moderate-to-heavy drainage and are good for filling cavities or tracts. An alternating pressure pad overlay would not treat the wound.

The nurse is providing discharge instructions for a client diagnosed with bacterial pneumonia. What is the most important information to convey to the client? A "Take your temperature every day." B "You will need another chest x-ray in six weeks." C "Complete all of the antibiotics as prescribed." Correct Answer (Blank) D "Take at least two weeks off from work."

Rationale: To avoid a recurrence of infection, the client must complete all the prescribed medications at the prescribed dosing intervals. It should be explained to the client that it may take two weeks or more for the energy level to return to normal, but one does not necessarily need to be off work for two weeks. The health care provider may order a follow-up chest x-ray, but this is not always done or a priority at this time. It is also not important to take the temperature daily unless symptoms (such as chills, shortness of breath, chest pain, night sweats) worsen or return.

The nurse is developing a plan of care for a client who underwent total hip arthroplasty 24 hours ago. Which interventions should the nurse include? Select all that apply. Assist the client with a clear liquid diet Encourage the use of an abduction pillow or splint between the legs Correct Answer (Blank) Encourage the client to use the incentive spirometer every 2 hours Correct Answer (Blank) Encourage the client to perform leg exercises while in bed Correct Answer (Blank) Remind the client to not bend the knee of the affected leg while seated Provide a seat riser for the toilet or commode Correct Answer (Blank)

Rationale: To prevent postoperative complications and complications related to immobility, the client should be up in a chair as soon as possible after surgery. While seated, the client should bend the affected leg at the knee. The nurse should reinforce the teaching of simple leg exercises while in bed and the use of an abduction pillow or foam wedge to prevent adduction. To prevent atelectasis and pneumonia the client should be encouraged to use an incentive spirometer every 2 hours. Once the client is alert after surgery and not experiencing nausea or vomiting, they can resume a regular diet.

The nurse is caring for a child who has just returned from surgery following a tonsillectomy and adenoidectomy. Which action by the nurse is most appropriate? A Allow the child to drink through a straw B Place the child in a supine position C Offer ice cream every two hours D Observe swallowing patterns Correct Answer (Blank)

Rationale: Tonsillectomies and adenoidectomies are the removals of a client's tonsils and adenoids. These procedures are routinely performed when a client suffers from frequent bouts of tonsillitis or resistant forms of tonsillitis. Complications of these procedures include bleeding, infection, and dehydration. In the postoperative area, clients should be positioned at a 45° angle. This position allows the client to maintain a patent airway. It also can prevent aspiration in the event that the client begins to hemorrhage. One manifestation of bleeding includes frequent swallowing. It is imperative that the nurse monitors the client's swallowing patterns. In the postoperative area, clients should drink from a glass, not a straw. Straws could disrupt the suture lines from the procedure. After a tonsillectomy, clients should gradually introduce fluids back into their diet. The literature suggests starting with clear fluids, not full liquids. Starting full liquids may cause nausea, vomiting, and frequent coughing.

A hospitalized 8-month-old infant is receiving digoxin to treat Tetralogy of Fallot. Prior to administering the next dose of the medication, the parent reports that the baby vomited one time, just after breakfast. The infant's heart rate is 92 bpm. What action should the nurse take? A Reduce the next dose by half and then resume the normal medication schedule. B Double the next dose to make up for the medication lost from vomiting. C Give the scheduled dose after the client is done eating lunch. D Hold the medication and notify the primary health care provider. Correct Answer (Blank)

Rationale: Toxic side effects of digoxin include bradycardia, dysrhythmia, nausea, vomiting, anorexia, dizziness, headache, weakness and fatigue. It isn't typically necessary to hold the medication for infants and children if there is only one episode of vomiting. However, it is appropriate to hold the medication and notify the primary health care provider (HCP) of the vomiting episode and the lower than normal heart rate. A digoxin level may need to be drawn. The normal resting heart rate for infants 1 to 11 months old is 100 to 160 bpm.

A client is transported to the emergency department after a motor vehicle accident. When assessing the client 30 minutes after arrival, the nurse notes several physical changes. Which finding requires immediate attention? A Tracheal deviation Correct Answer (Blank) B Increased restlessness C Tachycardia D Tachypnea

Rationale: Tracheal deviation is a sign that a mediastinal shift has occurred, most likely due to a tension pneumothorax. Air escaping from the injured lung into the pleural cavity causes pressure to build, collapsing the lung and shifting the mediastinum to the opposite side. This obstructs venous return to the heart, leading to circulatory instability and may result in cardiac arrest. This is a medical emergency, requiring emergency placement of a chest tube to remove air from the pleural cavity relieving the pressure. The other findings are most likely related to the potential pneumothorax.

An infant has just returned from surgery for placement of a gastrostomy tube as an initial treatment for a diagnosis of tracheoesophageal fistula (TEF). The mother asks, "When can the tube be used for feeding?" Which is the most appropriate response by the nurse? A "The stomach contents and air must be drained first." Correct Answer (Blank) B "The feeding tube can be used immediately." C "Healing of the anastomosis must be complete before feeding." D "Feedings can begin in five to seven days."

Rationale: Tracheoesophageal fistula (TEF) is an abnormal opening between the esophagus and trachea. After insertion of a gastrostomy tube for TEF, feeding will not begin for about 24 hours, and the tube will be connected to a drainage bag to empty the stomach contents and air. Giving tube feeding through a gastronomy tube is independent of healing of the esophageal anastomosis.

A correctional facility nurse is assessing a 67-year-old client who presents with anorexia, weight loss, and purulent, blood-tinged sputum. The nurse recognizes these symptoms are indicative of which disease process? A Tuberculosis Correct Answer (Blank) B Pneumonia C Influenza D Sinusitis

Rationale: Tuberculosis is an infectious disease that affects the respiratory system. Risk factors for tuberculosis include poorly ventilated and crowded environments, such as correctional facilities. Expected findings include blood-tinged, purulent sputum, weight loss, anorexia, and night sweats. Pneumonia is an inflammatory process that leads to excess fluid in the lungs. The symptoms displayed in the scenario are not indicative of pneumonia. Influenza is a viral infection that affects the respiratory system. Blood-tinged sputum is not an expected finding. Sinusitis is an inflammation of the mucous membranes in the nasal airway. Anorexia and weight loss are not associated with sinusitis.

The nurse is caring for a client who is undergoing transcutaneous pacing for symptomatic bradycardia. Which of the following instructions should the nurse provide to the client? A "The procedure will start with the highest energy and may be decreased as needed to restore a normal cardiac rhythm." B "Pacing electrodes will be applied to your chest and you will feel muscle contractions with each heartbeat." Correct Answer (Blank) C "This procedure will eliminate the possibility of future bradydysrhythmias from occurring." D "The energy delivery will be timed at a specific point within your current cardiac rhythm."

Rationale: Transcutaneous pacing involves delivering electrical pulses to stimulate ventricular depolarization in symptomatic bradycardia. The pacing stimulus is delivered using the lowest energy required to stimulate a QRS complex on the ECG, indicating ventricular depolarization. Transcutaneous pacing is an emergency procedure for profound bradycardia, it is a temporary measure until a more permanent device such as a permanent pacemaker can be inserted. Synchronized cardioversion involves delivering a dose of energy timed to the R wave of the QRS complex, this is not required for transcutaneous pacing.

The nurse is assigned to care for a client who was diagnosed with an intracranial aneurysm that has since resolved. To minimize the risk of another rupture, the nurse should plan to take which action? A Keep the client in a upright sitting position B Apply a warming blanket for temperatures of 98 °F (36.6 C°) or less C Treat any elevation in blood pressure Correct Answer (Blank) D Avoid arousal of the client except for family visits

Rationale: Treating any blood pressure elevation and reducing stress by maintaining a quiet environment, including during family visits, will assist in minimizing the risk of a cerebral bleed. An upright sitting position with the pressure on the hip area can lead to increased intracranial pressure; this position should be avoided. A warming blanket is inappropriate to use.

The nurse is caring for a symptomatic client diagnosed with a 2nd degree, Mobitz Type II heart block. Transcutaneous pacing has been initiated. Which of the following findings indicates that the intervention is having the intended effect? A Normal sinus rhythm shows on the electrocardiogram (EKG) B Ventricular pacing is evident on EKG with a HR of 65 Correct Answer (Blank) C Pacemaker spikes are present after the QRS complex on the EKG D Accelerated ventricular rhythm is present on the EKG with a HR of 68

Rationale: Treatment for a Mobitz Type II involves initiating pacing as soon as this rhythm is identified. Type II blocks imply structural damage to the AV conduction system. This rhythm often deteriorates into a complete heart block. These patients require transvenous pacing until a permanent pacemaker is placed. The line that represents pacing is called a pacemaker spike. The appropriate ECG complex should immediately follow the pacing spike; therefore, a P wave should follow an atrial pacing spike, and a QRS complex should follow a ventricular pacing spike. Capture is a term used to denote that the appropriate complex followed the pacing spike. Pacing does not result in normal sinus rhythm or accelerated ventricular rhythms.

The nurse is evaluating a client who was admitted for a small bowel obstruction and dehydration. Which observation by the nurse would indicate that the dehydration is improving? A The client reports the passing of flatus. B The client denies any nausea or vomiting. C The client has normoactive bowel sounds D The client voided 300 mL of urine in the past two hours. Correct Answer (Blank)

Rationale: Treatment for dehydration typically includes rehydration by increasing oral fluid intake and/or administering intravenous fluids. During dehydration, urine output usually decreases as the kidneys attempt to restore fluid volume by increasing water reabsorption. The fact that the client voided 300 mL in two hours indicates that fluid volume has been restored, the dehydration is resolved and the kidneys are eliminating excess water. The other findings pertain more to functioning of the client's bowel not the client's fluid balance.

A client is brought to the emergency department with a blood glucose level of 52 mg/dL. The client appears weak, tired, but is awake and talking. After drinking 4-ounces of juice, the client's blood glucose does not rise above 70 mg/dL. Which actions by the nurse would represent appropriate care of this client? Select all that apply. Determine blood sugar management medications Correct Answer (Blank) Offer a 12-ounce (355 mL) can of cola with added sugar Recheck blood sugar in 15 minutes Correct Answer (Blank) Offer 8-ounce (237 mL) glass of milk Correct Answer (Blank) Instruct the client to not take more insulin today

Rationale: Treatment for hypoglycemia is to consume approximately 15-20 grams of glucose or simple carbohydrates. Common examples of 15 grams of simple carbohydrates include: 2 tablespoons of raisins; 118 mL of juice or regular soda (not diet); 237 mL of nonfat or 1% milk; and 1 tablespoon of honey. In a clinical setting, the client may also be given glucose tablets. If after 15 minutes the blood sugar is still below 70 mg/dL (3.89 mmol/L), the client can be given another 15-20 grams of simple carbohydrates (this is also known as the "15 - 15 rule.") It's always a good idea to confirm how the client manages his/her diabetes.

The clinic nurse is following up with a client who was seen a few days ago for trigeminal neuralgia. Which action by the client indicates an understanding of how to manage the condition? A Keeps the environment at a moderate temperature and free from drafts. Correct Answer (Blank) B Eats a bowl of hot, steaming soup every day for lunch. C Takes an analgesic after performing household chores. D Performs vigorous brushing of teeth twice per day.

Rationale: Trigeminal neuralgia is a disruption in the cranial nerve and causes sudden, severe, brief stabbing pain. Keeping the environment at a moderate temperature and free from drafts can reduce the risk of triggering an acute attack. The client will often avoid oral care because of the potential for pain, but should be instructed to use a soft-bristles toothbrush and gently brush their teeth. Meals should be warm-to-cool, not hot, to avoid pain exacerbation. Analgesics should be taken before performing activities that can increase feelings of discomfort.

The home health nurse is reviewing information with a client who is being treated for pulmonary tuberculosis. Which statement by the nurse is correct? A "You can stop the medications once your symptoms have resolved." B "You should avoid public transportation and crowds in enclosed areas." Correct Answer (Blank) C "You should not leave your home until your cough is completely gone." D "Your family members should get the tuberculosis vaccine."

Rationale: Tuberculosis (TB) is an infectious disease that usually involves the lungs but can affect other organs. Treatment involves drug therapy and the prevention of transmission. Drug therapy typically consists of several medications that must be taken for several months, even if symptoms have subsided. Clients with pulmonary TB are not required to remain indoors, but should avoid travel on public transportation and trips to public places. Currently, there is no recommended vaccine for TB in the U.S.

A client with late-stage lung cancer was started on chemotherapy two days ago and might be experiencing tumor lysis syndrome. Which findings support this diagnosis? Select all that apply. A serum creatinine level of 2.4 mg/dL Correct Answer (Blank) A serum calcium level of 13.8 mg/dL A serum phosphorus level of 1.8 mg/dL A serum uric acid level of 22 mg/dL Correct Answer (Blank) Weakness and muscle cramps Correct Answer (Blank) A serum potassium level of 3.0 mg/dL

Rationale: Tumor lysis syndrome (TLS) is a metabolic complication in response to chemotherapy and is a medical emergency. Massive cell destruction releases intracellular components, such as potassium and phosphate, that are metabolized into uric acid. High levels of uric acid crystalize in the distal tubules of the kidneys and lead to acute kidney injury (AKI), as evidenced by the elevated creatinine level. TLS usually occurs within 24-48 hours after the initiation of chemotherapy and may persist for about 5-7 days. Hallmark signs of TLS include hyperuricemia, hyperphosphatemia, hyperkalemia, and hypocalcemia. In addition, the client might experience weakness, muscle cramps, nausea and vomiting (N/V), and diarrhea.

During a routine clinic visit, the nurse is providing education to a client with a history of Type 1 diabetes mellitus. The client's glycosylated hemoglobin (HbA1C) was 11%. Based on this result, which teaching concept should the nurse emphasize? A Proper storage of oral medication used to decrease glucose level B Rotate injection sites with every injection C Assess blood sugar and treat with insulin before meals and at bedtime Correct Answer (Blank) D Continue with the current effective regimen

Rationale: Type 1 diabetes mellitus is caused by an autoimmune destruction of the beta cells within the pancreas. These cells are responsible for making insulin. Because of this the client will be dependent on insulin and no oral antihyperglycemic agents will be effective. A glycosylated hemoglobin of 11% is elevated and indicates inadequate glucose control over a period of 2 to 3 months. Rotation of sites should be done regularly to prevent skin breakdown and to ensure proper delivery of the drug, but it is not a priority at this time.

The nurse is performing a primary assessment on an unconscious client who arrived at the emergency department after a motor vehicle accident. Which action will the nurse perform prior to assessing obstruction of the airway? A Perform in-line stabilization of the neck Correct Answer (Blank) B Place the client in a sitting position C Extend the client's head using a chin lift maneuver D Insert a nasopharyngeal airway

Rationale: Unconscious clients are unable to verbalize pain or neurovascular deficits. Trauma clients have a potential for cervical spine injuries. The cervical spine must be protected prior to performing all assessments. In-line stabilization of the neck provides cervical spine alignment. Placing the client in a sitting position may injure the cervical spine. Clients should be kept supine with the cervical spine protected. The chin lift maneuver can hyperextend the neck and cause injury to the cervical spine. The neck must be stabilized prior to clearing the airway. Inserting a nasopharyngeal airway can help maintain the client's airway. However, the cervical spine should be protected prior to this intervention.

A client presents to the clinic experiencing fatigue, nausea, and a recent unintentional weight loss of 6 pounds. The client tells the nurse they had unprotected sexual intercourse 2 weeks prior. The nurse identifies these manifestations as symptoms of which disease? A Hepatitis A B Syphilis C Viral meningitis D Human immunodeficiency virus Correct Answer (Blank)

Rationale: Unintentional weight loss, nausea, and fatigue are characteristic of a human immunodeficiency (HIV) viral infection. Symptoms occur between 2 to 4 weeks after initial exposure. Hepatitis A is a viral infection transmitted through the fecal-oral route. Unintentional weight loss is not an expected finding. The average incubation period for hepatitis A is 28 days. Syphilis is a sexually transmitted infection (STI) with an average incubation period of 21 days. Syphilis is characterized by a chancre sore at the site of viral entry. Viral meningitis is a viral illness caused by conditions, such as herpes, measles, and influenza. Sexual transmission is not a source of infection.

A community health nurse is vaccinating a group of children for protection against measles. The nurse knows that the vaccine will provide the clients with which type of immunity? A Passive B Natural C Acquired D Active Correct Answer (Blank)

Rationale: Vaccines provide active immunity. Vaccines signal the immune system to produce antibodies against the disease. Passive immunity occurs when antibodies are transferred to a person via transfusions or from a mother to a newborn via the placenta. Natural immunity is a natural defense mechanism to specific toxins or antigens. Acquired immunity is obtained after actual exposure and infection with the disease.

The nurse is taking a health history from the parents of a child who is admitted for Reye's syndrome. Which recent illness would the nurse identify as a significant risk of developing Reye's syndrome? A Hepatitis B Influenza Correct Answer (Blank) C Meningitis D Rubeola

Rationale: Varicella (chickenpox) and influenza are viral illnesses that have been identified as risks for the development of Reye's syndrome. It is important for nurses to educate parents to not use aspirin in children (birth to 19 years of age). The use of aspirin in the presence of viral infections can increase the risk of Reye's syndrome for children.

The nurse enters the room of an adult client in cardiac arrest with cardiopulmonary resuscitation already in progress. The client's bedside telemetry monitor shows ventricular fibrillation. What should the nurse do next? A Assist with the insertion of a large-bore IV catheter. B Quickly leave the room and notify the client's next-of-kin. C Assist with preparing the client for defibrillation. Correct Answer (Blank) D Prepare to administer two rescue breaths.

Rationale: Ventricular fibrillation (V-Fib) is a life-threatening dysrhythmia that requires immediate defibrillation to attempt to restore a viable cardiac rhythm. V-Fib will cause death within minutes due to the complete lack of cardiac output and tissue perfusion. The other actions should be implemented after defibrillation has been performed or attempted.

The nurse is assessing a client who is unresponsive and notes the absence of palpable pulses. When reviewing the cardiac rhythm strip, which abnormal rhythm should the nurse recognize the client is experiencing? Atrial fibrillation B Ventricular fibrillation Correct Answer (Blank) C Ventricular tachycardia D Atrial tachycardia

Rationale: Ventricular fibrillation (V-fib) is the abnormal cardiac rhythm associated with cardiac arrest. V-fib will have a rapid, disorganized waveform with no identifiable complexes. Atrial fibrillation will have a fast, irregular rate with no identifiable P waves. Atrial tachycardia will have a regular rhythm with a rate over 100 and identifiable P waves on the rhythm strip. Ventricular tachycardia will have a fast, regular rate with wide QRS complexes.

The nurse is planning care for a client with stage 2 human immunodeficiency virus (HIV). Which intervention will the nurse perform to help manage wasting syndrome? A Encourage independent activities of daily living. B Assist with range of motion exercises. C Serve foods with a high fat content. D Provide an oral rinse after every meal. Correct Answer (Blank)

Rationale: Wasting syndrome is a complication of HIV and is characterized by diarrhea, chronic weakness, and weight loss of more than 10% of body weight. Providing oral rinses increases appetite and decreases mouth pain associated with ulcerations. Activities of daily living (ADLs) should be encouraged for every client. ADLs alone will not help manage wasting syndrome. Assisting with range of motion is a musculoskeletal intervention and will not manage wasting syndrome. Wasting syndrome causes fat intolerance. Foods should contain low-fat content to decrease the complications of fat malabsorption, such as gallstones, abdominal pain, and gas.

The nurse is working in a long-term health care facility and assessing several clients. Which client is at highest risk for developing a pressure ulcer? A A 79-year-old malnourished client on bed rest Correct Answer (Blank) B An incontinent client who has had three diarrhea stools in the past hour C An 80-year-old ambulatory client with a history of diabetes mellitus D An obese client who uses a wheelchair

Rationale: Weighing significantly less than ideal body weight increases the number and surface area of bony prominences, which are susceptible to pressure ulcers. In addition, malnutrition is a major risk factor for pressure ulcers, from poor hydration and inadequate protein intake. Note that this is a priority question so that all of the clients are at risk for pressure ulcers. However, the question asks for the client with the highest risk.

The nurse is reviewing the chart of a client who was admitted after having been found lying on the bathroom floor in their home. The client's family reports that the client could have been lying on the floor for over 12 hours. Which laboratory result should be of greatest concern to the nurse? A Serum glucose level of 162 mg/dL B Serum white blood cell count of 14,000/mm3 C Serum hemoglobin level of 10.8 g/dL D Serum creatinine level of 4.2 mg/dL Correct Answer (Blank)

Rationale: When a person falls and lies immobile for an extended period of time, muscle tissue will start to break down. This is called rhabdomyolysis. Rhabdomyolysis leads to the release of myoglobin (muscle protein) into the bloodstream. Myoglobin breaks down into substances that will damage the kidneys, causing acute kidney injury (AKI) as evidenced by the client's severely elevated creatine level. (A normal range would be between 0.5 to 1.2 mg/dL). Although the client's other lab values are also outside of the normal range, the values are not as severely elevated or decreased as the creatinine level which represents the greatest concern to the client's condition at this time.

The nurse is assessing a client's Hemovac drainage system. Which of the following assessment findings should the nurse report to the provider immediately? A Serosanguineous drainage is present in the collection chamber. B The drain is secured to the client's gown with a safety pin. C An obstruction is present in the drainage tube. Correct Answer (Blank) D The collection chamber does not expand quickly after being compressed.

Rationale: When assessing the client's Hemovac drain, it is important to assess the insertion site, the drainage, and the drain tubing. Significant increases/decreases in the amount of drainage, insertion site abnormalities, or obstructions of the tube should be reported to the provider immediately. Serosanguineous drainage and securing the drain to the client's gown are normal findings. The collection chamber should not expand quickly after compressing it; this indicates negative pressure is not being maintained.

The nurse is assessing a client who has a closed chest drainage system. Which action by the nurse is appropriate? A Manually squeezing the tube to remove additional drainage B Marking the level of drainage on the output chamber with date and time Correct Answer (Blank) C Emptying the drainage from the collection chamber into a biohazard container D Clamping the drainage tube while the comprehensive assessment is completed

Rationale: When assessing the client's closed chest drainage system, it is important to assess the drainage in the collection chamber for color and amount. Significant increases/decreases in the amount of drainage or obstructions of the tube should be reported to the provider immediately. The collection chamber should not be emptied to measure output. Instead, the level of drainage should be marked on the chamber with the date and time to determine the volume change from the previous assessment. The tube should not be squeezed, milked, or stripped. Clamping the tube should only be done for very short periods of time such as changing the drainage unit.

The nurse observes an unlicensed assistive person (UAP) providing care to a client who had a total hip arthroplasty 24 hours ago. Which action by the UAP would require the nurse to intervene immediately? A Placing a raised toilet seat in the client's bathroom. B Reminding the client not to cross their legs. C Standing by the client's non-operative side during ambulation. Correct Answer (Blank) D Placing non-slip foot wear on the client prior to ambulation.

Rationale: When assisting the client during ambulation following a total hip arthroplasty, the UAP should stand on the operative side (i.e., the side of the surgery) to help provide support to the client because that is the client's weaker side. The other actions are appropriate for this client and do not require intervention by the nurse

The off-duty nurse witnesses a motor vehicle accident and is concerned that the driver of the automobile may be injured. What should the nurse do first? A Minimize movement of the driver's cervical spine. B Consider scene safety to prevent further injury. Correct Answer (Blank) C Check the driver's respiratory rate. D Check the driver's pulse.

Rationale: When attempting to render aid after a motor vehicle collision, it is critically important that the responder first consider scene safety. Responders should assess the scene for risks to safety to prevent further injury to themselves, the victim and other motorists on the road. Minimizing the movement of the driver's cervical spine, checking the driver's pulse and checking the driver's respiratory rate may all be indicated in the scenario, but scene safety should be considered first.

The nurse is caring for a client with Type I diabetes. Which finding requires immediate intervention by the nurse? A Reduced sensation in the periphery B Mild discomfort at the injection site C Intense thirst and increased urination D Diaphoresis and shakiness Correct Answer (Blank)

Rationale: When caring for a client with diabetes mellitus, the nurse must be knowledgeable about the clinical manifestations of hyperglycemia and hypoglycemia. Hyperglycemia is characterized by polyphagia, polydipsia, and polyuria (the 3 Ps). The client will also experience weight loss as the cells are not receiving adequate amounts of glucose for energy. Signs of hypoglycemia include diaphoresis (sweating) with cool skin. The client may shake and become confused. It is critical that the nurse recognize these signs and assess the client's blood sugar. Hypoglycemia will require immediate attempts to raise blood sugar and prevent diabetic coma. Hyperglycemia, while concerning, is not as critical as hypoglycemia. Decreased sensation in the periphery is a finding consistent with diabetic neuropathy, which develops over time. The client may describe sensations of tingling, pain, or numbness. The client may feel mild discomfort at the site of insulin injections, and this should be monitored.

A client admitted with congestive heart failure is experiencing severe dyspnea and states, "I feel like something is terribly wrong!" The client is restless and begins to cough up large amounts of pink, frothy sputum. The client's skin is a dusky, gray color. His oxygen saturation levels have decreased from 92% to 76% in the last hour. Which action should the nurse implement first? A Check vital signs B Call the health care provider C Place the bed in high Fowler's position D Administer the PRN ordered oxygen Correct Answer (Blank)

Rationale: When dealing with a medical emergency, the rule is to assess airway first, then breathing, and then circulation. Starting oxygen is the priority. The other actions should also be implemented as quickly as possible, including activation of the rapid response team. The client is experiencing an acute episode of fulminant pulmonary edema, likely as a result of a new and severe cardiac event and possible cardiogenic shock. Emergency assessment and intervention is indicated to prevent cardiac arrest and possible death.

The nurse is monitoring a client with fluid volume excess (FVE). Which of the following would provide the nurse with the most accurate evaluation of the client's fluid balance? A Strict intake documentation B Daily weights Correct Answer (Blank) C Weekly hemoglobin testing D Lower extremity edema measurements

Rationale: When monitoring clients with fluid volume excess (FVE), the nurse should evaluate the client's fluid balance for any increase or decrease in the balance. Physical assessments, such as measuring edema and auscultating lungs, provide information to the nurse if the client has an excess fluid, but does not indicate if the client has an increase in the balance. Laboratory results, such as hemoglobin and hematocrit or electrolytes, can determine if there is an excess in fluid volume but does not indicate if there is a change in the balance. Strict intake documentation is appropriate for a client with FVE but is not accurate. Daily weights are the most accurate way to evaluate fluid volume balance.

The nurse is planning care for a client with Alzheimer's disease. The client has episodes of bowel and bladder incontinence. Which intervention should the nurse include in the client's plan of care? A Limit PO fluid intake to reduce episodes of incontinence. B Schedule toileting for the client every two hours during the day. Correct Answer (Blank) C Place a picture of a toilet on the client's bathroom door. D Wake the client at night to ask about their toileting needs.

Rationale: When planning care for a client with Alzheimer's disease the nurse should promote bowel and bladder continence. The client may need prompting from health care personnel to complete the act of toileting. The client may remain continent of bowel and bladder for long periods if taken to the bathroom or given a bedpan or urinal every two hours or more often during the day. The nurse encourages the client to drink adequate fluids to promote optimal voiding. Placing a picture on the bathroom door may help the client identify the bathroom if they have been voiding in inappropriate places like the sink or a wastebasket. Clients with Alzheimer's disease often have difficulty sleeping, so their treatment and medication schedule should be adjusted to provide uninterrupted sleep.

The nurse in the primary care office is following up with a client who has been experiencing frequent constipation. Which statement by the client about using psyllium (Metamucil) indicates that additional teaching is needed? A "I will take it together with my other medications." Correct Answer (Blank) B "I will take it in the morning with lots of water." C "I may notice some bloating while I am taking it." D "I will only take it until my constipation is relieved."

Rationale: When using psyllium to manage constipation, it needs to be taken with at least 240 mL of water. The client may experience abdominal discomfort or bloating while taking psyllium. When using psyllium to treat constipation, it should not be used long-term and be discontinued when the constipation has resolved. Bulk laxatives can interfere with the absorption of some medications, so it should be taken two hours before or two hours after other medications. The client's statement about taking the psyllium with other medications requires additional teaching.

A nurse is caring for a child who underwent a tonsillectomy an hour ago. The child's parents report to the nurse that the child feels very warm. Which intervention should the nurse do first? A Offer the child cold oral fluids. B Administer the prescribed acetaminophen. C Reassure the parent that this is normal after surgery. D Measure the child's temperature. Correct Answer (Blank)

Rationale: While a low-grade fever (>101°F or 38.3°C) is common after surgery, the nurse should assess the child's temperature prior to any action. The health care provider (HCP) should be contacted if the temperature is higher than 101.5° (38.6°C). After evaluating the child's temperature, the other options may be implemented. However, the child should not drink fluids until they are alert and should not be given straws, acidic juices, or red/brown fluids. Straws and acidic juices may cause surgical site damage and red/brown fluids may be confused with blood in emesis.

The nurse is caring for a client admitted to the hospital with severe left-sided flank pain and hematuria. Diagnostic tests indicate a kidney stone partially obstructing the left ureter. Which outcome is the most important for this client? A Pain controlled with medication B Adequate urinary elimination is maintained Correct Answer (Blank) C Verbalizes understanding of the disease process D Tolerates diet without nausea and vomiting

Rationale: While all options are appropriate to the care of this client, urinary elimination is the nursing priority. A stone that completely obstructs the ureter can cause hydronephrosis and potential kidney damage. Remember Maslow - physiologic needs are more important than nutritional needs. Pain control and teaching are lower priorities

The nurse is caring for two children who have had surgical repair of congenital heart defects. For which defect is it the highest priority to assess for findings of heart conduction disturbance? A Ventricular septal defect Correct Answer (Blank) B Aortic valve stenosis C Patent ductus arteriosus D Atrial septal defect

Rationale: While assessments for conduction disturbance should be included following repair of any defect, it is a priority for ventricular septal defect. A ventricular septal defect is an abnormal opening between the right and left ventricles. The atrioventricular bundle (bundle of His) is a part of the electrical conduction system of the heart. It extends from the atrioventricular node along each side of the interventricular septum and then divides into right and left bundle branches. Surgical repair of a ventricular septal defect consists of a purse-string approach or a patch sewn over the opening. Either method involves manipulation of the ventricular septum, thereby increasing risk of interrupting the conduction pathway. Consequently, postoperative complications often include conduction disturbances.

The nurse is preparing a presentation focusing on the prevention of Lyme disease. Which statement by a participant would require further clarification by the nurse? A "Lyme disease can spread to my brain if I don't seek treatment." B "I will call the doctor if I see a rash that looks like a bull's eye." C "Lyme disease is caused by a virus similar to the flu." Correct Answer (Blank) D "I should wear light-colored clothing and long pants when gardening."

Rationale: While the symptoms of Lyme disease are similar to influenza, Lyme disease is not caused by a virus. Lyme disease is caused by the spirochete, Borrelia burgdorferi, which is transmitted to humans by deer ticks. Because the ticks are so small, it is easier to see them on light-colored clothing. Long pants and long-sleeved shirts help protect individuals from insect bites. After being outdoors, individuals should assess their body for any ticks or rashes. Parents should be instructed to check children for ticks and rashes. There may be a "bull's eye" rash at the site of the tick bite. Without antibiotics, the disease can spread to the brain, heart and joints of the body.

An unlicensed assistive person (UAP) is giving a bath to a 5-year-old client with Wilms tumor. The UAP asks the nurse why there is a sign above the bed that says, "Do not palpate/press on abdomen." What is the best response by the nurse? A "Pressing on the abdomen might cause a bowel obstruction." B "Pressing on the abdomen could cause the tumor to spread." Correct Answer (Blank) C "Pressing on the abdomen would be very painful for the child." D "Pressing on the abdomen will cause the tumor to bleed."

Rationale: Wilms tumor (nephroblastoma) is a childhood cancer. It is the most common kidney tumor of childhood. The most common presenting sign is swelling or mass within the abdomen. The mass is characteristically firm, nontender, confined to one side and deep within the flank. The mass usually is discovered during routine bathing or dressing of the child. When caring for a child with Wilms tumor, it is important not to palpate the tumor or press on the abdomen, because manipulation of the mass may cause dissemination of cancer cells. To reinforce the need for caution, a sign should be posted near the bed that reads "Do not palpate/press on abdomen."

The nurse is caring for a newborn with tracheoesophageal fistula (TEF). Which assessment is the highest priority? A Observe the newborn for cyanosis Correct Answer (Blank) B Observe the newborn for tachycardia with activity C Monitor for fever over 101°F (38.3°C) D Monitor intake and output

Rationale: With TEF, there is an abnormal opening between the trachea and esophagus. Fluids can easily be aspirated into the trachea and lungs. The 3 Cs of TEF are choking, coughing, and cyanosis. The priority is to prevent aspiration and maintain an open airway. The other options are appropriate when monitoring any newborn. However, they are not specific to TEF.

A client diagnosed with testicular cancer has undergone a unilateral orchiectomy. The client expresses fears about his prognosis. What should the nurse understand about this type of cancer? A This cancer has a five-year survival rate of 90% or greater with early diagnosis and treatment. Correct Answer (Blank) B This surgery causes impotence and infertility. C With early intervention, the cure rate for testicular cancer is about 50%. D Intensive chemotherapy is the treatment of choice following surgery.

Rationale: With aggressive treatment and early detection/diagnosis the cure rate is generally 90% or greater. The other options are incorrect information. After unilateral orchiectomy, the remaining testicle can produce adequate sperm for fertility and impotence is unlikely.

The clinic nurse is caring for a client with a cardiac history who has a pacemaker. The client has intractable upper back pain and cancer is suspected. Which of the prescriptions should the nurse clarify? A Administration of oxycontin on a routine schedule B Complete metabolic panel C MRI of the spine with contrast Correct Answer (Blank) D Cardiac enzymes

Rationale: With the introduction of "MRI safe" pacemakers, many clients can have MRIs done. However, this needs to be clarified with the provider. The client has intractable pain, so oxycontin around the clock is a reasonable therapy. A CMP and cardiac enzymes will tell the provider additional information to make an appropriate diagnosis.

The nurse is completing a surgical incision dressing change for a client that is two days postoperative. Which of the following actions is appropriate? A Administering prescribed pain medication after the dressing change B Applying tape over the dressing with moderate tension C Applying the new dressing using sterile technique Correct Answer (Blank) D Disposing of the soiled dressing in the sharps container

Rationale: Wound care of the surgical incision is sterile to prevent the introduction of pathogens to the fresh incision. Pain medication should be given prior to wound care, especially if the client is experiencing incisional pain. Soiled wound dressings should be discarded in a biohazard waste container, and tape should not be applied with tension. Pressure or tension from a dressing can cause skin shearing and breakdown.

A postoperative client returns to bed after ambulation and tells the nurse, "I feel something popping out of my abdomen." The nurse notes protruding bowel from the abdominal incision. Which action does the nurse perform next? A Leaves the room to immediately call the healthcare provider. B Pushes the bowel back into the abdomen. C Places the client in a low Fowler's position. Correct Answer (Blank) D Covers the wound with an abdominal binder.

Rationale: Wound evisceration is a medical emergency. The nurse should place the client in a low Fowler's position to avoid further straining of the abdominal muscles. Clients should not be left unattended during a medical emergency. The nurse should call for assistance. The protruding bowel should not be pushed back into the abdomen. Surgical repair is required to correct the problem. The protruding bowel should be kept moist using sterile towels moistened with sterile saline.

The nurse is taking care of a client with a viral infection who has an absolute neutrophil count of 900/mm³. Which action will the nurse perform to ensure a protective environment? Place the client in a negative pressure room Restrict fresh vegetables from the client's diet order Apply an N-95 mask prior to performing client care Prohibit the client from receiving visitors

Restrict fresh vegetables from the client's diet order Rationale: An absolute neutrophil count (ANC) below 1,000/mm³ places the client at significant risk for infection. Clients with low ANC counts should be placed on neutropenic precautions and provided with a protective environment. Fresh vegetables can contain bacteria and should be avoided in clients who are immunocompromised. Placing the client in a negative pressure room is not necessary. Negative pressure rooms are used for airborne precautions. An N-95 mask is not required for neutropenic precautions. N-95 masks are used for airborne precautions. The client's visitors should be restricted, not prohibited. Visitors with an active infection should not be allowed to visit with the client.

The nurse is monitoring a client with new onset of atrial fibrillation with rapid ventricular response who is receiving prescribed IV diltiazem. Which finding observed by the nurse would indicate the treatment is effective? The ventricular heart rate is now 110 The prothrombin time (PTT) is 50 The client reports improved breathing The blood pressure is now 150/62

The ventricular heart rate is now 110 Rationale: Treatment for atrial fibrillation is to terminate the rhythm or to control ventricular rate. This is a priority because it directly affects the cardiac output. Rapid ventricular response reduces the time for ventricular filling, resulting in a smaller stroke volume. Control of rhythm is the initial treatment followed by anticoagulation. As the HR slows, the client will likely report improved rhythm. Diltiazem is a calcium channel blocker and is also used to reduce blood pressure. This would be a secondary effect and not the intended effect.

The nurse is caring for a client who was admitted for rib fractures and a hemothorax from a motor vehicle collision. The client currently has an epidural catheter in place and is receiving continuous epidural analgesia with morphine. Which of the following findings indicates an adverse response to the client's therapy? Bleeding from IV sites, mottled skin, confusion, and fever Apprehension, anxiety, restlessness, and shortness of breath Frequent urination, salivation, excessive tearing, and diaphoresis Urinary retention, excessive somnolence, itching, and nausea

Urinary retention, excessive somnolence, itching, and nausea Rationale: Epidural analgesia is given by clinician-given bolus, continuous infusion (basal rate), and patient-controlled epidural analgesia (PCEA). The most common opioids given intraspinally are morphine, fentanyl, and hydromorphone. The most common adverse effects of opioids are constipation, nausea, vomiting, pruritus, hypotension, and sedation. Respiratory depression, while less common, is the most serious and feared of the opioid adverse effects. Urinary retention is a complication of epidural analgesia. The other options are concerning but none reflect adverse reactions to the epidural administration of morphine.

The nurse is monitoring a client who is receiving external beam radiation therapy to the upper chest. The nurse notes the area of radiation has redness and skin breakdown. Which action should the nurse take? Apply lotion to the red areas Cover with an occlusive dressing Wash the area daily with warm water Draw a circle around the area to monitor for changes

Wash the area daily with warm water Rationale: Clients who are receiving external beam radiation are at increased risk for skin breakdown. To prevent complications, the nurse should wash the area daily with warm water. Lotions should be avoided to prevent irritation or infection. Occlusive dressings can increase the risk of moisture causing further breakdown or infection. Marking the area to monitor for changes is not appropriate as the area will have markings for the radiation beam.


संबंधित स्टडी सेट्स

Siff Clinical Informatics Boards study set 1

View Set

4.6 The Extended Marketing Mix - Seven Ps Model

View Set

Practice Questions Communicable Diseases

View Set

التركيب والوظيفة في الكائنات الحية

View Set

Personal Finance Insurance Test Reveiw

View Set

Bus Orgs - Fiduciary Duties (Owed by Corp. Officers, Directors, and Controlling SHs); Indemnification and Insurance

View Set

South and Southeast Asia Questions

View Set

Family Med Aquifer Questions (2021)

View Set